Π­ΠΊΡΠΏΠ΅Ρ€ΠΈΠΌΠ΅Π½Ρ‚Π°Π»ΡŒΠ½Π°Ρ Ρ€Π°Π±ΠΎΡ‚Π° ΠΏΠΎ использованию элСктронных ΡƒΡ‡Π΅Π±Π½ΠΈΠΊΠΎΠ² Π² процСссС изучСния Ρ„ΠΈΠ·ΠΈΠΊΠ΅. Π˜Π·ΠΌΠ΅Ρ€Π΅Π½ΠΈΠ΅ ΠΌΠΎΠΌΠ΅Π½Ρ‚Π° силы

1.ΠŸΠΎΡΡΠ½ΠΈΡ‚Π΅Π»ΡŒΠ½Π°Ρ записка.

ΠžΠ±ΡƒΡ‡Π΅Π½ΠΈΠ΅ Ρ„ΠΈΠ·ΠΈΠΊΠ΅ Π² ΡΡ‚Π°Ρ€ΡˆΠ΅ΠΉ школС строится Π½Π° Π±Π°Π·Π΅ курса Ρ„ΠΈΠ·ΠΈΠΊΠΈ основной ΡˆΠΊΠΎΠ»Ρ‹ ΠΏΡ€ΠΈ условии Π΄ΠΈΡ„Ρ„Π΅Ρ€Π΅Π½Ρ†ΠΈΠ°Ρ†ΠΈΠΈ. Π‘ΠΎΠ΄Π΅Ρ€ΠΆΠ°Π½ΠΈΠ΅ образования Π΄ΠΎΠ»ΠΆΠ½ΠΎ ΡΠΏΠΎΡΠΎΠ±ΡΡ‚Π²ΠΎΠ²Π°Ρ‚ΡŒ ΠΎΡΡƒΡ‰Π΅ΡΡ‚Π²Π»Π΅Π½ΠΈΡŽ Ρ€Π°Π·Π½ΠΎΡƒΡ€ΠΎΠ²Π½Π΅Π²ΠΎΠ³ΠΎ ΠΏΠΎΠ΄Ρ…ΠΎΠ΄Π°. Π›ΠΈΡ†Π΅ΠΉ β„– 44 Π½Π°Ρ†Π΅Π»Π΅Π½ Π½Π° ΠΎΠΏΡ‚ΠΈΠΌΠ°Π»ΡŒΠ½ΠΎΠ΅ Ρ€Π°Π·Π²ΠΈΡ‚ΠΈΠ΅ творчСских способностСй учащихся, ΠΏΡ€ΠΎΡΠ²Π»ΡΡŽΡ‰ΠΈΡ… особый интСрСс Π² области Ρ„ΠΈΠ·ΠΈΠΊΠΈ; этот ΡƒΡ€ΠΎΠ²Π΅Π½ΡŒ прСподавания осущСствляСтся Π² классах с ΡƒΠ³Π»ΡƒΠ±Π»Π΅Π½Π½Ρ‹ΠΌ ΠΈΠ·ΡƒΡ‡Π΅Π½ΠΈΠ΅ΠΌ Ρ„ΠΈΠ·ΠΈΠΊΠΈ.

ΠžΠ±ΡŠΠ΅ΠΊΡ‚Π°ΠΌΠΈ изучСния Π² курсС Ρ„ΠΈΠ·ΠΈΠΊΠΈ Π½Π° доступном для учащихся ΡƒΡ€ΠΎΠ²Π½Π΅ наряду с Ρ„ΡƒΠ½Π΄Π°ΠΌΠ΅Π½Ρ‚Π°Π»ΡŒΠ½Ρ‹ΠΌΠΈ физичСскими понятиями ΠΈ Π·Π°ΠΊΠΎΠ½Π°ΠΌΠΈ Π΄ΠΎΠ»ΠΆΠ½Ρ‹ Π±Ρ‹Ρ‚ΡŒ экспСримСнт ΠΊΠ°ΠΊ ΠΌΠ΅Ρ‚ΠΎΠ΄ познания, ΠΌΠ΅Ρ‚ΠΎΠ΄ построСния ΠΌΠΎΠ΄Π΅Π»Π΅ΠΉ ΠΈ ΠΌΠ΅Ρ‚ΠΎΠ΄ ΠΈΡ… тСорСтичСского Π°Π½Π°Π»ΠΈΠ·Π°. Выпускники лицСя Π΄ΠΎΠ»ΠΆΠ½Ρ‹ ΠΏΠΎΠ½ΠΈΠΌΠ°Ρ‚ΡŒ, Π² Ρ‡Π΅ΠΌ ΡΡƒΡ‚ΡŒ ΠΌΠΎΠ΄Π΅Π»Π΅ΠΉ ΠΏΡ€ΠΈΡ€ΠΎΠ΄Π½Ρ‹Ρ… ΠΎΠ±ΡŠΠ΅ΠΊΡ‚ΠΎΠ² (процСссов) ΠΈ Π³ΠΈΠΏΠΎΡ‚Π΅Π·, ΠΊΠ°ΠΊ Π΄Π΅Π»Π°ΡŽΡ‚ΡΡ тСорСтичСскиС Π²Ρ‹Π²ΠΎΠ΄Ρ‹, ΠΊΠ°ΠΊ ΡΠΊΡΠΏΠ΅Ρ€ΠΈΠΌΠ΅Π½Ρ‚Π°Π»ΡŒΠ½ΠΎ ΠΏΡ€ΠΎΠ²Π΅Ρ€ΡΡ‚ΡŒ ΠΌΠΎΠ΄Π΅Π»ΠΈ, Π³ΠΈΠΏΠΎΡ‚Π΅Π·Ρ‹ ΠΈ тСорСтичСскиС Π²Ρ‹Π²ΠΎΠ΄Ρ‹.

Π’ Π»ΠΈΡ†Π΅Π΅ количСство часов ΠΏΠΎ Ρ„ΠΈΠ·ΠΈΠΊΠ΅ Π² ΡƒΠ³Π»ΡƒΠ±Π»Π΅Π½Π½Ρ‹Ρ… классах Π½Π΅ соотвСтствуСт Π½ΠΎΠ²ΠΎΠΌΡƒ статусу Ρ„ΠΈΠ·ΠΈΠΊΠΎ-матСматичСского лицСя: Π² 9 классах – 2 часа. Π’ связи с этим прСдлагаСтся ΡƒΡ€ΠΎΠΊΠΈ Ρ‚Π΅Ρ…Π½ΠΎΠ»ΠΎΠ³ΠΈΠΈ Π² 9 классС (1 час Π² нСдСлю с Π΄Π΅Π»Π΅Π½ΠΈΠ΅ΠΌ Π½Π° Π΄Π²Π΅ Π³Ρ€ΡƒΠΏΠΏΡ‹) Π·Π°ΠΌΠ΅Π½ΠΈΡ‚ΡŒ Π½Π° ΠΏΡ€Π°ΠΊΡ‚ΠΈΡ‡Π΅ΡΠΊΡƒΡŽ ΡΠΊΡΠΏΠ΅Ρ€ΠΈΠΌΠ΅Π½Ρ‚Π°Π»ΡŒΠ½ΡƒΡŽ Ρ„ΠΈΠ·ΠΈΠΊΡƒ Π΄ΠΎΠΏΠΎΠ»Π½ΠΈΡ‚Π΅Π»ΡŒΠ½ΠΎ ΠΊ основным ΡƒΡ€ΠΎΠΊΠ°ΠΌ ΠΏΠΎ сСткС часов.

ЦСль курса - прСдоставлСниС учащимся возмоТности ΡƒΠ΄ΠΎΠ²Π»Π΅Ρ‚Π²ΠΎΡ€ΠΈΡ‚ΡŒ ΠΈΠ½Π΄ΠΈΠ²ΠΈΠ΄ΡƒΠ°Π»ΡŒΠ½Ρ‹ΠΉ интСрСс ΠΊ ΠΈΠ·ΡƒΡ‡Π΅Π½ΠΈΡŽ практичСских ΠΏΡ€ΠΈΠ»ΠΎΠΆΠ΅Π½ΠΈΠΉ Ρ„ΠΈΠ·ΠΈΠΊΠΈ Π² процСссС ΠΏΠΎΠ·Π½Π°Π²Π°Ρ‚Π΅Π»ΡŒΠ½ΠΎΠΉ ΠΈ творчСской Π΄Π΅ΡΡ‚Π΅Π»ΡŒΠ½ΠΎΡΡ‚ΠΈ ΠΏΡ€ΠΈ ΠΏΡ€ΠΎΠ²Π΅Π΄Π΅Π½ΠΈΠΈ ΡΠ°ΠΌΠΎΡΡ‚ΠΎΡΡ‚Π΅Π»ΡŒΠ½Ρ‹Ρ… экспСримСнтов ΠΈ исслСдований.

Основная Π·Π°Π΄Π°Ρ‡Π° курса - ΠΏΠΎΠΌΠΎΡ‰ΡŒ учащимся Π² обоснованном Π²Ρ‹Π±ΠΎΡ€Π΅ профиля дальнСйшСго обучСния.

ΠŸΡ€ΠΎΠ³Ρ€Π°ΠΌΠΌΠ° состоит ΠΈΠ· ΡΠ»Π΅Π΄ΡƒΡŽΡ‰ΠΈΡ… частСй: Π°) ΠΏΠΎΠ³Ρ€Π΅ΡˆΠ½ΠΎΡΡ‚ΠΈ; Π±) Π»Π°Π±ΠΎΡ€Π°Ρ‚ΠΎΡ€Π½Ρ‹Π΅ Ρ€Π°Π±ΠΎΡ‚Ρ‹; Π²) ΡΠΊΡΠΏΠ΅Ρ€ΠΈΠΌΠ΅Π½Ρ‚Π°Π»ΡŒΠ½Ρ‹Π΅ Ρ€Π°Π±ΠΎΡ‚Ρ‹; Π³) ΡΠΊΡΠΏΠ΅Ρ€ΠΈΠΌΠ΅Π½Ρ‚Π°Π»ΡŒΠ½Ρ‹Π΅ Π·Π°Π΄Π°Ρ‡ΠΈ; Π΄) тСстированиС.

На элСктивных занятиях школьники Π½Π° ΠΏΡ€Π°ΠΊΡ‚ΠΈΠΊΠ΅ познакомятся с Ρ‚Π΅ΠΌΠΈ Π²ΠΈΠ΄Π°ΠΌΠΈ Π΄Π΅ΡΡ‚Π΅Π»ΡŒΠ½ΠΎΡΡ‚ΠΈ, ΠΊΠΎΡ‚ΠΎΡ€Ρ‹Π΅ ΡΠ²Π»ΡΡŽΡ‚ΡΡ Π²Π΅Π΄ΡƒΡ‰ΠΈΠΌΠΈ Π²ΠΎ ΠΌΠ½ΠΎΠ³ΠΈΡ… ΠΈΠ½ΠΆΠ΅Π½Π΅Ρ€Π½Ρ‹Ρ… ΠΈ тСхничСских профСссиях, связанных с практичСским ΠΏΡ€ΠΈΠΌΠ΅Π½Π΅Π½ΠΈΠ΅ΠΌ Ρ„ΠΈΠ·ΠΈΠΊΠΈ. ΠžΠΏΡ‹Ρ‚ ΡΠ°ΠΌΠΎΡΡ‚ΠΎΡΡ‚Π΅Π»ΡŒΠ½ΠΎΠ³ΠΎ выполнСния сначала простых физичСских экспСримСнтов, Π·Π°Ρ‚Π΅ΠΌ Π·Π°Π΄Π°Π½ΠΈΠΉ ΠΈΡΡΠ»Π΅Π΄ΠΎΠ²Π°Ρ‚Π΅Π»ΡŒΡΠΊΠΎΠ³ΠΎ ΠΈ конструкторского Ρ‚ΠΈΠΏΠ° ΠΏΠΎΠ·Π²ΠΎΠ»ΠΈΡ‚ Π»ΠΈΠ±ΠΎ ΡƒΠ±Π΅Π΄ΠΈΡ‚ΡŒΡΡ Π² ΠΏΡ€Π°Π²ΠΈΠ»ΡŒΠ½ΠΎΡΡ‚ΠΈ ΠΏΡ€Π΅Π΄Π²Π°Ρ€ΠΈΡ‚Π΅Π»ΡŒΠ½ΠΎΠ³ΠΎ Π²Ρ‹Π±ΠΎΡ€Π°, Π»ΠΈΠ±ΠΎ ΠΈΠ·ΠΌΠ΅Π½ΠΈΡ‚ΡŒ свой Π²Ρ‹Π±ΠΎΡ€ ΠΈ ΠΏΠΎΠΏΡ€ΠΎΠ±ΠΎΠ²Π°Ρ‚ΡŒ сСбя Π² ΠΊΠ°ΠΊΠΎΠΌ-Ρ‚ΠΎ ΠΈΠ½ΠΎΠΌ Π½Π°ΠΏΡ€Π°Π²Π»Π΅Π½ΠΈΠΈ.

ΠŸΡ€ΠΈ этом тСорСтичСскиС занятия цСлСсообразны лишь Π½Π° ΠΏΠ΅Ρ€Π²ΠΎΠΌ этапС ΠΏΡ€ΠΈ Ρ„ΠΎΡ€ΠΌΠΈΡ€ΠΎΠ²Π°Π½ΠΈΠΈ Π³Ρ€ΡƒΠΏΠΏΡ‹ ΠΈ ΠΎΠΏΡ€Π΅Π΄Π΅Π»Π΅Π½ΠΈΠΈ интСрСсов ΠΈ способностСй учащихся.

ΠžΡΠ½ΠΎΠ²Π½Ρ‹ΠΌΠΈ Ρ„ΠΎΡ€ΠΌΠ°ΠΌΠΈ занятий Π΄ΠΎΠ»ΠΆΠ½Ρ‹ ΡΡ‚Π°Ρ‚ΡŒ практичСскиС Ρ€Π°Π±ΠΎΡ‚Ρ‹ учащихся Π² физичСской Π»Π°Π±ΠΎΡ€Π°Ρ‚ΠΎΡ€ΠΈΠΈ ΠΈ Π²Ρ‹ΠΏΠΎΠ»Π½Π΅Π½ΠΈΠ΅ простых ΡΠΊΡΠΏΠ΅Ρ€ΠΈΠΌΠ΅Π½Ρ‚Π°Π»ΡŒΠ½Ρ‹Ρ… Π·Π°Π΄Π°Π½ΠΈΠΉ Π² Π΄ΠΎΠΌΠ°ΡˆΠ½ΠΈΡ… условиях.

На практичСских занятиях ΠΏΡ€ΠΈ Π²Ρ‹ΠΏΠΎΠ»Π½Π΅Π½ΠΈΠΈ Π»Π°Π±ΠΎΡ€Π°Ρ‚ΠΎΡ€Π½Ρ‹Ρ… Ρ€Π°Π±ΠΎΡ‚ учащиСся смогут приобрСсти Π½Π°Π²Ρ‹ΠΊΠΈ планирования физичСского экспСримСнта Π² соотвСтствии с поставлСнной Π·Π°Π΄Π°Ρ‡Π΅ΠΉ, научатся Π²Ρ‹Π±ΠΈΡ€Π°Ρ‚ΡŒ Ρ€Π°Ρ†ΠΈΠΎΠ½Π°Π»ΡŒΠ½Ρ‹ΠΉ ΠΌΠ΅Ρ‚ΠΎΠ΄ ΠΈΠ·ΠΌΠ΅Ρ€Π΅Π½ΠΈΠΉ, Π²Ρ‹ΠΏΠΎΠ»Π½ΡΡ‚ΡŒ экспСримСнт ΠΈ ΠΎΠ±Ρ€Π°Π±Π°Ρ‚Ρ‹Π²Π°Ρ‚ΡŒ Π΅Π³ΠΎ Ρ€Π΅Π·ΡƒΠ»ΡŒΡ‚Π°Ρ‚Ρ‹. Π’Ρ‹ΠΏΠΎΠ»Π½Π΅Π½ΠΈΠ΅ практичСских ΠΈ ΡΠΊΡΠΏΠ΅Ρ€ΠΈΠΌΠ΅Π½Ρ‚Π°Π»ΡŒΠ½Ρ‹Ρ… Π·Π°Π΄Π°Π½ΠΈΠΉ ΠΏΠΎΠ·Π²ΠΎΠ»ΠΈΡ‚ ΠΏΡ€ΠΈΠΌΠ΅Π½ΠΈΡ‚ΡŒ ΠΏΡ€ΠΈΠΎΠ±Ρ€Π΅Ρ‚Π΅Π½Π½Ρ‹Π΅ Π½Π°Π²Ρ‹ΠΊΠΈ Π² нСстандартной обстановкС, ΡΡ‚Π°Ρ‚ΡŒ ΠΊΠΎΠΌΠΏΠ΅Ρ‚Π΅Π½Ρ‚Π½Ρ‹ΠΌΠΈ Π²ΠΎ ΠΌΠ½ΠΎΠ³ΠΈΡ… практичСских вопросах.

ВсС Π²ΠΈΠ΄Ρ‹ практичСских Π·Π°Π΄Π°Π½ΠΈΠΉ рассчитаны Π½Π° использованиС Ρ‚ΠΈΠΏΠΎΠ²ΠΎΠ³ΠΎ оборудования ΠΊΠ°Π±ΠΈΠ½Π΅Ρ‚Π° Ρ„ΠΈΠ·ΠΈΠΊΠΈ ΠΈ ΠΌΠΎΠ³ΡƒΡ‚ Π²Ρ‹ΠΏΠΎΠ»Π½ΡΡ‚ΡŒΡΡ Π² Ρ„ΠΎΡ€ΠΌΠ΅ Π»Π°Π±ΠΎΡ€Π°Ρ‚ΠΎΡ€Π½Ρ‹Ρ… Ρ€Π°Π±ΠΎΡ‚ ΠΈΠ»ΠΈ Π² качСствС ΡΠΊΡΠΏΠ΅Ρ€ΠΈΠΌΠ΅Π½Ρ‚Π°Π»ΡŒΠ½Ρ‹Ρ… Π·Π°Π΄Π°Π½ΠΈΠΉ ΠΏΠΎ Π²Ρ‹Π±ΠΎΡ€Ρƒ.

Π­Π»Π΅ΠΊΡ‚ΠΈΠ²Π½Ρ‹ΠΉ курс Π½Π°ΠΏΡ€Π°Π²Π»Π΅Π½ Π½Π° воспитаниС Ρƒ школьников увСрСнности Π² своих силах ΠΈ ΡƒΠΌΠ΅Π½ΠΈΠ΅ ΠΈΡΠΏΠΎΠ»ΡŒΠ·ΠΎΠ²Π°Ρ‚ΡŒ Ρ€Π°Π·Π½ΠΎΠΎΠ±Ρ€Π°Π·Π½Ρ‹Π΅ ΠΏΡ€ΠΈΠ±ΠΎΡ€Ρ‹ ΠΈ устройства Π±Ρ‹Ρ‚ΠΎΠ²ΠΎΠΉ Ρ‚Π΅Ρ…Π½ΠΈΠΊΠΈ Π² повсСднСвной ΠΆΠΈΠ·Π½ΠΈ, Π° Ρ‚Π°ΠΊΠΆΠ΅ Π½Π° Ρ€Π°Π·Π²ΠΈΡ‚ΠΈΠ΅ интСрСса ΠΊ Π²Π½ΠΈΠΌΠ°Ρ‚Π΅Π»ΡŒΠ½ΠΎΠΌΡƒ Ρ€Π°ΡΡΠΌΠΎΡ‚Ρ€Π΅Π½ΠΈΡŽ ΠΏΡ€ΠΈΠ²Ρ‹Ρ‡Π½Ρ‹Ρ… явлСний, ΠΏΡ€Π΅Π΄ΠΌΠ΅Ρ‚ΠΎΠ². Π–Π΅Π»Π°Π½ΠΈΠ΅ ΠΏΠΎΠ½ΡΡ‚ΡŒ, Ρ€Π°Π·ΠΎΠ±Ρ€Π°Ρ‚ΡŒΡΡ Π² сущности явлСний, Π² устройствС Π²Π΅Ρ‰Π΅ΠΉ, ΠΊΠΎΡ‚ΠΎΡ€Ρ‹Π΅ слуТат Ρ‡Π΅Π»ΠΎΠ²Π΅ΠΊΡƒ всю Тизнь, Π½Π΅ΠΌΠΈΠ½ΡƒΠ΅ΠΌΠΎ ΠΏΠΎΡ‚Ρ€Π΅Π±ΡƒΠ΅Ρ‚ Π΄ΠΎΠΏΠΎΠ»Π½ΠΈΡ‚Π΅Π»ΡŒΠ½Ρ‹Ρ… Π·Π½Π°Π½ΠΈΠΉ, ΠΏΠΎΠ΄Ρ‚ΠΎΠ»ΠΊΠ½Π΅Ρ‚ ΠΊ ΡΠ°ΠΌΠΎΠΎΠ±Ρ€Π°Π·ΠΎΠ²Π°Π½ΠΈΡŽ, заставит Π½Π°Π±Π»ΡŽΠ΄Π°Ρ‚ΡŒ, Π΄ΡƒΠΌΠ°Ρ‚ΡŒ, Ρ‡ΠΈΡ‚Π°Ρ‚ΡŒ, ΠΈΠ·ΠΎΠ±Ρ€Π΅Ρ‚Π°Ρ‚ΡŒ.

ΠœΠ΅Ρ‚ΠΎΠ΄Ρ‹ измСрСния физичСских Π²Π΅Π»ΠΈΡ‡ΠΈΠ½ (2 часа).

ΠžΡΠ½ΠΎΠ²Π½Ρ‹Π΅ ΠΈ ΠΏΡ€ΠΎΠΈΠ·Π²ΠΎΠ΄Π½Ρ‹Π΅ физичСскиС Π²Π΅Π»ΠΈΡ‡ΠΈΠ½Ρ‹ ΠΈ ΠΈΡ… измСрСния. Π•Π΄ΠΈΠ½ΠΈΡ†Ρ‹ ΠΈ эталоны Π²Π΅Π»ΠΈΡ‡ΠΈΠ½. ΠΠ±ΡΠΎΠ»ΡŽΡ‚Π½Ρ‹Π΅ ΠΈ ΠΎΡ‚Π½ΠΎΡΠΈΡ‚Π΅Π»ΡŒΠ½Ρ‹Π΅ ΠΏΠΎΠ³Ρ€Π΅ΡˆΠ½ΠΎΡΡ‚ΠΈ прямых ΠΈΠ·ΠΌΠ΅Ρ€Π΅Π½ΠΈΠΉ. Π˜Π·ΠΌΠ΅Ρ€ΠΈΡ‚Π΅Π»ΡŒΠ½Ρ‹Π΅ ΠΏΡ€ΠΈΠ±ΠΎΡ€Ρ‹, инструмСнты, ΠΌΠ΅Ρ€Ρ‹. Π˜Π½ΡΡ‚Ρ€ΡƒΠΌΠ΅Π½Ρ‚Π°Π»ΡŒΠ½Ρ‹Π΅ ΠΏΠΎΠ³Ρ€Π΅ΡˆΠ½ΠΎΡΡ‚ΠΈ ΠΈ ΠΏΠΎΠ³Ρ€Π΅ΡˆΠ½ΠΎΡΡ‚ΠΈ отсчСта. ΠšΠ»Π°ΡΡΡ‹ точности ΠΏΡ€ΠΈΠ±ΠΎΡ€ΠΎΠ². Π“Ρ€Π°Π½ΠΈΡ†Ρ‹ систСматичСских ΠΏΠΎΠ³Ρ€Π΅ΡˆΠ½ΠΎΡΡ‚Π΅ΠΉ ΠΈ способы ΠΈΡ… ΠΎΡ†Π΅Π½ΠΊΠΈ. Π‘Π»ΡƒΡ‡Π°ΠΉΠ½Ρ‹Π΅ ΠΏΠΎΠ³Ρ€Π΅ΡˆΠ½ΠΎΡΡ‚ΠΈ ΠΈΠ·ΠΌΠ΅Ρ€Π΅Π½ΠΈΠΉ ΠΈ ΠΎΡ†Π΅Π½ΠΊΠ° ΠΈΡ… Π³Ρ€Π°Π½ΠΈΡ†.

Π­Ρ‚Π°ΠΏΡ‹ планирования ΠΈ выполнСния экспСримСнта. ΠœΠ΅Ρ€Ρ‹ прСдостороТности ΠΏΡ€ΠΈ ΠΏΡ€ΠΎΠ²Π΅Π΄Π΅Π½ΠΈΠΈ экспСримСнта. Π£Ρ‡Π΅Ρ‚ влияния ΠΈΠ·ΠΌΠ΅Ρ€ΠΈΡ‚Π΅Π»ΡŒΠ½Ρ‹Ρ… ΠΏΡ€ΠΈΠ±ΠΎΡ€ΠΎΠ² Π½Π° исслСдуСмый процСсс. Π’Ρ‹Π±ΠΎΡ€ ΠΌΠ΅Ρ‚ΠΎΠ΄Π° ΠΈΠ·ΠΌΠ΅Ρ€Π΅Π½ΠΈΠΉ ΠΈ ΠΈΠ·ΠΌΠ΅Ρ€ΠΈΡ‚Π΅Π»ΡŒΠ½Ρ‹Ρ… ΠΏΡ€ΠΈΠ±ΠΎΡ€ΠΎΠ².

Бпособы контроля Ρ€Π΅Π·ΡƒΠ»ΡŒΡ‚Π°Ρ‚ΠΎΠ² ΠΈΠ·ΠΌΠ΅Ρ€Π΅Π½ΠΈΠΉ. Π—Π°ΠΏΠΈΡΡŒ Ρ€Π΅Π·ΡƒΠ»ΡŒΡ‚Π°Ρ‚ΠΎΠ² ΠΈΠ·ΠΌΠ΅Ρ€Π΅Π½ΠΈΠΉ. Π’Π°Π±Π»ΠΈΡ†Ρ‹ ΠΈ Π³Ρ€Π°Ρ„ΠΈΠΊΠΈ. ΠžΠ±Ρ€Π°Π±ΠΎΡ‚ΠΊΠ° Ρ€Π΅Π·ΡƒΠ»ΡŒΡ‚Π°Ρ‚ΠΎΠ² ΠΈΠ·ΠΌΠ΅Ρ€Π΅Π½ΠΈΠΉ. ΠžΠ±ΡΡƒΠΆΠ΄Π΅Π½ΠΈΠ΅ ΠΈ прСдставлСниС ΠΏΠΎΠ»ΡƒΡ‡Π΅Π½Π½Ρ‹Ρ… Ρ€Π΅Π·ΡƒΠ»ΡŒΡ‚Π°Ρ‚ΠΎΠ².

Π›Π°Π±ΠΎΡ€Π°Ρ‚ΠΎΡ€Π½Ρ‹Π΅ Ρ€Π°Π±ΠΎΡ‚Ρ‹ (16 часов).

  1. РасчСт ΠΏΠΎΠ³Ρ€Π΅ΡˆΠ½ΠΎΡΡ‚Π΅ΠΉ ΠΈΠ·ΠΌΠ΅Ρ€Π΅Π½ΠΈΠΉ физичСских Π²Π΅Π»ΠΈΡ‡ΠΈΠ½.
  2. Π˜Π·ΡƒΡ‡Π΅Π½ΠΈΠ΅ равноускорСнного двиТСния.
  3. ΠžΠΏΡ€Π΅Π΄Π΅Π»Π΅Π½ΠΈΠ΅ ускорСния Ρ‚Π΅Π»Π° ΠΏΡ€ΠΈ равноускорСнном Π΄Π²ΠΈΠΆΠ΅Π½ΠΈΠΈ.
  4. Π˜Π·ΠΌΠ΅Ρ€Π΅Π½ΠΈΠ΅ массы Ρ‚Π΅Π»Π°.
  5. Π˜Π·ΡƒΡ‡Π΅Π½ΠΈΠ΅ Π²Ρ‚ΠΎΡ€ΠΎΠ³ΠΎ Π·Π°ΠΊΠΎΠ½Π° ΠΡŒΡŽΡ‚ΠΎΠ½Π°.
  6. ΠžΠΏΡ€Π΅Π΄Π΅Π»Π΅Π½ΠΈΠ΅ ТСсткости ΠΏΡ€ΡƒΠΆΠΈΠ½Ρ‹.
  7. ΠžΠΏΡ€Π΅Π΄Π΅Π»Π΅Π½ΠΈΠ΅ коэффициСнта трСния скольТСния.
  8. Π˜Π·ΡƒΡ‡Π΅Π½ΠΈΠ΅ двиТСния Ρ‚Π΅Π»Π°, Π±Ρ€ΠΎΡˆΠ΅Π½Π½ΠΎΠ³ΠΎ Π³ΠΎΡ€ΠΈΠ·ΠΎΠ½Ρ‚Π°Π»ΡŒΠ½ΠΎ.
  9. Π˜Π·ΡƒΡ‡Π΅Π½ΠΈΠ΅ двиТСния Ρ‚Π΅Π»Π° ΠΏΠΎ окруТности ΠΏΠΎΠ΄ дСйствиСм Π½Π΅ΡΠΊΠΎΠ»ΡŒΠΊΠΈΡ… сил.
  10. ВыяснСниС условий равновСсия Ρ‚Π΅Π» ΠΏΠΎΠ΄ дСйствиСм Π½Π΅ΡΠΊΠΎΠ»ΡŒΠΊΠΈΡ… сил.
  11. ΠžΠΏΡ€Π΅Π΄Π΅Π»Π΅Π½ΠΈΠ΅ Ρ†Π΅Π½Ρ‚Ρ€Π° тяТСсти плоской пластины.
  12. Π˜Π·ΡƒΡ‡Π΅Π½ΠΈΠ΅ Π·Π°ΠΊΠΎΠ½Π° сохранСния ΠΈΠΌΠΏΡƒΠ»ΡŒΡΠ°.
  13. Π˜Π·ΠΌΠ΅Ρ€Π΅Π½ΠΈΠ΅ ΠšΠŸΠ” Π½Π°ΠΊΠ»ΠΎΠ½Π½ΠΎΠΉ плоскости.
  14. Π‘Ρ€Π°Π²Π½Π΅Π½ΠΈΠ΅ ΠΏΡ€ΠΎΠΈΠ·Π²Π΅Π΄Π΅Π½Π½ΠΎΠΉ Ρ€Π°Π±ΠΎΡ‚Ρ‹ с ΠΈΠ·ΠΌΠ΅Π½Π΅Π½ΠΈΠ΅ΠΌ энСргии Ρ‚Π΅Π»Π°.
  15. Π˜Π·ΡƒΡ‡Π΅Π½ΠΈΠ΅ Π·Π°ΠΊΠΎΠ½Π° сохранСния энСргии.
  16. Π˜Π·ΠΌΠ΅Ρ€Π΅Π½ΠΈΠ΅ ускорСния свободного падСния с ΠΏΠΎΠΌΠΎΡ‰ΡŒΡŽ маятника.

Π­ΠΊΡΠΏΠ΅Ρ€ΠΈΠΌΠ΅Π½Ρ‚Π°Π»ΡŒΠ½Ρ‹Π΅ Ρ€Π°Π±ΠΎΡ‚Ρ‹ (4 часа).

  1. РасчСт срСднСй ΠΈ ΠΌΠ³Π½ΠΎΠ²Π΅Π½Π½ΠΎΠΉ скорости.
  2. Π˜Π·ΠΌΠ΅Ρ€Π΅Π½ΠΈΠ΅ скорости Π²Π½ΠΈΠ·Ρƒ Π½Π°ΠΊΠ»ΠΎΠ½Π½ΠΎΠΉ плоскости.
  3. РасчСт ΠΈ ΠΈΠ·ΠΌΠ΅Ρ€Π΅Π½ΠΈΠ΅ скорости ΡˆΠ°Ρ€Π°, ΡΠΊΠ°Ρ‚Ρ‹Π²Π°ΡŽΡ‰Π΅Π³ΠΎΡΡ ΠΏΠΎ Π½Π°ΠΊΠ»ΠΎΠ½Π½ΠΎΠΌΡƒ ΠΆΠ΅Π»ΠΎΠ±Ρƒ.
  4. Π˜Π·ΡƒΡ‡Π΅Π½ΠΈΠ΅ ΠΊΠΎΠ»Π΅Π±Π°Π½ΠΈΠΉ ΠΏΡ€ΡƒΠΆΠΈΠ½Π½ΠΎΠ³ΠΎ маятника.

Π­ΠΊΡΠΏΠ΅Ρ€ΠΈΠΌΠ΅Π½Ρ‚Π°Π»ΡŒΠ½Ρ‹Π΅ Π·Π°Π΄Π°Ρ‡ΠΈ(10 часов).

  1. РСшСниС ΡΠΊΡΠΏΠ΅Ρ€ΠΈΠΌΠ΅Π½Ρ‚Π°Π»ΡŒΠ½Ρ‹Ρ… Π·Π°Π΄Π°Ρ‡ 7 класса (2 часа).
  2. РСшСниС ΡΠΊΡΠΏΠ΅Ρ€ΠΈΠΌΠ΅Π½Ρ‚Π°Π»ΡŒΠ½Ρ‹Ρ… Π·Π°Π΄Π°Ρ‡ 8 класса (2 часа).
  3. РСшСниС ΡΠΊΡΠΏΠ΅Ρ€ΠΈΠΌΠ΅Π½Ρ‚Π°Π»ΡŒΠ½Ρ‹Ρ… Π·Π°Π΄Π°Ρ‡ 9 класса (2 часа).
  4. РСшСниС ΡΠΊΡΠΏΠ΅Ρ€ΠΈΠΌΠ΅Π½Ρ‚Π°Π»ΡŒΠ½Ρ‹Ρ… Π·Π°Π΄Π°Ρ‡ ΠΏΡ€ΠΈ ΠΏΠΎΠΌΠΎΡ‰ΠΈ ΠΊΠΎΠΌΠΏΡŒΡŽΡ‚Π΅Ρ€Π° (4 часа).

ВСстированноС Π·Π°Π΄Π°Π½ΠΈΠ΅ (1 час).

ΠžΠ±ΠΎΠ±Ρ‰Π°ΡŽΡ‰Π΅Π΅ занятиС (1 час).

3.АттСстация учащихся.

ΠžΡΠΎΠ±Π΅Π½Π½ΠΎΡΡ‚ΡΠΌ элСктивных занятий Π½Π°ΠΈΠ±ΠΎΠ»Π΅Π΅ соотвСтствуСт зачСтная Ρ„ΠΎΡ€ΠΌΠ° ΠΎΡ†Π΅Π½ΠΊΠΈ достиТСний учащихся. Π—Π°Ρ‡Π΅Ρ‚ ΠΏΠΎ Π²Ρ‹ΠΏΠΎΠ»Π½Π΅Π½Π½ΠΎΠΉ Π»Π°Π±ΠΎΡ€Π°Ρ‚ΠΎΡ€Π½ΠΎΠΉ Ρ€Π°Π±ΠΎΡ‚Π΅ цСлСсообразно Π²Ρ‹ΡΡ‚Π°Π²Π»ΡΡ‚ΡŒ ΠΏΠΎ прСдставлСнному ΠΏΠΈΡΡŒΠΌΠ΅Π½Π½ΠΎΠΌΡƒ ΠΎΡ‚Ρ‡Π΅Ρ‚Ρƒ, Π² ΠΊΠΎΡ‚ΠΎΡ€ΠΎΠΌ ΠΊΡ€Π°Ρ‚ΠΊΠΎ описаны условия экспСримСнта. Π’ систСматизированном Π²ΠΈΠ΄Π΅ прСдставлСны Ρ€Π΅Π·ΡƒΠ»ΡŒΡ‚Π°Ρ‚Ρ‹ ΠΈΠ·ΠΌΠ΅Ρ€Π΅Π½ΠΈΠΉ ΠΈ сдСланы Π²Ρ‹Π²ΠΎΠ΄Ρ‹.

По Ρ€Π΅Π·ΡƒΠ»ΡŒΡ‚Π°Ρ‚Π°ΠΌ выполнСния творчСских ΡΠΊΡΠΏΠ΅Ρ€ΠΈΠΌΠ΅Π½Ρ‚Π°Π»ΡŒΠ½Ρ‹Ρ… Π·Π°Π΄Π°Π½ΠΈΠΉ, ΠΊΡ€ΠΎΠΌΠ΅ ΠΏΠΈΡΡŒΠΌΠ΅Π½Π½Ρ‹Ρ… ΠΎΡ‚Ρ‡Π΅Ρ‚ΠΎΠ², ΠΏΠΎΠ»Π΅Π·Π½ΠΎ ΠΏΡ€Π°ΠΊΡ‚ΠΈΠΊΠΎΠ²Π°Ρ‚ΡŒ сообщСния Π½Π° ΠΎΠ±Ρ‰Π΅ΠΌ занятии Π³Ρ€ΡƒΠΏΠΏΡ‹ с дСмонстрациСй Π²Ρ‹ΠΏΠΎΠ»Π½Π΅Π½Π½Ρ‹Ρ… экспСримСнтов, ΠΈΠ·Π³ΠΎΡ‚ΠΎΠ²Π»Π΅Π½Π½Ρ‹Ρ… ΠΏΡ€ΠΈΠ±ΠΎΡ€ΠΎΠ². Для провСдСния ΠΎΠ±Ρ‰ΠΈΡ… ΠΈΡ‚ΠΎΠ³ΠΎΠ² занятий всСй Π³Ρ€ΡƒΠΏΠΏΡ‹ Π²ΠΎΠ·ΠΌΠΎΠΆΠ½ΠΎ ΠΏΡ€ΠΎΠ²Π΅Π΄Π΅Π½ΠΈΠ΅ конкурса творчСских Ρ€Π°Π±ΠΎΡ‚. На этом конкурсС учащиСся смогут Π½Π΅ Ρ‚ΠΎΠ»ΡŒΠΊΠΎ ΠΏΡ€ΠΎΠ΄Π΅ΠΌΠΎΠ½ΡΡ‚Ρ€ΠΈΡ€ΠΎΠ²Π°Ρ‚ΡŒ ΡΠΊΡΠΏΠ΅Ρ€ΠΈΠΌΠ΅Π½Ρ‚Π°Π»ΡŒΠ½ΡƒΡŽ установку Π² дСйствиС, Π½ΠΎ ΠΈ Ρ€Π°ΡΡΠΊΠ°Π·Π°Ρ‚ΡŒ ΠΎ Π΅Π΅ ΠΎΡ€ΠΈΠ³ΠΈΠ½Π°Π»ΡŒΠ½ΠΎΡΡ‚ΠΈ ΠΈ возмоТностях. Π—Π΄Π΅ΡΡŒ особСнно Π²Π°ΠΆΠ½ΠΎ ΠΎΡ„ΠΎΡ€ΠΌΠΈΡ‚ΡŒ свой Π΄ΠΎΠΊΠ»Π°Π΄ Π³Ρ€Π°Ρ„ΠΈΠΊΠ°ΠΌΠΈ, Ρ‚Π°Π±Π»ΠΈΡ†Π°ΠΌΠΈ, ΠΊΡ€Π°Ρ‚ΠΊΠΎ ΠΈ ΡΠΌΠΎΡ†ΠΈΠΎΠ½Π°Π»ΡŒΠ½ΠΎ Ρ€Π°ΡΡΠΊΠ°Π·Π°Ρ‚ΡŒ ΠΎ самом Π³Π»Π°Π²Π½ΠΎΠΌ. Π’ этом случаС появляСтся Π²ΠΎΠ·ΠΌΠΎΠΆΠ½ΠΎΡΡ‚ΡŒ ΡƒΠ²ΠΈΠ΄Π΅Ρ‚ΡŒ ΠΈ ΠΎΡ†Π΅Π½ΠΈΡ‚ΡŒ свой Ρ‚Ρ€ΡƒΠ΄ ΠΈ сСбя Π½Π° Ρ„ΠΎΠ½Π΅ Π΄Ρ€ΡƒΠ³ΠΈΡ… интСрСсных Ρ€Π°Π±ΠΎΡ‚ ΠΈ Ρ‚Π°ΠΊΠΈΡ… ΠΆΠ΅ ΡƒΠ²Π»Π΅Ρ‡Π΅Π½Π½Ρ‹Ρ… людСй.

Π˜Ρ‚ΠΎΠ³ΠΎΠ²Ρ‹ΠΉ Π·Π°Ρ‡Π΅Ρ‚ ΡƒΡ‡Π΅Π½ΠΈΠΊΠΎΠΌ ΠΏΠΎ всСму элСктивному курсу ΠΌΠΎΠΆΠ½ΠΎ Π²Ρ‹ΡΡ‚Π°Π²Π»ΡΡ‚ΡŒ, Π½Π°ΠΏΡ€ΠΈΠΌΠ΅Ρ€, ΠΏΠΎ Ρ‚Π°ΠΊΠΈΠΌ критСриям: Π²Ρ‹ΠΏΠΎΠ»Π½Π΅Π½ΠΈΠ΅ Π½Π΅ ΠΌΠ΅Π½Π΅Π΅ ΠΏΠΎΠ»ΠΎΠ²ΠΈΠ½Ρ‹ Π»Π°Π±ΠΎΡ€Π°Ρ‚ΠΎΡ€Π½Ρ‹Ρ… Ρ€Π°Π±ΠΎΡ‚; Π²Ρ‹ΠΏΠΎΠ»Π½Π΅Π½ΠΈΠ΅ Π½Π΅ ΠΌΠ΅Π½Π΅Π΅ ΠΎΠ΄Π½ΠΎΠ³ΠΎ ΡΠΊΡΠΏΠ΅Ρ€ΠΈΠΌΠ΅Π½Ρ‚Π°Π»ΡŒΠ½ΠΎΠ³ΠΎ задания ΠΈΡΡΠ»Π΅Π΄ΠΎΠ²Π°Ρ‚Π΅Π»ΡŒΡΠΊΠΎΠ³ΠΎ ΠΈΠ»ΠΈ конструкторского Ρ‚ΠΈΠΏΠ°; Π°ΠΊΡ‚ΠΈΠ²Π½ΠΎΠ΅ участиС Π² ΠΏΠΎΠ΄Π³ΠΎΡ‚ΠΎΠ²ΠΊΠ΅ ΠΈ ΠΏΡ€ΠΎΠ²Π΅Π΄Π΅Π½ΠΈΠΈ сСминаров, дискуссий, конкурсов.

ΠŸΡ€Π΅Π΄Π»Π°Π³Π°Π΅ΠΌΡ‹Π΅ ΠΊΡ€ΠΈΡ‚Π΅Ρ€ΠΈΠΈ ΠΎΡ†Π΅Π½ΠΊΠΈ достиТСния учащихся ΠΌΠΎΠ³ΡƒΡ‚ ΡΠ»ΡƒΠΆΠΈΡ‚ΡŒ лишь ΠΎΡ€ΠΈΠ΅Π½Ρ‚ΠΈΡ€ΠΎΠΌ, Π½ΠΎ Π½Π΅ ΡΠ²Π»ΡΡŽΡ‚ΡΡ ΠΎΠ±ΡΠ·Π°Ρ‚Π΅Π»ΡŒΠ½Ρ‹ΠΌΠΈ. На основС своСго ΠΎΠΏΡ‹Ρ‚Π° ΡƒΡ‡ΠΈΡ‚Π΅Π»ΡŒ ΠΌΠΎΠΆΠ΅Ρ‚ ΡƒΡΡ‚Π°Π½Π°Π²Π»ΠΈΠ²Π°Ρ‚ΡŒ ΠΈΠ½Ρ‹Π΅ ΠΊΡ€ΠΈΡ‚Π΅Ρ€ΠΈΠΈ.

4. Π›ΠΈΡ‚Π΅Ρ€Π°Ρ‚ΡƒΡ€Π°:

  1. ДСмонстрационный экспСримСнт ΠΏΠΎ Ρ„ΠΈΠ·ΠΈΠΊΠ΅ Π² срСднСй школС./Под Ρ€Π΅Π΄. А. А. ΠŸΠΎΠΊΡ€ΠΎΠ²
    ского. Π§. 1.- М.:ΠŸΡ€ΠΎΡΠ²Π΅Ρ‰Π΅Π½ΠΈΠ΅,1978.
  2. ΠœΠ΅Ρ‚ΠΎΠ΄ΠΈΠΊΠ° прСподавания Ρ„ΠΈΠ·ΠΈΠΊΠΈ Π² 7-11 классах срСднСй ΡˆΠΊΠΎΠ»Ρ‹./Под Ρ€Π΅Π΄Π°ΠΊΡ†ΠΈΠ΅ΠΉ Π’.П.
    ΠžΡ€Π΅Ρ…ΠΎΠ²Π° ΠΈ А.Π’. Усовой. - М.:ΠŸΡ€ΠΎΡΠ²Π΅Ρ‰Π΅Π½ΠΈΠ΅,1999.
  3. ΠœΠ°Ρ€Ρ‚Ρ‹Π½ΠΎΠ² И.М., Π₯озяинова Π­.Н. ДидактичСский ΠΌΠ°Ρ‚Π΅Ρ€ΠΈΠ°Π» ΠΏΠΎ Ρ„ΠΈΠ·ΠΈΠΊΠ΅. 9 класс. - М.:
    ΠŸΡ€ΠΎΡΠ²Π΅Ρ‰Π΅Π½ΠΈΠ΅,1995.
  4. Π’.А.Π‘ΡƒΡ€ΠΎΠ², А.И.Иванов, Π’.И.Π‘Π²ΠΈΡ€ΠΈΠ΄ΠΎΠ². Π€Ρ€ΠΎΠ½Ρ‚Π°Π»ΡŒΠ½Ρ‹Π΅ ΡΠΊΡΠΏΠ΅Ρ€ΠΈΠΌΠ΅Π½Ρ‚Π°Π»ΡŒΠ½Ρ‹Π΅ задания ΠΏΠΎ
    Ρ„ΠΈΠ·ΠΈΠΊΠ΅.9 класс.– М: ΠŸΡ€ΠΎΡΠ²Π΅Ρ‰Π΅Π½ΠΈΠ΅.1988.
  5. Π Ρ‹ΠΌΠΊΠ΅Π²ΠΈΡ‡ А.П., Π Ρ‹ΠΌΠΊΠ΅Π²ΠΈΡ‡ П.А. Π‘Π±ΠΎΡ€Π½ΠΈΠΊ Π·Π°Π΄Π°Ρ‡ ΠΏΠΎ Ρ„ΠΈΠ·ΠΈΠΊΠ΅ для 9 – 11 классов. – М.: ΠŸΡ€ΠΎ
    свСщСниС, 2000.
  6. Π‘Ρ‚Π΅ΠΏΠ°Π½ΠΎΠ²Π° Π“.Н. Π‘Π±ΠΎΡ€Π½ΠΈΠΊ Π·Π°Π΄Π°Ρ‡ ΠΏΠΎ Ρ„ΠΈΠ·ΠΈΠΊΠ΅: Для 9-11 классов ΠΎΠ±Ρ‰Π΅ΠΎΠ±Ρ€Π°Π·ΠΎΠ²Π°Ρ‚Π΅Π»ΡŒΠ½Ρ‹Ρ… ΡƒΡ‡
    Ρ€Π΅ΠΆΠ΄Π΅Π½ΠΈΠΉ. - М.: ΠŸΡ€ΠΎΡΠ²Π΅Ρ‰Π΅Π½ΠΈΠ΅,1998.
  7. Π“ΠΎΡ€ΠΎΠ΄Π΅Ρ†ΠΊΠΈΠΉ Π”.Н., ПСньков И.А. ΠŸΡ€ΠΎΠ²Π΅Ρ€ΠΎΡ‡Π½Ρ‹Π΅ Ρ€Π°Π±ΠΎΡ‚Ρ‹ ΠΏΠΎ Ρ„ΠΈΠ·ΠΈΠΊΠ΅. – Минск β€œΠ’Ρ‹ΡˆΡΠΉΡˆ
    ΡˆΠΊΠΎΠ»Π°β€, 1987
  8. Π’.А.Π‘ΡƒΡ€ΠΎΠ²,Π‘.Π€.Кабанов, Π’.И.Π‘Π²ΠΈΡ€ΠΈΠ΄ΠΎΠ². β€œΠ€Ρ€ΠΎΠ½Ρ‚Π°Π»ΡŒΠ½Ρ‹Π΅ ΡΠΊΡΠΏΠ΅Ρ€ΠΈΠΌΠ΅Π½Ρ‚Π°Π»ΡŒΠ½Ρ‹Π΅ задания ΠΏΠΎ
    физикС”. – М: ΠŸΡ€ΠΎΡΠ²Π΅Ρ‰Π΅Π½ΠΈΠ΅.1988
  9. Кикоин И.К.,Кикоин А.К.Π€ΠΈΠ·ΠΈΠΊΠ°: Π£Ρ‡Π΅Π±Π½ΠΈΠΊ для 10 классов – М.: ΠŸΡ€ΠΎΡΠ²Π΅Ρ‰Π΅Π½ΠΈΠ΅, 2003

Π’Π•ΠœΠΠ’Π˜Π§Π•Π‘ΠšΠžΠ• ΠŸΠ›ΠΠΠ˜Π ΠžΠ’ΠΠΠ˜Π• ПО Π€Π˜Π—Π˜ΠšΠ• Π’ 9 Π’ ΠšΠ›ΠΠ‘Π‘Π•

Π­Π»Π΅ΠΊΡ‚ΠΈΠ²Π½Ρ‹ΠΉ курс: β€œΠŸΡ€Π°ΠΊΡ‚ΠΈΡ‡Π΅ΡΠΊΠ°Ρ ΠΈ ΡΠΊΡΠΏΠ΅Ρ€ΠΈΠΌΠ΅Π½Ρ‚Π°Π»ΡŒΠ½Π°Ρ физика”

(ΡƒΠ³Π»ΡƒΠ±Π»Π΅Π½Π½ΠΎΠ΅ ΠΈΠ·ΡƒΡ‡Π΅Π½ΠΈΠ΅ - 34 часа)

Π‘Ρ‚ΡƒΠΏΠ΅Π½ΡŒ – Ρ‚Ρ€Π΅Ρ‚ΡŒΡ

Π£Ρ€ΠΎΠ²Π΅Π½ΡŒ – ΡƒΠ³Π»ΡƒΠ±Π»Π΅Π½Π½Ρ‹ΠΉ

β„– Π’ΠΈΠ΄ ΡƒΡ€ΠΎΠΊΠ° Часы Π‘ΠΎΠ΄Π΅Ρ€ΠΆΠ°Π½ΠΈΠ΅ ΡƒΡ€ΠΎΠΊΠ° Π”/Π·
1 ЛСкция 1Ρ‡ Π’Π΅Ρ…Π½ΠΈΠΊΠ° бСзопасности. ΠšΠΎΠ½ΡΠΏΠ΅ΠΊΡ‚
2 ЛСкция 1Ρ‡ ΠŸΠΎΠ³Ρ€Π΅ΡˆΠ½ΠΎΡΡ‚ΠΈ ΠΈΠ·ΠΌΠ΅Ρ€Π΅Π½ΠΈΠΉ физичСских Π²Π΅Π»ΠΈΡ‡ΠΈΠ½. ΠšΠΎΠ½ΡΠΏΠ΅ΠΊΡ‚
3 Лабораторная Ρ€Π°Π±ΠΎΡ‚Π° β„– 1 1Ρ‡ РасчСт ΠΏΠΎΠ³Ρ€Π΅ΡˆΠ½ΠΎΡΡ‚Π΅ΠΉ ΠΈΠ·ΠΌΠ΅Ρ€Π΅Π½ΠΈΠΉ физичСских Π²Π΅Π»ΠΈΡ‡ΠΈΠ½ Π—Π°ΠΊΠΎΠ½Ρ‡ΠΈΡ‚ΡŒ расчСты
4 1Ρ‡ Π·Π°Π΄Π°Ρ‡ΠΈ
5 Π­ΠΊΡΠΏΠ΅Ρ€ΠΈΠΌΠ΅Π½Ρ‚Π°Π»ΡŒΠ½Π°Ρ Ρ€Π°Π±ΠΎΡ‚Π° 1Ρ‡ РасчСт срСднСй ΠΈ ΠΌΠ³Π½ΠΎΠ²Π΅Π½Π½ΠΎΠΉ скорости Π—Π°ΠΊΠΎΠ½Ρ‡ΠΈΡ‚ΡŒ расчСты
6 Лабораторная Ρ€Π°Π±ΠΎΡ‚Π° β„– 2 1Ρ‡ Π˜Π·ΡƒΡ‡Π΅Π½ΠΈΠ΅ равноускорСнного двиТСния Π—Π°ΠΊΠΎΠ½Ρ‡ΠΈΡ‚ΡŒ расчСты
7 Лабораторная Ρ€Π°Π±ΠΎΡ‚Π° β„– 3. 1Ρ‡. ΠžΠΏΡ€Π΅Π΄Π΅Π»Π΅Π½ΠΈΠ΅ ускорСния Ρ‚Π΅Π»Π° ΠΏΡ€ΠΈ равноускорСнном Π΄Π²ΠΈΠΆΠ΅Π½ΠΈΠΈ. Π—Π°ΠΊΠΎΠ½Ρ‡ΠΈΡ‚ΡŒ расчСты
8 Π­ΠΊΡΠΏΠ΅Ρ€ΠΈΠΌΠ΅Π½Ρ‚Π°Π»ΡŒΠ½Π°Ρ Ρ€Π°Π±ΠΎΡ‚Π° 1Ρ‡. Π˜Π·ΠΌΠ΅Ρ€Π΅Π½ΠΈΠ΅ скорости Π²Π½ΠΈΠ·Ρƒ Π½Π°ΠΊΠ»ΠΎΠ½Π½ΠΎΠΉ плоскости. Π—Π°ΠΊΠΎΠ½Ρ‡ΠΈΡ‚ΡŒ расчСты
9 Лабораторная Ρ€Π°Π±ΠΎΡ‚Π° β„– 4 1Ρ‡ Π˜Π·ΠΌΠ΅Ρ€Π΅Π½ΠΈΠ΅ массы Ρ‚Π΅Π» Π—Π°ΠΊΠΎΠ½Ρ‡ΠΈΡ‚ΡŒ расчСты
10 Лабораторная Ρ€Π°Π±ΠΎΡ‚Π° β„– 5 1Ρ‡ Π˜Π·ΡƒΡ‡Π΅Π½ΠΈΠ΅ Π²Ρ‚ΠΎΡ€ΠΎΠ³ΠΎ Π·Π°ΠΊΠΎΠ½Π° ΠΡŒΡŽΡ‚ΠΎΠ½Π° Π—Π°ΠΊΠΎΠ½Ρ‡ΠΈΡ‚ΡŒ расчСты
11 Лабораторная Ρ€Π°Π±ΠΎΡ‚Π° β„– 6 1Ρ‡. ΠžΠΏΡ€Π΅Π΄Π΅Π»Π΅Π½ΠΈΠ΅ ТСсткости ΠΏΡ€ΡƒΠΆΠΈΠ½Ρ‹. Π—Π°ΠΊΠΎΠ½Ρ‡ΠΈΡ‚ΡŒ расчСты
12 Лабораторная Ρ€Π°Π±ΠΎΡ‚Π° β„– 7 1Ρ‡. ΠžΠΏΡ€Π΅Π΄Π΅Π»Π΅Π½ΠΈΠ΅ коэффициСнта трСния скольТСния. Π—Π°ΠΊΠΎΠ½Ρ‡ΠΈΡ‚ΡŒ расчСты
13 Лабораторная Ρ€Π°Π±ΠΎΡ‚Π° β„– 8 1Ρ‡. Π˜Π·ΡƒΡ‡Π΅Π½ΠΈΠ΅ двиТСния Ρ‚Π΅Π»Π°, Π±Ρ€ΠΎΡˆΠ΅Π½Π½ΠΎΠ³ΠΎ Π³ΠΎΡ€ΠΈΠ·ΠΎΠ½Ρ‚Π°Π»ΡŒΠ½ΠΎ. Π—Π°ΠΊΠΎΠ½Ρ‡ΠΈΡ‚ΡŒ расчСты
14 Лабораторная Ρ€Π°Π±ΠΎΡ‚Π° β„– 9 1Ρ‡. Π˜Π·ΡƒΡ‡Π΅Π½ΠΈΠ΅ двиТСния Ρ‚Π΅Π»Π° ΠΏΠΎ окруТности ΠΏΠΎΠ΄ дСйствиСм Π½Π΅ΡΠΊΠΎΠ»ΡŒΠΊΠΈΡ… сил”. Π—Π°ΠΊΠΎΠ½Ρ‡ΠΈΡ‚ΡŒ расчСты
15 РСшСниС ΡΠΊΡΠΏΠ΅Ρ€ΠΈΠΌΠ΅Π½Ρ‚Π°Π»ΡŒΠ½Ρ‹Ρ… Π·Π°Π΄Π°Ρ‡ 1Ρ‡ РСшСниС ΡΠΊΡΠΏΠ΅Ρ€ΠΈΠΌΠ΅Π½Ρ‚Π°Π»ΡŒΠ½Ρ‹Ρ… Π·Π°Π΄Π°Ρ‡ 7 класса Π·Π°Π΄Π°Ρ‡ΠΈ
16 Лабораторная Ρ€Π°Π±ΠΎΡ‚Π° β„– 10 1Ρ‡. ВыяснСниС условий равновСсия Ρ‚Π΅Π» ΠΏΠΎΠ΄ дСйствиСм Π½Π΅ΡΠΊΠΎΠ»ΡŒΠΊΠΈΡ… сил. Π—Π°ΠΊΠΎΠ½Ρ‡ΠΈΡ‚ΡŒ расчСты
17 Лабораторная Ρ€Π°Π±ΠΎΡ‚Π° β„– 11 1Ρ‡. ΠžΠΏΡ€Π΅Π΄Π΅Π»Π΅Π½ΠΈΠ΅ Ρ†Π΅Π½Ρ‚Ρ€Π° тяТСсти плоской пластины. Π—Π°ΠΊΠΎΠ½Ρ‡ΠΈΡ‚ΡŒ расчСты
18 РСшСниС ΡΠΊΡΠΏΠ΅Ρ€ΠΈΠΌΠ΅Π½Ρ‚Π°Π»ΡŒΠ½Ρ‹Ρ… Π·Π°Π΄Π°Ρ‡ 1Ρ‡ Π·Π°Π΄Π°Ρ‡ΠΈ
19 РСшСниС ΡΠΊΡΠΏΠ΅Ρ€ΠΈΠΌΠ΅Π½Ρ‚Π°Π»ΡŒΠ½Ρ‹Ρ… Π·Π°Π΄Π°Ρ‡ 1Ρ‡ РСшСниС ΡΠΊΡΠΏΠ΅Ρ€ΠΈΠΌΠ΅Π½Ρ‚Π°Π»ΡŒΠ½Ρ‹Ρ… Π·Π°Π΄Π°Ρ‡ 8 класса Π·Π°Π΄Π°Ρ‡ΠΈ
20 Лабораторная Ρ€Π°Π±ΠΎΡ‚Π° β„– 12 1Ρ‡ Π˜Π·ΡƒΡ‡Π΅Π½ΠΈΠ΅ Π·Π°ΠΊΠΎΠ½Π° сохранСния ΠΈΠΌΠΏΡƒΠ»ΡŒΡΠ° Π—Π°ΠΊΠΎΠ½Ρ‡ΠΈΡ‚ΡŒ расчСты
21 Лабораторная Ρ€Π°Π±ΠΎΡ‚Π° β„– 13 1Ρ‡ Π˜Π·ΠΌΠ΅Ρ€Π΅Π½ΠΈΠ΅ ΠšΠŸΠ” Π½Π°ΠΊΠ»ΠΎΠ½Π½ΠΎΠΉ плоскости Π—Π°ΠΊΠΎΠ½Ρ‡ΠΈΡ‚ΡŒ расчСты
22 Лабораторная Ρ€Π°Π±ΠΎΡ‚Π° β„–14 1Ρ‡. Π‘Ρ€Π°Π²Π½Π΅Π½ΠΈΠ΅ ΠΏΡ€ΠΎΠΈΠ·Π²Π΅Π΄Π΅Π½Π½ΠΎΠΉ Ρ€Π°Π±ΠΎΡ‚Ρ‹ с ΠΈΠ·ΠΌΠ΅Π½Π΅Π½ΠΈΠ΅ΠΌ энСргии тСла” Π—Π°ΠΊΠΎΠ½Ρ‡ΠΈΡ‚ΡŒ расчСты
23 Лабораторная Ρ€Π°Π±ΠΎΡ‚Π° β„– 15 1Ρ‡ Π˜Π·ΡƒΡ‡Π΅Π½ΠΈΠ΅ Π·Π°ΠΊΠΎΠ½Π° сохранСния энСргии Π—Π°ΠΊΠΎΠ½Ρ‡ΠΈΡ‚ΡŒ расчСты
24 Π­ΠΊΡΠΏΠ΅Ρ€ΠΈΠΌΠ΅Π½Ρ‚Π°Π»ΡŒΠ½Π°Ρ Ρ€Π°Π±ΠΎΡ‚Π° 1Ρ‡ РасчСт ΠΈ ΠΈΠ·ΠΌΠ΅Ρ€Π΅Π½ΠΈΠ΅ скорости ΡˆΠ°Ρ€Π°, ΡΠΊΠ°Ρ‚Ρ‹Π²Π°ΡŽΡ‰Π΅Π³ΠΎΡΡ ΠΏΠΎ Π½Π°ΠΊΠ»ΠΎΠ½Π½ΠΎΠΌΡƒ ΠΆΠ΅Π»ΠΎΠ±Ρƒ Π—Π°ΠΊΠΎΠ½Ρ‡ΠΈΡ‚ΡŒ расчСты
25 РСшСниС ΡΠΊΡΠΏΠ΅Ρ€ΠΈΠΌΠ΅Π½Ρ‚Π°Π»ΡŒΠ½Ρ‹Ρ… Π·Π°Π΄Π°Ρ‡ 1Ρ‡ Π—Π°Π΄Π°Ρ‡ΠΈ
26 РСшСниС ΡΠΊΡΠΏΠ΅Ρ€ΠΈΠΌΠ΅Π½Ρ‚Π°Π»ΡŒΠ½Ρ‹Ρ… Π·Π°Π΄Π°Ρ‡ 1Ρ‡ РСшСниС ΡΠΊΡΠΏΠ΅Ρ€ΠΈΠΌΠ΅Π½Ρ‚Π°Π»ΡŒΠ½Ρ‹Ρ… Π·Π°Π΄Π°Ρ‡ 9 класса Π·Π°Π΄Π°Ρ‡ΠΈ
27 Π­ΠΊΡΠΏΠ΅Ρ€ΠΈΠΌΠ΅Π½Ρ‚Π°Π»ΡŒΠ½Π°Ρ Ρ€Π°Π±ΠΎΡ‚Π° 1Ρ‡ Π˜Π·ΡƒΡ‡Π΅Π½ΠΈΠ΅ ΠΊΠΎΠ»Π΅Π±Π°Π½ΠΈΠΉ ΠΏΡ€ΡƒΠΆΠΈΠ½Π½ΠΎΠ³ΠΎ маятника Π—Π°ΠΊΠΎΠ½Ρ‡ΠΈΡ‚ΡŒ расчСты
28 Лабораторная Ρ€Π°Π±ΠΎΡ‚Π° β„– 16 1Ρ‡ Π˜Π·ΠΌΠ΅Ρ€Π΅Π½ΠΈΠ΅ ускорСния свободного падания с ΠΏΠΎΠΌΠΎΡ‰ΡŒΡŽ маятника Π—Π°ΠΊΠΎΠ½Ρ‡ΠΈΡ‚ΡŒ расчСты
29 1Ρ‡ РСшСниС ΡΠΊΡΠΏΠ΅Ρ€ΠΈΠΌΠ΅Π½Ρ‚Π°Π»ΡŒΠ½Ρ‹Ρ… Π·Π°Π΄Π°Ρ‡ 9 класса Π—Π°ΠΊΠΎΠ½Ρ‡ΠΈΡ‚ΡŒ расчСты
30 РСшСниС ΡΠΊΡΠΏΠ΅Ρ€ΠΈΠΌΠ΅Π½Ρ‚Π°Π»ΡŒΠ½Ρ‹Ρ… Π·Π°Π΄Π°Ρ‡ с ΠΏΠΎΠΌΠΎΡ‰ΡŒΡŽ ΠΊΠΎΠΌΠΏΡŒΡŽΡ‚Π΅Ρ€Π° 1Ρ‡ РСшСниС ΡΠΊΡΠΏΠ΅Ρ€ΠΈΠΌΠ΅Π½Ρ‚Π°Π»ΡŒΠ½Ρ‹Ρ… Π·Π°Π΄Π°Ρ‡ с ΠΏΠΎΠΌΠΎΡ‰ΡŒΡŽ ΠΊΠΎΠΌΠΏΡŒΡŽΡ‚Π΅Ρ€Π° Π—Π°ΠΊΠΎΠ½Ρ‡ΠΈΡ‚ΡŒ расчСты
31 РСшСниС ΡΠΊΡΠΏΠ΅Ρ€ΠΈΠΌΠ΅Π½Ρ‚Π°Π»ΡŒΠ½Ρ‹Ρ… Π·Π°Π΄Π°Ρ‡ с ΠΏΠΎΠΌΠΎΡ‰ΡŒΡŽ ΠΊΠΎΠΌΠΏΡŒΡŽΡ‚Π΅Ρ€Π° 1Ρ‡ РСшСниС ΡΠΊΡΠΏΠ΅Ρ€ΠΈΠΌΠ΅Π½Ρ‚Π°Π»ΡŒΠ½Ρ‹Ρ… Π·Π°Π΄Π°Ρ‡ с ΠΏΠΎΠΌΠΎΡ‰ΡŒΡŽ ΠΊΠΎΠΌΠΏΡŒΡŽΡ‚Π΅Ρ€Π° Π—Π°ΠΊΠΎΠ½Ρ‡ΠΈΡ‚ΡŒ расчСты
32 РСшСниС ΡΠΊΡΠΏΠ΅Ρ€ΠΈΠΌΠ΅Π½Ρ‚Π°Π»ΡŒΠ½Ρ‹Ρ… Π·Π°Π΄Π°Ρ‡ с ΠΏΠΎΠΌΠΎΡ‰ΡŒΡŽ ΠΊΠΎΠΌΠΏΡŒΡŽΡ‚Π΅Ρ€Π° 1Ρ‡ РСшСниС ΡΠΊΡΠΏΠ΅Ρ€ΠΈΠΌΠ΅Π½Ρ‚Π°Π»ΡŒΠ½Ρ‹Ρ… Π·Π°Π΄Π°Ρ‡ с ΠΏΠΎΠΌΠΎΡ‰ΡŒΡŽ ΠΊΠΎΠΌΠΏΡŒΡŽΡ‚Π΅Ρ€Π° Π—Π°ΠΊΠΎΠ½Ρ‡ΠΈΡ‚ΡŒ расчСты
33 ВСстированноС Π·Π°Π΄Π°Π½ΠΈΠ΅ 1Ρ‡ ВСст
34 ΠžΠ±ΠΎΠ±Ρ‰Π°ΡŽΡ‰Π΅Π΅ занятиС 1Ρ‡ ПодвСдСниС ΠΈΡ‚ΠΎΠ³ΠΎΠ² ΠΈ Π·Π°Π΄Π°Ρ‡ΠΈ Π½Π° ΡΠ»Π΅Π΄ΡƒΡŽΡ‰ΠΈΠΉ Π³ΠΎΠ΄

Π›Π˜Π’Π•Π ΠΠ’Π£Π Π:

  1. ДСмонстрационный экспСримСнт ΠΏΠΎ Ρ„ΠΈΠ·ΠΈΠΊΠ΅ Π² срСднСй школС./Под Ρ€Π΅Π΄. А. А. ΠŸΠΎΠΊΡ€ΠΎΠ²ΡΠΊΠΎΠ³ΠΎ. Π§. 1.- М.:ΠŸΡ€ΠΎΡΠ²Π΅Ρ‰Π΅Π½ΠΈΠ΅,1978.
  2. ΠœΠ΅Ρ‚ΠΎΠ΄ΠΈΠΊΠ° прСподавания Ρ„ΠΈΠ·ΠΈΠΊΠΈ Π² 7-11 классах срСднСй ΡˆΠΊΠΎΠ»Ρ‹./Под Ρ€Π΅Π΄Π°ΠΊΡ†ΠΈΠ΅ΠΉ Π’.П. ΠžΡ€Π΅Ρ…ΠΎΠ²Π° ΠΈ А.Π’. Усовой. - М.:ΠŸΡ€ΠΎΡΠ²Π΅Ρ‰Π΅Π½ΠΈΠ΅,1999.
  3. Π•Π½ΠΎΡ…ΠΎΠ²ΠΈΡ‡ А.Π‘. Π‘ΠΏΡ€Π°Π²ΠΎΡ‡Π½ΠΈΠΊ ΠΏΠΎ Ρ„ΠΈΠ·ΠΈΠΊΠ΅. - М.: ΠŸΡ€ΠΎΡΠ²Π΅Ρ‰Π΅Π½ΠΈΠ΅, 1978.
  4. ΠœΠ°Ρ€Ρ‚Ρ‹Π½ΠΎΠ² И.М., Π₯озяинова Π­.Н. ДидактичСский ΠΌΠ°Ρ‚Π΅Ρ€ΠΈΠ°Π» ΠΏΠΎ Ρ„ΠΈΠ·ΠΈΠΊΠ΅. 9 класс. - М.: ΠŸΡ€ΠΎΡΠ²Π΅Ρ‰Π΅Π½ΠΈΠ΅,1995.
  5. Π‘ΠΊΡ€Π΅Π»ΠΈΠ½ Π›.И. ДидактичСский ΠΌΠ°Ρ‚Π΅Ρ€ΠΈΠ°Π» ΠΏΠΎ Ρ„ΠΈΠ·ΠΈΠΊΠ΅. 9 класс. – М.: ΠŸΡ€ΠΎΡΠ²Π΅Ρ‰Π΅Π½ΠΈΠ΅, 1998.
  6. Π₯рСстоматия ΠΏΠΎ Ρ„ΠΈΠ·ΠΈΠΊΠ΅ /Под Ρ€Π΅Π΄. Π‘.И. Бпасского. – М.: ΠŸΡ€ΠΎΡΠ²Π΅Ρ‰Π΅Π½ΠΈΠ΅, 1982.
  7. Π Ρ‹ΠΌΠΊΠ΅Π²ΠΈΡ‡ А.П., Π Ρ‹ΠΌΠΊΠ΅Π²ΠΈΡ‡ П.А. Π‘Π±ΠΎΡ€Π½ΠΈΠΊ Π·Π°Π΄Π°Ρ‡ ΠΏΠΎ Ρ„ΠΈΠ·ΠΈΠΊΠ΅ для 9 – 11 классов. – М.: ΠŸΡ€ΠΎΡΠ²Π΅Ρ‰Π΅Π½ΠΈΠ΅, 2000.
  8. Π‘Ρ‚Π΅ΠΏΠ°Π½ΠΎΠ²Π° Π“.Н. Π‘Π±ΠΎΡ€Π½ΠΈΠΊ Π·Π°Π΄Π°Ρ‡ ΠΏΠΎ Ρ„ΠΈΠ·ΠΈΠΊΠ΅: Для 9-11 классов ΠΎΠ±Ρ‰Π΅ΠΎΠ±Ρ€Π°Π·ΠΎΠ²Π°Ρ‚Π΅Π»ΡŒΠ½Ρ‹Ρ… ΡƒΡ‡Ρ€Π΅ΠΆΠ΄Π΅Π½ΠΈΠΉ. - М.: ΠŸΡ€ΠΎΡΠ²Π΅Ρ‰Π΅Π½ΠΈΠ΅,1998.
  9. Π“ΠΎΡ€ΠΎΠ΄Π΅Ρ†ΠΊΠΈΠΉ Π”.Н., ПСньков И.А. ΠŸΡ€ΠΎΠ²Π΅Ρ€ΠΎΡ‡Π½Ρ‹Π΅ Ρ€Π°Π±ΠΎΡ‚Ρ‹ ΠΏΠΎ Ρ„ΠΈΠ·ΠΈΠΊΠ΅. – Минск β€œΠ’Ρ‹ΡˆΡΠΉΡˆΠ°Ρ ΡˆΠΊΠΎΠ»Π°β€, 1987.

ΠŸΡ€ΠΈΠ»ΠΎΠΆΠ΅Π½ΠΈΠ΅ 1

Π£Ρ€ΠΎΠΊ β„– 1: β€œΠ˜Π·ΠΌΠ΅Ρ€Π΅Π½ΠΈΠ΅ физичСских Π²Π΅Π»ΠΈΡ‡ΠΈΠ½ ΠΈ ΠΎΡ†Π΅Π½ΠΊΠ° ΠΏΠΎΠ³Ρ€Π΅ΡˆΠ½ΠΎΡΡ‚Π΅ΠΉ измСрСния”.

Π¦Π΅Π»ΠΈ ΡƒΡ€ΠΎΠΊΠ°: 1. ΠŸΠΎΠ·Π½Π°ΠΊΠΎΠΌΠΈΡ‚ΡŒ учащихся с матСматичСской ΠΎΠ±Ρ€Π°Π±ΠΎΡ‚ΠΊΠΎΠΉ Ρ€Π΅Π·ΡƒΠ»ΡŒΡ‚Π°Ρ‚ΠΎΠ² измСрСния ΠΈ Π½Π°ΡƒΡ‡ΠΈΡ‚ΡŒ ΠΏΡ€Π΅Π΄ΡΡ‚Π°Π²Π»ΡΡ‚ΡŒ ΡΠΊΡΠΏΠ΅Ρ€ΠΈΠΌΠ΅Π½Ρ‚Π°Π»ΡŒΠ½Ρ‹Π΅ Π΄Π°Π½Π½Ρ‹Π΅;

2. Π Π°Π·Π²ΠΈΡ‚ΠΈΠ΅ Π²Ρ‹Ρ‡ΠΈΡΠ»ΠΈΡ‚Π΅Π»ΡŒΠ½Ρ‹Ρ… способностСй, памяти ΠΈ внимания.

Π₯ΠΎΠ΄ ΡƒΡ€ΠΎΠΊΠ°

Π Π΅Π·ΡƒΠ»ΡŒΡ‚Π°Ρ‚Ρ‹ любого физичСского экспСримСнта Π½Π΅ΠΎΠ±Ρ…ΠΎΠ΄ΠΈΠΌΠΎ ΡƒΠΌΠ΅Ρ‚ΡŒ ΠΏΡ€ΠΎΠ°Π½Π°Π»ΠΈΠ·ΠΈΡ€ΠΎΠ²Π°Ρ‚ΡŒ. Π­Ρ‚ΠΎ Π·Π½Π°Ρ‡ΠΈΡ‚, Ρ‡Ρ‚ΠΎ Π² Π»Π°Π±ΠΎΡ€Π°Ρ‚ΠΎΡ€ΠΈΠΈ Π½Π΅ΠΎΠ±Ρ…ΠΎΠ΄ΠΈΠΌΠΎ Π½Π°ΡƒΡ‡ΠΈΡ‚ΡŒΡΡ Π½Π΅ Ρ‚ΠΎΠ»ΡŒΠΊΠΎ ΠΈΠ·ΠΌΠ΅Ρ€ΡΡ‚ΡŒ Ρ€Π°Π·Π»ΠΈΡ‡Π½Ρ‹Π΅ физичСскиС Π²Π΅Π»ΠΈΡ‡ΠΈΠ½Ρ‹, Π½ΠΎ ΠΈ ΠΏΡ€ΠΎΠ²Π΅Ρ€ΡΡ‚ΡŒ ΠΈ Π½Π°Ρ…ΠΎΠ΄ΠΈΡ‚ΡŒ связь ΠΌΠ΅ΠΆΠ΄Ρƒ Π½ΠΈΠΌΠΈ, ΡΠΎΠΏΠΎΡΡ‚Π°Π²Π»ΡΡ‚ΡŒ Ρ€Π΅Π·ΡƒΠ»ΡŒΡ‚Π°Ρ‚Ρ‹ экспСримСнта с Π²Ρ‹Π²ΠΎΠ΄Π°ΠΌΠΈ Ρ‚Π΅ΠΎΡ€ΠΈΠΈ.

Но Ρ‡Ρ‚ΠΎ Π·Π½Π°Ρ‡ΠΈΡ‚ ΠΈΠ·ΠΌΠ΅Ρ€ΠΈΡ‚ΡŒ Ρ„ΠΈΠ·ΠΈΡ‡Π΅ΡΠΊΡƒΡŽ Π²Π΅Π»ΠΈΡ‡ΠΈΠ½Ρƒ? Как Π±Ρ‹Ρ‚ΡŒ, Ссли ΠΈΡΠΊΠΎΠΌΡƒΡŽ Π²Π΅Π»ΠΈΡ‡ΠΈΠ½Ρƒ нСльзя ΠΈΠ·ΠΌΠ΅Ρ€ΠΈΡ‚ΡŒ нСпосрСдствСнно ΠΈ Π΅Π΅ Π·Π½Π°Ρ‡Π΅Π½ΠΈΠ΅ находится ΠΏΠΎ Π·Π½Π°Ρ‡Π΅Π½ΠΈΡŽ Π΄Ρ€ΡƒΠ³ΠΈΡ… Π²Π΅Π»ΠΈΡ‡ΠΈΠ½?

Под ΠΈΠ·ΠΌΠ΅Ρ€Π΅Π½ΠΈΠ΅ΠΌ ΠΏΠΎΠ½ΠΈΠΌΠ°ΡŽΡ‚ сравнСниС измСряСмой Π²Π΅Π»ΠΈΡ‡ΠΈΠ½Ρ‹ с Π΄Ρ€ΡƒΠ³ΠΎΠΉ Π²Π΅Π»ΠΈΡ‡ΠΈΠ½ΠΎΠΉ, принятой Π·Π° Π΅Π΄ΠΈΠ½ΠΈΡ†Ρƒ измСрСния.

Π˜Π·ΠΌΠ΅Ρ€Π΅Π½ΠΈΠ΅ ΠΏΠΎΠ΄Ρ€Π°Π·Π΄Π΅Π»ΡΡŽΡ‚ Π½Π° прямыС ΠΈ косвСнныС.

ΠŸΡ€ΠΈ прямых измСрСниях ΠΎΠΏΡ€Π΅Π΄Π΅Π»ΡΠ΅ΠΌΡƒΡŽ Π²Π΅Π»ΠΈΡ‡ΠΈΠ½Ρƒ ΡΡ€Π°Π²Π½ΠΈΠ²Π°ΡŽΡ‚ с Π΅Π΄ΠΈΠ½ΠΈΡ†Π΅ΠΉ измСрСния нСпосрСдствСнно ΠΈΠ»ΠΈ ΠΏΡ€ΠΈ ΠΏΠΎΠΌΠΎΡ‰ΠΈ ΠΈΠ·ΠΌΠ΅Ρ€ΠΈΡ‚Π΅Π»ΡŒΠ½ΠΎΠ³ΠΎ ΠΏΡ€ΠΈΠ±ΠΎΡ€Π°, ΠΏΡ€ΠΎΠ³Ρ€Π°Π΄ΡƒΠΈΡ€ΠΎΠ²Π°Π½Π½ΠΎΠ³ΠΎ Π² ΡΠΎΠΎΡ‚Π²Π΅Ρ‚ΡΡ‚Π²ΡƒΡŽΡ‰ΠΈΡ… Π΅Π΄ΠΈΠ½ΠΈΡ†Π°Ρ….

ΠŸΡ€ΠΈ косвСнных измСрСниях искомая Π²Π΅Π»ΠΈΡ‡ΠΈΠ½Π° опрСдСляСтся (вычисляСтся) ΠΈΠ· Ρ€Π΅Π·ΡƒΠ»ΡŒΡ‚Π°Ρ‚ΠΎΠ² прямых ΠΈΠ·ΠΌΠ΅Ρ€Π΅Π½ΠΈΠΉ Π΄Ρ€ΡƒΠ³ΠΈΡ… Π²Π΅Π»ΠΈΡ‡ΠΈΠ½, ΠΊΠΎΡ‚ΠΎΡ€Ρ‹Π΅ связаны с измСряСмой Π²Π΅Π»ΠΈΡ‡ΠΈΠ½ΠΎΠΉ ΠΎΠΏΡ€Π΅Π΄Π΅Π»Π΅Π½Π½ΠΎΠΉ Ρ„ΡƒΠ½ΠΊΡ†ΠΈΠΎΠ½Π°Π»ΡŒΠ½ΠΎΠΉ Π·Π°Π²ΠΈΡΠΈΠΌΠΎΡΡ‚ΡŒΡŽ.

ΠŸΡ€ΠΈ ΠΈΠ·ΠΌΠ΅Ρ€Π΅Π½ΠΈΠΈ любой физичСской Π²Π΅Π»ΠΈΡ‡ΠΈΠ½Ρ‹ ΠΎΠ±Ρ‹Ρ‡Π½ΠΎ приходится Π²Ρ‹ΠΏΠΎΠ»Π½ΡΡ‚ΡŒ Ρ‚Ρ€ΠΈ ΠΏΠΎΡΠ»Π΅Π΄ΠΎΠ²Π°Ρ‚Π΅Π»ΡŒΠ½Ρ‹Π΅ ΠΎΠΏΠ΅Ρ€Π°Ρ†ΠΈΠΈ:

  1. Π’Ρ‹Π±ΠΎΡ€, ΠΏΡ€ΠΎΠ²Π΅Ρ€ΠΊΡƒ ΠΈ установку ΠΏΡ€ΠΈΠ±ΠΎΡ€ΠΎΠ²;
  2. НаблюдСниС ΠΏΠΎΠΊΠ°Π·Π°Π½ΠΈΠΉ ΠΏΡ€ΠΈΠ±ΠΎΡ€ΠΎΠ² ΠΈ отсчСт;
  3. НычислСниС искомой Π²Π΅Π»ΠΈΡ‡ΠΈΠ½Ρ‹ ΠΈΠ· Ρ€Π΅Π·ΡƒΠ»ΡŒΡ‚Π°Ρ‚ΠΎΠ² ΠΈΠ·ΠΌΠ΅Ρ€Π΅Π½ΠΈΠΉ, ΠΏΡ€ΠΎΠ²Π΅Π΄Π΅Π½ΠΈΠ΅ ΠΎΡ†Π΅Π½ΠΊΠΈ ΠΏΠΎΠ³Ρ€Π΅ΡˆΠ½ΠΎΡΡ‚Π΅ΠΉ.

ΠŸΠΎΠ³Ρ€Π΅ΡˆΠ½ΠΎΡΡ‚ΠΈ Ρ€Π΅Π·ΡƒΠ»ΡŒΡ‚Π°Ρ‚ΠΎΠ² ΠΈΠ·ΠΌΠ΅Ρ€Π΅Π½ΠΈΠΉ.

Π˜ΡΡ‚ΠΈΠ½Π½ΠΎΠ΅ Π·Π½Π°Ρ‡Π΅Π½ΠΈΠ΅ физичСской Π²Π΅Π»ΠΈΡ‡ΠΈΠ½Ρ‹ ΠΎΠ±Ρ‹Ρ‡Π½ΠΎ Π°Π±ΡΠΎΠ»ΡŽΡ‚Π½ΠΎ Ρ‚ΠΎΡ‡Π½ΠΎ ΠΎΠΏΡ€Π΅Π΄Π΅Π»ΠΈΡ‚ΡŒ Π½Π΅Π²ΠΎΠ·ΠΌΠΎΠΆΠ½ΠΎ. КаТдоС ΠΈΠ·ΠΌΠ΅Ρ€Π΅Π½ΠΈΠ΅ Π΄Π°Π΅Ρ‚ Π·Π½Π°Ρ‡Π΅Π½ΠΈΠ΅ опрСдСляСмой Π²Π΅Π»ΠΈΡ‡ΠΈΠ½Ρ‹ Ρ… с Π½Π΅ΠΊΠΎΡ‚ΠΎΡ€ΠΎΠΉ ΠΏΠΎΠ³Ρ€Π΅ΡˆΠ½ΠΎΡΡ‚ΡŒΡŽ?Ρ…. Π­Ρ‚ΠΎ Π·Π½Π°Ρ‡ΠΈΡ‚, Ρ‡Ρ‚ΠΎ истинноС Π·Π½Π°Ρ‡Π΅Π½ΠΈΠ΅ Π»Π΅ΠΆΠΈΡ‚ Π² ΠΈΠ½Ρ‚Π΅Ρ€Π²Π°Π»Π΅

Ρ… ΠΈΠ·ΠΌ - dΡ… < Ρ… ист < Ρ… ΠΈΠ·ΠΌ + dΡ…, (1)

Π³Π΄Π΅ Ρ… ΠΈΠ·ΠΌ - Π·Π½Π°Ρ‡Π΅Π½ΠΈΠ΅ Π²Π΅Π»ΠΈΡ‡ΠΈΠ½Ρ‹ Ρ…, получСнная ΠΏΡ€ΠΈ ΠΈΠ·ΠΌΠ΅Ρ€Π΅Π½ΠΈΠΈ; ?Ρ… Ρ…Π°Ρ€Π°ΠΊΡ‚Π΅Ρ€ΠΈΠ·ΡƒΠ΅Ρ‚ Ρ‚ΠΎΡ‡Π½ΠΎΡΡ‚ΡŒ измСрСния Ρ…. Π’Π΅Π»ΠΈΡ‡ΠΈΠ½Ρƒ?Ρ… Π½Π°Π·Ρ‹Π²Π°ΡŽΡ‚ Π°Π±ΡΠΎΠ»ΡŽΡ‚Π½ΠΎΠΉ ΠΏΠΎΠ³Ρ€Π΅ΡˆΠ½ΠΎΡΡ‚ΡŒΡŽ, с ΠΊΠΎΡ‚ΠΎΡ€ΠΎΠΉ опрСдСляСтся Ρ….

ВсС ΠΏΠΎΠ³Ρ€Π΅ΡˆΠ½ΠΎΡΡ‚ΠΈ ΠΏΠΎΠ΄Ρ€Π°Π·Π΄Π΅Π»ΡΡŽΡ‚ Π½Π° систСматичСскиС, случайныС ΠΈ ΠΏΡ€ΠΎΠΌΠ°Ρ…ΠΈ(ошибки). ΠŸΡ€ΠΈΡ‡ΠΈΠ½Π° возникновСния ΠΏΠΎΠ³Ρ€Π΅ΡˆΠ½ΠΎΡΡ‚Π΅ΠΉ самыС Ρ€Π°Π·Π½ΠΎΠΎΠ±Ρ€Π°Π·Π½Ρ‹Π΅. ΠŸΠΎΠ½ΡΡ‚ΡŒ Π²ΠΎΠ·ΠΌΠΎΠΆΠ½Ρ‹Π΅ ΠΏΡ€ΠΈΡ‡ΠΈΠ½Ρ‹ ΠΏΠΎΠ³Ρ€Π΅ΡˆΠ½ΠΎΡΡ‚Π΅ΠΉ ΠΈ свСсти ΠΈΡ… ΠΊ ΠΌΠΈΠ½ΠΈΠΌΡƒΠΌΡƒ - это ΠΈ ΠΎΠ·Π½Π°Ρ‡Π°Π΅Ρ‚ Π³Ρ€Π°ΠΌΠΎΡ‚Π½ΠΎ ΠΏΠΎΡΡ‚Π°Π²ΠΈΡ‚ΡŒ экспСримСнт. Ясно, Ρ‡Ρ‚ΠΎ это нСпростая Π·Π°Π΄Π°Ρ‡Π°.

БистСматичСской Π½Π°Π·Ρ‹Π²Π°ΡŽΡ‚ Ρ‚Π°ΠΊΡƒΡŽ ΠΏΠΎΠ³Ρ€Π΅ΡˆΠ½ΠΎΡΡ‚ΡŒ, которая остаСтся постоянной ΠΈΠ»ΠΈ Π·Π°ΠΊΠΎΠ½ΠΎΠΌΠ΅Ρ€Π½ΠΎ измСняСтся ΠΏΡ€ΠΈ ΠΏΠΎΠ²Ρ‚ΠΎΡ€Π½Ρ‹Ρ… измСрСниях ΠΎΠ΄Π½ΠΎΠΉ ΠΎ Ρ‚ΠΎΠΉ ΠΆΠ΅ Π²Π΅Π»ΠΈΡ‡ΠΈΠ½Ρ‹.

Π’Π°ΠΊΠΈΠ΅ ΠΏΠΎΠ³Ρ€Π΅ΡˆΠ½ΠΎΡΡ‚ΠΈ Π²ΠΎΠ·Π½ΠΈΠΊΠ°ΡŽΡ‚ Π² Ρ€Π΅Π·ΡƒΠ»ΡŒΡ‚Π°Ρ‚Π΅ конструктивных особСнностСй ΠΈΠ·ΠΌΠ΅Ρ€ΠΈΡ‚Π΅Π»ΡŒΠ½Ρ‹Ρ… ΠΏΡ€ΠΈΠ±ΠΎΡ€ΠΎΠ², нСточности ΠΌΠ΅Ρ‚ΠΎΠ΄Π° исслСдования, ΠΊΠ°ΠΊΠΈΡ…-Π»ΠΈΠ±ΠΎ ΡƒΠΏΡƒΡ‰Π΅Π½ΠΈΠΉ экспСримСнтатора, Π° Ρ‚Π°ΠΊΠΆΠ΅ ΠΏΡ€ΠΈ ΠΏΡ€ΠΈΠΌΠ΅Π½Π΅Π½ΠΈΠΈ для вычислСний Π½Π΅Ρ‚ΠΎΡ‡Π½Ρ‹Ρ… Ρ„ΠΎΡ€ΠΌΡƒΠ», ΠΎΠΊΡ€ΡƒΠ³Π»Π΅Π½Π½Ρ‹Ρ… констант.

Π˜Π·ΠΌΠ΅Ρ€ΠΈΡ‚Π΅Π»ΡŒΠ½Ρ‹ΠΌ ΠΏΡ€ΠΈΠ±ΠΎΡ€ΠΎΠΌ Π½Π°Π·Ρ‹Π²Π°ΡŽΡ‚ Ρ‚Π°ΠΊΠΎΠ΅ устройство, с ΠΏΠΎΠΌΠΎΡ‰ΡŒΡŽ ΠΊΠΎΡ‚ΠΎΡ€ΠΎΠ³ΠΎ осущСствляСтся сравнСниС измСряСмой Π²Π΅Π»ΠΈΡ‡ΠΈΠ½Ρ‹ с Π΅Π΄ΠΈΠ½ΠΈΡ†Π΅ΠΉ измСрСния.

Π’ любом ΠΏΡ€ΠΈΠ±ΠΎΡ€Π΅ Π·Π°Π»ΠΎΠΆΠ΅Π½Π° Ρ‚Π° ΠΈΠ»ΠΈ иная систСматичСская ΠΏΠΎΠ³Ρ€Π΅ΡˆΠ½ΠΎΡΡ‚ΡŒ, ΠΊΠΎΡ‚ΠΎΡ€ΡƒΡŽ Π½Π΅Π²ΠΎΠ·ΠΌΠΎΠΆΠ½ΠΎ ΡƒΡΡ‚Ρ€Π°Π½ΠΈΡ‚ΡŒ, Π½ΠΎ порядок ΠΊΠΎΡ‚ΠΎΡ€ΠΎΠΉ ΠΌΠΎΠΆΠ½ΠΎ ΡƒΡ‡Π΅ΡΡ‚ΡŒ.

БистСматичСскиС ΠΏΠΎΠ³Ρ€Π΅ΡˆΠ½ΠΎΡΡ‚ΠΈ Π»ΠΈΠ±ΠΎ ΡƒΠ²Π΅Π»ΠΈΡ‡ΠΈΠ²Π°ΡŽΡ‚, Π»ΠΈΠ±ΠΎ ΡƒΠΌΠ΅Π½ΡŒΡˆΠ°ΡŽΡ‚ Ρ€Π΅Π·ΡƒΠ»ΡŒΡ‚Π°Ρ‚Ρ‹ ΠΈΠ·ΠΌΠ΅Ρ€Π΅Π½ΠΈΠΉ, Ρ‚ΠΎ Π΅ΡΡ‚ΡŒ эти ΠΏΠΎΠ³Ρ€Π΅ΡˆΠ½ΠΎΡΡ‚ΠΈ Ρ…Π°Ρ€Π°ΠΊΡ‚Π΅Ρ€ΠΈΠ·ΡƒΡŽΡ‚ΡΡ постоянством Π·Π½Π°ΠΊΠ°.

Π‘Π»ΡƒΡ‡Π°ΠΉΠ½Ρ‹Π΅ ΠΏΠΎΠ³Ρ€Π΅ΡˆΠ½ΠΎΡΡ‚ΠΈ-ошибки, появлСниС ΠΊΠΎΡ‚ΠΎΡ€Ρ‹Ρ… Π½Π΅ ΠΌΠΎΠΆΠ΅Ρ‚ Π±Ρ‹Ρ‚ΡŒ ΠΏΡ€Π΅Π΄ΡƒΠΏΡ€Π΅ΠΆΠ΄Π΅Π½ΠΎ.

ΠŸΠΎΡΡ‚ΠΎΠΌΡƒ ΠΎΠ½ΠΈ ΠΌΠΎΠ³ΡƒΡ‚ ΠΎΠΊΠ°Π·Π°Ρ‚ΡŒ ΠΎΠΏΡ€Π΅Π΄Π΅Π»Π΅Π½Π½ΠΎΠ΅ влияниС Π½Π° ΠΎΡ‚Π΄Π΅Π»ΡŒΠ½ΠΎΠ΅ ΠΈΠ·ΠΌΠ΅Ρ€Π΅Π½ΠΈΠ΅, Π½ΠΎ ΠΏΡ€ΠΈ ΠΌΠ½ΠΎΠ³ΠΎΠΊΡ€Π°Ρ‚Π½Ρ‹Ρ… измСрСниях ΠΎΠ½ΠΈ ΠΏΠΎΠ΄Ρ‡ΠΈΠ½ΡΡŽΡ‚ΡΡ статистичСским Π·Π°ΠΊΠΎΠ½Π°ΠΌ ΠΈ ΠΈΡ… влияниС Π½Π° Ρ€Π΅Π·ΡƒΠ»ΡŒΡ‚Π°Ρ‚Ρ‹ ΠΈΠ·ΠΌΠ΅Ρ€Π΅Π½ΠΈΠΉ ΠΌΠΎΠΆΠ½ΠΎ ΡƒΡ‡Π΅ΡΡ‚ΡŒ ΠΈΠ»ΠΈ Π·Π½Π°Ρ‡ΠΈΡ‚Π΅Π»ΡŒΠ½ΠΎ ΡƒΠΌΠ΅Π½ΡŒΡˆΠΈΡ‚ΡŒ.

ΠŸΡ€ΠΎΠΌΠ°Ρ…ΠΈ ΠΈ Π³Ρ€ΡƒΠ±Ρ‹Π΅ ΠΏΠΎΠ³Ρ€Π΅ΡˆΠ½ΠΎΡΡ‚ΠΈ - Ρ‡Ρ€Π΅Π·ΠΌΠ΅Ρ€Π½ΠΎ большиС ошибки, явно ΠΈΡΠΊΠ°ΠΆΠ°ΡŽΡ‰ΠΈΠ΅ Ρ€Π΅Π·ΡƒΠ»ΡŒΡ‚Π°Ρ‚ измСрСния.

Π­Ρ‚ΠΎΡ‚ класс ΠΏΠΎΠ³Ρ€Π΅ΡˆΠ½ΠΎΡΡ‚Π΅ΠΉ Π²Ρ‹Π·Π²Π°Π½ Ρ‡Π°Ρ‰Π΅ всСго Π½Π΅ΠΏΡ€Π°Π²ΠΈΠ»ΡŒΠ½Ρ‹ΠΌΠΈ дСйствиями Π½Π°Π±Π»ΡŽΠ΄Π°Ρ‚Π΅Π»Ρ. Π˜Π·ΠΌΠ΅Ρ€Π΅Π½ΠΈΡ, содСрТащиС ΠΏΡ€ΠΎΠΌΠ°Ρ…ΠΈ ΠΈ Π³Ρ€ΡƒΠ±Ρ‹Π΅ ΠΏΠΎΠ³Ρ€Π΅ΡˆΠ½ΠΎΡΡ‚ΠΈ, слСдуСт ΠΎΡ‚Π±Ρ€Π°ΡΡ‹Π²Π°Ρ‚ΡŒ.

Π˜Π·ΠΌΠ΅Ρ€Π΅Π½ΠΈΡ ΠΌΠΎΠ³ΡƒΡ‚ Π±Ρ‹Ρ‚ΡŒ ΠΏΡ€ΠΎΠ²Π΅Π΄Π΅Π½Ρ‹ с Ρ‚ΠΎΡ‡ΠΊΠΈ зрСния ΠΈΡ… точности тСхничСским ΠΈ Π»Π°Π±ΠΎΡ€Π°Ρ‚ΠΎΡ€Π½Ρ‹ΠΌ ΠΌΠ΅Ρ‚ΠΎΠ΄Π°ΠΌΠΈ.

Π’ этом случаС ΡƒΠ΄ΠΎΠ²Π»Π΅Ρ‚Π²ΠΎΡ€ΡΡŽΡ‚ΡΡ Ρ‚Π°ΠΊΠΎΠΉ Ρ‚ΠΎΡ‡Π½ΠΎΡΡ‚ΡŒΡŽ, ΠΏΡ€ΠΈ ΠΊΠΎΡ‚ΠΎΡ€ΠΎΠΉ ΠΏΠΎΠ³Ρ€Π΅ΡˆΠ½ΠΎΡΡ‚ΡŒ Π½Π΅ ΠΏΡ€Π΅Π²Ρ‹ΡˆΠ°Π΅Ρ‚ Π½Π΅ΠΊΠΎΡ‚ΠΎΡ€ΠΎΠ³ΠΎ ΠΎΠΏΡ€Π΅Π΄Π΅Π»Π΅Π½Π½ΠΎΠ³ΠΎ, Π½Π°ΠΏΠ΅Ρ€Π΅Π΄ Π·Π°Π΄Π°Π½Π½ΠΎΠ³ΠΎ значСния, опрСдСляСмого ΠΏΠΎΠ³Ρ€Π΅ΡˆΠ½ΠΎΡΡ‚ΡŒΡŽ ΠΏΡ€ΠΈΠΌΠ΅Π½Π΅Π½Π½ΠΎΠΉ ΠΈΠ·ΠΌΠ΅Ρ€ΠΈΡ‚Π΅Π»ΡŒΠ½ΠΎΠΉ Π°ΠΏΠΏΠ°Ρ€Π°Ρ‚ΡƒΡ€Ρ‹.

ΠŸΡ€ΠΈ Π»Π°Π±ΠΎΡ€Π°Ρ‚ΠΎΡ€Π½Ρ‹Ρ… ΠΌΠ΅Ρ‚ΠΎΠ΄Π°Ρ… ΠΈΠ·ΠΌΠ΅Ρ€Π΅Π½ΠΈΠΉ трСбуСтся Π±ΠΎΠ»Π΅Π΅ Ρ‚ΠΎΡ‡Π½ΠΎ ΡƒΠΊΠ°Π·Π°Ρ‚ΡŒ Π·Π½Π°Ρ‡Π΅Π½ΠΈΠ΅ измСряСмой Π²Π΅Π»ΠΈΡ‡ΠΈΠ½Ρ‹, Ρ‡Π΅ΠΌ это допускаСт ΠΎΠ΄Π½ΠΎΠΊΡ€Π°Ρ‚Π½ΠΎΠ΅ Π΅Π΅ ΠΈΠ·ΠΌΠ΅Ρ€Π΅Π½ΠΈΠ΅ тСхничСским ΠΌΠ΅Ρ‚ΠΎΠ΄ΠΎΠΌ.

Π’ΠΎΠ³Π΄Π° Π΄Π΅Π»Π°ΡŽΡ‚ нСсколько ΠΈΠ·ΠΌΠ΅Ρ€Π΅Π½ΠΈΠΉ ΠΈ Π²Ρ‹Ρ‡ΠΈΡΠ»ΡΡŽΡ‚ срСднСС арифмСтичСскоС ΠΏΠΎΠ»ΡƒΡ‡Π΅Π½Π½Ρ‹Ρ… Π·Π½Π°Ρ‡Π΅Π½ΠΈΠΉ, ΠΊΠΎΡ‚ΠΎΡ€ΠΎΠ΅ ΠΏΡ€ΠΈΠ½ΠΈΠΌΠ°ΡŽΡ‚ Π·Π° Π½Π°ΠΈΠ±ΠΎΠ»Π΅Π΅ достовСрноС Π·Π½Π°Ρ‡Π΅Π½ΠΈΠ΅ измСряСмой Π²Π΅Π»ΠΈΡ‡ΠΈΠ½Ρ‹. Π—Π°Ρ‚Π΅ΠΌ производят ΠΎΡ†Π΅Π½ΠΊΡƒ точности Ρ€Π΅Π·ΡƒΠ»ΡŒΡ‚Π°Ρ‚Π° измСрСния (ΡƒΡ‡Π΅Ρ‚ случайных ΠΏΠΎΠ³Ρ€Π΅ΡˆΠ½ΠΎΡΡ‚Π΅ΠΉ).

Из возмоТности провСдСния ΠΈΠ·ΠΌΠ΅Ρ€Π΅Π½ΠΈΠΉ двумя ΠΌΠ΅Ρ‚ΠΎΠ΄Π°ΠΌΠΈ Π²Ρ‹Ρ‚Π΅ΠΊΠ°Π΅Ρ‚ ΠΈ сущСствованиС Π΄Π²ΡƒΡ… ΠΌΠ΅Ρ‚ΠΎΠ΄ΠΎΠ² ΠΎΡ†Π΅Π½ΠΊΠΈ точности ΠΈΠ·ΠΌΠ΅Ρ€Π΅Π½ΠΈΠΉ: тСхничСского ΠΈ Π»Π°Π±ΠΎΡ€Π°Ρ‚ΠΎΡ€Π½ΠΎΠ³ΠΎ.

ΠšΠ»Π°ΡΡΡ‹ точности ΠΏΡ€ΠΈΠ±ΠΎΡ€ΠΎΠ².

Для характСристики Π±ΠΎΠ»ΡŒΡˆΠΈΠ½ΡΡ‚Π²Π° ΠΈΠ·ΠΌΠ΅Ρ€ΠΈΡ‚Π΅Π»ΡŒΠ½Ρ‹Ρ… ΠΏΡ€ΠΈΠ±ΠΎΡ€ΠΎΠ² часто ΠΈΡΠΏΠΎΠ»ΡŒΠ·ΡƒΡŽΡ‚ понятиС ΠΏΡ€ΠΈΠ²Π΅Π΄Π΅Π½Π½ΠΎΠΉ ΠΏΠΎΠ³Ρ€Π΅ΡˆΠ½ΠΎΡΡ‚ΠΈ Π• ΠΏ (класса точности).

ΠŸΡ€ΠΈΠ²Π΅Π΄Π΅Π½Π½Π°Ρ ΠΏΠΎΠ³Ρ€Π΅ΡˆΠ½ΠΎΡΡ‚ΡŒ это ΠΎΡ‚Π½ΠΎΡˆΠ΅Π½ΠΈΠ΅ Π°Π±ΡΠΎΠ»ΡŽΡ‚Π½ΠΎΠΉ ΠΏΠΎΠ³Ρ€Π΅ΡˆΠ½ΠΎΡΡ‚ΠΈ ?Ρ… ΠΊ ΠΏΡ€Π΅Π΄Π΅Π»ΡŒΠ½ΠΎΠΌΡƒ Π·Π½Π°Ρ‡Π΅Π½ΠΈΡŽ Ρ… ΠΏΡ€ измСряСмой Π²Π΅Π»ΠΈΡ‡ΠΈΠ½Ρ‹ (Ρ‚ΠΎ Π΅ΡΡ‚ΡŒ Π½Π°ΠΈΠ±ΠΎΠ»ΡŒΡˆΠ΅ΠΌΡƒ Π΅Π΅ Π·Π½Π°Ρ‡Π΅Π½ΠΈΡŽ, ΠΊΠΎΡ‚ΠΎΡ€ΠΎΠ΅ ΠΌΠΎΠΆΠ΅Ρ‚ Π±Ρ‹Ρ‚ΡŒ ΠΈΠ·ΠΌΠ΅Ρ€Π΅Π½ΠΎ ΠΏΠΎ шкалС ΠΏΡ€ΠΈΠ±ΠΎΡ€ΠΎΠ²).

ΠŸΡ€ΠΈΠ²Π΅Π΄Π΅Π½Π½Π°Ρ ΠΏΠΎΠ³Ρ€Π΅ΡˆΠ½ΠΎΡΡ‚ΡŒ, являясь ΠΏΠΎ сущСству ΠΎΡ‚Π½ΠΎΡΠΈΡ‚Π΅Π»ΡŒΠ½ΠΎΠΉ ΠΏΠΎΠ³Ρ€Π΅ΡˆΠ½ΠΎΡΡ‚ΡŒΡŽ, выраТаСтся Π² ΠΏΡ€ΠΎΡ†Π΅Π½Ρ‚Π°Ρ…:

Π• ΠΏ = /dΡ…/ Ρ… ΠΏΡ€ /*100%

По ΠΏΡ€ΠΈΠ²Π΅Π΄Π΅Π½Π½ΠΎΠΉ ΠΏΠΎΠ³Ρ€Π΅ΡˆΠ½ΠΎΡΡ‚ΠΈ ΠΏΡ€ΠΈΠ±ΠΎΡ€Ρ‹ Ρ€Π°Π·Π΄Π΅Π»ΡΡŽΡ‚ Π½Π° сСмь классов: 0,1; 0,2; 0,5; 1,0; 1,5; 2,5; 4.

ΠŸΡ€ΠΈΠ±ΠΎΡ€Ρ‹ класса точности 0,1; 0,2; 0,5 ΠΏΡ€ΠΈΠΌΠ΅Π½ΡΡŽΡ‚ для Ρ‚ΠΎΡ‡Π½Ρ‹Ρ… Π»Π°Π±ΠΎΡ€Π°Ρ‚ΠΎΡ€Π½Ρ‹Ρ… ΠΈΠ·ΠΌΠ΅Ρ€Π΅Π½ΠΈΠΉ ΠΈ Π½Π°Π·Ρ‹Π²Π°ΡŽΡ‚ ΠΏΡ€Π΅Ρ†ΠΈΠ·ΠΈΠΎΠ½Π½Ρ‹ΠΌΠΈ.

Π’ Ρ‚Π΅Ρ…Π½ΠΈΠΊΠ΅ ΠΏΡ€ΠΈΠΌΠ΅Π½ΡΡŽΡ‚ ΠΏΡ€ΠΈΠ±ΠΎΡ€Ρ‹ классов 1, 0; 1,5; 2,5 ΠΈ 4 (тСхничСскиС). Класс точности ΠΏΡ€ΠΈΠ±ΠΎΡ€Π° ΡƒΠΊΠ°Π·Ρ‹Π²Π°ΡŽΡ‚ Π½Π° шкалС ΠΏΡ€ΠΈΠ±ΠΎΡ€Π°. Если Π½Π° шкалС Ρ‚Π°ΠΊΠΎΠ³ΠΎ обозначСния Π½Π΅Ρ‚, Π½ΠΎ Π΄Π°Π½Π½Ρ‹ΠΉ ΠΏΡ€ΠΈΠ±ΠΎΡ€ внСклассный, Ρ‚ΠΎ Π΅ΡΡ‚ΡŒ Π΅Π³ΠΎ привСдСнная ΠΏΠΎΠ³Ρ€Π΅ΡˆΠ½ΠΎΡΡ‚ΡŒ Π±ΠΎΠ»Π΅Π΅ 4%. Π’ Ρ‚Π΅Ρ… случаях, ΠΊΠΎΠ³Π΄Π° Π½Π° ΠΏΡ€ΠΈΠ±ΠΎΡ€Π΅ класс точности Π½Π΅ ΡƒΠΊΠ°Π·Π°Π½, Π°Π±ΡΠΎΠ»ΡŽΡ‚Π½Π°Ρ ΠΏΠΎΠ³Ρ€Π΅ΡˆΠ½ΠΎΡΡ‚ΡŒ принимаСтся Ρ€Π°Π²Π½ΠΎΠΉ ΠΏΠΎΠ»ΠΎΠ²ΠΈΠ½Π΅ Ρ†Π΅Π½Ρ‹ наимСньшСго дСлСния.

Π’Π°ΠΊ, ΠΏΡ€ΠΈ ΠΈΠ·ΠΌΠ΅Ρ€Π΅Π½ΠΈΠΈ Π»ΠΈΠ½Π΅ΠΉΠΊΠΎΠΉ, наимСньшСС Π΄Π΅Π»Π΅Π½ΠΈΠ΅ ΠΊΠΎΡ‚ΠΎΡ€ΠΎΠΉ 1 ΠΌΠΌ, допускаСтся ошибка Π΄ΠΎ 0,5 ΠΌΠΌ. Для ΠΏΡ€ΠΈΠ±ΠΎΡ€ΠΎΠ², оснащСнных нониусом, Π·Π° ΠΏΡ€ΠΈΠ±ΠΎΡ€Π½ΡƒΡŽ ΠΏΠΎΠ³Ρ€Π΅ΡˆΠ½ΠΎΡΡ‚ΡŒ ΠΏΡ€ΠΈΠ½ΠΈΠΌΠ°ΡŽΡ‚ ΠΏΠΎΠ³Ρ€Π΅ΡˆΠ½ΠΎΡΡ‚ΡŒ, ΠΎΠΏΡ€Π΅Π΄Π΅Π»ΡΠ΅ΠΌΡƒΡŽ нониусом (для ΡˆΡ‚Π°Π½Π³Π΅Π½Ρ†ΠΈΡ€ΠΊΡƒΠ»Ρ-0,1 ΠΌΠΌ ΠΈΠ»ΠΈ 0,05 ΠΌΠΌ; для ΠΌΠΈΠΊΡ€ΠΎΠΌΠ΅Ρ‚Ρ€Π°-0,01 ΠΌΠΌ).

ΠŸΡ€ΠΈΠ»ΠΎΠΆΠ΅Π½ΠΈΠ΅ 2

Лабораторная Ρ€Π°Π±ΠΎΡ‚Π°: β€œΠ˜Π·ΠΌΠ΅Ρ€Π΅Π½ΠΈΠ΅ ΠšΠŸΠ” Π½Π°ΠΊΠ»ΠΎΠ½Π½ΠΎΠΉ плоскости”.

ΠžΠ±ΠΎΡ€ΡƒΠ΄ΠΎΠ²Π°Π½ΠΈΠ΅: дСрСвянная доска, дСрСвянный брусок, ΡˆΡ‚Π°Ρ‚ΠΈΠ², Π΄ΠΈΠ½Π°ΠΌΠΎΠΌΠ΅Ρ‚Ρ€, Π»ΠΈΠ½Π΅ΠΉΠΊΠ° ΠΈΠ·ΠΌΠ΅Ρ€ΠΈΡ‚Π΅Π»ΡŒΠ½Π°Ρ.

Π—Π°Π΄Π°Π½ΠΈΠ΅.Π˜ΡΡΠ»Π΅Π΄ΡƒΠΉΡ‚Π΅ Π·Π°Π²ΠΈΡΠΈΠΌΠΎΡΡ‚ΡŒ ΠšΠŸΠ” Π½Π°ΠΊΠ»ΠΎΠ½Π½ΠΎΠΉ плоскости ΠΈ Π²Ρ‹ΠΈΠ³Ρ€Ρ‹Ρˆ Π² силС, ΠΏΠΎΠ»ΡƒΡ‡Π°Π΅ΠΌΠΎΠ³ΠΎ с Π΅Π΅ ΠΏΠΎΠΌΠΎΡ‰ΡŒΡŽ ΠΎΡ‚ ΡƒΠ³Π»Π° Π½Π°ΠΊΠ»ΠΎΠ½Π° плоскости ΠΊ Π³ΠΎΡ€ΠΈΠ·ΠΎΠ½Ρ‚Ρƒ.

ΠšΠŸΠ” любого простого ΠΌΠ΅Ρ…Π°Π½ΠΈΠ·ΠΌΠ° Ρ€Π°Π²Π΅Π½ ΠΎΡ‚Π½ΠΎΡˆΠ΅Π½ΠΈΡŽ ΠΏΠΎΠ»Π΅Π·Π½ΠΎΠΉ Ρ€Π°Π±ΠΎΡ‚Ρ‹ А ΠΏΠΎΠ», ΠΊ ΡΠΎΠ²Π΅Ρ€ΡˆΠ΅Π½Π½ΠΎΠΉ Ρ€Π°Π±ΠΎΡ‚Π΅ А сов ΠΈ выраТаСтся Π² ΠΏΡ€ΠΎΡ†Π΅Π½Ρ‚Π°Ρ…:

n = А пол / А сов *100% (1).

ΠŸΡ€ΠΈ отсутствии трСния ΠšΠŸΠ” простого ΠΌΠ΅Ρ…Π°Π½ΠΈΠ·ΠΌΠ°, Π² Ρ‚ΠΎΠΌ числС ΠΈ Π½Π°ΠΊΠ»ΠΎΠ½Π½ΠΎΠΉ плоскости, Ρ€Π°Π²Π΅Π½ Π΅Π΄ΠΈΠ½ΠΈΡ†Π΅. Π’ этом случаС ΡΠΎΠ²Π΅Ρ€ΡˆΠ΅Π½Π½Π°Ρ Ρ€Π°Π±ΠΎΡ‚Π° А сов силы F Ρ‚, ΠΏΡ€ΠΈΠ»ΠΎΠΆΠ΅Π½Π½ΠΎΠΉ ΠΊ Ρ‚Π΅Π»Ρƒ ΠΈ Π½Π°ΠΏΡ€Π°Π²Π»Π΅Π½Π½ΠΎΠΉ Π²Π²Π΅Ρ€Ρ… вдоль Π½Π°ΠΊΠ»ΠΎΠ½Π½ΠΎΠΉ плоскости, Ρ€Π°Π²Π½Π° ΠΏΠΎΠ»Π΅Π·Π½ΠΎΠΉ Ρ€Π°Π±ΠΎΡ‚Π΅ А ΠΏΠΎΠ».

А пол = А сов.

ΠžΠ±ΠΎΠ·Π½Π°Ρ‡ΠΈΠ² ΠΏΡƒΡ‚ΡŒ, ΠΏΡ€ΠΎΠΉΠ΄Π΅Π½Π½Ρ‹ΠΉ Ρ‚Π΅Π»ΠΎΠΌ вдоль Π½Π°ΠΊΠ»ΠΎΠ½Π½ΠΎΠΉ плоскости Π±ΡƒΠΊΠ²ΠΎΠΉ S , высоту подъСма? , ΠΏΠΎΠ»ΡƒΡ‡ΠΈΠΌ F*S=hgm.

ΠŸΡ€ΠΈ этом Π²Ρ‹ΠΈΠ³Ρ€Ρ‹Ρˆ Π² силС Π±ΡƒΠ΄Π΅Ρ‚ Ρ€Π°Π²Π΅Π½: ΠΊ = gm/F=l/h.

Π’ Ρ€Π΅Π°Π»ΡŒΠ½Ρ‹Ρ… условиях дСйствиС силы трСния сниТаСт ΠšΠŸΠ” Π½Π°ΠΊΠ»ΠΎΠ½Π½ΠΎΠΉ плоскости ΠΈ ΡƒΠΌΠ΅Π½ΡŒΡˆΠ°Π΅Ρ‚ Π²Ρ‹ΠΈΠ³Ρ€Ρ‹Ρˆ Π² силС.

Для опрСдСлСния ΠšΠŸΠ” Π½Π°ΠΊΠ»ΠΎΠ½Π½ΠΎΠΉ плоскости Π²Ρ‹ΠΈΠ³Ρ€Ρ‹ΡˆΠ° Π² силС, ΠΏΠΎΠ»ΡƒΡ‡Π΅Π½Π½ΠΎΠ³ΠΎ с Π΅Π΅ ΠΏΠΎΠΌΠΎΡ‰ΡŒΡŽ, слСдуСт ΠΈΡΠΏΠΎΠ»ΡŒΠ·ΠΎΠ²Π°Ρ‚ΡŒ Π²Ρ‹Ρ€Π°ΠΆΠ΅Π½ΠΈΠ΅:

n = hgm/ F Ρ‚ l*100% (2), ΠΊ= gm/F Ρ‚ (3).

ЦСлью Ρ€Π°Π±ΠΎΡ‚Ρ‹ являСтся ΠΈΠ·ΠΌΠ΅Ρ€ΠΈΡ‚ΡŒ ΠšΠŸΠ” Π½Π°ΠΊΠ»ΠΎΠ½Π½ΠΎΠΉ плоскости ΠΈ Π²Ρ‹ΠΈΠ³Ρ€Ρ‹Ρˆ Π² силС ΠΏΡ€ΠΈ Ρ€Π°Π·Π½Ρ‹Ρ… ΡƒΠ³Π»Π°Ρ…? Π΅Π΅ Π½Π°ΠΊΠ»ΠΎΠ½Π° ΠΊ Π³ΠΎΡ€ΠΈΠ·ΠΎΠ½Ρ‚Ρƒ ΠΈ ΠΎΠ±ΡŠΡΡΠ½ΠΈΡ‚ΡŒ ΠΏΠΎΠ»ΡƒΡ‡Π΅Π½Π½Ρ‹ΠΉ Ρ€Π΅Π·ΡƒΠ»ΡŒΡ‚Π°Ρ‚.

ΠŸΠΎΡ€ΡΠ΄ΠΎΠΊ выполнСния Ρ€Π°Π±ΠΎΡ‚Ρ‹.

1. Π‘ΠΎΠ±Π΅Ρ€ΠΈΡ‚Π΅ установку ΠΏΠΎ рис1. Π˜Π·ΠΌΠ΅Ρ€ΡŒΡ‚Π΅ высоту? ΠΈ Π΄Π»ΠΈΠ½Ρƒ l Π½Π°ΠΊΠ»ΠΎΠ½Π½ΠΎΠΉ плоскости (рис.2).

2. ВычислитС максимально Π²ΠΎΠ·ΠΌΠΎΠΆΠ½ΠΎΠ΅ Π·Π½Π°Ρ‡Π΅Π½ΠΈΠ΅ Π²Ρ‹ΠΈΠ³Ρ€Ρ‹ΡˆΠ° Π² силС, ΠΏΠΎΠ»ΡƒΡ‡Π°Π΅ΠΌΠΎΠ΅ ΠΏΡ€ΠΈ Π·Π°Π΄Π°Π½Π½ΠΎΠΌ Π½Π°ΠΊΠ»ΠΎΠ½Π΅ плоскости (a=30).

3. ΠŸΠΎΠ»ΠΎΠΆΠΈΡ‚Π΅ брусок Π½Π° Π½Π°ΠΊΠ»ΠΎΠ½Π½ΡƒΡŽ ΠΏΠ»ΠΎΡΠΊΠΎΡΡ‚ΡŒ. ΠŸΡ€ΠΈΠΊΡ€Π΅ΠΏΠΈΠ² ΠΊ Π½Π΅ΠΌΡƒ Π΄ΠΈΠ½Π°ΠΌΠΎΠΌΠ΅Ρ‚Ρ€, Ρ€Π°Π²Π½ΠΎΠΌΠ΅Ρ€Π½ΠΎ тянитС Π΅Π³ΠΎ Π²Π²Π΅Ρ€Ρ… вдоль Π½Π°ΠΊΠ»ΠΎΠ½Π½ΠΎΠΉ плоскости. Π˜Π·ΠΌΠ΅Ρ€ΡŒΡ‚Π΅ силу тяги F Ρ‚.

4. Π˜Π·ΠΌΠ΅Ρ€ΡŒΡ‚Π΅ с ΠΏΠΎΠΌΠΎΡ‰ΡŒΡŽ Π΄ΠΈΠ½Π°ΠΌΠΎΠΌΠ΅Ρ‚Ρ€Π° силу тяТСсти mg бруска ΠΈ Π½Π°ΠΉΠ΄ΠΈΡ‚Π΅ ΡΠΊΡΠΏΠ΅Ρ€ΠΈΠΌΠ΅Π½Ρ‚Π°Π»ΡŒΠ½ΠΎΠ΅ Π·Π½Π°Ρ‡Π΅Π½ΠΈΠ΅ Π²Ρ‹ΠΈΠ³Ρ€Ρ‹ΡˆΠ° Π² силС, ΠΏΠΎΠ»ΡƒΡ‡Π΅Π½Π½ΠΎΠ³ΠΎ с ΠΏΠΎΠΌΠΎΡ‰ΡŒΡŽ Π½Π°ΠΊΠ»ΠΎΠ½Π½ΠΎΠΉ плоскости: ΠΊ= gm/F Ρ‚.

5. ВычислитС ΠšΠŸΠ” Π½Π°ΠΊΠ»ΠΎΠ½Π½ΠΎΠΉ плоскости ΠΏΡ€ΠΈ Π·Π°Π΄Π°Π½Π½ΠΎΠΌ ΡƒΠ³Π»Π΅ Π΅Π΅ Π½Π°ΠΊΠ»ΠΎΠ½Π°

n = hgm/ F Ρ‚ l*100%

6. ΠŸΠΎΠ²Ρ‚ΠΎΡ€ΠΈΡ‚Π΅ измСрСния ΠΏΡ€ΠΈ Π΄Ρ€ΡƒΠ³ΠΈΡ… ΡƒΠ³Π»Π°Ρ… Π½Π°ΠΊΠ»ΠΎΠ½Π° плоскости: a 2 =45?, a 3 =60 ?.

7. Π Π΅Π·ΡƒΠ»ΡŒΡ‚Π°Ρ‚Ρ‹ ΠΈΠ·ΠΌΠ΅Ρ€Π΅Π½ΠΈΠΉ ΠΈ вычислСний занСситС Π² Ρ‚Π°Π±Π»ΠΈΡ†Ρƒ:

β„– a m, ΠΊΠ³ h, ΠΌ l, ΠΌ F , Н ΠΊ n ,%
1 30
2 45
3 60

8. Π”ΠΎΠΏΠΎΠ»Π½ΠΈΡ‚Π΅Π»ΡŒΠ½ΠΎΠ΅ Π·Π°Π΄Π°Π½ΠΈΠ΅

ΠŸΠΎΠ»ΡƒΡ‡Π΅Π½Π½ΡƒΡŽ Ρ‚Π΅ΠΎΡ€Π΅Ρ‚ΠΈΡ‡Π΅ΡΠΊΡƒΡŽ Π·Π°Π²ΠΈΡΠΈΠΌΠΎΡΡ‚ΡŒ n(a) ΠΈ ΠΊ(a) сравнитС с Ρ€Π΅Π·ΡƒΠ»ΡŒΡ‚Π°Ρ‚Π°ΠΌΠΈ экспСримСнта.

ΠšΠΎΠ½Ρ‚Ρ€ΠΎΠ»ΡŒΠ½Ρ‹Π΅ вопросы.

  1. Π‘ ΠΊΠ°ΠΊΠΎΠΉ Ρ†Π΅Π»ΡŒΡŽ ΠΏΡ€ΠΈΠΌΠ΅Π½ΡΡŽΡ‚ Π½Π°ΠΊΠ»ΠΎΠ½Π½ΡƒΡŽ ΠΏΠ»ΠΎΡΠΊΠΎΡΡ‚ΡŒ?
  2. Каким ΠΎΠ±Ρ€Π°Π·ΠΎΠΌ ΠΌΠΎΠΆΠ½ΠΎ ΡƒΠ²Π΅Π»ΠΈΡ‡ΠΈΡ‚ΡŒ ΠšΠŸΠ” Π½Π°ΠΊΠ»ΠΎΠ½Π½ΠΎΠΉ плоскости?
  3. Каким ΠΎΠ±Ρ€Π°Π·ΠΎΠΌ ΠΌΠΎΠΆΠ½ΠΎ ΡƒΠ²Π΅Π»ΠΈΡ‡ΠΈΡ‚ΡŒ Π²Ρ‹ΠΈΠ³Ρ€Ρ‹Ρˆ Π² силС, ΠΏΠΎΠ»ΡƒΡ‡Π΅Π½Π½Ρ‹ΠΉ с ΠΏΠΎΠΌΠΎΡ‰ΡŒΡŽ Π½Π°ΠΊΠ»ΠΎΠ½Π½ΠΎΠΉ плоскости?
  4. Зависит Π»ΠΈ ΠšΠŸΠ” Π½Π°ΠΊΠ»ΠΎΠ½Π½ΠΎΠΉ плоскости ΠΎΡ‚ массы Π³Ρ€ΡƒΠ·Π°?
  5. ΠžΠ±ΡŠΡΡΠ½ΠΈΡ‚Π΅ качСствСнно Π·Π°Π²ΠΈΡΠΈΠΌΠΎΡΡ‚ΡŒ ΠšΠŸΠ” Π½Π°ΠΊΠ»ΠΎΠ½Π½ΠΎΠΉ плоскости ΠΈ Π²Ρ‹ΠΈΠ³Ρ€Ρ‹ΡˆΠ° Π² силС, ΠΏΠΎΠ»ΡƒΡ‡Π°Π΅ΠΌΠΎΠ³ΠΎ с Π΅Π΅ ΠΏΠΎΠΌΠΎΡ‰ΡŒΡŽ, ΠΎΡ‚ ΡƒΠ³Π»Π° Π½Π°ΠΊΠ»ΠΎΠ½Π° плоскости.

ΠŸΡ€ΠΈΠ»ΠΎΠΆΠ΅Π½ΠΈΠ΅ 3

ΠŸΠ΅Ρ€Π΅Ρ‡Π½ΡŒ ΡΠΊΡΠΏΠ΅Ρ€Π΅ΠΌΠ΅Π½Ρ‚Π°Π»ΡŒΠ½Ρ‹Ρ… Π·Π°Π΄Π°Π½ΠΈΠΉ для 7 класса

  1. Π˜Π·ΠΌΠ΅Ρ€Π΅Π½ΠΈΠ΅ Ρ€Π°Π·ΠΌΠ΅Ρ€ΠΎΠ² бруска.
  2. Π˜Π·ΠΌΠ΅Ρ€Π΅Π½ΠΈΠ΅ объСма Тидкости ΠΏΡ€ΠΈ ΠΏΠΎΠΌΠΎΡ‰ΠΈ ΠΌΠ΅Π½Π·ΡƒΡ€ΠΊΠΈ.
  3. Π˜Π·ΠΌΠ΅Ρ€Π΅Π½ΠΈΠ΅ плотности Тидкости.
  4. Π˜Π·ΠΌΠ΅Ρ€Π΅Π½ΠΈΠ΅ плотности Ρ‚Π²Π΅Ρ€Π΄ΠΎΠ³ΠΎ Ρ‚Π΅Π»Π°.

ВсС Ρ€Π°Π±ΠΎΡ‚Ρ‹ проводятся с расчСтом ΠΏΠΎΠ³Ρ€Π΅ΡˆΠ½ΠΎΡΡ‚Π΅ΠΉ ΠΈ ΠΏΡ€ΠΎΠ²Π΅Ρ€ΠΊΠΎΠΉ

размСрностСй.

  1. Π˜Π·ΠΌΠ΅Ρ€Π΅Π½ΠΈΠ΅ вСса Ρ‚Π΅Π»Π° ΠΏΡ€ΠΈ ΠΏΠΎΠΌΠΎΡ‰ΠΈ Ρ€Ρ‹Ρ‡Π°Π³Π°.
  2. ВычислСния Π²Ρ‹ΠΈΠ³Ρ€Ρ‹ΡˆΠ° Π² силС инструмСнтов, Π² ΠΊΠΎΡ‚ΠΎΡ€Ρ‹Ρ… ΠΏΡ€ΠΈΠΌΠ΅Π½Π΅Π½ (Π½ΠΎΠΆΠ½ΠΈΡ†Ρ‹, кусачки, плоскогубцы)
  3. НаблюдСниС зависимости кинСтичСской энСргии Ρ‚Π΅Π»Π° ΠΎΡ‚ Π΅Π³ΠΎ скорости ΠΈ массы.
  4. Π’Ρ‹ΡΡΠ½ΠΈΡ‚ΡŒ ΠΎΡ‚ Ρ‡Π΅Π³ΠΎ зависит сила трСния ΡΠΊΡΠΏΠ΅Ρ€ΠΈΠΌΠ΅Π½Ρ‚Π°Π»ΡŒΠ½ΠΎ.

ΠŸΠ΅Ρ€Π΅Ρ‡Π΅Π½ΡŒ ΡΠΊΡΠΏΠ΅Ρ€ΠΈΠΌΠ΅Π½Ρ‚Π°Π»ΡŒΠ½Ρ‹Ρ… Π·Π°Π΄Π°Π½ΠΈΠΉ для 8 класса

  1. НаблюдСниС дСйствий элСктричСского Ρ‚ΠΎΠΊΠ° (Ρ‚Π΅ΠΏΠ»ΠΎΠ²ΠΎΠ³ΠΎ, Ρ…ΠΈΠΌΠΈΡ‡Π΅cΠΊΠΎΠ³ΠΎ, ΠΌΠ°Π³Π½ΠΈΡ‚Π½ΠΎΠ³ΠΎ ΠΈ ΠΏΠΎ возмоТности физиологичСского).
  2. РасчСт характСристик смСшанного соСдинСния ΠΏΡ€ΠΎΠ²ΠΎΠ΄Π½ΠΈΠΊΠΎΠ².
  3. ΠžΠΏΡ€Π΅Π΄Π΅Π»Π΅Π½ΠΈΠ΅ ΡƒΠ΄Π΅Π»ΡŒΠ½ΠΎΠ³ΠΎ сопротивлСния ΠΏΡ€ΠΎΠ²ΠΎΠ΄Π½ΠΈΠΊΠ° с ΠΎΡ†Π΅Π½ΠΊΠΎΠΉ ΠΏΠΎΠ³Ρ€Π΅ΡˆΠ½ΠΎΡΡ‚Π΅ΠΉ.
  4. НаблюдСниС явлСния элСктромагнитной ΠΈΠ½Π΄ΡƒΠΊΡ†ΠΈΠΈ.
  1. НаблюдСниС поглощСния энСргии ΠΏΡ€ΠΈ ΠΏΠ»Π°Π²Π»Π΅Π½ΠΈΠΈ льда.
  2. НаблюдСниС выдСлСния энСргии ΠΏΡ€ΠΈ кристаллизации Π³ΠΈΠΏΠΎΡΡƒΠ»ΡŒΡ„ΠΈΡ‚Π°.
  3. НаблюдСниС поглощСния энСргии ΠΏΡ€ΠΈ испарСнии ТидкостСй.
  4. НаблюдСниС зависимости скорости испарСния Тидкости ΠΎΡ‚ Ρ€ΠΎΠ΄Π° Тидкости, ΠΏΠ»ΠΎΡ‰Π°Π΄ΠΈ Π΅Π΅ свободной повСрхности, Ρ‚Π΅ΠΌΠΏΠ΅Ρ€Π°Ρ‚ΡƒΡ€Ρ‹ ΠΈ скорости удалСния ΠΏΠ°Ρ€ΠΎΠ².
  5. ΠžΠΏΡ€Π΅Π΄Π΅Π»Π΅Π½ΠΈΠ΅ влаТности Π²ΠΎΠ·Π΄ΡƒΡ…Π° Π² ΠΊΠ°Π±ΠΈΠ½Π΅Ρ‚Π΅.

ΠŸΠ΅Ρ€Π΅Ρ‡Π΅Π½ΡŒ ΡΠΊΡΠΏΠ΅Ρ€ΠΈΠΌΠ΅Π½Ρ‚Π°Π»ΡŒΠ½Ρ‹Ρ… Ρ€Π°Π±ΠΎΡ‚ 9 класса

  1. 1.Π˜Π·ΠΌΠ΅Ρ€Π΅Π½ΠΈΠ΅ ΠΌΠΎΠ΄ΡƒΠ»Π΅ΠΉ ΡƒΠ³Π»ΠΎΠ²ΠΎΠΉ ΠΈ Π»ΠΈΠ½Π΅ΠΉΠ½ΠΎΠΉ скоростСй Ρ‚Π΅Π»Π° ΠΏΡ€ΠΈ Ρ€Π°Π²Π½ΠΎΠΌΠ΅Ρ€Π½ΠΎΠΌ Π΄Π²ΠΈΠΆΠ΅Π½ΠΈΠΈ ΠΏΠΎ окруТности.
  2. 2.Π˜Π·ΠΌΠ΅Ρ€Π΅Π½ΠΈΠ΅ модуля Ρ†Π΅Π½Ρ‚Ρ€ΠΎΡΡ‚Ρ€Π΅ΠΌΠΈΡ‚Π΅Π»ΡŒΠ½ΠΎΠ³ΠΎ ускорСния Ρ‚Π΅Π»Π° ΠΏΡ€ΠΈ Ρ€Π°Π²Π½ΠΎΠΌΠ΅Ρ€Π½ΠΎΠΌ Π΄Π²ΠΈΠΆΠ΅Π½ΠΈΠΈ ΠΏΠΎ окруТности.
  3. 3.НаблюдСниС зависимости ΠΌΠΎΠ΄ΡƒΠ»Π΅ΠΉ сил натяТСния Π½ΠΈΡ‚Π΅ΠΉ ΠΎΡ‚ ΡƒΠ³Π»Π° ΠΌΠ΅ΠΆΠ΄Ρƒ Π½ΠΈΠΌΠΈ ΠΏΡ€ΠΈ постоянной Ρ€Π°Π²Π½ΠΎΠ΄Π΅ΠΉΡΡ‚Π²ΡƒΡŽΡ‰Π΅ΠΉ силС.
  4. 4.Π˜Π·ΡƒΡ‡Π΅Π½ΠΈΠ΅ Ρ‚Ρ€Π΅Ρ‚ΡŒΠ΅Π³ΠΎ Π·Π°ΠΊΠΎΠ½Π° ΠΡŒΡŽΡ‚ΠΎΠ½Π°.
  1. НаблюдСниС измСнСния модуля вСса Ρ‚Π΅Π»Π°, двиТущСгося с ускорСниСм.
  2. ВыяснСниС условий равновСсия Ρ‚Π΅Π»Π°, ΠΈΠΌΠ΅ΡŽΡ‰Π΅Π³ΠΎ ось вращСния, ΠΏΡ€ΠΈ дСйствии Π½Π° Π½Π΅Π³ΠΎ сил.
  3. Π˜Π·ΡƒΡ‡Π΅Π½ΠΈΠ΅ Π·Π°ΠΊΠΎΠ½Π° сохранСния ΠΈΠΌΠΏΡƒΠ»ΡŒΡΠ° ΠΏΡ€ΠΈ ΡƒΠΏΡ€ΡƒΠ³ΠΎΠΌ соударСнии Ρ‚Π΅Π».
  4. Π˜Π·ΠΌΠ΅Ρ€Π΅Π½ΠΈΠ΅ ΠšΠŸΠ” ΠΏΠΎΠ΄Π²ΠΈΠΆΠ½ΠΎΠ³ΠΎ Π±Π»ΠΎΠΊΠ°.

ΠŸΡ€ΠΈΠ»ΠΎΠΆΠ΅Π½ΠΈΠ΅ 4

Π­ΠΊΡΠΏΠ΅Ρ€ΠΈΠΌΠ΅Π½Ρ‚Π°Π»ΡŒΠ½Ρ‹Π΅ задания

Π˜Π·ΠΌΠ΅Ρ€Π΅Π½ΠΈΠ΅ Ρ€Π°Π·ΠΌΠ΅Ρ€ΠΎΠ² бруска

ΠŸΡ€ΠΈΠ±ΠΎΡ€Ρ‹ ΠΈ ΠΌΠ°Ρ‚Π΅Ρ€ΠΈΠ°Π»Ρ‹ (рис. 2): 1) Π»ΠΈΠ½Π΅ΠΉΠΊΠ° ΠΈΠ·ΠΌΠ΅Ρ€ΠΈΡ‚Π΅Π»ΡŒΠ½Π°Ρ, 2) брусок дСрСвянный.

ΠŸΠΎΡ€ΡΠ΄ΠΎΠΊ выполнСния Ρ€Π°Π±ΠΎΡ‚Ρ‹:

  • ВычислитС Ρ†Π΅Π½Ρƒ дСлСния ΡˆΠΊΠ°Π»Ρ‹ Π»ΠΈΠ½Π΅ΠΉΠΊΠΈ.
  • Π£ΠΊΠ°ΠΆΠΈΡ‚Π΅ ΠΏΡ€Π΅Π΄Π΅Π» этой ΡˆΠΊΠ°Π»Ρ‹.
  • Π˜Π·ΠΌΠ΅Ρ€ΡŒΡ‚Π΅ Π»ΠΈΠ½Π΅ΠΉΠΊΠΎΠΉ Π΄Π»ΠΈΠ½Ρƒ, ΡˆΠΈΡ€ΠΈΠ½Ρƒ, высоту бруска.
  • Π Π΅Π·ΡƒΠ»ΡŒΡ‚Π°Ρ‚Ρ‹ всСх ΠΈΠ·ΠΌΠ΅Ρ€Π΅Π½ΠΈΠΉ Π·Π°ΠΏΠΈΡˆΠΈΡ‚Π΅ Π² Ρ‚Π΅Ρ‚Ρ€Π°Π΄ΡŒ.

Π˜Π·ΠΌΠ΅Ρ€Π΅Π½ΠΈΠ΅ объСма Тидкости ΠΏΡ€ΠΈ ΠΏΠΎΠΌΠΎΡ‰ΠΈ ΠΌΠ΅Π½Π·ΡƒΡ€ΠΊΠΈ

ΠŸΡ€ΠΈΠ±ΠΎΡ€Ρ‹ ΠΈ ΠΌΠ°Ρ‚Π΅Ρ€ΠΈΠ°Π»Ρ‹ (рис. 3):

  • Ρ†ΠΈΠ»ΠΈΠ½Π΄Ρ€ ΠΈΠ·ΠΌΠ΅Ρ€ΠΈΡ‚Π΅Π»ΡŒΠ½Ρ‹ΠΉ (ΠΌΠ΅Π½Π·ΡƒΡ€ΠΊΠ°),
  • стакан с Π²ΠΎΠ΄ΠΎΠΉ.

ΠŸΠΎΡ€ΡΠ΄ΠΎΠΊ выполнСния Ρ€Π°Π±ΠΎΡ‚Ρ‹

  1. ВычислитС Ρ†Π΅Π½Ρƒ дСлСния ΡˆΠΊΠ°Π»Ρ‹ ΠΌΠ΅Π½Π·ΡƒΡ€ΠΊΠΈ.
  2. ЗарисуйтС Π² Ρ‚Π΅Ρ‚Ρ€Π°Π΄ΠΈ Ρ‡Π°ΡΡ‚ΡŒ ΡˆΠΊΠ°Π»Ρ‹ ΠΌΠ΅Π½Π·ΡƒΡ€ΠΊΠΈ ΠΈ сдСлайтС запись, ΠΏΠΎΡΡΠ½ΡΡŽΡ‰ΡƒΡŽ порядок вычислСния Ρ†Π΅Π½Ρ‹ дСлСния ΡˆΠΊΠ°Π»Ρ‹.
  3. Π£ΠΊΠ°ΠΆΠΈΡ‚Π΅ ΠΏΡ€Π΅Π΄Π΅Π» этой ΡˆΠΊΠ°Π»Ρ‹.
  4. Π˜Π·ΠΌΠ΅Ρ€ΡŒΡ‚Π΅ объСм Π²ΠΎΠ΄Ρ‹ Π² стаканС ΠΏΡ€ΠΈ ΠΏΠΎΠΌΠΎΡ‰ΠΈ ΠΌΠ΅Π½Π·ΡƒΡ€ΠΊΠΈ. " "
  5. Π Π΅Π·ΡƒΠ»ΡŒΡ‚Π°Ρ‚ измСрСния Π·Π°ΠΏΠΈΡˆΠΈΡ‚Π΅ Π² Ρ‚Π΅Ρ‚Ρ€Π°Π΄ΡŒ.
  6. Π’Ρ‹Π»Π΅ΠΉΡ‚Π΅ Π²ΠΎΠ΄Ρƒ ΠΎΠ±Ρ€Π°Ρ‚Π½ΠΎ Π² стакан.

НалСйтС Π² ΠΌΠ΅Π½Π·ΡƒΡ€ΠΊΡƒ, Π½Π°ΠΏΡ€ΠΈΠΌΠ΅Ρ€, 20 ΠΌΠ» Π²ΠΎΠ΄Ρ‹. ПослС ΠΏΡ€ΠΎΠ²Π΅Ρ€ΠΊΠΈ ΡƒΡ‡ΠΈΡ‚Π΅Π»Π΅ΠΌ Π΄ΠΎΠ»Π΅ΠΉΡ‚Π΅ Π² Π½Π΅Π΅ Π΅Ρ‰Π΅ Π²ΠΎΠ΄Ρ‹, довСдя ΡƒΡ€ΠΎΠ²Π΅Π½ΡŒ Π΄ΠΎ дСлСния, Π½Π°ΠΏΡ€ΠΈΠΌΠ΅Ρ€, 50 ΠΌΠ». Бколько Π²ΠΎΠ΄Ρ‹ Π±Ρ‹Π»ΠΎ Π΄ΠΎΠ»ΠΈΡ‚ΠΎ Π² ΠΌΠ΅Π½Π·ΡƒΡ€ΠΊΡƒ

Π˜Π·ΠΌΠ΅Ρ€Π΅Π½ΠΈΠ΅ плотности Тидкости

ΠŸΡ€ΠΈΠ±ΠΎΡ€Ρ‹ ΠΈ ΠΌΠ°Ρ‚Π΅Ρ€ΠΈΠ°Π»Ρ‹ (рис. 14): 1) вСсы ΡƒΡ‡Π΅Π±Π½Ρ‹Π΅, 2) Π³ΠΈΡ€ΠΈ, 3) Ρ†ΠΈΠ»ΠΈΠ½Π΄Ρ€ ΠΈΠ·ΠΌΠ΅Ρ€ΠΈΡ‚Π΅Π»ΡŒΠ½Ρ‹ΠΉ (ΠΌΠ΅Π½Π·ΡƒΡ€ΠΊΠ°), 4) стакан с Π²ΠΎΠ΄ΠΎΠΉ.

ΠŸΠΎΡ€ΡΠ΄ΠΎΠΊ выполнСния Ρ€Π°Π±ΠΎΡ‚Ρ‹

  1. Π—Π°ΠΏΠΈΡˆΠΈΡ‚Π΅:Ρ†Π΅Π½Ρƒ дСлСния ΡˆΠΊΠ°Π»Ρ‹ ΠΌΠ΅Π½Π·ΡƒΡ€ΠΊΠΈ; Π²Π΅Ρ€Ρ…Π½ΠΈΠΉ ΠΏΡ€Π΅Π΄Π΅Π» ΡˆΠΊΠ°Π»Ρ‹ ΠΌΠ΅Π½Π·ΡƒΡ€ΠΊΠΈ.
  2. Π˜Π·ΠΌΠ΅Ρ€ΡŒΡ‚Π΅ массу стакана с Π²ΠΎΠ΄ΠΎΠΉ ΠΏΡ€ΠΈ ΠΏΠΎΠΌΠΎΡ‰ΠΈ вСсов.
  3. ΠŸΠ΅Ρ€Π΅Π»Π΅ΠΉΡ‚Π΅ Π²ΠΎΠ΄Ρƒ ΠΈΠ· стакана Π² ΠΌΠ΅Π½Π·ΡƒΡ€ΠΊΡƒ ΠΈ ΠΈΠ·ΠΌΠ΅Ρ€ΡŒΡ‚Π΅ массу пустого стакана.
  4. ВычислитС массу Π²ΠΎΠ΄Ρ‹ Π² ΠΌΠ΅Π½Π·ΡƒΡ€ΠΊΠ΅.
  5. Π˜Π·ΠΌΠ΅Ρ€ΡŒΡ‚Π΅ объСм Π²ΠΎΠ΄Ρ‹ Π² ΠΌΠ΅Π½Π·ΡƒΡ€ΠΊΠ΅.
  6. ВычислитС ΠΏΠ»ΠΎΡ‚Π½ΠΎΡΡ‚ΡŒ Π²ΠΎΠ΄Ρ‹.

ВычислСниС массы Ρ‚Π΅Π»Π° ΠΏΠΎ Π΅Π³ΠΎ плотности ΠΈ ΠΎΠ±ΡŠΠ΅ΠΌΡƒ

ΠŸΡ€ΠΈΠ±ΠΎΡ€Ρ‹ ΠΈ ΠΌΠ°Ρ‚Π΅Ρ€ΠΈΠ°Π»Ρ‹ (рис. 15): 1) вСсы ΡƒΡ‡Π΅Π±Π½Ρ‹Π΅, 2) Π³ΠΈΡ€ΠΈ, 3) Ρ†ΠΈΠ»ΠΈΠ½Π΄Ρ€ ΠΈΠ·ΠΌΠ΅Ρ€ΠΈΡ‚Π΅Π»ΡŒΠ½Ρ‹ΠΉ (ΠΌΠ΅Π½Π·ΡƒΡ€ΠΊΠ°) с Π²ΠΎΠ΄ΠΎΠΉ, 4)Ρ‚Π΅Π»ΠΎ Π½Π΅ΠΏΡ€Π°Π²ΠΈΠ»ΡŒΠ½ΠΎΠΉ Ρ„ΠΎΡ€ΠΌΡ‹ Π½Π° Π½ΠΈΡ‚ΠΈ, 5)Ρ‚Π°Π±Π»ΠΈΡ†Π° плотностСй.

ΠŸΠΎΡ€ΡΠ΄ΠΎΠΊ выполнСния Ρ€Π°Π±ΠΎΡ‚Ρ‹ (Рис. 15)

  1. Π˜Π·ΠΌΠ΅Ρ€ΡŒΡ‚Π΅ объСм Ρ‚Π΅Π»Π° ΠΏΡ€ΠΈ ΠΏΠΎΠΌΠΎΡ‰ΠΈ ΠΌΠ΅Π½Π·ΡƒΡ€ΠΊΠΈ.
  2. ВычислитС массу Ρ‚Π΅Π»Π°.
  3. ΠŸΡ€ΠΎΠ²Π΅Ρ€ΡŒΡ‚Π΅ Ρ€Π΅Π·ΡƒΠ»ΡŒΡ‚Π°Ρ‚ вычислСния массы Ρ‚Π΅Π»Π° ΠΏΡ€ΠΈ ΠΏΠΎΠΌΠΎΡ‰ΠΈ вСсов.
  4. Π Π΅Π·ΡƒΠ»ΡŒΡ‚Π°Ρ‚Ρ‹ ΠΈΠ·ΠΌΠ΅Ρ€Π΅Π½ΠΈΠΉ ΠΈ вычислСний Π·Π°ΠΏΠΈΡˆΠΈΡ‚Π΅ Π² Ρ‚Π΅Ρ‚Ρ€Π°Π΄ΡŒ.

ВычислСниС объСма Ρ‚Π΅Π»Π° ΠΏΠΎ Π΅Π³ΠΎ плотности ΠΈ массС

ΠŸΡ€ΠΈΠ±ΠΎΡ€Ρ‹ ΠΈ ΠΌΠ°Ρ‚Π΅Ρ€ΠΈΠ°Π»Ρ‹ (рис. 15): 1) вСсы ΡƒΡ‡Π΅Π±Π½Ρ‹Π΅, 2) Π³ΠΈΡ€ΠΈ, 3) Ρ†ΠΈΠ»ΠΈΠ½Π΄Ρ€ ΠΈΠ·ΠΌΠ΅Ρ€ΠΈΡ‚Π΅Π»ΡŒΠ½Ρ‹ΠΉ (ΠΌΠ΅Π½Π·ΡƒΡ€ΠΊΠ°) с Π²ΠΎΠ΄ΠΎΠΉ, 4) Ρ‚Π΅Π»ΠΎ Π½Π΅ΠΏΡ€Π°Π²ΠΈΠ»ΡŒΠ½ΠΎΠΉ Ρ„ΠΎΡ€ΠΌΡ‹ Π½Π° Π½ΠΈΡ‚ΠΈ, Π±) Ρ‚Π°Π±Π»ΠΈΡ†Π° плотностСй.

ΠŸΠΎΡ€ΡΠ΄ΠΎΠΊ выполнСния Ρ€Π°Π±ΠΎΡ‚Ρ‹

  1. Π—Π°ΠΏΠΈΡˆΠΈΡ‚Π΅ вСщСство, ΠΈΠ· ΠΊΠΎΡ‚ΠΎΡ€ΠΎΠ³ΠΎ состоит Ρ‚Π΅Π»ΠΎ Π½Π΅ΠΏΡ€Π°Π²ΠΈΠ»ΡŒΠ½ΠΎΠΉ Ρ„ΠΎΡ€ΠΌΡ‹.
  2. НайдитС Π² Ρ‚Π°Π±Π»ΠΈΡ†Π΅ Π·Π½Π°Ρ‡Π΅Π½ΠΈΠ΅ плотности этого вСщСства.
  3. Π˜Π·ΠΌΠ΅Ρ€ΡŒΡ‚Π΅ массу Ρ‚Π΅Π»Π° ΠΏΡ€ΠΈ ΠΏΠΎΠΌΠΎΡ‰ΠΈ вСсов.
  4. ВычислитС объСм Ρ‚Π΅Π»Π°.
  5. ΠŸΡ€ΠΎΠ²Π΅Ρ€ΡŒΡ‚Π΅ Ρ€Π΅Π·ΡƒΠ»ΡŒΡ‚Π°Ρ‚ вычислСния объСма Ρ‚Π΅Π»Π° ΠΏΡ€ΠΈ ΠΏΠΎΠΌΠΎΡ‰ΠΈ ΠΌΠ΅Π½Π·ΡƒΡ€ΠΊΠΈ.
  6. Π Π΅Π·ΡƒΠ»ΡŒΡ‚Π°Ρ‚Ρ‹ ΠΈΠ·ΠΌΠ΅Ρ€Π΅Π½ΠΈΠΉ ΠΈ вычислСний Π·Π°ΠΏΠΈΡˆΠΈΡ‚Π΅ Π² Ρ‚Π΅Ρ‚Ρ€Π°Π΄ΡŒ.

Π˜Π·ΡƒΡ‡Π΅Π½ΠΈΠ΅ зависимости силы трСния скольТСния ΠΎΡ‚ Ρ€ΠΎΠ΄Π° трущихся повСрхностСй

ΠŸΡ€ΠΈΠ±ΠΎΡ€Ρ‹ ΠΈ ΠΌΠ°Ρ‚Π΅Ρ€ΠΈΠ°Π»Ρ‹ (рис. 23): 1) Π΄ΠΈΠ½Π°ΠΌΠΎΠΌΠ΅Ρ‚Ρ€, 2) Ρ‚Ρ€ΠΈΠ±ΠΎΠΌΠ΅Ρ‚Ρ€ 3) Π³Ρ€ΡƒΠ·Ρ‹ с двумя ΠΊΡ€ΡŽΡ‡ΠΊΠ°ΠΌΠΈ -2 ΡˆΡ‚., 4) лист Π±ΡƒΠΌΠ°Π³ΠΈ, 5) лист Π½Π°ΠΆΠ΄Π°Ρ‡Π½ΠΎΠΉ Π±ΡƒΠΌΠ°Π³ΠΈ.

ΠŸΠΎΡ€ΡΠ΄ΠΎΠΊ выполнСния Ρ€Π°Π±ΠΎΡ‚Ρ‹

1. ΠŸΠΎΠ΄Π³ΠΎΡ‚ΠΎΠ²ΡŒΡ‚Π΅ Π² Ρ‚Π΅Ρ‚Ρ€Π°Π΄ΠΈ Ρ‚Π°Π±Π»ΠΈΡ†Ρƒ для записи Ρ€Π΅Π·ΡƒΠ»ΡŒΡ‚Π°Ρ‚ΠΎΠ² ΠΈΠ·ΠΌΠ΅Ρ€Π΅Π½ΠΈΠΉ:

2.ВычислитС Ρ†Π΅Π½Ρƒ дСлСния ΡˆΠΊΠ°Π»Ρ‹ Π΄ΠΈΠ½Π°ΠΌΠΎΠΌΠ΅Ρ‚Ρ€Π°.
3.Π˜Π·ΠΌΠ΅Ρ€ΡŒΡ‚Π΅ силу трСния скольТСния бруска с двумя Π³Ρ€ΡƒΠ·Π°ΠΌΠΈ:

4. Π Π΅Π·ΡƒΠ»ΡŒΡ‚Π°Ρ‚Ρ‹ ΠΈΠ·ΠΌΠ΅Ρ€Π΅Π½ΠΈΠΉ Π·Π°ΠΏΠΈΡˆΠΈΡ‚Π΅ Π² Ρ‚Π°Π±Π»ΠΈΡ†Ρƒ.

5. ΠžΡ‚Π²Π΅Ρ‚ΡŒΡ‚Π΅ Π½Π° вопросы:

  1. Зависит Π»ΠΈ сила трСния скольТСния:
    Π°) ΠΎΡ‚ Ρ€ΠΎΠ΄Π° трущихся повСрхностСй?
    Π±) ΠΎΡ‚ ΡˆΠ΅Ρ€ΠΎΡ…ΠΎΠ²Π°Ρ‚ΠΎΡΡ‚ΠΈ трущихся повСрхностСй?
  2. Какими способами ΠΌΠΎΠΆΠ½ΠΎ ΡƒΠ²Π΅Π»ΠΈΡ‡ΠΈΡ‚ΡŒ ΠΈ ΡƒΠΌΠ΅Π½ΡŒΡˆΠΈΡ‚ΡŒ силу трСния скольТСния?(рис. 24):
    1) Π΄ΠΈΠ½Π°ΠΌΠΎΠΌΠ΅Ρ‚Ρ€, 2) Ρ‚Ρ€ΠΈΠ±ΠΎΠΌΠ΅Ρ‚Ρ€.

Π˜Π·ΡƒΡ‡Π΅Π½ΠΈΠ΅ зависимости силы трСния скольТСния ΠΎΡ‚ силы давлСния ΠΈ нСзависимости ΠΎΡ‚ ΠΏΠ»ΠΎΡ‰Π°Π΄ΠΈ трущихся повСрхностСй

ΠŸΡ€ΠΈΠ±ΠΎΡ€Ρ‹ ΠΈ ΠΌΠ°Ρ‚Π΅Ρ€ΠΈΠ°Π»Ρ‹: 1) Π΄ΠΈΠ½Π°ΠΌΠΎΠΌΠ΅Ρ‚Ρ€,2) Ρ‚Ρ€ΠΈΠ±ΠΎΠΌΠ΅Ρ‚Ρ€;3) Π³Ρ€ΡƒΠ·Ρ‹ с сдвумя ΠΊΡ€ΡŽΡ‡ΠΊΠ°ΠΌΠΈ - 2 ΡˆΡ‚.

ΠŸΠΎΡ€ΡΠ΄ΠΎΠΊ выполнСния Ρ€Π°Π±ΠΎΡ‚Ρ‹

  1. ВычислитС Ρ†Π΅Π½Ρƒ дСлСния ΡˆΠΊΠ°Π»Ρ‹ Π΄ΠΈΠ½Π°ΠΌΠΎΠΌΠ΅Ρ‚Ρ€Π°.
  2. ΠŸΠΎΠ»ΠΎΠΆΠΈΡ‚Π΅ Π½Π° Π»ΠΈΠ½Π΅ΠΉΠΊΡƒ Ρ‚Ρ€ΠΈΠ±ΠΎΠΌΠ΅Ρ‚Ρ€Π° брусок большой Π³Ρ€Π°Π½ΡŒΡŽ, Π° Π½Π° Π½Π΅Π³ΠΎ - Π³Ρ€ΡƒΠ· ΠΈ ΠΈΠ·ΠΌΠ΅Ρ€ΡŒΡ‚Π΅ силу трСния скольТСния бруска ΠΏΠΎ Π»ΠΈΠ½Π΅ΠΉΠΊΠ΅ (рис. 24, Π°).
  3. ΠŸΠΎΠ»ΠΎΠΆΠΈΡ‚Π΅ Π½Π° брусок Π²Ρ‚ΠΎΡ€ΠΎΠΉ Π³Ρ€ΡƒΠ· ΠΈ снова ΠΈΠ·ΠΌΠ΅Ρ€ΡŒΡ‚Π΅ силу трСния скольТСния бруска ΠΏΠΎ Π»ΠΈΠ½Π΅ΠΉΠΊΠ΅ (рис. 24, Π±).
  4. ΠŸΠΎΠ»ΠΎΠΆΠΈΡ‚Π΅ Π½Π° Π»ΠΈΠ½Π΅ΠΉΠΊΡƒ брусок мСньшСй Π³Ρ€Π°Π½ΡŒΡŽ, ΠΏΠΎΡΡ‚Π°Π²ΡŒΡ‚Π΅ Π½Π° Π½Π΅Π³ΠΎ ΠΎΠΏΡΡ‚ΡŒ Π΄Π²Π° Π³Ρ€ΡƒΠ·Π° ΠΈ снова ΠΈΠ·ΠΌΠ΅Ρ€ΡŒΡ‚Π΅ силу трСния скольТСния бруска ΠΏΠΎ Π»ΠΈΠ½Π΅ΠΉΠΊΠ΅ (рис. 24, Π²)
  5. 5. ΠžΡ‚Π²Π΅Ρ‚ΡŒΡ‚Π΅ Π½Π° вопрос: зависит Π»ΠΈ сила трСния скольТСния:
    Π°) ΠΎΡ‚ силы давлСния, ΠΈ Ссли зависит, Ρ‚ΠΎ ΠΊΠ°ΠΊ?
    Π±) ΠΎΡ‚ ΠΏΠ»ΠΎΡ‰Π°Π΄ΠΈ трущихся повСрхностСй ΠΏΡ€ΠΈ постоянной силС давлСния?

Π˜Π·ΠΌΠ΅Ρ€Π΅Π½ΠΈΠ΅ вСса Ρ‚Π΅Π»Π° ΠΏΡ€ΠΈ ΠΏΠΎΠΌΠΎΡ‰ΠΈ Ρ€Ρ‹Ρ‡Π°Π³Π°

ΠŸΡ€ΠΈΠ±ΠΎΡ€Ρ‹ ΠΈ ΠΌΠ°Ρ‚Π΅Ρ€ΠΈΠ°Π»Ρ‹: 1) Ρ€Ρ‹Ρ‡Π°Π³-Π»ΠΈΠ½Π΅ΠΉΠΊΠ°, 2) Π»ΠΈΠ½Π΅ΠΉΠΊΠ° ΠΈΠ·ΠΌΠ΅Ρ€ΠΈΡ‚Π΅Π»ΡŒΠ½Π°Ρ, 3) Π΄ΠΈΠ½Π°ΠΌΠΎΠΌΠ΅Ρ‚Ρ€, 4) Π³Ρ€ΡƒΠ·Π΅ двумя ΠΊΡ€ΡŽΡ‡ΠΊΠ°ΠΌΠΈ, 5) Ρ†ΠΈΠ»ΠΈΠ½Π΄Ρ€ мСталличСский, 6) ΡˆΡ‚Π°Ρ‚ΠΈΠ².

ΠŸΠΎΡ€ΡΠ΄ΠΎΠΊ выполнСния Ρ€Π°Π±ΠΎΡ‚Ρ‹

  1. ΠŸΠΎΠ΄Π²Π΅ΡΡŒΡ‚Π΅ Ρ€Ρ‹Ρ‡Π°Π³ Π½Π° оси, Π·Π°ΠΊΡ€Π΅ΠΏΠ»Π΅Π½Π½ΠΎΠΉ Π² ΠΌΡƒΡ„Ρ‚Π΅ ΡˆΡ‚Π°Ρ‚ΠΈΠ²Π°. Вращая Π³Π°ΠΉΠΊΠΈ Π½Π° ΠΊΠΎΠ½Ρ†Π°Ρ… Ρ€Ρ‹Ρ‡Π°Π³Π°, установитС Π΅Π³ΠΎ Π² Π³ΠΎΡ€ΠΈΠ·ΠΎΠ½Ρ‚Π°Π»ΡŒΠ½ΠΎΠ΅ ΠΏΠΎΠ»ΠΎΠΆΠ΅Π½ΠΈΠ΅.
  2. ΠŸΠΎΠ΄Π²Π΅ΡΡŒΡ‚Π΅ ΠΊ Π»Π΅Π²ΠΎΠΉ части Ρ€Ρ‹Ρ‡Π°Π³Π° мСталличСский Ρ†ΠΈΠ»ΠΈΠ½Π΄Ρ€, Π° ΠΊ ΠΏΡ€Π°Π²ΠΎΠΉ - Π³Ρ€ΡƒΠ·, ΠΏΡ€Π΅Π΄Π²Π°Ρ€ΠΈΡ‚Π΅Π»ΡŒΠ½ΠΎ ΠΈΠ·ΠΌΠ΅Ρ€ΠΈΠ² Π΄ΠΈΠ½Π°ΠΌΠΎΠΌΠ΅Ρ‚Ρ€ΠΎΠΌ Π΅Π³ΠΎ вСс. ΠžΠΏΡ‹Ρ‚Π½Ρ‹ΠΌ ΠΏΡƒΡ‚Π΅ΠΌ Π΄ΠΎΠ±Π΅ΠΉΡ‚Π΅ΡΡŒ равновСсия Ρ€Ρ‹Ρ‡Π°Π³Π° с Π³Ρ€ΡƒΠ·ΠΎΠΌ.
  3. Π˜Π·ΠΌΠ΅Ρ€ΡŒΡ‚Π΅ ΠΏΠ»Π΅Ρ‡ΠΈ сил, Π΄Π΅ΠΉΡΡ‚Π²ΡƒΡŽΡ‰ΠΈΡ… Π½Π° Ρ€Ρ‹Ρ‡Π°Π³.
  4. Π˜ΡΠΏΠΎΠ»ΡŒΠ·ΡƒΡ ΠΏΡ€Π°Π²ΠΈΠ»ΠΎ равновСсия Ρ€Ρ‹Ρ‡Π°Π³Π°, вычислитС вСс мСталличСского Ρ†ΠΈΠ»ΠΈΠ½Π΄Ρ€Π°.
  5. Π˜Π·ΠΌΠ΅Ρ€ΡŒΡ‚Π΅ вСс мСталличСского Ρ†ΠΈΠ»ΠΈΠ½Π΄Ρ€Π° ΠΏΡ€ΠΈ ΠΏΠΎΠΌΠΎΡ‰ΠΈ Π΄ΠΈΠ½Π°ΠΌΠΎΠΌΠ΅Ρ‚Ρ€Π° ΠΈ ΠΏΠΎΠ»ΡƒΡ‡Π΅Π½Π½Ρ‹ΠΉ Ρ€Π΅Π·ΡƒΠ»ΡŒΡ‚Π°Ρ‚ сравнитС с расчСтным.
  6. Π Π΅Π·ΡƒΠ»ΡŒΡ‚Π°Ρ‚Ρ‹ ΠΈΠ·ΠΌΠ΅Ρ€Π΅Π½ΠΈΠΉ ΠΈ вычислСний Π·Π°ΠΏΠΈΡˆΠΈΡ‚Π΅ Π² Ρ‚Π΅Ρ‚Ρ€Π°Π΄ΡŒ.
  7. ΠžΡ‚Π²Π΅Ρ‚ΡŒΡ‚Π΅ Π½Π° вопросы: измСнится Π»ΠΈ Ρ€Π΅Π·ΡƒΠ»ΡŒΡ‚Π°Ρ‚ ΠΎΠΏΡ‹Ρ‚Π°, Ссли:
  • Ρ€Ρ‹Ρ‡Π°Π³ ΡƒΡ€Π°Π²Π½ΠΎΠ²Π΅ΡΠΈΡ‚ΡŒ ΠΏΡ€ΠΈ Π΄Ρ€ΡƒΠ³ΠΎΠΉ Π΄Π»ΠΈΠ½Π΅ ΠΏΠ»Π΅Ρ‡ сил, Π΄Π΅ΠΉΡΡ‚Π²ΡƒΡŽΡ‰ΠΈΡ… Π½Π° Π½Π΅Π³ΠΎ?
  • Ρ†ΠΈΠ»ΠΈΠ½Π΄Ρ€ ΠΏΠΎΠ΄Π²Π΅ΡΠΈΡ‚ΡŒ ΠΊ ΠΏΡ€Π°Π²ΠΎΠΉ части Ρ€Ρ‹Ρ‡Π°Π³Π°, Π° ΡƒΡ€Π°Π²Π½ΠΎΠ²Π΅ΡˆΠΈΠ²Π°ΡŽΡ‰ΠΈΠΉ Π³Ρ€ΡƒΠ· - ΠΊ Π»Π΅Π²ΠΎΠΉ?

ВычислСниС Π²Ρ‹ΠΈΠ³Ρ€Ρ‹ΡˆΠ° Π² силС инструмСнтов, Π² ΠΊΠΎΡ‚ΠΎΡ€Ρ‹Ρ… ΠΏΡ€ΠΈΠΌΠ΅Π½Π΅Π½ Ρ€Ρ‹Ρ‡Π°Π³

"ΠŸΡ€ΠΈΠ±ΠΎΡ€Ρ‹ ΠΈ ΠΌΠ°Ρ‚Π΅Ρ€ΠΈΠ°Π»Ρ‹ (рис. 45): 1) Π½ΠΎΠΆΠ½ΠΈΡ†Ρ‹, 2) кусачки, 3) плоскогубцы, 4) Π»ΠΈΠ½Π΅ΠΉΠΊΠ° ΠΈΠ·ΠΌΠ΅Ρ€ΠΈΡ‚Π΅Π»ΡŒΠ½Π°Ρ.

ΠŸΠΎΡ€ΡΠ΄ΠΎΠΊ выполнСния Ρ€Π°Π±ΠΎΡ‚Ρ‹

  1. ΠžΠ·Π½Π°ΠΊΠΎΠΌΡŒΡ‚Π΅ΡΡŒ с устройством ΠΏΡ€Π΅Π΄Π»ΠΎΠΆΠ΅Π½Π½ΠΎΠ³ΠΎ Π²Π°ΠΌ инструмСнта, Π² ΠΊΠΎΡ‚ΠΎΡ€ΠΎΠΌ ΠΏΡ€ΠΈΠΌΠ΅Π½Π΅Π½ Ρ€Ρ‹Ρ‡Π°Π³: Π½Π°ΠΉΠ΄ΠΈΡ‚Π΅ ось вращСния, Ρ‚ΠΎΡ‡ΠΊΠΈ прилоТСния сил.
  2. Π˜Π·ΠΌΠ΅Ρ€ΡŒΡ‚Π΅ ΠΏΠ»Π΅Ρ‡ΠΈ сил.
  3. ВычислитС ΠΏΡ€ΠΈΠΌΠ΅Ρ€Π½ΠΎ, Π² ΠΊΠ°ΠΊΠΈΡ… ΠΏΡ€Π΅Π΄Π΅Π»Π°Ρ… ΠΌΠΎΠΆΠ΅Ρ‚ ΠΈΠ·ΠΌΠ΅Π½ΡΡ‚ΡŒΡΡ Π²Ρ‹-
    ΠΈΠ³Ρ€Ρ‹Ρˆ Π² силС ΠΏΡ€ΠΈ пользовании Π΄Π°Π½Π½Ρ‹ΠΌ инструмСнтом.
  4. Π Π΅Π·ΡƒΠ»ΡŒΡ‚Π°Ρ‚Ρ‹ ΠΈΠ·ΠΌΠ΅Ρ€Π΅Π½ΠΈΠΉ ΠΈ вычислСний Π·Π°ΠΏΠΈΡˆΠΈΡ‚Π΅ Π² Ρ‚Π΅Ρ‚Ρ€Π°Π΄ΡŒ.
  5. ΠžΡ‚Π²Π΅Ρ‚ΡŒΡ‚Π΅ Π½Π° вопросы:
  • Как Π½ΡƒΠΆΠ½ΠΎ Ρ€Π°ΡΠΏΠΎΠ»Π°Π³Π°Ρ‚ΡŒ Ρ€Π°Π·Ρ€Π΅Π·Π°Π΅ΠΌΡ‹ΠΉ ΠΌΠ°Ρ‚Π΅Ρ€ΠΈΠ°Π» Π² Π½ΠΎΠΆΠ½ΠΈΡ†Π°Ρ…, Ρ‡Ρ‚ΠΎΠ±Ρ‹ ΠΏΠΎΠ»ΡƒΡ‡ΠΈΡ‚ΡŒ наибольший Π²Ρ‹ΠΈΠ³Ρ€Ρ‹Ρˆ Π² силС?
  • Как Π½ΡƒΠΆΠ½ΠΎ Π΄Π΅Ρ€ΠΆΠ°Ρ‚ΡŒ кусачки Π² Ρ€ΡƒΠΊΠ΅, Ρ‡Ρ‚ΠΎΠ±Ρ‹ ΠΏΠΎΠ»ΡƒΡ‡ΠΈΡ‚ΡŒ наибольший Π²Ρ‹ΠΈΠ³Ρ€Ρ‹Ρˆ Π² силС?

НаблюдСниС зависимости кинСтичСской энСргии Ρ‚Π΅Π»Π° ΠΎΡ‚ Π΅Π³ΠΎ скорости ΠΈ массы

ΠŸΡ€ΠΈΠ±ΠΎΡ€Ρ‹ ΠΈ ΠΌΠ°Ρ‚Π΅Ρ€ΠΈΠ°Π»Ρ‹ (рис. 50): I) ΡˆΠ°Ρ€Ρ‹ Ρ€Π°Π·Π½ΠΎΠΉ массы - 2 ΡˆΡ‚., 2) ΠΆΠ΅Π»ΠΎΠ±, 3) брусок, 4) Π»Π΅Π½Ρ‚Π° ΠΈΠ·ΠΌΠ΅Ρ€ΠΈΡ‚Π΅Π»ΡŒΠ½Π°Ρ, 5) ΡˆΡ‚Π°Ρ‚ΠΈΠ². Рис. 50.

ΠŸΠΎΡ€ΡΠ΄ΠΎΠΊ выполнСния Ρ€Π°Π±ΠΎΡ‚Ρ‹

  1. Π£ΠΊΡ€Π΅ΠΏΠΈΡ‚Π΅ ΠΆΡ‘Π»ΠΎΠ± Π² Π½Π°ΠΊΠ»ΠΎΠ½Π½ΠΎΠΌ ΠΏΠΎΠ»ΠΎΠΆΠ΅Π½ΠΈΠΈ ΠΏΡ€ΠΈ ΠΏΠΎΠΌΠΎΡ‰ΠΈ ΡˆΡ‚Π°Ρ‚ΠΈΠ²Π°, ΠΊΠ°ΠΊ ΠΏΠΎΠΊΠ°Π·Π°Π½ΠΎ Π½Π° рисункС 50. К Π½ΠΈΠΆΠ½Π΅ΠΌΡƒ ΠΊΠΎΠ½Ρ†Ρƒ ΠΆΠ΅Π»ΠΎΠ±Π° ΠΏΡ€ΠΈΡΡ‚Π°Π²ΡŒΡ‚Π΅ дСрСвянный брусок
  2. ΠŸΠΎΠ»ΠΎΠΆΠΈΡ‚Π΅ Π½Π° сСрСдину ΠΆΠ΅Π»ΠΎΠ±Π° ΡˆΠ°Ρ€ΠΈΠΊ мСньшСй массы ΠΈ, отпустив Π΅Π³ΠΎ, Π½Π°Π±Π»ΡŽΠ΄Π°ΠΉΡ‚Π΅, ΠΊΠ°ΠΊ ΡˆΠ°Ρ€ΠΈΠΊ, ΡΠΊΠ°Ρ‚ΠΈΠ²ΡˆΠΈΡΡŒ с ΠΆΠ΅Π»ΠΎΠ±Π° ΠΈ ΡƒΠ΄Π°Ρ€ΠΈΠ²ΡˆΠΈΡΡŒ ΠΎ дСрСвянный брусок, ΠΏΠ΅Ρ€Π΅Π΄Π²ΠΈΠ½Π΅Ρ‚ послСдний Π½Π° Π½Π΅ΠΊΠΎΡ‚ΠΎΡ€ΠΎΠ΅ расстояниС,ΡΠΎΠ²Π΅Ρ€ΡˆΠ°Ρ Ρ€Π°Π±ΠΎΡ‚Ρƒ ΠΏΠΎ ΠΏΡ€Π΅ΠΎΠ΄ΠΎΠ»Π΅Π½ΠΈΡŽ силы трСния.
  3. Π˜Π·ΠΌΠ΅Ρ€ΡŒΡ‚Π΅ расстояниС, Π½Π° ΠΊΠΎΡ‚ΠΎΡ€ΠΎΠ΅ пСрСмСстился брусок.
  4. ΠŸΠΎΠ²Ρ‚ΠΎΡ€ΠΈΡ‚Π΅ ΠΎΠΏΡ‹Ρ‚, пустив ΡˆΠ°Ρ€ΠΈΠΊ с Π²Π΅Ρ€Ρ…Π½Π΅Π³ΠΎ ΠΊΠΎΠ½Ρ†Π° ΠΆΠ΅Π»ΠΎΠ±Π°, ΠΈ снова ΠΈΠ·ΠΌΠ΅Ρ€ΡŒΡ‚Π΅ расстояниС, Π½Π° ΠΊΠΎΡ‚ΠΎΡ€ΠΎΠ΅ пСрСмСстился брусок.
  5. ΠŸΡƒΡΡ‚ΠΈΡ‚Π΅ с сСрСдины ΠΆΠ΅Π»ΠΎΠ±Π° ΡˆΠ°Ρ€ΠΈΠΊ большСй массы ΠΈ снова ΠΈΠ·ΠΌΠ΅Ρ€ΡŒΡ‚Π΅ ΠΏΠ΅Ρ€Π΅ΠΌΠ΅Ρ‰Π΅Π½ΠΈΠ΅ бруска.

Π˜Π·ΠΌΠ΅Ρ€Π΅Π½ΠΈΠ΅ ΠΌΠΎΠ΄ΡƒΠ»Π΅ΠΉ ΡƒΠ³Π»ΠΎΠ²ΠΎΠΉ ΠΈ Π»ΠΈΠ½Π΅ΠΉΠ½ΠΎΠΉ скоростСй Ρ‚Π΅Π»Π° ΠΏΡ€ΠΈ Ρ€Π°Π²Π½ΠΎΠΌΠ΅Ρ€Π½ΠΎΠΌ Π΄Π²ΠΈΠΆΠ΅Π½ΠΈΠΈ ΠΏΠΎ окруТности

ΠŸΡ€ΠΈΠ±ΠΎΡ€Ρ‹ ΠΈ ΠΌΠ°Ρ‚Π΅Ρ€ΠΈΠ°Π»Ρ‹* 1) ΡˆΠ°Ρ€ΠΈΠΊ Π΄ΠΈΠ°ΠΌΠ΅Ρ‚Ρ€ΠΎΠΌ 25 ΠΌΠΌ Π½Π° Π½ΠΈΡ‚ΠΈ Π΄Π»ΠΈΠ½ΠΎΠΉ 200 ΠΌΠΌ, 2) Π»ΠΈΠ½Π΅ΠΉΠΊΠ° ΠΈΠ·ΠΌΠ΅Ρ€ΠΈΡ‚Π΅Π»ΡŒΠ½Π°Ρ 30-35 см с ΠΌΠΈΠ»Π»ΠΈΠΌΠ΅Ρ‚Ρ€ΠΎΠ²Ρ‹ΠΌΠΈ дСлСниями, 3) часы с сСкундной стрСлкой ΠΈΠ»ΠΈ ΠΌΠ΅Ρ‚Ρ€ΠΎΠ½ΠΎΠΌ мСханичСский (ΠΎΠ΄ΠΈΠ½ Π½Π° класс).

ΠŸΠΎΡ€ΡΠ΄ΠΎΠΊ выполнСния Ρ€Π°Π±ΠΎΡ‚Ρ‹

  1. ΠŸΠΎΠ΄Π½ΠΈΠΌΠΈΡ‚Π΅ ΡˆΠ°Ρ€ΠΈΠΊ Π·Π° ΠΊΠΎΠ½Π΅Ρ† Π½ΠΈΡ‚ΠΈ Π½Π°Π΄ Π»ΠΈΠ½Π΅ΠΉΠΊΠΎΠΉ ΠΈ ΠΏΡ€ΠΈΠ²Π΅Π΄ΠΈΡ‚Π΅ Π΅Π³ΠΎ Π² Ρ€Π°Π²Π½ΠΎΠΌΠ΅Ρ€Π½ΠΎΠ΅ Π΄Π²ΠΈΠΆΠ΅Π½ΠΈΠ΅ ΠΏΠΎ окруТности Ρ‚Π°ΠΊ, Ρ‡Ρ‚ΠΎΠ±Ρ‹ ΠΎΠ½ ΠΏΡ€ΠΈ Π²Ρ€Π°Ρ‰Π΅Π½ΠΈΠΈ ΠΊΠ°ΠΆΠ΄Ρ‹ΠΉ Ρ€Π°Π· ΠΏΡ€ΠΎΡ…ΠΎΠ΄ΠΈΠ» Ρ‡Π΅Ρ€Π΅Π· Π½ΡƒΠ»Π΅Π²ΠΎΠ΅ ΠΈ, Π½Π°ΠΏΡ€ΠΈΠΌΠ΅Ρ€, дСсятоС Π΄Π΅Π»Π΅Π½ΠΈΠ΅ ΡˆΠΊΠ°Π»Ρ‹ (рис. 9). Для получСния устойчивого двиТСния ΡˆΠ°Ρ€ΠΈΠΊΠ° Π»ΠΎΠΊΠΎΡ‚ΡŒ Ρ€ΡƒΠΊΠΈ, ΡƒΠ΄Π΅Ρ€ΠΆΠΈΠ²Π°ΡŽΡ‰Π΅ΠΉ Π½ΠΈΡ‚ΡŒ, ΠΏΠΎΡΡ‚Π°Π²ΡŒΡ‚Π΅ Π½Π° стол
  2. Π˜Π·ΠΌΠ΅Ρ€ΡŒΡ‚Π΅ врСмя, Π½Π°ΠΏΡ€ΠΈΠΌΠ΅Ρ€, 30 ΠΏΠΎΠ»Π½Ρ‹Ρ… ΠΎΠ±ΠΎΡ€ΠΎΡ‚ΠΎΠ² ΡˆΠ°Ρ€ΠΈΠΊΠ°.
  3. Зная врСмя двиТСния, число ΠΎΠ±ΠΎΡ€ΠΎΡ‚ΠΎΠ² ΠΈ радиус вращСния, вычислитС ΠΌΠΎΠ΄ΡƒΠ»ΠΈ ΡƒΠ³Π»ΠΎΠ²ΠΎΠΉ ΠΈ Π»ΠΈΠ½Π΅ΠΉΠ½ΠΎΠΉ скоростСй ΡˆΠ°Ρ€ΠΈΠΊΠ° ΠΎΡ‚Π½ΠΎΡΠΈΡ‚Π΅Π»ΡŒΠ½ΠΎ стола.
  4. Π Π΅Π·ΡƒΠ»ΡŒΡ‚Π°Ρ‚Ρ‹ ΠΈΠ·ΠΌΠ΅Ρ€Π΅Π½ΠΈΠΉ ΠΈ вычислСний Π·Π°ΠΏΠΈΡˆΠΈΡ‚Π΅ Π² Ρ‚Π΅Ρ‚Ρ€Π°Π΄ΡŒ.
  5. ΠžΡ‚Π²Π΅Ρ‚ΡŒΡ‚Π΅ Π½Π° вопросы:

Π˜Π·ΠΌΠ΅Ρ€Π΅Π½ΠΈΠ΅ модуля Ρ†Π΅Π½Ρ‚Ρ€ΠΎΡΡ‚Ρ€Π΅ΠΌΠΈΡ‚Π΅Π»ΡŒΠ½ΠΎΠ³ΠΎ ускорСния Ρ‚Π΅Π»Π° ΠΏΡ€ΠΈ Ρ€Π°Π²Π½ΠΎΠΌΠ΅Ρ€Π½ΠΎΠΌ Π΄Π²ΠΈΠΆΠ΅Π½ΠΈΠΈ ΠΏΠΎ окруТности

ΠŸΡ€ΠΈΠ±ΠΎΡ€Ρ‹ ΠΈ ΠΌΠ°Ρ‚Π΅Ρ€ΠΈΠ°Π»Ρ‹ Ρ‚Π΅ ΠΆΠ΅, Ρ‡Ρ‚ΠΎ Π² Π·Π°Π΄Π°Π½ΠΈΠΈ 11.

ΠŸΠΎΡ€ΡΠ΄ΠΎΠΊ выполнСния Ρ€Π°Π±ΠΎΡ‚Ρ‹

  1. Π’Ρ‹ΠΏΠΎΠ»Π½ΠΈΡ‚Π΅ ΠΏΠΏ. 1, 2 задания 11.
  2. Зная врСмя двиТСния, число ΠΎΠ±ΠΎΡ€ΠΎΡ‚ΠΎΠ² ΠΈ радиус вращСния, вычислитС ΠΌΠΎΠ΄ΡƒΠ»ΡŒ Ρ†Π΅Π½Ρ‚Ρ€ΠΎΡΡ‚Ρ€Π΅ΠΌΠΈΡ‚Π΅Π»ΡŒΠ½ΠΎΠ³ΠΎ ускорСния ΡˆΠ°Ρ€ΠΈΠΊΠ°.
  3. Π Π΅Π·ΡƒΠ»ΡŒΡ‚Π°Ρ‚Ρ‹ ΠΈΠ·ΠΌΠ΅Ρ€Π΅Π½ΠΈΠΉ ΠΈ вычислСний Π·Π°ΠΏΠΈΡˆΠΈΡ‚Π΅ Π² Ρ‚Π΅Ρ‚Ρ€Π°Π΄ΡŒ:
  4. ΠžΡ‚Π²Π΅Ρ‚ΡŒΡ‚Π΅ Π½Π° вопросы:
  • Как измСнится ΠΌΠΎΠ΄ΡƒΠ»ΡŒ Ρ†Π΅Π½Ρ‚Ρ€ΠΎΡΡ‚Ρ€Π΅ΠΌΠΈΡ‚Π΅Π»ΡŒΠ½ΠΎΠ³ΠΎ ускорСния ΡˆΠ°Ρ€ΠΈΠΊΠ°, Ссли число Π΅Π³ΠΎ ΠΎΠ±ΠΎΡ€ΠΎΡ‚ΠΎΠ² Π² Π΅Π΄ΠΈΠ½ΠΈΡ†Ρƒ Π²Ρ€Π΅ΠΌΠ΅Π½ΠΈ ΡƒΠ²Π΅Π»ΠΈΡ‡ΠΈΡ‚ΡŒ Π² 2 Ρ€Π°Π·Π°?
  • Как измСнится ΠΌΠΎΠ΄ΡƒΠ»ΡŒ Ρ†Π΅Π½Ρ‚Ρ€ΠΎΡΡ‚Ρ€Π΅ΠΌΠΈΡ‚Π΅Π»ΡŒΠ½ΠΎΠ³ΠΎ ускорСния ΡˆΠ°Ρ€ΠΈΠΊΠ°, Ссли радиус Π΅Π³ΠΎ вращСния ΡƒΠ²Π΅Π»ΠΈΡ‡ΠΈΡ‚ΡŒ Π² 2 Ρ€Π°Π·Π°?

НаблюдСниС зависимости ΠΌΠΎΠ΄ΡƒΠ»Π΅ΠΉ сил натяТСния Π½ΠΈΡ‚Π΅ΠΉ ΠΎΡ‚ ΡƒΠ³Π»Π° ΠΌΠ΅ΠΆΠ΄Ρƒ Π½ΠΈΠΌΠΈ ΠΏΡ€ΠΈ постоянной Ρ€Π°Π²Π½ΠΎΠ΄Π΅ΠΉΡΡ‚Π²ΡƒΡŽΡ‰Π΅ΠΉ силС

ΠŸΡ€ΠΈΠ±ΠΎΡ€Ρ‹ ΠΈ ΠΌΠ°Ρ‚Π΅Ρ€ΠΈΠ°Π»Ρ‹: 1) Π³Ρ€ΡƒΠ· массой 100 Π³ с двумя ΠΊΡ€ΡŽΡ‡ΠΊΠ°ΠΌΠΈ, 2) Π΄ΠΈΠ½Π°ΠΌΠΎΠΌΠ΅Ρ‚Ρ€Ρ‹ ΡƒΡ‡Π΅Π±Π½Ρ‹Π΅ - 2 ΡˆΡ‚., 3) Π½ΠΈΡ‚ΡŒ Π΄Π»ΠΈΠ½ΠΎΠΉ 200 ΠΌΠΌ с пСтлями Π½Π° ΠΊΠΎΠ½Ρ†Π°Ρ….

ΠŸΠΎΡ€ΡΠ΄ΠΎΠΊ выполнСния Ρ€Π°Π±ΠΎΡ‚Ρ‹


  • Π§Π΅ΠΌΡƒ Ρ€Π°Π²Π½Ρ‹ ΠΌΠΎΠ΄ΡƒΠ»ΠΈ сил натяТСния Π½ΠΈΡ‚Π΅ΠΉ? ИзмСнялись Π»ΠΈ ΠΎΠ½ΠΈ Π²ΠΎ врСмя ΠΎΠΏΡ‹Ρ‚Π°?
  • Π§Π΅ΠΌΡƒ Ρ€Π°Π²Π΅Π½ ΠΌΠΎΠ΄ΡƒΠ»ΡŒ Ρ€Π°Π²Π½ΠΎΠ΄Π΅ΠΉΡΡ‚Π²ΡƒΡŽΡ‰Π΅ΠΉ Π΄Π²ΡƒΡ… сил натяТСния Π½ΠΈΡ‚Π΅ΠΉ? ИзмСнялся Π»ΠΈ ΠΎΠ½ Π²ΠΎ врСмя ΠΎΠΏΡ‹Ρ‚Π°?
  • Π§Ρ‚ΠΎ ΠΌΠΎΠΆΠ½ΠΎ ΡΠΊΠ°Π·Π°Ρ‚ΡŒ ΠΎ зависимости ΠΌΠΎΠ΄ΡƒΠ»Π΅ΠΉ сил натяТСния Π½ΠΈΡ‚Π΅ΠΉ ΠΎΡ‚ ΡƒΠ³Π»Π° ΠΌΠ΅ΠΆΠ΄Ρƒ Π½ΠΈΠΌΠΈ ΠΏΡ€ΠΈ постоянной Ρ€Π°Π²Π½ΠΎΠ΄Π΅ΠΉΡΡ‚Π²ΡƒΡŽΡ‰Π΅ΠΉ силС?

Π˜Π·ΡƒΡ‡Π΅Π½ΠΈΠ΅ Ρ‚Ρ€Π΅Ρ‚ΡŒΠ΅Π³ΠΎ Π·Π°ΠΊΠΎΠ½Π° ΠΡŒΡŽΡ‚ΠΎΠ½Π°

ΠŸΡ€ΠΈΠ±ΠΎΡ€Ρ‹ ΠΈ ΠΌΠ°Ρ‚Π΅Ρ€ΠΈΠ°Π»Ρ‹: I) Π΄ΠΈΠ½Π°ΠΌΠΎΠΌΠ΅Ρ‚Ρ€Ρ‹ ΡƒΡ‡Π΅Π±Π½Ρ‹Π΅ - 2 ΡˆΡ‚., 2) Π½ΠΈΡ‚ΡŒ Π΄Π»ΠΈΠ½ΠΎΠΉ 200 ΠΌΠΌ с пСтлями Π½Π° ΠΊΠΎΠ½Ρ†Π°Ρ….

ΠŸΠΎΡ€ΡΠ΄ΠΎΠΊ выполнСния Ρ€Π°Π±ΠΎΡ‚Ρ‹


  • Π‘ ΠΊΠ°ΠΊΠΎΠΉ силой ΠΏΠΎ ΠΌΠΎΠ΄ΡƒΠ»ΡŽ Π»Π΅Π²Ρ‹ΠΉ Π΄ΠΈΠ½Π°ΠΌΠΎΠΌΠ΅Ρ‚Ρ€ дСйствуСт Π½Π° ΠΏΡ€Π°Π²Ρ‹ΠΉ? Π’ ΠΊΠ°ΠΊΡƒΡŽ сторону Π½Π°ΠΏΡ€Π°Π²Π»Π΅Π½Π° эта сила? К ΠΊΠ°ΠΊΠΎΠΌΡƒ Π΄ΠΈΠ½Π°ΠΌΠΎΠΌΠ΅Ρ‚Ρ€Ρƒ ΠΎΠ½Π° ΠΏΡ€ΠΈΠ»ΠΎΠΆΠ΅Π½Π°?
  • Π‘ ΠΊΠ°ΠΊΠΎΠΉ силой ΠΏΠΎ ΠΌΠΎΠ΄ΡƒΠ»ΡŽ ΠΏΡ€Π°Π²Ρ‹ΠΉ Π΄ΠΈΠ½Π°ΠΌΠΎΠΌΠ΅Ρ‚Ρ€ дСйствуСт Π½Π° Π»Π΅Π²Ρ‹ΠΉ? Π’ ΠΊΠ°ΠΊΡƒΡŽ сторону Π½Π°ΠΏΡ€Π°Π²Π»Π΅Π½Π° эта сила? К ΠΊΠ°ΠΊΠΎΠΌΡƒ Π΄ΠΈΠ½Π°ΠΌΠΎΠΌΠ΅Ρ‚Ρ€Ρƒ ΠΎΠ½Π° ΠΏΡ€ΠΈΠ»ΠΎΠΆΠ΅Π½Π°?

3. Π£Π²Π΅Π»ΠΈΡ‡ΡŒΡ‚Π΅ взаимодСйствиС Π΄ΠΈΠ½Π°ΠΌΠΎΠΌΠ΅Ρ‚Ρ€ΠΎΠ². Π—Π°ΠΌΠ΅Ρ‚ΡŒΡ‚Π΅ ΠΈΡ… Π½ΠΎΠ²Ρ‹Π΅ показания.

4. Π‘ΠΎΠ΅Π΄ΠΈΠ½ΠΈΡ‚Π΅ Π΄ΠΈΠ½Π°ΠΌΠΎΠΌΠ΅Ρ‚Ρ€Ρ‹ Π½ΠΈΡ‚ΡŒΡŽ ΠΈ натянитС Π΅Π΅.

5. ΠžΡ‚Π²Π΅Ρ‚ΡŒΡ‚Π΅ Π½Π° вопросы:

  • Π‘ ΠΊΠ°ΠΊΠΎΠΉ силой ΠΏΠΎ ΠΌΠΎΠ΄ΡƒΠ»ΡŽ Π»Π΅Π²Ρ‹ΠΉ Π΄ΠΈΠ½Π°ΠΌΠΎΠΌΠ΅Ρ‚Ρ€ дСйствуСт Π½Π° Π½ΠΈΡ‚ΡŒ?
  • Π‘ ΠΊΠ°ΠΊΠΎΠΉ силой ΠΏΠΎ ΠΌΠΎΠ΄ΡƒΠ»ΡŽ ΠΏΡ€Π°Π²Ρ‹ΠΉ Π΄ΠΈΠ½Π°ΠΌΠΎΠΌΠ΅Ρ‚Ρ€ дСйствуСт Π½Π° Π½ΠΈΡ‚ΡŒ?
  • Π‘ ΠΊΠ°ΠΊΠΎΠΉ силой ΠΏΠΎ ΠΌΠΎΠ΄ΡƒΠ»ΡŽ растягиваСтся Π½ΠΈΡ‚ΡŒ?

6. Π‘Π΄Π΅Π»Π°ΠΉΡ‚Π΅ ΠΎΠ±Ρ‰ΠΈΠΉ Π²Ρ‹Π²ΠΎΠ΄ ΠΈΠ· ΠΏΡ€ΠΎΠ΄Π΅Π»Π°Π½Π½Ρ‹Ρ… ΠΎΠΏΡ‹Ρ‚ΠΎΠ².

НаблюдСниС измСнСния модуля вСса Ρ‚Π΅Π»Π°, двиТущСгося с ускорСниСм

ΠŸΡ€ΠΈΠ±ΠΎΡ€Ρ‹ ΠΈ ΠΌΠ°Ρ‚Π΅Ρ€ΠΈΠ°Π»Ρ‹: 1) Π΄ΠΈΠ½Π°ΠΌΠΎΠΌΠ΅Ρ‚Ρ€ ΡƒΡ‡Π΅Π±Π½Ρ‹ΠΉ, 2) Π³Ρ€ΡƒΠ· массой 100 Π³ с двумя ΠΊΡ€ΡŽΡ‡ΠΊΠ°ΠΌΠΈ, 3) Π½ΠΈΡ‚ΡŒ Π΄Π»ΠΈΠ½ΠΎΠΉ 200 ΠΌΠΌ с пСтлями Π½Π° ΠΊΠΎΠ½Ρ†Π°Ρ….

ΠŸΠΎΡ€ΡΠ΄ΠΎΠΊ выполнСния Ρ€Π°Π±ΠΎΡ‚Ρ‹

  • ИзмСнялась Π»ΠΈ ΡΠΊΠΎΡ€ΠΎΡΡ‚ΡŒ двиТСния Π³Ρ€ΡƒΠ·Π° ΠΏΡ€ΠΈ Π΅Π³ΠΎ Π΄Π²ΠΈΠΆΠ΅Π½ΠΈΠΈ Π²Π²Π΅Ρ€Ρ… ΠΈ Π²Π½ΠΈΠ·?
  • Как измСнялся ΠΌΠΎΠ΄ΡƒΠ»ΡŒ вСса Π³Ρ€ΡƒΠ·Π° ΠΏΡ€ΠΈ Π΅Π³ΠΎ ускорСнном Π΄Π²ΠΈΠΆΠ΅Π½ΠΈΠΈ Π²Π²Π΅Ρ€Ρ… ΠΈ Π²Π½ΠΈΠ·?

4. ΠŸΠΎΡΡ‚Π°Π²ΡŒΡ‚Π΅ Π΄ΠΈΠ½Π°ΠΌΠΎΠΌΠ΅Ρ‚Ρ€ Π½Π° ΠΊΡ€Π°ΠΉ стола. ΠžΡ‚ΠΊΠ»ΠΎΠ½ΠΈΡ‚Π΅ Π³Ρ€ΡƒΠ· Π² сторону Π½Π° Π½Π΅ΠΊΠΎΡ‚ΠΎΡ€Ρ‹ΠΉ ΡƒΠ³ΠΎΠ» ΠΈ отпуститС (рис. 18). ΠΠ°Π±Π»ΡŽΠ΄Π°ΠΉΡ‚Π΅ Π·Π° показаниями Π΄ΠΈΠ½Π°ΠΌΠΎ ΠΌΠ΅Ρ‚Ρ€Π° Π²ΠΎ врСмя ΠΊΠΎΠ»Π΅Π±Π°Π½ΠΈΠΉ Π³Ρ€ΡƒΠ·Π°.

5. ΠžΡ‚Π²Π΅Ρ‚ΡŒΡ‚Π΅ Π½Π° вопросы:

  • Π˜Π·ΠΌΠ΅Π½ΡΠ΅Ρ‚ΡΡ Π»ΠΈ ΡΠΊΠΎΡ€ΠΎΡΡ‚ΡŒ Π³Ρ€ΡƒΠ·Π° ΠΏΡ€ΠΈ Π΅Π³ΠΎ колСбаниях?
  • Π˜Π·ΠΌΠ΅Π½ΡΡŽΡ‚ΡΡ Π»ΠΈ ускорСниС ΠΈ вСс Π³Ρ€ΡƒΠ·Π° ΠΏΡ€ΠΈ Π΅Π³ΠΎ колСбаниях?
  • Как ΠΈΠ·ΠΌΠ΅Π½ΡΡŽΡ‚ΡΡ Ρ†Π΅Π½Ρ‚Ρ€ΠΎ ΡΡ‚Ρ€Π΅ΠΌΠΈΡ‚Π΅Π»ΡŒΠ½ΠΎΠ΅ ускорСниС ΠΈ вСс Π³Ρ€ΡƒΠ·Π° ΠΏΡ€ΠΈ Π΅Π³ΠΎ колСбаниях?
  • Π’ ΠΊΠ°ΠΊΠΈΡ… Ρ‚ΠΎΡ‡ΠΊΠ°Ρ… Ρ‚Ρ€Π°Π΅ΠΊΡ‚ΠΎΡ€ΠΈΠΈ Ρ†Π΅Π½Ρ‚Ρ€ΠΎΡΡ‚Ρ€Π΅ΠΌΠΈΡ‚Π΅Π»ΡŒΠ½ΠΎΠ΅ ускорСниС ΠΈ вСс Π³Ρ€ΡƒΠ·Π° ΠΏΠΎ ΠΌΠΎΠ΄ΡƒΠ»ΡŽ наибольшиС, Π² ΠΊΠ°ΠΊΠΈΡ… наимСньшиС? Рис 18.

ВыяснСниС условий равновСсия Ρ‚Π΅Π»Π°, ΠΈΠΌΠ΅ΡŽΡ‰Π΅Π³ΠΎ ось вращСния, ΠΏΡ€ΠΈ дСйствии Π½Π° Π½Π΅Π³ΠΎ сил

ΠŸΡ€ΠΈΠ±ΠΎΡ€Ρ‹ ΠΈ ΠΌΠ°Ρ‚Π΅Ρ€ΠΈΠ°Π»Ρ‹: 1) лист ΠΊΠ°Ρ€Ρ‚ΠΎΠ½Π° Ρ€Π°Π·ΠΌΠ΅Ρ€ΠΎΠΌ 150Π₯ 150 ΠΌΠΌ с,вумя нитяными пСтлями, 2) Π΄ΠΈΠ½Π°ΠΌΠΎΠΌΠ΅Ρ‚Ρ€Ρ‹ ΡƒΡ‡Π΅Π±Π½Ρ‹Π΅-2 ΡˆΡ‚., 3) лист ΠΊΠ°Ρ€Ρ‚ΠΎΠ½Π° Ρ€Π°Π·ΠΌΠ΅Ρ€ΠΎΠΌ 240X340 Мм с Π²Π±ΠΈΡ‚Ρ‹ΠΌ Π³Π²ΠΎΠ·Π΄Π΅ΠΌ, 4) ΡƒΠ³ΠΎΠ»ΡŒΠ½ΠΈΠΊ учСничСский, 5) Π»ΠΈΠ½Π΅ΠΉΠΊΠ° ΠΈΠ·ΠΌΠ΅Ρ€ΠΈΡ‚Π΅Π»ΡŒΠ½Π°Ρ 30-35 см с ΠΌΠΈΠ»Π»ΠΈΠΌΠ΅Ρ‚Ρ€ΠΎΠ²Ρ‹ΠΌΠΈ дСлСниями, 6) ΠΊΠ°Ρ€Π°Π½Π΄Π°Ρˆ.

ΠŸΠΎΡ€ΡΠ΄ΠΎΠΊ выполнСния Ρ€Π°Π±ΠΎΡ‚Ρ‹

1. ΠΠ°Π΄Π΅Π½ΡŒΡ‚Π΅ Π½Π° гвоздь лист ΠΊΠ°Ρ€Ρ‚ΠΎΠ½Π°. Π—Π°Ρ†Π΅ΠΏΠΈΡ‚Π΅ Π΄ΠΈΠ½Π°ΠΌΠΎΠΌΠ΅Ρ‚Ρ€Ρ‹ Π·Π° ΠΏΠ΅Ρ‚Π»ΠΈ, натянитС ΠΈΡ… с силами ΠΏΡ€ΠΈΠΌΠ΅Ρ€Π½ΠΎ 2 ΠΈ 3 Н ΠΈ располоТитС ΠΏΠ΅Ρ‚Π»ΠΈ ΠΏΠΎΠ΄ ΡƒΠ³Π»ΠΎΠΌ 100-120Β° Π΄Ρ€ΡƒΠ³ ΠΊ Π΄Ρ€ΡƒΠ³Ρƒ, ΠΊΠ°ΠΊ ΠΏΠΎΠΊΠ°Π·Π°Π½ΠΎ Π½Π° рисункС 27. Π£Π±Π΅Π΄ΠΈΡ‚Π΅ΡΡŒ, Ρ‡Ρ‚ΠΎ лист ΠΊΠ°Ρ€Ρ‚ΠΎΠ½Π° ΠΏΡ€ΠΈ Π΅Π³ΠΎ ΠΎΡ‚ΠΊΠ»ΠΎΠ½Π΅Π½ΠΈΠΈ Π² сторону возвращаСтся Π² состояниС

Рис. 27. Π˜Π·ΠΌΠ΅Ρ€ΡŒΡ‚Π΅ ΠΌΠΎΠ΄ΡƒΠ»ΠΈ ΠΏΡ€ΠΈΠ»ΠΎΠΆΠ΅Π½Π½Ρ‹Ρ… сил (силой тяТСсти ΠΊΠ°Ρ€Ρ‚ΠΎΠ½Π° ΠΏΡ€Π΅Π½Π΅Π±Ρ€Π΅Π³ΠΈΡ‚Π΅).

2. ΠžΡ‚Π²Π΅Ρ‚ΡŒΡ‚Π΅ Π½Π° вопросы:

  • Бколько сил дСйствуСт Π½Π° ΠΊΠ°Ρ€Ρ‚ΠΎΠ½?
  • Π§Π΅ΠΌΡƒ Ρ€Π°Π²Π΅Π½ ΠΌΠΎΠ΄ΡƒΠ»ΡŒ Ρ€Π°Π²Π½ΠΎΠ΄Π΅ΠΉΡΡ‚Π²ΡƒΡŽΡ‰Π΅ΠΉ ΠΏΡ€ΠΈΠ»ΠΎΠΆΠ΅Π½Π½Ρ‹Ρ… ΠΊ ΠΊΠ°Ρ€Ρ‚ΠΎΠ½Ρƒ сил?

3. На листС ΠΊΠ°Ρ€Ρ‚ΠΎΠ½Π° ΠΏΡ€ΠΎΠ²Π΅Π΄ΠΈΡ‚Π΅ ΠΎΡ‚Ρ€Π΅Π·ΠΊΠΈ прямых Π»ΠΈΠ½ΠΈΠΉ, вдоль ΠΊΠΎΡ‚ΠΎΡ€Ρ‹Ρ… Π΄Π΅ΠΉΡΡ‚Π²ΡƒΡŽΡ‚ силы, ΠΈ ΠΏΡ€ΠΈ ΠΏΠΎΠΌΠΎΡ‰ΠΈ ΡƒΠ³ΠΎΠ»ΡŒΠ½ΠΈΠΊΠ° постройтС ΠΏΠ»Π΅Ρ‡ΠΈ этих сил, ΠΊΠ°ΠΊ ΠΏΠΎΠΊΠ°Π·Π°Π½ΠΎ Π½Π° рисункС 28.

4. Π˜Π·ΠΌΠ΅Ρ€ΡŒΡ‚Π΅ ΠΏΠ»Π΅Ρ‡ΠΈ сил.

5. ВычислитС ΠΌΠΎΠΌΠ΅Π½Ρ‚Ρ‹ Π΄Π΅ΠΉΡΡ‚Π²ΡƒΡŽΡ‰ΠΈΡ… сил ΠΈ ΠΈΡ… Π°Π»Π³Π΅Π±Ρ€Π°ΠΈΡ‡Π΅ΡΠΊΡƒΡŽ сумму. ΠŸΡ€ΠΈ ΠΊΠ°ΠΊΠΎΠΌ условии Ρ‚Π΅Π»ΠΎ с Π·Π°ΠΊΡ€Π΅ΠΏΠ»Π΅Π½Π½ΠΎΠΉ осью вращСния находится Π² состоянии равновСсия? Рис. 28. ΠžΡ‚Π²Π΅Ρ‚ Π·Π°ΠΏΠΈΡˆΠΈΡ‚Π΅ Π² Ρ‚Π΅Ρ‚Ρ€Π°Π΄ΡŒ.

Π˜Π·ΡƒΡ‡Π΅Π½ΠΈΠ΅ Π·Π°ΠΊΠΎΠ½Π° сохранСния ΠΈΠΌΠΏΡƒΠ»ΡŒΡΠ° ΠΏΡ€ΠΈ ΡƒΠΏΡ€ΡƒΠ³ΠΎΠΌ соударСнии Ρ‚Π΅Π»

ΠŸΡ€ΠΈΠ±ΠΎΡ€Ρ‹ ΠΈ ΠΌΠ°Ρ‚Π΅Ρ€ΠΈΠ°Π»Ρ‹: 1) ΡˆΠ°Ρ€ΠΈΠΊΠΈ Π΄ΠΈΠ°ΠΌΠ΅Ρ‚Ρ€ΠΎΠΌ 25 ΠΌΠΌ - 2 ΡˆΡ‚., 2) Π½ΠΈΡ‚ΡŒ Π΄Π»ΠΈΠ½ΠΎΠΉ 500 ΠΌΠΌ, 3) ΡˆΡ‚Π°Ρ‚ΠΈΠ² для Ρ„Ρ€ΠΎΠ½Ρ‚Π°Π»ΡŒΠ½Ρ‹Ρ… Ρ€Π°Π±ΠΎΡ‚.

ΠŸΠΎΡ€ΡΠ΄ΠΎΠΊ выполнСния Ρ€Π°Π±ΠΎΡ‚Ρ‹

  • Π§Π΅ΠΌΡƒ Ρ€Π°Π²Π΅Π½ ΠΎΠ±Ρ‰ΠΈΠΉ ΠΈΠΌΠΏΡƒΠ»ΡŒΡ ΡˆΠ°Ρ€ΠΈΠΊΠΎΠ² Π΄ΠΎ взаимодСйствия?
  • ΠžΠ΄ΠΈΠ½Π°ΠΊΠΎΠ²Ρ‹Π΅ Π»ΠΈ ΠΈΠΌΠΏΡƒΠ»ΡŒΡΡ‹ ΠΏΠΎ ΠΌΠΎΠ΄ΡƒΠ»ΡŽ ΠΏΡ€ΠΈΠΎΠ±Ρ€Π΅Π»ΠΈ ΡˆΠ°Ρ€ΠΈΠΊΠΈ послС взаимодСйствия?
  • Π§Π΅ΠΌΡƒ Ρ€Π°Π²Π΅Π½ ΠΎΠ±Ρ‰ΠΈΠΉ ΠΈΠΌΠΏΡƒΠ»ΡŒΡ ΡˆΠ°Ρ€ΠΈΠΊΠΎΠ² послС взаимодСйствия?

4. ΠžΡ‚ΠΏΡƒΡΡ‚ΠΈΡ‚Π΅ ΠΎΡ‚Π²Π΅Π΄Π΅Π½Π½Ρ‹ΠΉ ΡˆΠ°Ρ€ΠΈΠΊ ΠΈ Π·Π°ΠΌΠ΅Ρ‚ΡŒΡ‚Π΅ отклонСния ΡˆΠ°Ρ€ΠΈΠΊΠΎΠ² послС ΡƒΠ΄Π°Ρ€Π°. ΠžΠΏΡ‹Ρ‚ ΠΏΠΎΠ²Ρ‚ΠΎΡ€ΠΈΡ‚Π΅ 2-3 Ρ€Π°Π·Π°.ΠžΡ‚ΠΊΠ»ΠΎΠ½ΠΈΡ‚Π΅ ΠΎΠ΄ΠΈΠ½ ΠΈΠ· ΡˆΠ°Ρ€ΠΈΠΊΠΎΠ² Π½Π° 4-5 см ΠΎΡ‚ полоТСния равновСсия, Π° Π²Ρ‚ΠΎΡ€ΠΎΠΉ ΠΎΡΡ‚Π°Π²ΡŒΡ‚Π΅ Π² ΠΏΠΎΠΊΠΎΠ΅.

5. ΠžΡ‚Π²Π΅Ρ‚ΡŒΡ‚Π΅ Π½Π° вопросы ΠΏ. 3.

6. Π‘Π΄Π΅Π»Π°ΠΉΡ‚Π΅ Π²Ρ‹Π²ΠΎΠ΄ ΠΈΠ· ΠΏΡ€ΠΎΠ΄Π΅Π»Π°Π½Π½Ρ‹Ρ… ΠΎΠΏΡ‹Ρ‚ΠΎΠ²

Π˜Π·ΠΌΠ΅Ρ€Π΅Π½ΠΈΠ΅ ΠšΠŸΠ” ΠΏΠΎΠ΄Π²ΠΈΠΆΠ½ΠΎΠ³ΠΎ Π±Π»ΠΎΠΊΠ°

ΠŸΡ€ΠΈΠ±ΠΎΡ€Ρ‹ ΠΈ ΠΌΠ°Ρ‚Π΅Ρ€ΠΈΠ°Π»Ρ‹: 1) Π±Π»ΠΎΠΊ, 2) Π΄ΠΈΠ½Π°ΠΌΠΎΠΌΠ΅Ρ‚Ρ€ ΡƒΡ‡Π΅Π±Π½Ρ‹ΠΉ, 3) Π»Π΅Π½Ρ‚Π° ΠΈΠ·ΠΌΠ΅Ρ€ΠΈΡ‚Π΅Π»ΡŒΠ½Π°Ρ с сантимСтровыми дСлСниями, 4) Π³Ρ€ΡƒΠ·Ρ‹ массой ΠΏΠΎ 100 Π³ с двумя ΠΊΡ€ΡŽΡ‡ΠΊΠ°ΠΌΠΈ - 3 ΡˆΡ‚., 5) ΡˆΡ‚Π°Ρ‚ΠΈΠ² для Ρ„Ρ€ΠΎΠ½Ρ‚Π°Π»ΡŒΠ½Ρ‹Ρ… Ρ€Π°Π±ΠΎΡ‚, 6) Π½ΠΈΡ‚ΡŒ Π΄Π»ΠΈΠ½ΠΎΠΉ 50 см с пСтлями Π½Π° ΠΊΠΎΠ½Ρ†Π°Ρ….

ΠŸΠΎΡ€ΡΠ΄ΠΎΠΊ выполнСния Ρ€Π°Π±ΠΎΡ‚Ρ‹

  1. Π‘ΠΎΠ±Π΅Ρ€ΠΈΡ‚Π΅ установку с ΠΏΠΎΠ΄Π²ΠΈΠΆΠ½Ρ‹ΠΌ Π±Π»ΠΎΠΊΠΎΠΌ, ΠΊΠ°ΠΊ ΠΏΠΎΠΊΠ°Π·Π°Π½ΠΎ Π½Π° рисункС 42. Π§Π΅Ρ€Π΅Π· Π±Π»ΠΎΠΊ ΠΏΠ΅Ρ€Π΅Π±Ρ€ΠΎΡΡŒΡ‚Π΅ Π½ΠΈΡ‚ΡŒ. Один ΠΊΠΎΠ½Π΅Ρ† Π½ΠΈΡ‚ΠΈ Π·Π°Ρ†Π΅ΠΏΠΈΡ‚Π΅ Π·Π° Π»Π°ΠΏΠΊΡƒ ΡˆΡ‚Π°Ρ‚ΠΈΠ²Π°, Π²Ρ‚ΠΎΡ€ΠΎΠΉ - Π·Π° ΠΊΡ€ΡŽΡ‡ΠΎΠΊ Π΄ΠΈΠ½Π°ΠΌΠΎΠΌΠ΅Ρ‚Ρ€Π°. К ΠΎΠ±ΠΎΠΉΠΌΠ΅ Π±Π»ΠΎΠΊΠ° ΠΏΠΎΠ΄Π²Π΅ΡΡŒΡ‚Π΅ Ρ‚Ρ€ΠΈ Π³Ρ€ΡƒΠ·Π° массой ΠΏΠΎ 100 Π³.
  2. Π’ΠΎΠ·ΡŒΠΌΠΈΡ‚Π΅ Π΄ΠΈΠ½Π°ΠΌΠΎΠΌΠ΅Ρ‚Ρ€ Π² Ρ€ΡƒΠΊΡƒ, располоТитС Π΅Π³ΠΎ Π²Π΅Ρ€Ρ‚ΠΈΠΊΠ°Π»ΡŒΠ½ΠΎ Ρ‚Π°ΠΊ, Ρ‡Ρ‚ΠΎΠ±Ρ‹ Π±Π»ΠΎΠΊ с Π³Ρ€ΡƒΠ·Π°ΠΌΠΈ повис Π½Π° нитях, ΠΈ ΠΈΠ·ΠΌΠ΅Ρ€ΡŒΡ‚Π΅ ΠΌΠΎΠ΄ΡƒΠ»ΡŒ силы натяТСния Π½ΠΈΡ‚ΠΈ.
  3. ΠŸΠΎΠ΄Π½ΠΈΠΌΠΈΡ‚Π΅ Ρ€Π°Π²Π½ΠΎΠΌΠ΅Ρ€Π½ΠΎ Π³Ρ€ΡƒΠ·Ρ‹ Π½Π° Π½Π΅ΠΊΠΎΡ‚ΠΎΡ€ΡƒΡŽ высоту ΠΈ ΠΈΠ·ΠΌΠ΅Ρ€ΡŒΡ‚Π΅ ΠΌΠΎΠ΄ΡƒΠ»ΠΈ ΠΏΠ΅Ρ€Π΅ΠΌΠ΅Ρ‰Π΅Π½ΠΈΠΉ Π³Ρ€ΡƒΠ·ΠΎΠ² ΠΈ Π΄ΠΈΠ½Π°ΠΌΠΎΠΌΠ΅Ρ‚Ρ€Π° ΠΎΡ‚Π½ΠΎΡΠΈΡ‚Π΅Π»ΡŒΠ½ΠΎ стола.
  4. ВычислитС ΠΏΠΎΠ»Π΅Π·Π½ΡƒΡŽ ΠΈ ΡΠΎΠ²Π΅Ρ€ΡˆΠ΅Π½Π½ΡƒΡŽ Ρ€Π°Π±ΠΎΡ‚Ρ‹ ΠΎΡ‚Π½ΠΎΡΠΈΡ‚Π΅Π»ΡŒΠ½ΠΎ стола.
  5. ВычислитС ΠšΠŸΠ” ΠΏΠΎΠ΄Π²ΠΈΠΆΠ½ΠΎΠ³ΠΎ Π±Π»ΠΎΠΊΠ°.
  6. ΠžΡ‚Π²Π΅Ρ‚ΡŒΡ‚Π΅ Π½Π° вопросы:
  • Какой Π²Ρ‹ΠΈΠ³Ρ€Ρ‹Ρˆ Π² силС Π΄Π°Π΅Ρ‚ ΠΏΠΎΠ΄Π²ΠΈΠΆΠ½Ρ‹ΠΉ Π±Π»ΠΎΠΊ?
  • МоТно Π»ΠΈ ΠΏΡ€ΠΈ ΠΏΠΎΠΌΠΎΡ‰ΠΈ ΠΏΠΎΠ΄Π²ΠΈΠΆΠ½ΠΎΠ³ΠΎ Π±Π»ΠΎΠΊΠ° ΠΏΠΎΠ»ΡƒΡ‡ΠΈΡ‚ΡŒ Π²Ρ‹ΠΈΠ³Ρ€Ρ‹Ρˆ Π² Ρ€Π°Π±ΠΎΡ‚Π΅?
  • Как ΠΏΠΎΠ²Ρ‹ΡΠΈΡ‚ΡŒ ΠšΠŸΠ” ΠΏΠΎΠ΄Π²ΠΈΠΆΠ½ΠΎΠ³ΠΎ Π±Π»ΠΎΠΊΠ°?

ΠŸΡ€ΠΈΠ»ΠΎΠΆΠ΅Π½ΠΈΠ΅5

ВрСбования ΠΊ ΡƒΡ€ΠΎΠ²Π½ΡŽ ΠΏΠΎΠ΄Π³ΠΎΡ‚ΠΎΠ²ΠΊΠΈ выпускников основной ΡˆΠΊΠΎΠ»Ρ‹.

1. Π’Π»Π°Π΄Π΅Ρ‚ΡŒ ΠΌΠ΅Ρ‚ΠΎΠ΄Π°ΠΌΠΈ Π½Π°ΡƒΡ‡Π½ΠΎΠ³ΠΎ познания.

1.1. Π‘ΠΎΠ±ΠΈΡ€Π°Ρ‚ΡŒ установки для экспСримСнта ΠΏΠΎ описанию, рисунку ΠΈΠ»ΠΈ схСмС ΠΈ ΠΏΡ€ΠΎΠ²ΠΎΠ΄ΠΈΡ‚ΡŒ наблюдСния ΠΈΠ·ΡƒΡ‡Π°Π΅ΠΌΡ‹Ρ… явлСний.

1.2. Π˜Π·ΠΌΠ΅Ρ€ΡΡ‚ΡŒ: Ρ‚Π΅ΠΌΠΏΠ΅Ρ€Π°Ρ‚ΡƒΡ€Ρƒ, массу, объСм, силу (упругости, тяТСсти, трСния скольТСния), расстояниС, ΠΏΡ€ΠΎΠΌΠ΅ΠΆΡƒΡ‚ΠΎΠΊ Π²Ρ€Π΅ΠΌΠ΅Π½ΠΈ, силу Ρ‚ΠΎΠΊΠ°, напряТСниС, ΠΏΠ»ΠΎΡ‚Π½ΠΎΡΡ‚ΡŒ, ΠΏΠ΅Ρ€ΠΈΠΎΠ΄ ΠΊΠΎΠ»Π΅Π±Π°Π½ΠΈΠΉ маятника, фокусноС расстояниС ΡΠΎΠ±ΠΈΡ€Π°ΡŽΡ‰Π΅ΠΉ Π»ΠΈΠ½Π·Ρ‹.

1.3. ΠŸΡ€Π΅Π΄ΡΡ‚Π°Π²Π»ΡΡ‚ΡŒ Ρ€Π΅Π·ΡƒΠ»ΡŒΡ‚Π°Ρ‚Ρ‹ ΠΈΠ·ΠΌΠ΅Ρ€Π΅Π½ΠΈΠΉ Π² Π²ΠΈΠ΄Π΅ Ρ‚Π°Π±Π»ΠΈΡ†, Π³Ρ€Π°Ρ„ΠΈΠΊΠΎΠ² ΠΈ Π²Ρ‹ΡΠ²Π»ΡΡ‚ΡŒ эмпиричСскиС закономСрности:

  • измСнСния ΠΊΠΎΠΎΡ€Π΄ΠΈΠ½Π°Ρ‚Ρ‹ Ρ‚Π΅Π»Π° ΠΎΡ‚ Π²Ρ€Π΅ΠΌΠ΅Π½ΠΈ;
  • силы упругости ΠΎΡ‚ удлинСния ΠΏΡ€ΡƒΠΆΠΈΠ½Ρ‹;
  • силы Ρ‚ΠΎΠΊΠ° Π² рСзисторС ΠΎΡ‚ напряТСния;
  • массы вСщСства ΠΎΡ‚ Π΅Π³ΠΎ объСма;
  • Ρ‚Π΅ΠΌΠΏΠ΅Ρ€Π°Ρ‚ΡƒΡ€Ρ‹ Ρ‚Π΅Π»Π° ΠΎΡ‚ Π²Ρ€Π΅ΠΌΠ΅Π½ΠΈ ΠΏΡ€ΠΈ Ρ‚Π΅ΠΏΠ»ΠΎΠΎΠ±ΠΌΠ΅Π½Π΅.

1.4. ΠžΠ±ΡŠΡΡΠ½ΡΡ‚ΡŒ Ρ€Π΅Π·ΡƒΠ»ΡŒΡ‚Π°Ρ‚Ρ‹ наблюдСний ΠΈ экспСримСнтов:

  • смСну дня ΠΈ Π½ΠΎΡ‡ΠΈ Π² систСмС отсчСта, связанной с Π—Π΅ΠΌΠ»Π΅ΠΉ, ΠΈ Π² систСмС отсчСта, связанной с Π‘ΠΎΠ»Π½Ρ†Π΅ΠΌ;
  • Π±ΠΎΠ»ΡŒΡˆΡƒΡŽ ΡΠΆΠΈΠΌΠ°Π΅ΠΌΠΎΡΡ‚ΡŒ Π³Π°Π·ΠΎΠ²;
  • ΠΌΠ°Π»ΡƒΡŽ ΡΠΆΠΈΠΌΠ°Π΅ΠΌΠΎΡΡ‚ΡŒ ТидкостСй ΠΈ Ρ‚Π²Π΅Ρ€Π΄Ρ‹Ρ… Ρ‚Π΅Π»;
  • процСссы испарСния ΠΈ плавлСния вСщСства;
  • испарСниС ТидкостСй ΠΏΡ€ΠΈ любой Ρ‚Π΅ΠΌΠΏΠ΅Ρ€Π°Ρ‚ΡƒΡ€Π΅ ΠΈ Π΅Π΅ ΠΎΡ…Π»Π°ΠΆΠ΄Π΅Π½ΠΈΠ΅ ΠΏΡ€ΠΈ испарСнии.

1.5. ΠŸΡ€ΠΈΠΌΠ΅Π½ΡΡ‚ΡŒ ΡΠΊΡΠΏΠ΅Ρ€ΠΈΠΌΠ΅Π½Ρ‚Π°Π»ΡŒΠ½Ρ‹Π΅ Ρ€Π΅Π·ΡƒΠ»ΡŒΡ‚Π°Ρ‚Ρ‹ для прСдсказания значСния Π²Π΅Π»ΠΈΡ‡ΠΈΠ½, Ρ…Π°Ρ€Π°ΠΊΡ‚Π΅Ρ€ΠΈΠ·ΡƒΡŽΡ‰ΠΈΡ… Ρ…ΠΎΠ΄ физичСских явлСний:

  • ΠΏΠΎΠ»ΠΎΠΆΠ΅Π½ΠΈΠ΅ Ρ‚Π΅Π»Π° ΠΏΡ€ΠΈ Π΅Π³ΠΎ Π΄Π²ΠΈΠΆΠ΅Π½ΠΈΠΈ ΠΏΠΎΠ΄ дСйствиСм силы;
  • ΡƒΠ΄Π»ΠΈΠ½Π΅Π½ΠΈΠ΅ ΠΏΡ€ΡƒΠΆΠΈΠ½Ρ‹ ΠΏΠΎΠ΄ дСйствиСм подвСшСнного Π³Ρ€ΡƒΠ·Π°;
  • силу Ρ‚ΠΎΠΊΠ° ΠΏΡ€ΠΈ Π·Π°Π΄Π°Π½Π½ΠΎΠΌ напряТСнии;
  • Π·Π½Π°Ρ‡Π΅Π½ΠΈΠ΅ Ρ‚Π΅ΠΌΠΏΠ΅Ρ€Π°Ρ‚ΡƒΡ€Ρ‹ ΠΎΡΡ‚Ρ‹Π²Π°ΡŽΡ‰Π΅ΠΉ Π²ΠΎΠ΄Ρ‹ Π² Π·Π°Π΄Π°Π½Π½Ρ‹ΠΉ ΠΌΠΎΠΌΠ΅Π½Ρ‚ Π²Ρ€Π΅ΠΌΠ΅Π½ΠΈ.

2. Π’Π»Π°Π΄Π΅Ρ‚ΡŒ основными понятиями ΠΈ Π·Π°ΠΊΠΎΠ½Π°ΠΌΠΈ Ρ„ΠΈΠ·ΠΈΠΊΠΈ.

2.1. Π”Π°Π²Π°Ρ‚ΡŒ ΠΎΠΏΡ€Π΅Π΄Π΅Π»Π΅Π½ΠΈΠ΅ физичСских Π²Π΅Π»ΠΈΡ‡ΠΈΠ½ ΠΈ Ρ„ΠΎΡ€ΠΌΡƒΠ»ΠΈΡ€ΠΎΠ²Π°Ρ‚ΡŒ физичСскиС Π·Π°ΠΊΠΎΠ½Ρ‹.

2.2. ΠžΠΏΠΈΡΡ‹Π²Π°Ρ‚ΡŒ:

  • физичСскиС явлСния ΠΈ процСссы;
  • измСнСния ΠΈ прСобразования энСргии ΠΏΡ€ΠΈ Π°Π½Π°Π»ΠΈΠ·Π΅: свободного падСния Ρ‚Π΅Π», двиТСния Ρ‚Π΅Π» ΠΏΡ€ΠΈ Π½Π°Π»ΠΈΡ‡ΠΈΠΈ трСния, ΠΊΠΎΠ»Π΅Π±Π°Π½ΠΈΠΉ нитяного ΠΈ ΠΏΡ€ΡƒΠΆΠΈΠ½Π½ΠΎΠ³ΠΎ маятников, нагрСвания ΠΏΡ€ΠΎΠ²ΠΎΠ΄Π½ΠΈΠΊΠΎΠ² элСктричСским Ρ‚ΠΎΠΊΠΎΠΌ, плавлСния ΠΈ испарСния вСщСства.

2.3. Π’Ρ‹Ρ‡ΠΈΡΠ»ΡΡ‚ΡŒ:

  • Ρ€Π°Π²Π½ΠΎΠ΄Π΅ΠΉΡΡ‚Π²ΡƒΡŽΡ‰ΡƒΡŽ силу, ΠΈΡΠΏΠΎΠ»ΡŒΠ·ΡƒΡ Π²Ρ‚ΠΎΡ€ΠΎΠΉ Π·Π°ΠΊΠΎΠ½ ΠΡŒΡŽΡ‚ΠΎΠ½Π°;
  • ΠΈΠΌΠΏΡƒΠ»ΡŒΡ Ρ‚Π΅Π»Π°, Ссли извСстны ΡΠΊΠΎΡ€ΠΎΡΡ‚ΡŒ Ρ‚Π΅Π»Π° ΠΈ Π΅Π³ΠΎ масса;
  • расстояниС, Π½Π° ΠΊΠΎΡ‚ΠΎΡ€ΠΎΠ΅ распространяСтся Π·Π²ΡƒΠΊ Π·Π° ΠΎΠΏΡ€Π΅Π΄Π΅Π»Π΅Π½Π½ΠΎΠ΅ врСмя ΠΏΡ€ΠΈ Π·Π°Π΄Π°Π½Π½ΠΎΠΉ скорости;
  • ΠΊΠΈΠ½Π΅Ρ‚ΠΈΡ‡Π΅ΡΠΊΡƒΡŽ ΡΠ½Π΅Ρ€Π³ΠΈΡŽ Ρ‚Π΅Π»Π° ΠΏΡ€ΠΈ Π·Π°Π΄Π°Π½Π½Ρ‹Ρ… массС ΠΈ скорости;
  • ΠΏΠΎΡ‚Π΅Π½Ρ†ΠΈΠ°Π»ΡŒΠ½ΡƒΡŽ ΡΠ½Π΅Ρ€Π³ΠΈΡŽ взаимодСйствия Ρ‚Π΅Π»Π° с Π—Π΅ΠΌΠ»Π΅ΠΉ ΠΈ силу тяТСсти ΠΏΡ€ΠΈ Π·Π°Π΄Π°Π½Π½ΠΎΠΉ массС Ρ‚Π΅Π»Π°;
  • ΡΠ½Π΅Ρ€Π³ΠΈΡŽ, Π²Ρ‹Π΄Π΅Π»ΡΠ΅ΠΌΡƒΡŽ Π² ΠΏΡ€ΠΎΠ²ΠΎΠ΄Π½ΠΈΠΊΠ΅ ΠΏΡ€ΠΈ ΠΏΡ€ΠΎΡ…ΠΎΠΆΠ΄Π΅Π½ΠΈΠΈ элСктричСского Ρ‚ΠΎΠΊΠ° (ΠΏΡ€ΠΈ Π·Π°Π΄Π°Π½Π½Ρ‹Ρ… силС Ρ‚ΠΎΠΊΠ° ΠΈ напряТСнии);
  • ΡΠ½Π΅Ρ€Π³ΠΈΡŽ, ΠΏΠΎΠ³Π»ΠΎΡ‰Π°Π΅ΠΌΡƒΡŽ (Π²Ρ‹Π΄Π΅Π»ΡΠ΅ΠΌΡƒΡŽ) ΠΏΡ€ΠΈ Π½Π°Π³Ρ€Π΅Π²Π°Π½ΠΈΠΈ (ΠΎΡ…Π»Π°ΠΆΠ΄Π΅Π½ΠΈΠΈ) Ρ‚Π΅Π»;

2.4. Π‘Ρ‚Ρ€ΠΎΠΈΡ‚ΡŒ ΠΈΠ·ΠΎΠ±Ρ€Π°ΠΆΠ΅Π½ΠΈΠ΅ Ρ‚ΠΎΡ‡ΠΊΠΈ Π² плоском Π·Π΅Ρ€ΠΊΠ°Π»Π΅ ΠΈ ΡΠΎΠ±ΠΈΡ€Π°ΡŽΡ‰Π΅ΠΉ Π»ΠΈΠ½Π·Π΅.

3. Π’ΠΎΡΠΏΡ€ΠΈΠ½ΠΈΠΌΠ°Ρ‚ΡŒ, ΠΏΠ΅Ρ€Π΅Ρ€Π°Π±Π°Ρ‚Ρ‹Π²Π°Ρ‚ΡŒ ΠΈ ΠΏΡ€Π΅Π΄ΡŠΡΠ²Π»ΡΡ‚ΡŒ ΡƒΡ‡Π΅Π±Π½ΡƒΡŽ ΠΈΠ½Ρ„ΠΎΡ€ΠΌΠ°Ρ†ΠΈΡŽ Π² Ρ€Π°Π·Π»ΠΈΡ‡Π½Ρ‹Ρ… Ρ„ΠΎΡ€ΠΌΠ°Ρ… (словСсной, ΠΎΠ±Ρ€Π°Π·Π½ΠΎΠΉ, символичСской).

3.1. ΠΠ°Π·Ρ‹Π²Π°Ρ‚ΡŒ:

  • источники элСктростатичСского ΠΈ ΠΌΠ°Π³Π½ΠΈΡ‚Π½ΠΎΠ³ΠΎ ΠΏΠΎΠ»Π΅ΠΉ, способы ΠΈΡ… обнаруТСния;
  • прСобразования энСргии Π² двигатСлях Π²Π½ΡƒΡ‚Ρ€Π΅Π½Π½Π΅Π³ΠΎ сгорания, элСктрогСнСраторах, ΡΠ»Π΅ΠΊΡ‚Ρ€ΠΎΠ½Π°Π³Ρ€Π΅Π²Π°Ρ‚Π΅Π»ΡŒΠ½Ρ‹Ρ… ΠΏΡ€ΠΈΠ±ΠΎΡ€Π°Ρ….

3.2. ΠŸΡ€ΠΈΠ²ΠΎΠ΄ΠΈΡ‚ΡŒ ΠΏΡ€ΠΈΠΌΠ΅Ρ€Ρ‹:

  • ΠΎΡ‚Π½ΠΎΡΠΈΡ‚Π΅Π»ΡŒΠ½ΠΎΡΡ‚ΠΈ скорости ΠΈ Ρ‚Ρ€Π°Π΅ΠΊΡ‚ΠΎΡ€ΠΈΠΈ двиТСния ΠΎΠ΄Π½ΠΎΠ³ΠΎ ΠΈ Ρ‚ΠΎΠ³ΠΎ ΠΆΠ΅ Ρ‚Π΅Π»Π° Π² Ρ€Π°Π·Π½Ρ‹Ρ… систСмах отсчСта;
  • ΠΈΠ·ΠΌΠ΅Π½Π΅Π½ΠΈΠ΅ скорости Ρ‚Π΅Π» ΠΏΠΎΠ΄ дСйствиСм силы;
  • дСформация Ρ‚Π΅Π» ΠΏΡ€ΠΈ взаимодСйствии;
  • проявлСниС Π·Π°ΠΊΠΎΠ½Π° сохранСния ΠΈΠΌΠΏΡƒΠ»ΡŒΡΠ° Π² ΠΏΡ€ΠΈΡ€ΠΎΠ΄Π΅ ΠΈ Ρ‚Π΅Ρ…Π½ΠΈΠΊΠ΅;
  • ΠΊΠΎΠ»Π΅Π±Π°Ρ‚Π΅Π»ΡŒΠ½Ρ‹Ρ… ΠΈ Π²ΠΎΠ»Π½ΠΎΠ²Ρ‹Ρ… Π΄Π²ΠΈΠΆΠ΅Π½ΠΈΠΉ Π² ΠΏΡ€ΠΈΡ€ΠΎΠ΄Π΅ ΠΈ Ρ‚Π΅Ρ…Π½ΠΈΠΊΠ΅;
  • экологичСских послСдствий Ρ€Π°Π±ΠΎΡ‚Ρ‹ Π΄Π²ΠΈΠ³Π°Ρ‚Π΅Π»Π΅ΠΉ Π²Π½ΡƒΡ‚Ρ€Π΅Π½Π½Π΅Π³ΠΎ сгорания, Ρ‚Π΅ΠΏΠ»ΠΎΠ²Ρ‹Ρ…, Π°Ρ‚ΠΎΠΌΠ½Ρ‹Ρ… ΠΈ гидроэлСктростанций;
  • ΠΎΠΏΡ‹Ρ‚ΠΎΠ², ΠΏΠΎΠ΄Ρ‚Π²Π΅Ρ€ΠΆΠ΄Π°ΡŽΡ‰ΠΈΡ… основныС полоТСния молСкулярно-кинСтичСской Ρ‚Π΅ΠΎΡ€ΠΈΠΈ.

3.4. Π’Ρ‹Π΄Π΅Π»ΡΡ‚ΡŒ Π³Π»Π°Π²Π½ΡƒΡŽ ΠΌΡ‹ΡΠ»ΡŒ Π² ΠΏΡ€ΠΎΡ‡ΠΈΡ‚Π°Π½Π½ΠΎΠΌ тСкстС.

3.5. ΠΠ°Ρ…ΠΎΠ΄ΠΈΡ‚ΡŒ Π² ΠΏΡ€ΠΎΡ‡ΠΈΡ‚Π°Π½Π½ΠΎΠΌ тСкстС ΠΎΡ‚Π²Π΅Ρ‚Ρ‹ Π½Π° поставлСнныС вопросы.

3.6. ΠšΠΎΠ½ΡΠΏΠ΅ΠΊΡ‚ΠΈΡ€ΠΎΠ²Π°Ρ‚ΡŒ ΠΏΡ€ΠΎΡ‡ΠΈΡ‚Π°Π½Π½Ρ‹ΠΉ тСкст.

3.7. ΠžΠΏΡ€Π΅Π΄Π΅Π»ΡΡ‚ΡŒ:

  • ΠΏΡ€ΠΎΠΌΠ΅ΠΆΡƒΡ‚ΠΎΡ‡Π½Ρ‹Π΅ значСния Π²Π΅Π»ΠΈΡ‡ΠΈΠ½ ΠΏΠΎ Ρ‚Π°Π±Π»ΠΈΡ†Π°ΠΌ Ρ€Π΅Π·ΡƒΠ»ΡŒΡ‚Π°Ρ‚ΠΎΠ² ΠΈΠ·ΠΌΠ΅Ρ€Π΅Π½ΠΈΠΉ ΠΈ построСнным Π³Ρ€Π°Ρ„ΠΈΠΊΠ°ΠΌ;
  • Ρ…Π°Ρ€Π°ΠΊΡ‚Π΅Ρ€ Ρ‚Π΅ΠΏΠ»ΠΎΠ²Ρ‹Ρ… процСссов: Π½Π°Π³Ρ€Π΅Π²Π°Π½ΠΈΠ΅, ΠΎΡ…Π»Π°ΠΆΠ΄Π΅Π½ΠΈΠ΅, ΠΏΠ»Π°Π²Π»Π΅Π½ΠΈΠ΅, ΠΊΠΈΠΏΠ΅Π½ΠΈΠ΅ (ΠΏΠΎ Π³Ρ€Π°Ρ„ΠΈΠΊΠ°ΠΌ измСнСния Ρ‚Π΅ΠΌΠΏΠ΅Ρ€Π°Ρ‚ΡƒΡ€Ρ‹ Ρ‚Π΅Π»Π° со Π²Ρ€Π΅ΠΌΠ΅Π½Π΅ΠΌ);
  • сопротивлСниС мСталличСского ΠΏΡ€ΠΎΠ²ΠΎΠ΄Π½ΠΈΠΊΠ° (ΠΏΠΎ Π³Ρ€Π°Ρ„ΠΈΠΊΡƒ ΠΊΠΎΠ»Π΅Π±Π°Π½ΠΈΠΉ);
  • ΠΏΠΎ Π³Ρ€Π°Ρ„ΠΈΠΊΡƒ зависимости ΠΊΠΎΠΎΡ€Π΄ΠΈΠ½Π°Ρ‚Ρ‹ ΠΎΡ‚ Π²Ρ€Π΅ΠΌΠ΅Π½ΠΈ: Π² ΠΊΠΎΠΎΡ€Π΄ΠΈΠ½Π°Ρ‚Ρƒ Ρ‚Π΅Π»Π° Π² Π·Π°Π΄Π°Π½Π½Ρ‹ΠΉ ΠΌΠΎΠΌΠ΅Π½Ρ‚ Π²Ρ€Π΅ΠΌΠ΅Π½ΠΈ; ΠΏΡ€ΠΎΠΌΠ΅ΠΆΡƒΡ‚ΠΊΠΈ Π²Ρ€Π΅ΠΌΠ΅Π½ΠΈ, Π² Ρ‚Π΅Ρ‡Π΅Π½ΠΈΠ΅ ΠΊΠΎΡ‚ΠΎΡ€Ρ‹Ρ… Ρ‚Π΅Π»ΠΎ двигалось с постоянной, ΡƒΠ²Π΅Π»ΠΈΡ‡ΠΈΠ²Π°ΡŽΡ‰Π΅ΠΉΡΡ, ΡƒΠΌΠ΅Π½ΡŒΡˆΠ°ΡŽΡ‰Π΅ΠΉΡΡ ΡΠΊΠΎΡ€ΠΎΡΡ‚ΡŒΡŽ; ΠΏΡ€ΠΎΠΌΠ΅ΠΆΡƒΡ‚ΠΊΠΈ Π²Ρ€Π΅ΠΌΠ΅Π½ΠΈ дСйствия силы.

3.8. Π‘Ρ€Π°Π²Π½ΠΈΠ²Π°Ρ‚ΡŒ сопротивлСния мСталличСских ΠΏΡ€ΠΎΠ²ΠΎΠ΄Π½ΠΈΠΊΠΎΠ² (большС - мСньшС) ΠΏΠΎ Π³Ρ€Π°Ρ„ΠΈΠΊΠ°ΠΌ зависимости силы Ρ‚ΠΎΠΊΠ° ΠΎΡ‚ напряТСния.

Π­Ρ„Ρ„Π΅ΠΊΡ‚ΠΈΠ²Π½ΠΎΡΡ‚ΡŒ использования ΡΠΊΡΠΏΠ΅Ρ€ΠΈΠΌΠ΅Π½Ρ‚Π°Π»ΡŒΠ½Ρ‹Ρ… Π·Π°Π΄Π°Ρ‡ Π½Π° ΡƒΡ€ΠΎΠΊΠ°Ρ… Π² Π·Π½Π°Ρ‡ΠΈΡ‚Π΅Π»ΡŒΠ½ΠΎΠΉ стСпСни опрСдСляСтся ΠΈΡ… Ρ‚Π΅Ρ…Π½ΠΎΠ»ΠΎΠ³ΠΈΡ‡Π½ΠΎΡΡ‚ΡŒΡŽ, Π½Π΅ΠΏΡ€ΠΈΡ‚ΡΠ·Π°Ρ‚Π΅Π»ΡŒΠ½ΠΎΡΡ‚ΡŒΡŽ Π² ΠΎΠ±ΠΎΡ€ΡƒΠ΄ΠΎΠ²Π°Π½ΠΈΠΈ, ΡˆΠΈΡ€ΠΎΡ‚ΠΎΠΉ рассматриваСмых явлСний. Π‘Π°Π·ΠΈΡ€ΡƒΡΡΡŒ Π½Π° самом ΠΏΡ€ΠΎΡΡ‚Π΅ΠΉΡˆΠ΅ΠΌ ΠΎΠ±ΠΎΡ€ΡƒΠ΄ΠΎΠ²Π°Π½ΠΈΠΈ ΠΈ Π΄Π°ΠΆΠ΅ Π½Π° ΠΏΡ€Π΅Π΄ΠΌΠ΅Ρ‚Π°Ρ… ΠΎΠ±ΠΈΡ…ΠΎΠ΄Π°, ΡΠΊΡΠΏΠ΅Ρ€ΠΈΠΌΠ΅Π½Ρ‚Π°Π»ΡŒΠ½Π°Ρ Π·Π°Π΄Π°Ρ‡Π° ΠΏΡ€ΠΈΠ±Π»ΠΈΠΆΠ°Π΅Ρ‚ Ρ„ΠΈΠ·ΠΈΠΊΡƒ ΠΊ Π½Π°ΠΌ, прСвращая Π΅Π΅ Π² прСдставлСниях учащихся ΠΈΠ· абстрактной систСмы Π·Π½Π°Π½ΠΈΠΉ Π² Π½Π°ΡƒΠΊΠ΅, ΠΈΠ·ΡƒΡ‡Π°ΡŽΡ‰ΡƒΡŽ Β«ΠΌΠΈΡ€ Π²ΠΎΠΊΡ€ΡƒΠ³ нас».

ΠœΠ΅Ρ…Π°Π½ΠΈΠΊΠ°

Π—Π°Π΄Π°Ρ‡Π° 1. ΠšΠΎΡΡ„Ρ„ΠΈΡ†ΠΈΠ΅Π½Ρ‚ трСния

Π—Π°Π΄Π°Π½ΠΈΠ΅. Π˜Π·ΠΌΠ΅Ρ€ΡŒΡ‚Π΅ коэффициСнт трСния скольТСния дСрСвянного бруска ΠΏΠΎ повСрхности доски (Π»ΠΈΠ½Π΅ΠΉΠΊΠΈ).

ΠžΠ±ΠΎΡ€ΡƒΠ΄ΠΎΠ²Π°Π½ΠΈΠ΅: брусок, доска, ΡˆΡ‚Π°Ρ‚ΠΈΠ² с Π»Π°ΠΏΠΊΠΎΠΉ, Π»ΠΈΠ½Π΅ΠΉΠΊΠ° Π΄Π»ΠΈΠ½ΠΎΠΉ 30(40) см .

Π’ΠΎΠ·ΠΌΠΎΠΆΠ½Ρ‹ΠΉ способ Ρ€Π΅ΡˆΠ΅Π½ΠΈΡ. КладСм брусок Π½Π° Π΄ΠΎΡ‰Π΅Ρ‡ΠΊΡƒ, Π² соотвСтствии с рисунком 4. ΠŸΠΎΡΡ‚Π΅ΠΏΠ΅Π½Π½ΠΎ поднимая ΠΎΠ΄ΠΈΠ½ ΠΊΠΎΠ½Π΅Ρ† доски, ΠΏΠΎΠ»ΡƒΡ‡Π°Π΅ΠΌ Π½Π°ΠΊΠ»ΠΎΠ½Π½ΡƒΡŽ ΠΏΠ»ΠΎΡΠΊΠΎΡΡ‚ΡŒ ΠΈ добиваСмся Ρ€Π°Π²Π½ΠΎΠΌΠ΅Ρ€Π½ΠΎΠ³ΠΎ скольТСния бруска. Π’Π°ΠΊ ΠΊΠ°ΠΊ сила трСния покоя Π½Π°ΠΌΠ½ΠΎΠ³ΠΎ большС силы трСния скольТСния, Π½Π΅ΠΎΠ±Ρ…ΠΎΠ΄ΠΈΠΌΠΎ Π½Π΅ΠΌΠ½ΠΎΠ³ΠΎ ΠΏΠΎΠ΄Ρ‚Π°Π»ΠΊΠΈΠ²Π°Ρ‚ΡŒ бусок Π² Π½Π°Ρ‡Π°Π»Π΅ скольТСния. Для фиксации Π½ΡƒΠΆΠ½ΠΎΠ³ΠΎ Π½Π°ΠΊΠ»ΠΎΠ½Π° ΠΈΡΠΏΠΎΠ»ΡŒΠ·ΡƒΠ΅ΠΌ ΡˆΡ‚Π°Ρ‚ΠΈΠ². Π˜Π·ΠΌΠ΅Ρ€ΡΠ΅ΠΌ высоту Π° ΠΈ Π΄Π»ΠΈΠ½Ρƒ основания Π½Π°ΠΊΠ»ΠΎΠ½Π½ΠΎΠΉ плоскости b .

Π˜Π·ΠΌΠ΅Ρ€Π΅Π½ΠΈΡ ΠΈ Π°Π½Π°Π»ΠΈΠ· ΠΏΠΎΠ³Ρ€Π΅ΡˆΠ½ΠΎΡΡ‚Π΅ΠΉ:

ΠžΠΏΡ‹Ρ‚ повторяСм нСсколько Ρ€Π°Π·. Π’ Π΄Π°Π½Π½ΠΎΠΌ случаС это Π½Π΅ΠΎΠ±Ρ…ΠΎΠ΄ΠΈΠΌΠΎ ΡΠ΄Π΅Π»Π°Ρ‚ΡŒ Π³Π»Π°Π²Π½Ρ‹ΠΌ ΠΎΠ±Ρ€Π°Π·ΠΎΠΌ ΠΏΠΎΡ‚ΠΎΠΌΡƒ, Ρ‡Ρ‚ΠΎ Ρ‚Ρ€ΡƒΠ΄Π½ΠΎ Π΄ΠΎΠ±ΠΈΡ‚ΡŒΡΡ ΠΈΠΌΠ΅Π½Π½ΠΎ Ρ€Π°Π²Π½ΠΎΠΌΠ΅Ρ€Π½ΠΎΠ³ΠΎ скольТСния бруска ΠΏΠΎ плоскости. Π Π΅Π·ΡƒΠ»ΡŒΡ‚Π°Ρ‚Ρ‹ заносим Π² Ρ‚Π°Π±Π»ΠΈΡ†Ρƒ 2.

Π’Π°Π±Π»ΠΈΡ†Π° 2

ΠŸΠΎΠ³Ρ€Π΅ΡˆΠ½ΠΎΡΡ‚ΠΈ ΠΈΠ·ΠΌΠ΅Ρ€Π΅Π½ΠΈΠΉ

а, см

Π”a, см

(Π”a) 2 ,см 2

в, см

Π”b, см

(Π”b) 2 ,см 2

<a >=12,2

Π£(a ) 2 = 1,81

Π£(b ) 2 = 0,32

ΠšΡ€ΠΎΠΌΠ΅ случайных ΠΏΠΎΠ³Ρ€Π΅ΡˆΠ½ΠΎΡΡ‚Π΅ΠΉ Π² ΠΎΠ±Ρ‰ΡƒΡŽ ΠΏΠΎΠ³Ρ€Π΅ΡˆΠ½ΠΎΡΡ‚ΡŒ, ΠΊΠΎΠ½Π΅Ρ‡Π½ΠΎ, входят ΠΈ ΠΎΠ±Ρ‹Ρ‡Π½Ρ‹Π΅ ΠΏΠΎΠ³Ρ€Π΅ΡˆΠ½ΠΎΡΡ‚ΠΈ отслСта: Π”Π° = Π”b = 0,5 см .Π­Ρ‚ΠΎ составляСт:

Π’Π°ΠΊΠΈΠΌ ΠΎΠ±Ρ€Π°Π·ΠΎΠΌ, ΠΏΠΎΠ»ΡƒΡ‡Π°Π΅ΠΌ:

a = 12,2 ± 1,1 см, д = 8,6%

b = 27,4 ± 0,7 см, д = 2,6%

По Ρ€Π΅Π·ΡƒΠ»ΡŒΡ‚Π°Ρ‚Π°ΠΌ ΠΏΠ΅Ρ€Π²ΠΎΠ³ΠΎ ΠΎΠΏΡ‹Ρ‚Π°:

ΠžΠΊΠΎΠ½Ρ‡Π°Ρ‚Π΅Π»ΡŒΠ½Ρ‹ΠΉ Ρ€Π΅Π·ΡƒΠ»ΡŒΡ‚Π°Ρ‚ измСрСния коэффициСнта трСния:

ΠΌ = 0,46 Β± 0,05 Π΄ = 10,9%

Π—Π°Π΄Π°Ρ‡Π° 2. Π˜Π·ΠΌΠ΅Ρ€Π΅Π½ΠΈΠ΅ высоты Π΄ΠΎΠΌΠ°

Π—Π°Π΄Π°Π½ΠΈΠ΅. ΠŸΡ€Π΅Π΄ΡΡ‚Π°Π²ΡŒΡ‚Π΅, Ρ‡Ρ‚ΠΎ для измСрСния высоты Π΄ΠΎΠΌΠ° Π²Π°ΠΌ Π±Ρ‹Π»ΠΎ ΠΏΡ€Π΅Π΄Π»ΠΎΠΆΠ΅Π½ΠΎ Π²ΠΎΡΠΏΠΎΠ»ΡŒΠ·ΠΎΠ²Π°Ρ‚ΡŒΡΡ пустой консСрвной Π±Π°Π½ΠΊΠΎΠΉ ΠΈ сСкундомСром. Π‘ΡƒΠΌΠ΅Π»ΠΈ Π±Ρ‹ Π²Ρ‹ ΡΠΏΡ€Π°Π²ΠΈΡ‚ΡŒΡΡ с Π·Π°Π΄Π°Π½ΠΈΠ΅ΠΌ? РасскаТитС, ΠΊΠ°ΠΊ Π½ΡƒΠΆΠ½ΠΎ Π΄Π΅ΠΉΡΡ‚Π²ΠΎΠ²Π°Ρ‚ΡŒ.

Подсказка. Если Π±Π°Π½ΠΊΡƒ ΡΠ±Ρ€ΠΎΡΠΈΡ‚ΡŒ с ΠΊΡ€Ρ‹ΡˆΠΈ Π΄ΠΎΠΌΠ°, Ρ‚ΠΎ Π·Π²ΡƒΠΊ ΡƒΠ΄Π°Ρ€Π° Π±Π°Π½ΠΊΠΈ ΠΎ Π·Π΅ΠΌΠ½ΡƒΡŽ ΠΏΠΎΠ²Π΅Ρ€Ρ…Π½ΠΎΡΡ‚ΡŒ Π±ΡƒΠ΄Π΅Ρ‚ ΠΎΡ‚Ρ‡Π΅Ρ‚Π»ΠΈΠ²ΠΎ ΡΠ»Ρ‹ΡˆΠ΅Π½.

РСшСниС. Встав Π½Π° ΠΊΡ€Ρ‹ΡˆΡƒ Π΄ΠΎΠΌΠ°, Π½ΡƒΠΆΠ½ΠΎ Π²Ρ‹ΠΏΡƒΡΡ‚ΠΈΡ‚ΡŒ Π±Π°Π½ΠΊΡƒ ΠΈΠ· Ρ€ΡƒΠΊ, ΠΎΠ΄Π½ΠΎΠ²Ρ€Π΅ΠΌΠ΅Π½Π½ΠΎ Π½Π°ΠΆΠ°Π² Π½Π° ΠΏΡƒΡΠΊΠΎΠ²ΡƒΡŽ ΠΊΠ½ΠΎΠΏΠΊΡƒ сСкундомСра. Π£ΡΠ»Ρ‹ΡˆΠ°Π² Π·Π²ΡƒΠΊ ΡƒΠ΄Π°Ρ€Π° Π±Π°Π½ΠΊΠΈ ΠΎ зСмлю, слСдуСт ΠΎΡΡ‚Π°Π½ΠΎΠ²ΠΈΡ‚ΡŒ сСкундомСр. Показания сСкундомСра t ΡΠΊΠ»Π°Π΄Ρ‹Π²Π°ΡŽΡ‚ΡΡ ΠΈΠ· Π²Ρ€Π΅ΠΌΠ΅Π½ΠΈ падСния Π±Π°Π½ΠΊΠΈ t 1 ΠΈ Π²Ρ€Π΅ΠΌΠ΅Π½ΠΈ t 2 , Π·Π° ΠΊΠΎΡ‚ΠΎΡ€ΠΎΠ΅ Π·Π²ΡƒΠΊ ΡƒΠ΄Π°Ρ€Π° Π΅Π΅ ΠΎ Π·Π΅ΠΌΠ½ΡƒΡŽ ΠΏΠΎΠ²Π΅Ρ€Ρ…Π½ΠΎΡΡ‚ΡŒ Π΄ΠΎΠΉΠ΄Π΅Ρ‚ Π΄ΠΎ Π½Π°Π±Π»ΡŽΠ΄Π°Ρ‚Π΅Π»Ρ.

ΠŸΠ΅Ρ€Π²ΠΎΠ΅ врСмя связано с высотой Π΄ΠΎΠΌΠ° h ΡΠ»Π΅Π΄ΡƒΡŽΡ‰ΠΈΠΌ ΠΎΠ±Ρ€Π°Π·ΠΎΠΌ:

Ρ‚ΠΎΠ³Π΄Π° ΠΊΠ°ΠΊ связь ΠΌΠ΅ΠΆΠ΄Ρƒ h ΠΈ t 2 ΠΈΠΌΠ΅Π΅Ρ‚ Π²ΠΈΠ΄

Π³Π΄Π΅ с - ΡΠΊΠΎΡ€ΠΎΡΡ‚ΡŒ Π·Π²ΡƒΠΊΠ°, ΠΊΠΎΡ‚ΠΎΡ€ΡƒΡŽ ΠΏΡ€ΠΈ расчСтах ΠΌΡ‹ ΠΏΠΎΠ»ΠΎΠΆΠΈΠΌ Ρ€Π°Π²Π½ΠΎΠΉ 340 ΠΌ/сСк .

ΠžΠΏΡ€Π΅Π΄Π΅Π»ΡΡ t 1 ΠΈ t 2 ΠΈΠ· этих Π²Ρ‹Ρ€Π°ΠΆΠ΅Π½ΠΈΠΉ ΠΈ подставляя ΠΈΡ… значСния Π² Ρ„ΠΎΡ€ΠΌΡƒΠ»Ρƒ, ΡΠ²ΡΠ·Ρ‹Π²Π°ΡŽΡ‰ΡƒΡŽ t 1 , t 2 ΠΈ t , ΠΏΠΎΠ»ΡƒΡ‡ΠΈΠΌ ΠΈΡ€Ρ€Π°Ρ†ΠΈΠΎΠ½Π°Π»ΡŒΠ½ΠΎΠ΅ ΡƒΡ€Π°Π²Π½Π΅Π½ΠΈΠ΅

Из ΠΊΠΎΡ‚ΠΎΡ€ΠΎΠ³ΠΎ ΠΌΠΎΠΆΠ½ΠΎ Π½Π°ΠΉΡ‚ΠΈ высоту Π΄ΠΎΠΌΠ°.

ΠŸΡ€ΠΈ ΠΏΡ€ΠΈΠ±Π»ΠΈΠΆΠ΅Π½Π½ΠΎΠΌ вычислСнии (Π² особСнности, Ссли Π΄ΠΎΠΌ нСвысок) Π²Ρ‚ΠΎΡ€ΠΎΠ΅ слагаСмоС слСва ΠΌΠΎΠΆΠ½ΠΎ ΡΡ‡ΠΈΡ‚Π°Ρ‚ΡŒ ΠΌΠ°Π»Ρ‹ΠΌ ΠΈ ΠΎΡ‚Π±Ρ€ΠΎΡΠΈΡ‚ΡŒ. Π’ΠΎΠ³Π΄Π°

ΠœΠΎΠ»Π΅ΠΊΡƒΠ»ΡΡ€Π½Π°Ρ Ρ„ΠΈΠ·ΠΈΠΊΠ°

Π—Π°Π΄Π°Ρ‡Π° 3. ΠšΠ°Ρ€Π°Π½Π΄Π°Ρˆ

Π—Π°Π΄Π°Π½ΠΈΠ΅. ΠžΡ†Π΅Π½ΠΈΡ‚Π΅ ΠΌΠ΅Ρ…Π°Π½ΠΈΡ‡Π΅ΡΠΊΡƒΡŽ Ρ€Π°Π±ΠΎΡ‚Ρƒ, ΠΊΠΎΡ‚ΠΎΡ€ΡƒΡŽ Π½Π΅ΠΎΠ±Ρ…ΠΎΠ΄ΠΈΠΌΠΎ ΡΠΎΠ²Π΅Ρ€ΡˆΠΈΡ‚ΡŒ для Ρ‚ΠΎΠ³ΠΎ, Ρ‡Ρ‚ΠΎΠ±Ρ‹ Ρ€Π°Π²Π½ΠΎΠΌΠ΅Ρ€Π½ΠΎ ΠΏΠΎΠ΄Π½ΡΡ‚ΡŒ ΠΏΠ»Π°Π²Π°ΡŽΡ‰ΠΈΠΉ Π² сосудС ΠΊΠ°Ρ€Π°Π½Π΄Π°Ρˆ Π΄ΠΎ уровня касания Π½ΠΈΠΆΠ½ΠΈΠΌ Π΅Π³ΠΎ Ρ‚ΠΎΡ€Ρ†ΠΎΠΌ повСрхности Π²ΠΎΠ΄Ρ‹. Π‘Ρ‡ΠΈΡ‚Π°ΠΉΡ‚Π΅ ΠΏΠΎΠ»ΠΎΠΆΠ΅Π½ΠΈΠ΅ ΠΊΠ°Ρ€Π°Π½Π΄Π°ΡˆΠ° Π²Π΅Ρ€Ρ‚ΠΈΠΊΠ°Π»ΡŒΠ½Ρ‹ΠΌ. ΠŸΠ»ΠΎΡ‚Π½ΠΎΡΡ‚ΡŒ Π²ΠΎΠ΄Ρ‹ с 0 = 1000 ΠΊΠ³/ΠΌ 3 .

ΠžΠ±ΠΎΡ€ΡƒΠ΄ΠΎΠ²Π°Π½ΠΈΠ΅: ΠΊΡ€ΡƒΠ³Π»Ρ‹ΠΉ ΠΊΠ°Ρ€Π°Π½Π΄Π°Ρˆ, ΠΏΠΎΡ‡Ρ‚ΠΈ полная Π±ΡƒΡ‚Ρ‹Π»ΠΊΠ° с Π²ΠΎΠ΄ΠΎΠΉ, Π»ΠΈΠ½Π΅ΠΉΠΊΠ°.

Π’ΠΎΠ·ΠΌΠΎΠΆΠ½Ρ‹ΠΉ способ Ρ€Π΅ΡˆΠ΅Π½ΠΈΡ. ΠžΠΏΡƒΡΠΊΠ°Π΅ΠΌ ΠΊΠ°Ρ€Π°Π½Π΄Π°Ρˆ Π² Π±ΡƒΡ‚Ρ‹Π»ΠΊΡƒ - ΠΎΠ½ Π±ΡƒΠ΄Π΅Ρ‚ ΠΏΠ»Π°Π²Π°Ρ‚ΡŒ, ΠΊΠ°ΠΊ ΠΏΠΎΠΏΠ»Π°Π²ΠΎΠΊ, Π² соотвСтствии с рисунком 5. ΠŸΡƒΡΡ‚ΡŒ L - Π΄Π»ΠΈΠ½Π° всСго ΠΊΠ°Ρ€Π°Π½Π΄Π°ΡˆΠ°, V - Π΅Π³ΠΎ объСм, h - Π΄Π»ΠΈΠ½Π° ΠΏΠΎΠ³Ρ€ΡƒΠΆΠ΅Π½Π½ΠΎΠΉ Π² Π²ΠΎΠ΄Ρƒ части ΠΊΠ°Ρ€Π°Π½Π΄Π°ΡˆΠ°, V 1 - Π΅Π΅ объСм, S - ΠΏΠ»ΠΎΡ‰Π°Π΄ΡŒ сСчСния ΠΈ d - Π΄ΠΈΠ°ΠΌΠ΅Ρ‚Ρ€ ΠΊΠ°Ρ€Π°Π½Π΄Π°ΡˆΠ°. НайдСм ΡΡ€Π΅Π΄Π½ΡŽΡŽ ΠΏΠ»ΠΎΡ‚Π½ΠΎΡΡ‚ΡŒ ΠΊΠ°Ρ€Π°Π½Π΄Π°ΡˆΠ° с ΠΈΠ· условия плавания Ρ‚Π΅Π»Π°:

с 0 gSh = сgSL , ΠΎΡ‚ΠΊΡƒΠ΄Π° с = с 0 hL .

ΠŸΡ€Π΅Π΄ΠΏΠΎΠ»ΠΎΠΆΠΈΠΌ, Ρ‡Ρ‚ΠΎ ΠΌΡ‹ с постоянной ΡΠΊΠΎΡ€ΠΎΡΡ‚ΡŒΡŽ вытаскиваСм ΠΊΠ°Ρ€Π°Π½Π΄Π°Ρˆ ΠΈΠ· Π²ΠΎΠ΄Ρ‹, ΠΈΡΠΏΠΎΠ»ΡŒΠ·ΡƒΡ Π΄ΠΈΠ½Π°ΠΌΠΎΠΌΠ΅Ρ‚Ρ€. Когда ΠΊΠ°Ρ€Π°Π½Π΄Π°Ρˆ свободно ΠΏΠ»Π°Π²Π°Π΅Ρ‚, Π΄ΠΈΠ½Π°ΠΌΠΎΠΌΠ΅Ρ‚Ρ€ ΠΏΠΎΠΊΠ°Π·Ρ‹Π²Π°Π΅Ρ‚ ноль. Если ΠΆΠ΅ ΠΊΠ°Ρ€Π°Π½Π΄Π°Ρˆ ΠΏΠΎΠ»Π½ΠΎΡΡ‚ΡŒΡŽ Π²Ρ‹Ρ‚Π°Ρ‰ΠΈΡ‚ΡŒ ΠΈΠ· Π²ΠΎΠ΄Ρ‹, Ρ‚ΠΎ Π΄ΠΈΠ½Π°ΠΌΠΎΠΌΠ΅Ρ‚Ρ€ ΠΏΠΎΠΊΠ°ΠΆΠ΅Ρ‚ силу, Ρ€Π°Π²Π½ΡƒΡŽ вСсу Π  ΠΊΠ°Ρ€Π°Π½Π΄Π°ΡˆΠ°:

F = P = mg = сgV = с0hLgSL = с0hgΡ€d24

ΠŸΠΎΠ»ΡƒΡ‡Π°Π΅Ρ‚ΡΡ, Ρ‡Ρ‚ΠΎ показания Π΄ΠΈΠ½Π°ΠΌΠΎΠΌΠ΅Ρ‚Ρ€Π° ΠΏΡ€ΠΈ вытаскивании ΠΊΠ°Ρ€Π°Π½Π΄Π°ΡˆΠ° ΠΈΠ· Π²ΠΎΠ΄Ρ‹ ΠΈΠ·ΠΌΠ΅Π½ΡΡŽΡ‚ΡΡ ΠΎΡ‚ 0 Π΄ΠΎ P ΠΏΠΎ Π»ΠΈΠ½Π΅ΠΉΠ½ΠΎΠΌΡƒ Π·Π°ΠΊΠΎΠ½Ρƒ, Π² соотвСтствии с рисунком 6. ΠŸΡ€ΠΈ этом мСханичСская Ρ€Π°Π±ΠΎΡ‚Π° А Π±ΡƒΠ΄Π΅Ρ‚ Ρ€Π°Π²Π½Π° ΠΏΠ»ΠΎΡ‰Π°Π΄ΠΈ Π²Ρ‹Π΄Π΅Π»Π΅Π½Π½ΠΎΠ³ΠΎ Ρ‚Ρ€Π΅ΡƒΠ³ΠΎΠ»ΡŒΠ½ΠΈΠΊΠ°:

A = 12Ph = с 0 h 2gΡ€d 2 8.

НапримСр, ΠΏΡ€ΠΈ h = 13,4 см ΠΈ d = 7,5 ΠΌΠΌ Ρ€Π°Π±ΠΎΡ‚Π° составляСт ΠΎΠΊΠΎΠ»ΠΎ 0,004 Π”ΠΆ .

Π—Π°Π΄Π°Ρ‡Π° 4. Π‘ΠΏΠ»Π°Π²

Π—Π°Π΄Π°Π½ΠΈΠ΅. ΠžΠΏΡ€Π΅Π΄Π΅Π»ΠΈΡ‚Π΅ ΠΏΡ€ΠΎΡ†Π΅Π½Ρ‚Π½ΠΎΠ΅ содСрТаниС (ΠΏΠΎ массС) ΠΎΠ»ΠΎΠ²Π° Π² оловянно-свинцовом ΠΏΡ€ΠΈΠΏΠΎΠ΅. ΠŸΡ€Π΅Π΄ΠΏΠΎΠ»ΠΎΠΆΠΈΡ‚Π΅, Ρ‡Ρ‚ΠΎ ΠΎΠ±ΡŠΠ΅ΠΌΡ‹ свинца ΠΈ ΠΎΠ»ΠΎΠ²Π° Π² сплавС ΡΠΎΡ…Ρ€Π°Π½ΡΡŽΡ‚ΡΡ. ΠŸΠ»ΠΎΡ‚Π½ΠΎΡΡ‚ΡŒ свинца с c = 11350 ΠΊΠ³/ΠΌ 3 , ΠΎΠ»ΠΎΠ²Π° с 0 = 7300 ΠΊΠ³/ΠΌ 3 .

ΠžΠ±ΠΎΡ€ΡƒΠ΄ΠΎΠ²Π°Π½ΠΈΠ΅: Π»ΠΈΠ½Π΅ΠΉΠΊΠ°, Π³Ρ€ΡƒΠ· (Π³Π°ΠΉΠΊΠ°), цилиндричСский кусок припоя, ΡˆΡ‚Π°Π½Π³Π΅Π½Ρ†ΠΈΡ€ΠΊΡƒΠ»ΡŒ ΠΈΠ»ΠΈ ΠΌΠΈΠΊΡ€ΠΎΠΌΠ΅Ρ‚Ρ€. Π’ΠΎΠ·ΠΌΠΎΠΆΠ½Ρ‹ΠΉ способ Ρ€Π΅ΡˆΠ΅Π½ΠΈΡ. Π­Ρ‚Π° Π·Π°Π΄Π°Ρ‡Π° Π°Π½Π°Π»ΠΎΠ³ΠΈΡ‡Π½Π° Π·Π°Π΄Π°Ρ‡Π΅ АрхимСда ΠΏΠΎ ΠΎΠΏΡ€Π΅Π΄Π΅Π»Π΅Π½ΠΈΡŽ Π΄ΠΎΠ»ΠΈ Π·ΠΎΠ»ΠΎΡ‚Π° Π² царской ΠΊΠΎΡ€ΠΎΠ½Π΅. Однако для ΠΎΠΏΡ‹Ρ‚ΠΎΠ² оловянно-свинцовый ΠΏΡ€ΠΈΠΏΠΎΠΉ Π΄ΠΎΡΡ‚Π°Ρ‚ΡŒ ΠΏΡ€ΠΎΡ‰Π΅, Ρ‡Π΅ΠΌ ΠΊΠΎΡ€ΠΎΠ½Ρƒ.

Π˜Π·ΠΌΠ΅Ρ€ΠΈΠ² Π΄ΠΈΠ°ΠΌΠ΅Ρ‚Ρ€ куска припоя D ΠΈ Π΅Π³ΠΎ Π΄Π»ΠΈΠ½Ρƒ L , Π½Π°ΠΉΠ΄Π΅ΠΌ объСм цилиндричСского куска припоя:

V = Ρ€D 2 L 4

ΠœΠ°ΡΡΡƒ припоя ΠΎΠΏΡ€Π΅Π΄Π΅Π»ΠΈΠΌ, ΠΈΠ·Π³ΠΎΡ‚ΠΎΠ²ΠΈΠ² Ρ€Ρ‹Ρ‡Π°ΠΆΠ½Ρ‹Π΅ вСсы. Для этого уравновСсим Π»ΠΈΠ½Π΅ΠΉΠΊΡƒ Π½Π° ΠΊΡ€Π°ΡŽ стола (Π½Π° ΠΊΠ°Ρ€Π°Π½Π΄Π°ΡˆΠ΅, Π½Π° стСрТнС ΠΎΡ‚ ΡˆΠ°Ρ€ΠΈΠΊΠΎΠ²ΠΎΠΉ Ρ€ΡƒΡ‡ΠΊΠΈ ΠΈ Ρ‚.ΠΏ.). Π—Π°Ρ‚Π΅ΠΌ, ΠΈΡΠΏΠΎΠ»ΡŒΠ·ΡƒΡ Π³Π°ΠΉΠΊΡƒ извСстной массы, уравновСсим кусок припоя Π½Π° Π»ΠΈΠ½Π΅ΠΉΠΊΠ΅ ΠΈ с ΠΏΠΎΠΌΠΎΡ‰ΡŒΡŽ равСнства ΠΌΠΎΠΌΠ΅Π½Ρ‚ΠΎΠ² сил Π½Π°ΠΉΠ΄Π΅ΠΌ массу припоя m . Π—Π°ΠΏΠΈΡˆΠ΅ΠΌ ΠΎΡ‡Π΅Π²ΠΈΠ΄Π½Ρ‹Π΅ равСнства для масс, объСмов ΠΈ плотностСй свинца ΠΈ ΠΎΠ»ΠΎΠ²Π°:

m = m c +m o = сcV c +с o V o , V = V c +V o .

РСшая эти уравнСния совмСстно, Π½Π°ΠΉΠ΄Π΅ΠΌ объСм ΠΎΠ»ΠΎΠ²Π°, Π΅Π³ΠΎ массу ΠΈ долю Π² ΠΎΠ±Ρ‰Π΅ΠΉ массС:

V o = rh o cV?mrh o c?rh oo , mo = с o V o , m o m = rh oo V o m

Π—Π°Π΄Π°Ρ‡Π° 5. ΠŸΠΎΠ²Π΅Ρ€Ρ…Π½ΠΎΡΡ‚Π½ΠΎΠ΅ натяТСниС

Π—Π°Π΄Π°Π½ΠΈΠ΅. ΠžΠΏΡ€Π΅Π΄Π΅Π»ΠΈΡ‚Π΅ коэффициСнт повСрхностного натяТСния Π²ΠΎΠ΄Ρ‹.

ΠžΠ±ΠΎΡ€ΡƒΠ΄ΠΎΠ²Π°Π½ΠΈΠ΅: Ρ‚Π°Ρ€Π΅Π»ΠΊΠ°, Π²ΠΎΠ΄Π°, Π»ΠΎΠΆΠΊΠ°, Π»ΠΈΠ½Π΅ΠΉΠΊΠ°, кусок Ρ€ΠΎΠ²Π½ΠΎΠΉ алюминиСвой ΠΏΡ€ΠΎΠ²ΠΎΠ»ΠΎΠΊΠΈ Π΄Π»ΠΈΠ½ΠΎΠΉ 15-20 см ΠΈ ΠΏΠ»ΠΎΡ‚Π½ΠΎΡΡ‚ΡŒΡŽ 2700 ΠΊΠ³/ΠΌ 3 , ΠΌΠΈΠΊΡ€ΠΎΠΌΠ΅Ρ‚Ρ€, спирт, Π²Π°Ρ‚Π°.

Π’ΠΎΠ·ΠΌΠΎΠΆΠ½Ρ‹ΠΉ способ Ρ€Π΅ΡˆΠ΅Π½ΠΈΡ. НальСм ΠΏΠΎΡ‡Ρ‚ΠΈ ΠΏΠΎΠ»Π½ΡƒΡŽ Ρ‚Π°Ρ€Π΅Π»ΠΊΡƒ Π²ΠΎΠ΄Ρ‹. ПолоТим Π½Π° ΠΊΡ€Π°ΠΉ Ρ‚Π°Ρ€Π΅Π»ΠΊΠΈ ΠΏΡ€ΠΎΠ²ΠΎΠ»ΠΎΠΊΡƒ Ρ‚Π°ΠΊ, Ρ‡Ρ‚ΠΎΠ±Ρ‹ ΠΎΠ΄ΠΈΠ½ ΠΊΠΎΠ½Π΅Ρ† Π΅Π΅ касался Π²ΠΎΠ΄Ρ‹, Π° Π΄Ρ€ΡƒΠ³ΠΎΠΉ Π±Ρ‹Π» Π·Π° ΠΏΡ€Π΅Π΄Π΅Π»Π°ΠΌΠΈ Ρ‚Π°Ρ€Π΅Π»ΠΊΠΈ. ΠŸΡ€ΠΎΠ²ΠΎΠ»ΠΎΠΊΠ° выполняСт Π΄Π²Π΅ Ρ„ΡƒΠ½ΠΊΡ†ΠΈΠΈ: ΠΎΠ½Π° являСтся Ρ€Ρ‹Ρ‡Π°ΠΆΠ½Ρ‹ΠΌΠΈ вСсами ΠΈ Π°Π½Π°Π»ΠΎΠ³ΠΎΠΌ ΠΏΡ€ΠΎΠ²ΠΎΠ»ΠΎΡ‡Π½ΠΎΠΉ Ρ€Π°ΠΌΠΊΠΈ, ΠΊΠΎΡ‚ΠΎΡ€ΡƒΡŽ ΠΎΠ±Ρ‹Ρ‡Π½ΠΎ Π²Ρ‹Ρ‚Π°ΡΠΊΠΈΠ²Π°ΡŽΡ‚ ΠΈΠ· Π²ΠΎΠ΄Ρ‹ для измСрСния повСрхностного натяТСния. Π’ зависимости ΠΎΡ‚ уровня Π²ΠΎΠ΄Ρ‹ ΠΌΠΎΠ³ΡƒΡ‚ Π½Π°Π±Π»ΡŽΠ΄Π°Ρ‚ΡŒΡΡ Ρ€Π°Π·Π»ΠΈΡ‡Π½Ρ‹Π΅ полоТСния ΠΏΡ€ΠΎΠ²ΠΎΠ»ΠΎΠΊΠΈ. НаиболСС ΡƒΠ΄ΠΎΠ±Π½ΠΎ для расчСтов ΠΈ ΠΈΠ·ΠΌΠ΅Ρ€Π΅Π½ΠΈΠΉ Π³ΠΎΡ€ΠΈΠ·ΠΎΠ½Ρ‚Π°Π»ΡŒΠ½ΠΎΠ΅ располоТСниС ΠΏΡ€ΠΎΠ²ΠΎΠ»ΠΎΠΊΠΈ ΠΏΡ€ΠΈ ΡƒΡ€ΠΎΠ²Π½Π΅ Π²ΠΎΠ΄Ρ‹ Π½Π° 1-1,5 ΠΌΠΌ Π½ΠΈΠΆΠ΅ края Ρ‚Π°Ρ€Π΅Π»ΠΊΠΈ, Π² соотвСтствии с рисунком 7. Π‘ ΠΏΠΎΠΌΠΎΡ‰ΡŒΡŽ Π»ΠΎΠΆΠΊΠΈ ΠΌΠΎΠΆΠ½ΠΎ Ρ€Π΅Π³ΡƒΠ»ΠΈΡ€ΠΎΠ²Π°Ρ‚ΡŒ ΡƒΡ€ΠΎΠ²Π΅Π½ΡŒ, доливая ΠΈΠ»ΠΈ отливая Π²ΠΎΠ΄Ρƒ. ΠŸΡ€ΠΎΠ²ΠΎΠ»ΠΎΠΊΡƒ слСдуСт Π²Ρ‹Π΄Π²ΠΈΠ³Π°Ρ‚ΡŒ ΠΈΠ· Ρ‚Π°Ρ€Π΅Π»ΠΊΠΈ Π΄ΠΎ Ρ‚Π΅Ρ… ΠΏΠΎΡ€, ΠΏΠΎΠΊΠ° ΠΏΠ»Π΅Π½ΠΊΠ° Π²ΠΎΠ΄Ρ‹ ΠΏΠΎΠ΄ ΠΏΡ€ΠΎΠ²ΠΎΠ»ΠΎΠΊΠΎΠΉ Π½Π΅ Π½Π°Ρ‡Π½Π΅Ρ‚ Ρ€Π°Π·Ρ€Ρ‹Π²Π°Ρ‚ΡŒΡΡ. Π’ этом ΠΊΡ€Π°ΠΉΠ½Π΅ΠΌ ΠΏΠΎΠ»ΠΎΠΆΠ΅Π½ΠΈΠΈ ΠΏΠ»Π΅Π½ΠΊΠ° ΠΈΠΌΠ΅Π΅Ρ‚ высоту 1,5-2 ΠΌΠΌ , ΠΈ ΠΌΠΎΠΆΠ½ΠΎ ΡΠΊΠ°Π·Π°Ρ‚ΡŒ, Ρ‡Ρ‚ΠΎ силы повСрхностного натяТСния, ΠΏΡ€ΠΈΠ»ΠΎΠΆΠ΅Π½Π½Ρ‹Π΅ ΠΊ ΠΏΡ€ΠΎΠ²ΠΎΠ»ΠΎΠΊΠ΅, Π½Π°ΠΏΡ€Π°Π²Π»Π΅Π½Ρ‹ практичСски Π²Π΅Ρ€Ρ‚ΠΈΠΊΠ°Π»ΡŒΠ½ΠΎ Π²Π½ΠΈΠ·.

ΠŸΡƒΡΡ‚ΡŒ m - масса ΠΏΡ€ΠΎΠ²ΠΎΠ»ΠΎΠΊΠΈ, L = L 1 + L 2 - Π΄Π»ΠΈΠ½Π° ΠΏΡ€ΠΎΠ²ΠΎΠ»ΠΎΠΊΠΈ, m/L - масса Π΅Π΄ΠΈΠ½ΠΈΡ†Ρ‹ Π΄Π»ΠΈΠ½Ρ‹ ΠΏΡ€ΠΎΠ²ΠΎΠ»ΠΎΠΊΠΈ. Π—Π°ΠΏΠΈΡˆΠ΅ΠΌ условиС равновСсия ΠΏΡ€ΠΎΠ²ΠΎΠ»ΠΎΠΊΠΈ ΠΎΡ‚Π½ΠΎΡΠΈΡ‚Π΅Π»ΡŒΠ½ΠΎ края Ρ‚Π°Ρ€Π΅Π»ΠΊΠΈ, Ρ‚.Π΅. равСнство ΠΌΠΎΠΌΠ΅Π½Ρ‚ΠΎΠ² сил:

F p (L 1 ?x 2)+m 1 gL 12 = m 2 gL 22 .

ΠŸΠΎΠ΄ΡΡ‚Π°Π²ΠΈΠΌ сюда силу повСрхностного натяТСния F p =2x Ρƒ , массы

m 1 =L 1 mL , m 2 = L 2 mL , m = сV = срd 2 L 4

ΠΈ Π²Ρ‹Ρ€Π°Π·ΠΈΠΌ коэффициСнт повСрхностного натяТСния Ρƒ . Π˜Π·ΠΌΠ΅Ρ€Π΅Π½ΠΈΡ ΠΈ вычислСния упростятся, Ссли Π²ΠΎΠ΄Π° Π±ΡƒΠ΄Π΅Ρ‚ ΡΠΌΠ°Ρ‡ΠΈΠ²Π°Ρ‚ΡŒ всю Π΄Π»ΠΈΠ½Ρƒ L 1 . ΠžΠΊΠΎΠ½Ρ‡Π°Ρ‚Π΅Π»ΡŒΠ½ΠΎ ΠΏΠΎΠ»ΡƒΡ‡ΠΈΠΌ

Ρƒ = срd 2 g 8((LL 1 ?1) 2 ?1).

Π’Π΅Π»ΠΈΡ‡ΠΈΠ½Ρ‹ L ΠΈ L 1 ΠΈΠ·ΠΌΠ΅Ρ€ΡΡŽΡ‚ΡΡ Π»ΠΈΠ½Π΅ΠΉΠΊΠΎΠΉ, Π° Π΄ΠΈΠ°ΠΌΠ΅Ρ‚Ρ€ ΠΏΡ€ΠΎΠ²ΠΎΠ»ΠΎΠΊΠΈ d - ΠΌΠΈΠΊΡ€ΠΎΠΌΠ΅Ρ‚Ρ€ΠΎΠΌ.

НапримСр, ΠΏΡ€ΠΈ L = 15 см , L 1 = 5,4 см , d = 1,77 ΠΌΠΌ ΠΏΠΎΠ»ΡƒΡ‡Π°Π΅ΠΌ O = 0,0703 Н/ΠΌ , Ρ‡Ρ‚ΠΎ Π±Π»ΠΈΠ·ΠΊΠΎ ΠΊ Ρ‚Π°Π±Π»ΠΈΡ‡Π½ΠΎΠΌΡƒ Π·Π½Π°Ρ‡Π΅Π½ΠΈΡŽ 0,0728 Н/ΠΌ .

Π—Π°Π΄Π°Ρ‡Π° 6. Π’Π»Π°ΠΆΠ½ΠΎΡΡ‚ΡŒ Π²ΠΎΠ·Π΄ΡƒΡ…Π°

Π—Π°Π΄Π°Π½ΠΈΠ΅. ΠžΠΏΡ€Π΅Π΄Π΅Π»ΠΈΡ‚Π΅ ΠΎΡ‚Π½ΠΎΡΠΈΡ‚Π΅Π»ΡŒΠ½ΡƒΡŽ Π²Π»Π°ΠΆΠ½ΠΎΡΡ‚ΡŒ Π²ΠΎΠ·Π΄ΡƒΡ…Π° Π² ΠΊΠΎΠΌΠ½Π°Ρ‚Π΅.

ΠžΠ±ΠΎΡ€ΡƒΠ΄ΠΎΠ²Π°Π½ΠΈΠ΅: стСклянный ΠΊΠΎΠΌΠ½Π°Ρ‚Π½Ρ‹ΠΉ Ρ‚Π΅Ρ€ΠΌΠΎΠΌΠ΅Ρ‚Ρ€, Π±Ρ‹Ρ‚ΠΎΠ²ΠΎΠΉ Ρ…ΠΎΠ»ΠΎΠ΄ΠΈΠ»ΡŒΠ½ΠΈΠΊ, Ρ‚Π°Π±Π»ΠΈΡ†Π° Π΄Π°Π²Π»Π΅Π½ΠΈΠΉ насыщСнных ΠΏΠ°Ρ€ΠΎΠ² Π²ΠΎΠ΄Ρ‹ ΠΏΡ€ΠΈ Ρ€Π°Π·Π»ΠΈΡ‡Π½Ρ‹Ρ… Ρ‚Π΅ΠΌΠΏΠ΅Ρ€Π°Ρ‚ΡƒΡ€Π°Ρ….

Π’ΠΎΠ·ΠΌΠΎΠΆΠ½Ρ‹ΠΉ способ Ρ€Π΅ΡˆΠ΅Π½ΠΈΡ. ΠŸΡ€ΠΈ ΠΎΠ±Ρ‹Ρ‡Π½ΠΎΠΌ ΠΌΠ΅Ρ‚ΠΎΠ΄Π΅ измСрСния влаТности ΠΎΠ±ΡŠΠ΅ΠΊΡ‚ ΠΎΡ…Π»Π°ΠΆΠ΄Π°ΡŽΡ‚ Π½ΠΈΠΆΠ΅ Ρ‚ΠΎΡ‡ΠΊΠΈ росы ΠΈ ΠΎΠ½ Β«Π·Π°ΠΏΠΎΡ‚Π΅Π²Π°Π΅Ρ‚Β». Π‘Π΄Π΅Π»Π°Π΅ΠΌ Π½Π°ΠΎΠ±ΠΎΡ€ΠΎΡ‚. Π’Π΅ΠΌΠΏΠ΅Ρ€Π°Ρ‚ΡƒΡ€Π° Π² Ρ…ΠΎΠ»ΠΎΠ΄ΠΈΠ»ΡŒΠ½ΠΈΠΊΠ΅ (ΠΎΠΊΠΎΠ»ΠΎ +5 Β°C ) Π½Π°ΠΌΠ½ΠΎΠ³ΠΎ Π½ΠΈΠΆΠ΅ Ρ‚ΠΎΡ‡ΠΊΠΈ росы для ΠΊΠΎΠΌΠ½Π°Ρ‚Π½ΠΎΠ³ΠΎ Π²ΠΎΠ·Π΄ΡƒΡ…Π°. ΠŸΠΎΡΡ‚ΠΎΠΌΡƒ, Ссли Π²Ρ‹Ρ‚Π°Ρ‰ΠΈΡ‚ΡŒ ΠΎΡ…Π»Π°ΠΆΠ΄Π΅Π½Π½Ρ‹ΠΉ стСклянный Ρ‚Π΅Ρ€ΠΌΠΎΠΌΠ΅Ρ‚Ρ€ ΠΈΠ· Ρ…ΠΎΠ»ΠΎΠ΄ΠΈΠ»ΡŒΠ½ΠΈΠΊΠ°, Ρ‚ΠΎ ΠΎΠ½ сразу Β«Π·Π°ΠΏΠΎΡ‚Π΅Π΅Ρ‚Β» - стСклянный корпус станСт Π½Π΅ΠΏΡ€ΠΎΠ·Ρ€Π°Ρ‡Π½Ρ‹ΠΌ ΠΎΡ‚ Π²Π»Π°Π³ΠΈ. Π—Π°Ρ‚Π΅ΠΌ Ρ‚Π΅Ρ€ΠΌΠΎΠΌΠ΅Ρ‚Ρ€ Π½Π°Ρ‡Π½Π΅Ρ‚ Π½Π°Π³Ρ€Π΅Π²Π°Ρ‚ΡŒΡΡ, ΠΈ Π² ΠΊΠ°ΠΊΠΎΠΉ-Ρ‚ΠΎ ΠΌΠΎΠΌΠ΅Π½Ρ‚ ΡΠΊΠΎΠ½Π΄Π΅Π½ΡΠΈΡ€ΠΎΠ²Π°Π²ΡˆΠ°ΡΡΡ Π²Π»Π°Π³Π° Π½Π° Π½Π΅ΠΌ испарится - стСкло станСт ΠΏΡ€ΠΎΠ·Ρ€Π°Ρ‡Π½Ρ‹ΠΌ. Π­Ρ‚ΠΎ ΠΈ Π΅ΡΡ‚ΡŒ Ρ‚Π΅ΠΌΠΏΠ΅Ρ€Π°Ρ‚ΡƒΡ€Π° Ρ‚ΠΎΡ‡ΠΊΠΈ росы, ΠΏΠΎ ΠΊΠΎΡ‚ΠΎΡ€ΠΎΠΉ с ΠΏΠΎΠΌΠΎΡ‰ΡŒΡŽ Ρ‚Π°Π±Π»ΠΈΡ†Ρ‹ ΠΌΠΎΠΆΠ½ΠΎ Ρ€Π°ΡΡΡ‡ΠΈΡ‚Π°Ρ‚ΡŒ ΠΎΡ‚Π½ΠΎΡΠΈΡ‚Π΅Π»ΡŒΠ½ΡƒΡŽ Π²Π»Π°ΠΆΠ½ΠΎΡΡ‚ΡŒ.

Π—Π°Π΄Π°Ρ‡Π° 7. Π˜ΡΠΏΠ°Ρ€Π΅Π½ΠΈΠ΅

Π—Π°Π΄Π°Π½ΠΈΠ΅. НалСйтС ΠΏΠΎΡ‡Ρ‚ΠΈ ΠΏΠΎΠ»Π½Ρ‹ΠΉ стакан Π²ΠΎΠ΄Ρ‹ ΠΈ ΠΏΠΎΡΡ‚Π°Π²ΡŒΡ‚Π΅ Π΅Π³ΠΎ Π² ΠΊΠΎΠΌΠ½Π°Ρ‚Π΅ Π² Ρ‚Π΅ΠΏΠ»ΠΎΠ΅ мСсто - для Ρ‚ΠΎΠ³ΠΎ Ρ‡Ρ‚ΠΎΠ±Ρ‹ Π²ΠΎΠ΄Π° быстрСС ΠΈΡΠΏΠ°Ρ€ΡΠ»Π°ΡΡŒ. Π˜Π·ΠΌΠ΅Ρ€ΡŒΡ‚Π΅ Π»ΠΈΠ½Π΅ΠΉΠΊΠΎΠΉ Π½Π°Ρ‡Π°Π»ΡŒΠ½Ρ‹ΠΉ ΡƒΡ€ΠΎΠ²Π΅Π½ΡŒ Π²ΠΎΠ΄Ρ‹ ΠΈ Π·Π°ΠΏΠΈΡˆΠΈΡ‚Π΅ врСмя Π½Π°Ρ‡Π°Π»Π° ΠΎΠΏΡ‹Ρ‚Π°. Π§Π΅Ρ€Π΅Π· нСсколько Π΄Π½Π΅ΠΉ ΡƒΡ€ΠΎΠ²Π΅Π½ΡŒ Π²ΠΎΠ΄Ρ‹ понизится Π·Π° счСт испарСния. Π˜Π·ΠΌΠ΅Ρ€ΡŒΡ‚Π΅ Π½ΠΎΠ²Ρ‹ΠΉ ΡƒΡ€ΠΎΠ²Π΅Π½ΡŒ Π²ΠΎΠ΄Ρ‹ ΠΈ Π·Π°ΠΏΠΈΡˆΠΈΡ‚Π΅ врСмя окончания ΠΎΠΏΡ‹Ρ‚Π°. ΠžΠΏΡ€Π΅Π΄Π΅Π»ΠΈΡ‚Π΅ массу ΠΈΡΠΏΠ°Ρ€ΠΈΠ²ΡˆΠ΅ΠΉΡΡ Π²ΠΎΠ΄Ρ‹. Бколько Π² срСднСм ΠΌΠΎΠ»Π΅ΠΊΡƒΠ» Π²Ρ‹Π»Π΅Ρ‚Π°Π»ΠΎ с повСрхности Π²ΠΎΠ΄Ρ‹ Π·Π° 1 сСкунду? Бколько ΠΏΡ€ΠΈΠ±Π»ΠΈΠ·ΠΈΡ‚Π΅Π»ΡŒΠ½ΠΎ ΠΌΠΎΠ»Π΅ΠΊΡƒΠ» находится Π½Π° повСрхности Π²ΠΎΠ΄Ρ‹ Π² стаканС? Π‘Ρ€Π°Π²Π½ΠΈΡ‚Π΅ эти Π΄Π²Π° числа. Π”ΠΈΠ°ΠΌΠ΅Ρ‚Ρ€ ΠΌΠΎΠ»Π΅ΠΊΡƒΠ»Ρ‹ Π²ΠΎΠ΄Ρ‹ ΠΏΡ€ΠΈΠΌΠΈΡ‚Π΅ Ρ€Π°Π²Π½Ρ‹ΠΌ d 0 = 0,3 Π½ΠΌ . Зная ΡƒΠ΄Π΅Π»ΡŒΠ½ΡƒΡŽ Ρ‚Π΅ΠΏΠ»ΠΎΡ‚Ρƒ парообразования, ΠΎΠΏΡ€Π΅Π΄Π΅Π»ΠΈΡ‚Π΅ ΡΠΊΠΎΡ€ΠΎΡΡ‚ΡŒ ΠΏΠ΅Ρ€Π΅Π΄Π°Ρ‡ΠΈ Ρ‚Π΅ΠΏΠ»Π° (Π”ΠΆ/с ) Π²ΠΎΠ΄Π΅ ΠΎΡ‚ ΠΎΠΊΡ€ΡƒΠΆΠ°ΡŽΡ‰Π΅ΠΉ срСды.

Π’ΠΎΠ·ΠΌΠΎΠΆΠ½Ρ‹ΠΉ способ Ρ€Π΅ΡˆΠ΅Π½ΠΈΡ. ΠŸΡƒΡΡ‚ΡŒ d - Π²Π½ΡƒΡ‚Ρ€Π΅Π½Π½ΠΈΠΉ Π΄ΠΈΠ°ΠΌΠ΅Ρ‚Ρ€ стакана, с - ΠΏΠ»ΠΎΡ‚Π½ΠΎΡΡ‚ΡŒ Π²ΠΎΠ΄Ρ‹, М - молярная масса Π²ΠΎΠ΄Ρ‹, r - ΡƒΠ΄Π΅Π»ΡŒΠ½Π°Ρ Ρ‚Π΅ΠΏΠ»ΠΎΡ‚Π° парообразования, Π”h - ΠΏΠΎΠ½ΠΈΠΆΠ΅Π½ΠΈΠ΅ уровня Π²ΠΎΠ΄Ρ‹ Π·Π° врСмя t . Π’ΠΎΠ³Π΄Π° масса ΠΈΡΠΏΠ°Ρ€ΠΈΠ²ΡˆΠ΅ΠΉΡΡ Π²ΠΎΠ΄Ρ‹ Ρ€Π°Π²Π½Π°

m = сv = с Π”hS = с Π”hΡ€d 2 4.

Π’ этой массС содСрТится N = mN A /М ΠΌΠΎΠ»Π΅ΠΊΡƒΠ», Π³Π΄Π΅ N A - постоянная Авогадро. Число ΠΈΡΠΏΠ°Ρ€ΠΈΠ²ΡˆΠΈΡ…ΡΡ Π·Π° 1 сСкунду ΠΌΠΎΠ»Π΅ΠΊΡƒΠ» Ρ€Π°Π²Π½ΠΎ

N 1 = Nt = mN A Mt .

Если S = Ρ€d 2 /4 - ΠΏΠ»ΠΎΡ‰Π°Π΄ΡŒ повСрхности Π²ΠΎΠ΄Ρ‹ Π² стаканС, Π° S 0 = Ρ€d 2 0 /4 - ΠΏΠ»ΠΎΡ‰Π°Π΄ΡŒ сСчСния ΠΎΠ΄Π½ΠΎΠΉ ΠΌΠΎΠ»Π΅ΠΊΡƒΠ»Ρ‹, Ρ‚ΠΎ Π½Π° повСрхности Π²ΠΎΠ΄Ρ‹ Π² стаканС находится ΠΏΡ€ΠΈΠ±Π»ΠΈΠ·ΠΈΡ‚Π΅Π»ΡŒΠ½ΠΎ

N 2 = SS 0 = (dd 0) 2 .

Π’ΠΎΠ΄Π° для испарСния ΠΏΠΎΠ»ΡƒΡ‡Π°Π΅Ρ‚ Π² Π΅Π΄ΠΈΠ½ΠΈΡ†Ρƒ Π²Ρ€Π΅ΠΌΠ΅Π½ΠΈ количСство Ρ‚Π΅ΠΏΠ»ΠΎΡ‚Ρ‹

Qt = rmt .

Если ΠΏΡ€ΠΎΠΈΠ·Π²ΠΎΠ΄ΠΈΡ‚ΡŒ ΠΊΠ°ΠΊΠΈΠ΅-Π»ΠΈΠ±ΠΎ расчСты, связанныС с ΠΌΠΎΠ»Π΅ΠΊΡƒΠ»Π°ΠΌΠΈ, Ρ‚ΠΎ всСгда ΠΏΠΎΠ»ΡƒΡ‡Π°ΡŽΡ‚ΡΡ интСрСсныС Ρ€Π΅Π·ΡƒΠ»ΡŒΡ‚Π°Ρ‚Ρ‹. НапримСр, ΠΏΡƒΡΡ‚ΡŒ Π·Π° врСмя t = 5 суток Π² стаканС Π΄ΠΈΠ°ΠΌΠ΅Ρ‚Ρ€ΠΎΠΌ d = 65 ΠΌΠΌ ΡƒΡ€ΠΎΠ²Π΅Π½ΡŒ Π²ΠΎΠ΄Ρ‹ понизился Π½Π° Π”h = 1 см . Π’ΠΎΠ³Π΄Π° ΠΏΠΎΠ»ΡƒΡ‡ΠΈΠΌ, Ρ‡Ρ‚ΠΎ Π² ΠΏΠ°Ρ€ ΠΏΡ€Π΅Π²Ρ€Π°Ρ‚ΠΈΠ»ΠΎΡΡŒ 33 Π³ Π²ΠΎΠ΄Ρ‹, Π·Π° 1 с ΠΈΡΠΏΠ°Ρ€ΠΈΠ»ΠΎΡΡŒ N 1 = 2,56?10 18 ΠΌΠΎΠ»Π΅ΠΊΡƒΠ», Π½Π° повСрхности Π²ΠΎΠ΄Ρ‹ Π² стаканС Π½Π°Ρ…ΠΎΠ΄ΠΈΠ»ΠΎΡΡŒ N 2 = 4,69?1016 ΠΌΠΎΠ»Π΅ΠΊΡƒΠ», Π° ΠΈΠ· ΠΎΠΊΡ€ΡƒΠΆΠ°ΡŽΡ‰Π΅ΠΉ срСды поступило 0,19 Π’Ρ‚ Ρ‚Π΅ΠΏΠ»Π°. Π˜Π½Ρ‚Π΅Ρ€Π΅ΡΠ½Ρ‹ΠΌ являСтся ΠΎΡ‚Π½ΠΎΡˆΠ΅Π½ΠΈΠ΅ N 1 /N 2 ? 54, ΠΈΠ· ΠΊΠΎΡ‚ΠΎΡ€ΠΎΠ³ΠΎ Π²ΠΈΠ΄Π½ΠΎ, Ρ‡Ρ‚ΠΎ Π·Π° 1 с ΠΈΡΠΏΠ°Ρ€ΡΠ»ΠΎΡΡŒ ΡΡ‚ΠΎΠ»ΡŒΠΊΠΎ ΠΌΠΎΠ»Π΅ΠΊΡƒΠ», сколько ΠΏΠΎΠΌΠ΅Ρ‰Π°Π»ΠΎΡΡŒ Π² стаканС Π² 54 слоях Π²ΠΎΠ΄Ρ‹.

Π—Π°Π΄Π°Ρ‡Π° 8. РастворСниС

Π—Π°Π΄Π°Π½ΠΈΠ΅. Высыпая соль ΠΈΠ»ΠΈ сахар Π² ΠΊΠΈΠΏΡΡ‰ΡƒΡŽ Π²ΠΎΠ΄Ρƒ, ΠΌΠΎΠΆΠ½ΠΎ Π·Π°ΠΌΠ΅Ρ‚ΠΈΡ‚ΡŒ, Ρ‡Ρ‚ΠΎ ΠΊΠΈΠΏΠ΅Π½ΠΈΠ΅ Π½Π΅Π½Π°Π΄ΠΎΠ»Π³ΠΎ прСкращаСтся Π·Π° счСт сниТСния Ρ‚Π΅ΠΌΠΏΠ΅Ρ€Π°Ρ‚ΡƒΡ€Ρ‹ Π²ΠΎΠ΄Ρ‹. ΠžΠΏΡ€Π΅Π΄Π΅Π»ΠΈΡ‚Π΅ количСство Ρ‚Π΅ΠΏΠ»ΠΎΡ‚Ρ‹, Π½Π΅ΠΎΠ±Ρ…ΠΎΠ΄ΠΈΠΌΠΎΠ΅ для растворСния 1 ΠΊΠ³ ΠΏΠΈΡ‰Π΅Π²ΠΎΠΉ соды Π² Π²ΠΎΠ΄Π΅ ΠΊΠΎΠΌΠ½Π°Ρ‚Π½ΠΎΠΉ Ρ‚Π΅ΠΌΠΏΠ΅Ρ€Π°Ρ‚ΡƒΡ€Ρ‹.

ΠžΠ±ΠΎΡ€ΡƒΠ΄ΠΎΠ²Π°Π½ΠΈΠ΅: ΡΠ°ΠΌΠΎΠ΄Π΅Π»ΡŒΠ½Ρ‹ΠΉ ΠΊΠ°Π»ΠΎΡ€ΠΈΠΌΠ΅Ρ‚Ρ€, Ρ‚Π΅Ρ€ΠΌΠΎΠΌΠ΅Ρ‚Ρ€, Π²ΠΎΠ΄Π°, сода, ΠΌΠ΅Ρ€Π½Ρ‹ΠΉ Ρ†ΠΈΠ»ΠΈΠ½Π΄Ρ€ (стакан), Π³Ρ€ΡƒΠ· извСстной массы (Π³Π°ΠΉΠΊΠ° массой 10 Π³ ), пластиковая Π»ΠΎΠΆΠΊΠ°.

Π’ΠΎΠ·ΠΌΠΎΠΆΠ½Ρ‹ΠΉ способ Ρ€Π΅ΡˆΠ΅Π½ΠΈΡ. Π’ Π·Π°Π΄Π°Ρ‡Ρƒ Π²Ρ…ΠΎΠ΄ΠΈΡ‚ Π΄ΠΎΠΏΠΎΠ»Π½ΠΈΡ‚Π΅Π»ΡŒΠ½ΠΎΠ΅ конструкторскоС Π·Π°Π΄Π°Π½ΠΈΠ΅ ΠΏΠΎ ΠΈΠ·Π³ΠΎΡ‚ΠΎΠ²Π»Π΅Π½ΠΈΡŽ простого самодСльного ΠΊΠ°Π»ΠΎΡ€ΠΈΠΌΠ΅Ρ‚Ρ€Π°. Для Π²Π½ΡƒΡ‚Ρ€Π΅Π½Π½Π΅Π³ΠΎ сосуда ΠΊΠ°Π»ΠΎΡ€ΠΈΠΌΠ΅Ρ‚Ρ€Π° слСдуСт Π²Π·ΡΡ‚ΡŒ ΠΎΠ±Ρ‹Ρ‡Π½ΡƒΡŽ Π°Π»ΡŽΠΌΠΈΠ½ΠΈΠ΅Π²ΡƒΡŽ Π±Π°Π½ΠΊΡƒ объСмом 0,33 Π». Π£ Π±Π°Π½ΠΊΠΈ удаляСтся вСрхняя ΠΊΡ€Ρ‹ΡˆΠΊΠ° Ρ‚Π°ΠΊ, Ρ‡Ρ‚ΠΎΠ±Ρ‹ получился Π°Π»ΡŽΠΌΠΈΠ½ΠΈΠ΅Π²Ρ‹ΠΉ стакан (массой всСго 12 Π³ ) с ТСстким Π²Π΅Ρ€Ρ…Π½ΠΈΠΌ ΠΎΠ±ΠΎΠ΄ΠΊΠΎΠΌ. Π’Π½ΡƒΡ‚Ρ€ΠΈ Π²Π΅Ρ€Ρ…Π½Π΅Π³ΠΎ ΠΎΠ±ΠΎΠ΄ΠΊΠ° дСлаСтся ΠΏΡ€ΠΎΡ€Π΅Π·ΡŒ для Ρ‚ΠΎΠ³ΠΎ, Ρ‡Ρ‚ΠΎΠ±Ρ‹ Π²ΠΎΠ΄Π° ΠΏΠΎΠ»Π½ΠΎΡΡ‚ΡŒΡŽ Π²Ρ‹Π»ΠΈΠ²Π°Π»Π°ΡΡŒ ΠΈΠ· Π±Π°Π½ΠΊΠΈ. Π’Π½Π΅ΡˆΠ½ΡΡ пластмассовая ΠΎΠ±ΠΎΠ»ΠΎΡ‡ΠΊΠ° изготавливаСтся Π½Π° основС пластиковой Π±ΡƒΡ‚Ρ‹Π»ΠΊΠΈ объСмом 1,5 Π» . Π‘ΡƒΡ‚Ρ‹Π»ΠΊΠ° разрСзаСтся Π½Π° Ρ‚Ρ€ΠΈ части, вСрхняя Ρ‡Π°ΡΡ‚ΡŒ удаляСтся, Π° срСдняя ΠΈ ниТняя части с Π½Π΅ΠΊΠΎΡ‚ΠΎΡ€Ρ‹ΠΌ усилиСм Π²ΡΡ‚Π°Π²Π»ΡΡŽΡ‚ΡΡ Π΄Ρ€ΡƒΠ³ Π² Π΄Ρ€ΡƒΠ³Π° ΠΈ ΠΏΠ»ΠΎΡ‚Π½ΠΎ Ρ„ΠΈΠΊΡΠΈΡ€ΡƒΡŽΡ‚ Π²Π½ΡƒΡ‚Ρ€Π΅Π½Π½ΡŽΡŽ Π°Π»ΡŽΠΌΠΈΠ½ΠΈΠ΅Π²ΡƒΡŽ Π±Π°Π½ΠΊΡƒ Π² Π²Π΅Ρ€Ρ‚ΠΈΠΊΠ°Π»ΡŒΠ½ΠΎΠΌ ΠΏΠΎΠ»ΠΎΠΆΠ΅Π½ΠΈΠΈ. (Если Π½Π΅Ρ‚ ΠΊΠ°Π»ΠΎΡ€ΠΈΠΌΠ΅Ρ‚Ρ€Π°, Ρ‚ΠΎ ΠΎΠΏΡ‹Ρ‚Ρ‹ ΠΌΠΎΠΆΠ½ΠΎ ΠΏΡ€ΠΎΠ²ΠΎΠ΄ΠΈΡ‚ΡŒ ΠΈ Π² ΠΎΠ΄Π½ΠΎΡ€Π°Π·ΠΎΠ²ΠΎΠΌ пластиковом стаканчикС, массой ΠΈ Ρ‚Π΅ΠΏΠ»ΠΎΠΏΠ΅Ρ€Π΅Π΄Π°Ρ‡Π΅ΠΉ ΠΊΠΎΡ‚ΠΎΡ€ΠΎΠ³ΠΎ ΠΌΠΎΠΆΠ½ΠΎ ΠΏΡ€Π΅Π½Π΅Π±Ρ€Π΅Ρ‡ΡŒ).

ΠŸΡ€Π΅Π΄Π²Π°Ρ€ΠΈΡ‚Π΅Π»ΡŒΠ½ΠΎ слСдуСт ΡΠ΄Π΅Π»Π°Ρ‚ΡŒ Π΄Π²Π° измСрСния: 1) ΠΎΠΏΡ€Π΅Π΄Π΅Π»ΠΈΡ‚ΡŒ, сколько соды помСщаСтся Π² Π»ΠΎΠΆΠΊΡƒ (для этого Π½Π°Π΄ΠΎ Π·Π°Π³Π»ΡΠ½ΡƒΡ‚ΡŒ Π² ΠΊΡƒΠ»ΠΈΠ½Π°Ρ€Π½Ρ‹ΠΉ справочник ΠΈΠ»ΠΈ Β«Π²Ρ‹Ρ‡Π΅Ρ€ΠΏΠ°Ρ‚ΡŒΒ» этой Π»ΠΎΠΆΠΊΠΎΠΉ ΠΏΠ°ΠΊΠ΅Ρ‚ соды извСстной массы); 2) ΠΎΠΏΡ€Π΅Π΄Π΅Π»ΠΈΡ‚ΡŒΡΡ с количСством Π²ΠΎΠ΄Ρ‹ - Π² ΠΌΠ°Π»ΠΎΠΌ количСствС Π²ΠΎΠ΄Ρ‹ раствор сразу ΠΆΠ΅ станСт насыщСнным ΠΈ Ρ‡Π°ΡΡ‚ΡŒ соды Π½Π΅ растворится, Π² большом количСствС Π²ΠΎΠ΄Ρ‹ Ρ‚Π΅ΠΌΠΏΠ΅Ρ€Π°Ρ‚ΡƒΡ€Π° измСнится Π½Π° Π΄ΠΎΠ»ΠΈ градуса, Ρ‡Ρ‚ΠΎ Π·Π°Ρ‚Ρ€ΡƒΠ΄Π½ΠΈΡ‚ измСрСния.

ΠžΡ‡Π΅Π²ΠΈΠ΄Π½ΠΎ, Ρ‡Ρ‚ΠΎ количСство Ρ‚Π΅ΠΏΠ»ΠΎΡ‚Ρ‹, Π½Π΅ΠΎΠ±Ρ…ΠΎΠ΄ΠΈΠΌΠΎΠ΅ для растворСния вСщСства, ΠΏΡ€ΠΎΠΏΠΎΡ€Ρ†ΠΈΠΎΠ½Π°Π»ΡŒΠ½ΠΎ массС этого вСщСства: Q ~ m . Для записи равСнства слСдуСт ввСсти коэффициСнт ΠΏΡ€ΠΎΠΏΠΎΡ€Ρ†ΠΈΠΎΠ½Π°Π»ΡŒΠ½ΠΎΡΡ‚ΠΈ, Π½Π°ΠΏΡ€ΠΈΠΌΠ΅Ρ€ z , ΠΊΠΎΡ‚ΠΎΡ€Ρ‹ΠΉ ΠΌΠΎΠΆΠ½ΠΎ Π½Π°Π·Π²Π°Ρ‚ΡŒ Β«ΡƒΠ΄Π΅Π»ΡŒΠ½ΠΎΠΉ Ρ‚Π΅ΠΏΠ»ΠΎΡ‚ΠΎΠΉ растворСния». Π’ΠΎΠ³Π΄Π°

Q = zm .

РастворСниС соды осущСствляСтся Π·Π° счСт энСргии, Π²Ρ‹Π΄Π΅Π»ΡΡŽΡ‰Π΅ΠΉΡΡ ΠΏΡ€ΠΈ ΠΎΡ…Π»Π°ΠΆΠ΄Π΅Π½ΠΈΠΈ сосуда с Π²ΠΎΠ΄ΠΎΠΉ. Π’Π΅Π»ΠΈΡ‡ΠΈΠ½Π° z находится ΠΈΠ· ΡΠ»Π΅Π΄ΡƒΡŽΡ‰Π΅Π³ΠΎ уравнСния Ρ‚Π΅ΠΏΠ»ΠΎΠ²ΠΎΠ³ΠΎ баланса:

mvcv(t 2 -t 1 )+ma cc (t 2 -t 1 ) = zm .

Π³Π΄Π΅ m v - масса Π²ΠΎΠ΄Ρ‹ Π² ΠΊΠ°Π»ΠΎΡ€ΠΈΠΌΠ΅Ρ‚Ρ€Π΅, m a - масса Π²Π½ΡƒΡ‚Ρ€Π΅Π½Π½Π΅Π³ΠΎ алюминиСвого стакана ΠΊΠ°Π»ΠΎΡ€ΠΈΠΌΠ΅Ρ‚Ρ€Π°, m - масса растворСнной соды, (t 2 -t 1) - ΠΏΠΎΠ½ΠΈΠΆΠ΅Π½ΠΈΠ΅ Ρ‚Π΅ΠΌΠΏΠ΅Ρ€Π°Ρ‚ΡƒΡ€Ρ‹ Π² ΠΊΠ°Π»ΠΎΡ€ΠΈΠΌΠ΅Ρ‚Ρ€Π΅. ΠœΠ°ΡΡΡƒ Π²Π½ΡƒΡ‚Ρ€Π΅Π½Π½Π΅Π³ΠΎ сосуда ΠΊΠ°Π»ΠΎΡ€ΠΈΠΌΠ΅Ρ‚Ρ€Π° ΠΌΠΎΠΆΠ½ΠΎ Π»Π΅Π³ΠΊΠΎ Π½Π°ΠΉΡ‚ΠΈ, ΠΈΡΠΏΠΎΠ»ΡŒΠ·ΡƒΡ ΠΏΡ€Π°Π²ΠΈΠ»ΠΎ ΠΌΠΎΠΌΠ΅Π½Ρ‚ΠΎΠ² сил, уравновСсив сосуд ΠΈ Π³Ρ€ΡƒΠ· извСстной массы ΠΏΡ€ΠΈ ΠΏΠΎΠΌΠΎΡ‰ΠΈ Π»ΠΈΠ½Π΅ΠΉΠΊΠΈ ΠΈ Π½ΠΈΡ‚ΠΎΠΊ.

Π˜Π·ΠΌΠ΅Ρ€Π΅Π½ΠΈΡ ΠΈ расчСты ΠΏΠΎΠΊΠ°Π·Ρ‹Π²Π°ΡŽΡ‚, Ρ‡Ρ‚ΠΎ ΠΏΡ€ΠΈ m = 6 Π³ ΠΈ m v = 100 Π³ Π²ΠΎΠ΄Π° остываСт Π½Π° 2-2,5 Ρ”C , Π° Π²Π΅Π»ΠΈΡ‡ΠΈΠ½Π° z оказываСтся Ρ€Π°Π²Π½ΠΎΠΉ 144-180 ΠΊΠ”ΠΆ/ΠΊΠ³ .

Π—Π°Π΄Π°Ρ‡Π° 9. Π•ΠΌΠΊΠΎΡΡ‚ΡŒ ΠΊΠ°ΡΡ‚Ρ€ΡŽΠ»ΠΈ

Π—Π°Π΄Π°Π½ΠΈΠ΅. Каким ΠΎΠ±Ρ€Π°Π·ΠΎΠΌ ΠΌΠΎΠΆΠ½ΠΎ Π½Π°ΠΉΡ‚ΠΈ Π΅ΠΌΠΊΠΎΡΡ‚ΡŒ ΠΊΠ°ΡΡ‚Ρ€ΡŽΠ»ΠΈ, ΠΏΠΎΠ»ΡŒΠ·ΡƒΡΡΡŒ вСсами ΠΈ Π½Π°Π±ΠΎΡ€ΠΎΠΌ Π³ΠΈΡ€ΡŒ?

Подсказка. Π’Π·Π²Π΅ΡΡŒΡ‚Π΅ ΠΏΡƒΡΡ‚ΡƒΡŽ ΠΊΠ°ΡΡ‚Ρ€ΡŽΠ»ΡŽ, Π° ΠΏΠΎΡ‚ΠΎΠΌ - ΠΊΠ°ΡΡ‚Ρ€ΡŽΠ»ΡŽ с Π²ΠΎΠ΄ΠΎΠΉ.

РСшСниС. ΠŸΡƒΡΡ‚ΡŒ масса пустой ΠΊΠ°ΡΡ‚Ρ€ΡŽΠ»ΠΈ Ρ€Π°Π²Π½Π° m 1 , Π° послС наполнСния Π²ΠΎΠ΄ΠΎΠΉ ΠΎΠ½Π° составляСт m 2 . Π’ΠΎΠ³Π΄Π° Ρ€Π°Π·Π½ΠΎΡΡ‚ΡŒ m 2 -m 1 Π΄Π°Π΅Ρ‚ массу Π²ΠΎΠ΄Ρ‹ Π² объСмС ΠΊΠ°ΡΡ‚Ρ€ΡŽΠ»ΠΈ. ПодСлив эту Ρ€Π°Π·Π½ΠΎΡΡ‚ΡŒ Π½Π° ΠΏΠ»ΠΎΡ‚Π½ΠΎΡΡ‚ΡŒ Π²ΠΎΠ΄Ρ‹ с , Π½Π°Ρ…ΠΎΠ΄ΠΈΠΌ объСм ΠΊΠ°ΡΡ‚Ρ€ΡŽΠ»ΠΈ:

Π—Π°Π΄Π°Ρ‡Π° 10. Как Ρ€Π°Π·Π΄Π΅Π»ΠΈΡ‚ΡŒ содСрТимоС стакана

Π—Π°Π΄Π°Π½ΠΈΠ΅. Π˜ΠΌΠ΅Π΅Ρ‚ΡΡ цилиндричСский стакан, Π΄ΠΎ ΠΊΡ€Π°Π΅Π² Π½Π°ΠΏΠΎΠ»Π½Π΅Π½Π½Ρ‹ΠΉ ΠΆΠΈΠ΄ΠΊΠΎΡΡ‚ΡŒΡŽ. Как Ρ€Π°Π·Π΄Π΅Π»ΠΈΡ‚ΡŒ содСрТимоС стакана Π½Π° Π΄Π²Π΅ ΡΠΎΠ²Π΅Ρ€ΡˆΠ΅Π½Π½ΠΎ Ρ€Π°Π²Π½Ρ‹Π΅ части, располагая Π΅Ρ‰Π΅ ΠΎΠ΄Π½ΠΈΠΌ сосудом, Π½ΠΎ ΡƒΠΆΠ΅ ΠΈΠ½ΠΎΠΉ Ρ„ΠΎΡ€ΠΌΡ‹ ΠΈ нСсколько мСньшСго Ρ€Π°Π·ΠΌΠ΅Ρ€Π°?

Подсказка. ΠŸΠΎΠ΄ΡƒΠΌΠ°ΠΉΡ‚Π΅, ΠΊΠ°ΠΊ ΠΌΠΎΠΆΠ½ΠΎ провСсти ΠΏΠ»ΠΎΡΠΊΠΎΡΡ‚ΡŒ, Ρ€Π°Π·Π΄Π΅Π»ΡΡŽΡ‰ΡƒΡŽ Ρ†ΠΈΠ»ΠΈΠ½Π΄Ρ€ Π½Π° Π΄Π²Π΅ Ρ€Π°Π²Π½Ρ‹Π΅ ΠΏΠΎ ΠΎΠ±ΡŠΠ΅ΠΌΡƒ части.

РСшСниС. Если Ρ‡Π΅Ρ€Π΅Π· Ρ‚ΠΎΡ‡ΠΊΠΈ М ΠΈ N мыслСнно провСсти ΠΏΠ»ΠΎΡΠΊΠΎΡΡ‚ΡŒ Ρ‚Π°ΠΊ, ΠΊΠ°ΠΊ это ΠΏΠΎΠΊΠ°Π·Π°Π½ΠΎ Π½Π° рисункС 1Π° , Ρ‚ΠΎ ΠΎΠ½Π° рассСчСт Ρ†ΠΈΠ»ΠΈΠ½Π΄Ρ€ Π½Π° Π΄Π²Π΅ симмСтричныС ΠΈ поэтому Ρ€Π°Π²Π½Ρ‹Π΅ ΠΏΠΎ ΠΎΠ±ΡŠΠ΅ΠΌΡƒ Ρ„ΠΈΠ³ΡƒΡ€Ρ‹, Π² соотвСтствии с рисунком 8. ΠžΡ‚ΡΡŽΠ΄Π° Π²Ρ‹Ρ‚Π΅ΠΊΠ°Π΅Ρ‚ Ρ€Π΅ΡˆΠ΅Π½ΠΈΠ΅ Π·Π°Π΄Π°Ρ‡ΠΈ.

ΠŸΠΎΡΡ‚Π΅ΠΏΠ΅Π½Π½ΠΎ наклоняя стакан, Π½ΡƒΠΆΠ½ΠΎ ΠΎΡ‚Π»ΠΈΠ²Π°Ρ‚ΡŒ ΡΠΎΠ΄Π΅Ρ€ΠΆΠ°Ρ‰ΡƒΡŽΡΡ Π² Π½Π΅ΠΌ ΠΆΠΈΠ΄ΠΊΠΎΡΡ‚ΡŒ Π΄ΠΎ Ρ‚Π΅Ρ… ΠΏΠΎΡ€, ΠΏΠΎΠΊΠ° Ρ‡ΡƒΡ‚ΡŒ-Ρ‡ΡƒΡ‚ΡŒ Π½Π΅ покаТСтся Π΄Π½ΠΎ (рисунок 1Π± ). Π’ этот ΠΌΠΎΠΌΠ΅Π½Ρ‚ Π² стаканС останСтся Ρ€ΠΎΠ²Π½ΠΎ ΠΏΠΎΠ»ΠΎΠ²ΠΈΠ½Π° Тидкости.

ЭлСктричСство

Π—Π°Π΄Π°Ρ‡Π° 11. ЭлСктричСский Β«Ρ‡Π΅Ρ€Π½Ρ‹ΠΉ ящик»

Β«Π§Π΅Ρ€Π½Ρ‹ΠΉ ящик» прСдставляСт собой Π½Π΅ΠΏΡ€ΠΎΠ·Ρ€Π°Ρ‡Π½ΡƒΡŽ Π·Π°ΠΊΡ€Ρ‹Ρ‚ΡƒΡŽ ΠΊΠΎΡ€ΠΎΠ±ΠΊΡƒ, ΠΊΠΎΡ‚ΠΎΡ€ΡƒΡŽ нСльзя Π²ΡΠΊΡ€Ρ‹Π²Π°Ρ‚ΡŒ, Ρ‡Ρ‚ΠΎΠ±Ρ‹ ΠΈΠ·ΡƒΡ‡ΠΈΡ‚ΡŒ Π΅Π΅ Π²Π½ΡƒΡ‚Ρ€Π΅Π½Π½Π΅Π΅ устройство. Π’Π½ΡƒΡ‚Ρ€ΠΈ ящика находятся нСсколько элСктричСских элСмСнтов, соСдинСнных ΠΌΠ΅ΠΆΠ΄Ρƒ собой Π² ΠΏΡ€ΠΎΡΡ‚ΡƒΡŽ ΡΠ»Π΅ΠΊΡ‚Ρ€ΠΈΡ‡Π΅ΡΠΊΡƒΡŽ Ρ†Π΅ΠΏΡŒ. ΠžΠ±Ρ‹Ρ‡Π½ΠΎ Ρ‚Π°ΠΊΠΈΠΌΠΈ элСмСнтами ΡΠ²Π»ΡΡŽΡ‚ΡΡ: источники Ρ‚ΠΎΠΊΠ°, постоянныС ΠΈ ΠΏΠ΅Ρ€Π΅ΠΌΠ΅Π½Π½Ρ‹Π΅ рСзисторы, кондСнсаторы, ΠΊΠ°Ρ‚ΡƒΡˆΠΊΠΈ индуктивности, ΠΏΠΎΠ»ΡƒΠΏΡ€ΠΎΠ²ΠΎΠ΄Π½ΠΈΠΊΠΎΠ²Ρ‹Π΅ Π΄ΠΈΠΎΠ΄Ρ‹. Π‘Π½Π°Ρ€ΡƒΠΆΠΈ ящика находятся нСсколько Π²Ρ‹Π²ΠΎΠ΄ΠΎΠ².

Основная Ρ†Π΅Π»ΡŒ задания Β«Ρ‡Π΅Ρ€Π½Ρ‹ΠΉ ящик»: сдСлав минимальноС число элСктричСских ΠΈΠ·ΠΌΠ΅Ρ€Π΅Π½ΠΈΠΉ с использованиСм Π²Π½Π΅ΡˆΠ½ΠΈΡ… Π²Ρ‹Π²ΠΎΠ΄ΠΎΠ², Β«Ρ€Π°ΡΡˆΠΈΡ„Ρ€ΠΎΠ²Π°Ρ‚ΡŒΒ» Β«Ρ‡Π΅Ρ€Π½Ρ‹ΠΉ ящик», Ρ‚.Π΅.:

  • - ΡƒΡΡ‚Π°Π½ΠΎΠ²ΠΈΡ‚ΡŒ, ΠΊΠ°ΠΊΠΈΠ΅ ΠΈΠΌΠ΅Π½Π½ΠΎ элСктричСскиС ΠΏΡ€ΠΈΠ±ΠΎΡ€Ρ‹ находятся Π²Π½ΡƒΡ‚Ρ€ΠΈ Β«Ρ‡Π΅Ρ€Π½ΠΎΠ³ΠΎ ящика».
  • - ΡƒΡΡ‚Π°Π½ΠΎΠ²ΠΈΡ‚ΡŒ схСму ΠΈΡ… соСдинСния.
  • - ΠΎΠΏΡ€Π΅Π΄Π΅Π»ΠΈΡ‚ΡŒ Π½ΠΎΠΌΠΈΠ½Π°Π»Ρ‹ (Π²Π΅Π»ΠΈΡ‡ΠΈΠ½Ρ‹ сопротивлСний рСзисторов, Смкости кондСнсаторов ΠΈ Ρ‚.Π΄.)

Π—Π°Π΄Π°Π½ΠΈΠ΅. Π’Ρ€ΠΈ рСзистора соСдинСны ΠΌΠ΅ΠΆΠ΄Ρƒ собой ΠΈ ΠΏΠΎΠΌΠ΅Ρ‰Π΅Π½Ρ‹ Π² Β«Ρ‡Π΅Ρ€Π½Ρ‹ΠΉ ящик» с трСмя Π²Ρ‹Π²ΠΎΠ΄Π°ΠΌΠΈ, Π² соотвСтствии с рисунком 9. Π’ΠΎΡ‡Π½ΠΎ Ρ‚Π°ΠΊΠΈΠ΅ ΠΆΠ΅ рСзисторы соСдинСны ΠΌΠ΅ΠΆΠ΄Ρƒ собой ΠΏΠΎ-Π΄Ρ€ΡƒΠ³ΠΎΠΌΡƒ ΠΈ ΠΏΠΎΠΌΠ΅Ρ‰Π΅Π½Ρ‹ Π²ΠΎ Π²Ρ‚ΠΎΡ€ΠΎΠΉ Β«Ρ‡Π΅Ρ€Π½Ρ‹ΠΉ ящик» с трСмя Π²Ρ‹Π²ΠΎΠ΄Π°ΠΌΠΈ. ΠžΠΏΡ€Π΅Π΄Π΅Π»ΠΈΡ‚ΡŒ сопротивлСниС ΠΊΠ°ΠΆΠ΄ΠΎΠ³ΠΎ рСзистора. ΠŸΠ΅Ρ€Π΅ΠΌΡ‹Ρ‡ΠΊΠΈ ΠΏΡ€ΠΈΠΌΠ΅Π½ΡΡ‚ΡŒ Π·Π°ΠΏΡ€Π΅Ρ‰Π΅Π½ΠΎ.

ΠžΠ±ΠΎΡ€ΡƒΠ΄ΠΎΠ²Π°Π½ΠΈΠ΅: ΠΌΡƒΠ»ΡŒΡ‚ΠΈΠΌΠ΅Ρ‚Ρ€.

Π˜Π·ΠΌΠ΅Ρ€Π΅Π½ΠΈΠ΅ сопротивлСния ΠΌΠ΅ΠΆΠ΄Ρƒ Π²Ρ‹Π²ΠΎΠ΄Π°ΠΌΠΈ Π΄Π°Π»ΠΈ Ρ€Π΅Π·ΡƒΠ»ΡŒΡ‚Π°Ρ‚Ρ‹:

Π―Ρ‰ΠΈΠΊ β„– 1: R 1-2 = 12 Ом, R 2-3 = 25 Ом, R 1-3 = 37 Ом

Π―Ρ‰ΠΈΠΊ β„– 2: R 1-2 = 5,45 Ом, R 2-3 = 15 Ом, R 1-3 = 20,45 Ом

Π’ΠΎΠ·ΠΌΠΎΠΆΠ½Ρ‹ΠΉ способ Ρ€Π΅ΡˆΠ΅Π½ΠΈΡ. Π’ΠΎΠ·ΠΌΠΎΠΆΠ½Ρ‹ Ρ‡Π΅Ρ‚Ρ‹Ρ€Π΅ способа соСдинСния Ρ‚Ρ€Π΅Ρ… рСзисторов с трСмя Π½Π°Ρ€ΡƒΠΆΠ½Ρ‹ΠΌΠΈ Π²Ρ‹Π²ΠΎΠ΄Π°ΠΌΠΈ Ρ‚Π°ΠΊ, Ρ‡Ρ‚ΠΎΠ±Ρ‹ Ρ‚Ρ€ΠΈ измСрСния Π΄Π°Π²Π°Π»ΠΈ Ρ€Π°Π·Π½ΠΎΠ΅ Π·Π½Π°Ρ‡Π΅Π½ΠΈΠ΅ сопротивлСний:

1) ΠΏΠΎΡΠ»Π΅Π΄ΠΎΠ²Π°Ρ‚Π΅Π»ΡŒΠ½ΠΎΠ΅, 2) смСшанноС, 3) Π·Π²Π΅Π·Π΄ΠΎΠΉ, 4) Ρ‚Ρ€Π΅ΡƒΠ³ΠΎΠ»ΡŒΠ½ΠΈΠΊΠΎΠΌ, Π² соотвСтствии с рисунком 10.


ПокаТСм ΠΏΠΎΡΠ»Π΅Π΄ΠΎΠ²Π°Ρ‚Π΅Π»ΡŒΠ½ΠΎΡΡ‚ΡŒ поиска ΠΎΡ‚Π²Π΅Ρ‚ΠΎΠ².

Π₯Π°Ρ€Π°ΠΊΡ‚Π΅Ρ€Π½Ρ‹ΠΌ ΠΏΡ€ΠΈΠ·Π½Π°ΠΊΠΎΠΌ Π΄Π²ΡƒΡ… ΠΏΠ΅Ρ€Π²Ρ‹Ρ… схСм являСтся Ρ‚ΠΎ, Ρ‡Ρ‚ΠΎ ΠΎΠ΄Π½ΠΎ ΠΈΠ· ΠΈΠ·ΠΌΠ΅Ρ€Π΅Π½ΠΈΠΉ Ρ€Π°Π²Π½ΠΎ суммС Π΄Π²ΡƒΡ… Π΄Ρ€ΡƒΠ³ΠΈΡ…, Ρ‡Ρ‚ΠΎ ΠΈ соотвСтствуСт ΡƒΡΠ»ΠΎΠ²ΠΈΡŽ Π·Π°Π΄Π°Ρ‡ΠΈ:

Π‘Π»Π΅Π΄ΠΎΠ²Π°Ρ‚Π΅Π»ΡŒΠ½ΠΎ, Π² ΠΎΠ΄Π½ΠΎΠΌ ящикС ΠΏΠΎΡΠ»Π΅Π΄ΠΎΠ²Π°Ρ‚Π΅Π»ΡŒΠ½ΠΎΠ΅ соСдинСниС, Π½ΠΎ Ρ‚ΠΎΠ³Π΄Π° Π² Π΄Ρ€ΡƒΠ³ΠΎΠΌ - смСшанноС, ΠΏΠΎΡΠΊΠΎΠ»ΡŒΠΊΡƒ Ρ€Π΅Π·ΡƒΠ»ΡŒΡ‚Π°Ρ‚Ρ‹ ΠΈΠ·ΠΌΠ΅Ρ€Π΅Π½ΠΈΠΉ Π½Π΅ ΡΠΎΠ²ΠΏΠ°Π΄Π°ΡŽΡ‚, хотя Π½ΠΎΠΌΠΈΠ½Π°Π»Ρ‹ рСзисторов Ρ‚Π΅ ΠΆΠ΅ самыС.

Π˜Π·Π²Π΅ΡΡ‚Π½ΠΎ, Ρ‡Ρ‚ΠΎ всСгда выполняСтся ΡΠΎΠΎΡ‚Π½ΠΎΡˆΠ΅Π½ΠΈΠ΅

А ΠΏΠΎΡΠΊΠΎΠ»ΡŒΠΊΡƒ R 1-3 cΠ»Π΅Π²Π° большС, Ρ‡Π΅ΠΌ R 1-3 справа, Ρ‚ΠΎ Π² Π»Π΅Π²ΠΎΠΌ ящикС (β„–1) находится ΠΏΠΎΡΠ»Π΅Π΄ΠΎΠ²Π°Ρ‚Π΅Π»ΡŒΠ½ΠΎΠ΅ соСдинСниС, Π° Π² ΠΏΡ€Π°Π²ΠΎΠΌ (β„–2) - смСшанноС.

Π’ состав ΠΏΠΎΡΠ»Π΅Π΄ΠΎΠ²Π°Ρ‚Π΅Π»ΡŒΠ½ΠΎΠ³ΠΎ соСдинСния Π² Π»Π΅Π²ΠΎΠΌ ящикС входят рСзисторы с Π½ΠΎΠΌΠΈΠ½Π°Π»Π°ΠΌΠΈ 12 ΠΈΠ»ΠΈ 25 Ом . Π’Π°ΠΊ ΠΊΠ°ΠΊ Π½ΠΈ Ρ‚ΠΎ, Π½ΠΈ Π΄Ρ€ΡƒΠ³ΠΎΠ΅ Π·Π½Π°Ρ‡Π΅Π½ΠΈΠ΅ Π½Π΅ Π½Π°Π±Π»ΡŽΠ΄Π°Π΅Ρ‚ΡΡ Π² составС смСшанного соСдинСния, ΡΠ»Π΅Π΄ΠΎΠ²Π°Ρ‚Π΅Π»ΡŒΠ½ΠΎ, Π½ΠΎΠΌΠΈΠ½Π°Π» ΠΎΠ΄Π½ΠΎΠ³ΠΎ ΠΈΠ· рСзисторов R 1 = 15 Ом .

ΠžΡΡ‚Π°Π»ΡŒΠ½Ρ‹Π΅ Π½ΠΎΠΌΠΈΠ½Π°Π»Ρ‹: R 2 = 12 Ом ΠΈ R 3 = 10 Ом .

ΠžΡ‡Π΅Π²ΠΈΠ΄Π½ΠΎ, ΠΊ Ρ‚Π΅ΠΌ ΠΆΠ΅ Ρ€Π΅Π·ΡƒΠ»ΡŒΡ‚Π°Ρ‚Π°ΠΌ ΠΌΠΎΠΆΠ½ΠΎ ΠΏΡ€Π΅ΠΉΡ‚ΠΈ ΠΈ с ΠΏΠΎΠΌΠΎΡ‰ΡŒΡŽ ΠΈΠ½ΠΎΠΉ Ρ†Π΅ΠΏΠΎΡ‡ΠΊΠΈ рассуТдСний.

ΠžΡ‚ΠΌΠ΅Ρ‚ΠΈΠΌ Ρ‚Π°ΠΊΠΆΠ΅, Ρ‡Ρ‚ΠΎ Π²ΠΎΠ·ΠΌΠΎΠΆΠ½Ρ‹ Π΅Ρ‰Π΅ 5 ΠΊΠΎΠΌΠ±ΠΈΠ½Π°Ρ†ΠΈΠΉ схСм ΠΏΠΎ Π΄Π²Π° Β«Ρ‡Π΅Ρ€Π½Ρ‹Ρ… ящика» ΠΈΠ· ΠΏΡ€ΠΈΠ²Π΅Π΄Ρ‘Π½Π½Ρ‹Ρ… Ρ‡Π΅Ρ‚Ρ‹Ρ€Π΅Ρ…. НаиболСС Π³Ρ€ΠΎΠΌΠΎΠ·Π΄ΠΊΠ° матСматичСская Ρ‡Π°ΡΡ‚ΡŒ Π·Π°Π΄Π°Ρ‡ΠΈ ΠΏΠΎ Β«Ρ€Π°ΡΡˆΠΈΡ„Ρ€ΠΎΠ²ΠΊΠ΅Β» Ρ‡Π΅Ρ€Π½ΠΎΠ³ΠΎ ящика, ΠΎ ΠΊΠΎΡ‚ΠΎΡ€ΠΎΠΌ извСстно, Ρ‡Ρ‚ΠΎ Ρ‚Π°ΠΌ находится Ρ‚Ρ€Π΅ΡƒΠ³ΠΎΠ»ΡŒΠ½ΠΈΠΊ.

Π’ Π·Π°ΠΊΠ»ΡŽΡ‡Π΅Π½ΠΈΠΈ ΠΎΡ‚ΠΌΠ΅Ρ‚ΠΈΠΌ, Ρ‡Ρ‚ΠΎ Π½Π΅ всС ΠΌΠΎΠΆΠ΅Ρ‚ ΠΈΠ΄Ρ‚ΠΈ Ρ‚Π°ΠΊ Π³Π»Π°Π΄ΠΊΠΎ, ΠΊΠ°ΠΊ Π² Π΄Π°Π½Π½ΠΎΠΌ ΠΏΡ€ΠΈΠΌΠ΅Ρ€Π΅. ЗначСния сопротивлСний ΠΈΠ»ΠΈ Π΄Ρ€ΡƒΠ³ΠΈΡ… элСктричСских Π²Π΅Π»ΠΈΡ‡ΠΈΠ½, СстСствСнно, содСрТат ΠΏΠΎΠ³Ρ€Π΅ΡˆΠ½ΠΎΡΡ‚ΠΈ. И, Π½Π°ΠΏΡ€ΠΈΠΌΠ΅Ρ€, ΡΠΎΠΎΡ‚Π½ΠΎΡˆΠ΅Π½ΠΈΠ΅ ΠΌΠΎΠΆΠ΅Ρ‚ Π²Ρ‹ΠΏΠΎΠ»Π½ΡΡ‚ΡŒΡΡ Ρ‚ΠΎΠ»ΡŒΠΊΠΎ ΠΏΡ€ΠΈΠ±Π»ΠΈΠ·ΠΈΡ‚Π΅Π»ΡŒΠ½ΠΎ.

Π—Π°Π΄Π°Ρ‡Π° 12. Π’Π΅ΠΌΠΏΠ΅Ρ€Π°Ρ‚ΡƒΡ€Π° Π²ΠΎΠ·Π΄ΡƒΡ…Π° Π² ΠΊΠΎΠΌΠ½Π°Ρ‚Π΅

Π—Π°Π΄Π°Π½ΠΈΠ΅. Π—Π° ΠΎΠΊΠ½ΠΎΠΌ снСг, Π° Π² ΠΊΠΎΠΌΠ½Π°Ρ‚Π΅ Ρ‚Π΅ΠΏΠ»ΠΎ. К соТалСнию, ΠΈΠ·ΠΌΠ΅Ρ€ΠΈΡ‚ΡŒ Ρ‚Π΅ΠΌΠΏΠ΅Ρ€Π°Ρ‚ΡƒΡ€Ρƒ Π½Π΅Ρ‡Π΅ΠΌ - Π½Π΅Ρ‚ Ρ‚Π΅Ρ€ΠΌΠΎΠΌΠ΅Ρ‚Ρ€Π°. Но Π·Π°Ρ‚ΠΎ Π΅ΡΡ‚ΡŒ батарСя, ΠΎΡ‡Π΅Π½ΡŒ Ρ‚ΠΎΡ‡Π½Ρ‹ΠΉ Π²ΠΎΠ»ΡŒΡ‚ΠΌΠ΅Ρ‚Ρ€ ΠΈ Ρ‚Π°ΠΊΠΎΠΉ ΠΆΠ΅ Π°ΠΌΠΏΠ΅Ρ€ΠΌΠ΅Ρ‚Ρ€, сколько ΡƒΠ³ΠΎΠ΄Π½ΠΎ ΠΌΠ΅Π΄Π½ΠΎΠΉ ΠΏΡ€ΠΎΠ²ΠΎΠ»ΠΎΠΊΠΈ ΠΈ ΠΏΠΎΠ΄Ρ€ΠΎΠ±Π½Ρ‹ΠΉ физичСский справочник. НСльзя Π»ΠΈ с ΠΈΡ… ΠΏΠΎΠΌΠΎΡ‰ΡŒΡŽ Π½Π°ΠΉΡ‚ΠΈ Ρ‚Π΅ΠΌΠΏΠ΅Ρ€Π°Ρ‚ΡƒΡ€Ρƒ Π²ΠΎΠ·Π΄ΡƒΡ…Π° Π² ΠΊΠΎΠΌΠ½Π°Ρ‚Π΅?

Подсказка. ΠŸΡ€ΠΈ Π½Π°Π³Ρ€Π΅Π²Π°Π½ΠΈΠΈ ΠΌΠ΅Ρ‚Π°Π»Π»Π° Π΅Π³ΠΎ сопротивлСниС возрастаСт ΠΏΠΎ Π»ΠΈΠ½Π΅ΠΉΠ½ΠΎΠΌΡƒ Π·Π°ΠΊΠΎΠ½Ρƒ.

РСшСниС. Π‘ΠΎΠ΅Π΄ΠΈΠ½ΠΈΠΌ ΠΏΠΎΡΠ»Π΅Π΄ΠΎΠ²Π°Ρ‚Π΅Π»ΡŒΠ½ΠΎ Π±Π°Ρ‚Π°Ρ€Π΅ΡŽ, ΠΌΠΎΡ‚ΠΎΠΊ ΠΏΡ€ΠΎΠ²ΠΎΠ»ΠΎΠΊΠΈ ΠΈ Π°ΠΌΠΏΠ΅Ρ€ΠΌΠ΅Ρ‚Ρ€ Π²ΠΊΠ»ΡŽΡ‡ΠΈΠΌ Ρ‚Π°ΠΊ, Ρ‡Ρ‚ΠΎΠ±Ρ‹ ΠΎΠ½ ΠΏΠΎΠΊΠ°Π·Ρ‹Π²Π°Π» напряТСниС Π½Π° ΠΌΠΎΡ‚ΠΊΠ΅, Π² соотвСтствии с рисунком 11. Π—Π°ΠΏΠΈΡˆΠ΅ΠΌ показания ΠΏΡ€ΠΈΠ±ΠΎΡ€ΠΎΠ² ΠΈ рассчитаСм сопротивлСниС ΠΌΠΎΡ‚ΠΊΠ° ΠΏΡ€ΠΈ ΠΊΠΎΠΌΠ½Π°Ρ‚Π½ΠΎΠΉ Ρ‚Π΅ΠΌΠΏΠ΅Ρ€Π°Ρ‚ΡƒΡ€Π΅:

ПослС этого принСсСм с ΡƒΠ»ΠΈΡ†Ρ‹ снСг, ΠΏΠΎΠ³Ρ€ΡƒΠ·ΠΈΠΌ Π² Π½Π΅Π³ΠΎ ΠΌΠΎΡ‚ΠΎΠΊ ΠΈ, ΠΏΠΎΠ΄ΠΎΠΆΠ΄Π°Π² Π½Π΅ΠΌΠ½ΠΎΠ³ΠΎ, Ρ‡Ρ‚ΠΎΠ±Ρ‹ снСг Π½Π°Ρ‡Π°Π» Ρ‚Π°ΡΡ‚ΡŒ, Π° ΠΏΡ€ΠΎΠ²ΠΎΠ»ΠΎΠΊΠ° Π΅Π³ΠΎ Ρ‚Π΅ΠΌΠΏΠ΅Ρ€Π°Ρ‚ΡƒΡ€Ρƒ, Ρ‚Π΅ΠΌ ΠΆΠ΅ способом ΠΎΠΏΡ€Π΅Π΄Π΅Π»ΠΈΠΌ сопротивлСниС ΠΏΡ€ΠΎΠ²ΠΎΠ»ΠΎΠΊΠΈ R 0 ΠΏΡ€ΠΈ Ρ‚Π΅ΠΌΠΏΠ΅Ρ€Π°Ρ‚ΡƒΡ€Π΅ Ρ‚Π°ΡŽΡ‰Π΅Π³ΠΎ снСга, Ρ‚.Π΅. ΠΏΡ€ΠΈ 0 Ρ” Π‘ . ΠŸΠΎΠ»ΡŒΠ·ΡƒΡΡΡŒ Π·Π°Ρ‚Π΅ΠΌ Π·Π°Π²ΠΈΡΠΈΠΌΠΎΡΡ‚ΡŒΡŽ ΠΌΠ΅ΠΆΠ΄Ρƒ сопротивлСниСм ΠΏΡ€ΠΎΠ²ΠΎΠ΄Π½ΠΈΠΊΠ° ΠΈ Π΅Π³ΠΎ Ρ‚Π΅ΠΌΠΏΠ΅Ρ€Π°Ρ‚ΡƒΡ€ΠΎΠΉ

Π½Π°Ρ…ΠΎΠ΄ΠΈΠΌ Ρ‚Π΅ΠΌΠΏΠ΅Ρ€Π°Ρ‚ΡƒΡ€Ρƒ Π²ΠΎΠ·Π΄ΡƒΡ…Π° Π² ΠΊΠΎΠΌΠ½Π°Ρ‚Π΅:

ΠŸΡ€ΠΈ расчСтС ΠΈΡΠΏΠΎΠ»ΡŒΠ·ΡƒΠ΅Ρ‚ΡΡ Π·Π½Π°Ρ‡Π΅Π½ΠΈΠ΅ Ρ‚Π΅ΠΌΠΏΠ΅Ρ€Π°Ρ‚ΡƒΡ€Π½ΠΎΠ³ΠΎ коэффициСнта сопротивлСния Π± , взятоС ΠΈΠ· справочника. Π’ области ΠΊΠΎΠΌΠ½Π°Ρ‚Π½Ρ‹Ρ… Ρ‚Π΅ΠΌΠΏΠ΅Ρ€Π°Ρ‚ΡƒΡ€ для чистой ΠΌΠ΅Π΄ΠΈ Π± = 0,0043 Π³Ρ€Π°Π΄ - 1 . Если содСрТаниС примСсСй Π² ΠΌΠ΅Π΄ΠΈ, ΠΈΠ· ΠΊΠΎΡ‚ΠΎΡ€ΠΎΠΉ ΠΈΠ·Π³ΠΎΡ‚ΠΎΠ²Π»Π΅Π½Π° ΠΏΡ€ΠΎΠ²ΠΎΠ»ΠΎΠΊΠ°, Π½Π΅ особСнно Π²Π΅Π»ΠΈΠΊΠΎ, Π° ΡΠ»Π΅ΠΊΡ‚Ρ€ΠΎΠΈΠ·ΠΌΠ΅Ρ€ΠΈΡ‚Π΅Π»ΡŒΠ½Ρ‹Π΅ ΠΏΡ€ΠΈΠ±ΠΎΡ€Ρ‹ ΠΈΠΌΠ΅ΡŽΡ‚ класс точности 0,1, Ρ‚ΠΎ Ρ‚Π΅ΠΌΠΏΠ΅Ρ€Π°Ρ‚ΡƒΡ€Ρƒ Π²ΠΎΠ·Π΄ΡƒΡ…Π° ΠΌΠΎΠΆΠ½ΠΎ ΠΎΠΏΡ€Π΅Π΄Π΅Π»ΠΈΡ‚ΡŒ с ΠΏΠΎΠ³Ρ€Π΅ΡˆΠ½ΠΎΡΡ‚ΡŒΡŽ, Π·Π½Π°Ρ‡ΠΈΡ‚Π΅Π»ΡŒΠ½ΠΎ мСньшСй ΠΎΠ΄Π½ΠΎΠ³ΠΎ градуса.

ΠžΠΏΡ‚ΠΈΠΊΠ°

Π—Π°Π΄Π°Ρ‡Π° 13.

Π—Π°Π΄Π°Π½ΠΈΠ΅. ВрСбуСтся Π½Π°ΠΉΡ‚ΠΈ радиус сфСричСского Π·Π΅Ρ€ΠΊΠ°Π»Π° (ΠΈΠ»ΠΈ радиус ΠΊΡ€ΠΈΠ²ΠΈΠ·Π½Ρ‹ Π²ΠΎΠ³Π½ΡƒΡ‚ΠΎΠΉ Π»ΠΈΠ½Π·Ρ‹) с ΠΏΠΎΠΌΠΎΡ‰ΡŒΡŽ сСкундомСра ΠΈ ΡΡ‚Π°Π»ΡŒΠ½ΠΎΠ³ΠΎ ΡˆΠ°Ρ€ΠΈΠΊΠ° извСстного радиуса. Как это ΡΠ΄Π΅Π»Π°Ρ‚ΡŒ?

Подсказка. Π¦Π΅Π½Ρ‚Ρ€ ΠΊΠ°Ρ‚Π°ΡŽΡ‰Π΅Π³ΠΎΡΡ ΠΏΠΎ повСрхности Π·Π΅Ρ€ΠΊΠ°Π»Π° ΡˆΠ°Ρ€ΠΈΠΊΠ° ΡΠΎΠ²Π΅Ρ€ΡˆΠ°Π΅Ρ‚ Ρ‚Π°ΠΊΠΎΠ΅ ΠΆΠ΅ Π΄Π²ΠΈΠΆΠ΅Π½ΠΈΠ΅, ΠΊΠ°ΠΊ маятник.

РСшСниС. Π‘Π»Π΅Π΄ΡƒΠ΅Ρ‚ Ρ€Π°ΡΠΏΠΎΠ»ΠΎΠΆΠΈΡ‚ΡŒ Π·Π΅Ρ€ΠΊΠ°Π»ΠΎ Π³ΠΎΡ€ΠΈΠ·ΠΎΠ½Ρ‚Π°Π»ΡŒΠ½ΠΎ ΠΈ ΠΎΠΏΡƒΡΡ‚ΠΈΡ‚ΡŒ Π½Π° Π½Π΅Π³ΠΎ ΡˆΠ°Ρ€ΠΈΠΊ. Если ΡˆΠ°Ρ€ΠΈΠΊ ΠΎΠΏΡƒΡ‰Π΅Π½ Π½Π΅ Π² ΡΠ°ΠΌΡƒΡŽ ниТнюю Ρ‚ΠΎΡ‡ΠΊΡƒ, ΠΎΠ½ Π½Π°Ρ‡Π½Π΅Ρ‚ Π΄Π²ΠΈΠ³Π°Ρ‚ΡŒΡΡ ΠΏΠΎ повСрхности Π·Π΅Ρ€ΠΊΠ°Π»Π°. НСтрудно Π΄ΠΎΠ³Π°Π΄Π°Ρ‚ΡŒΡΡ, Ρ‡Ρ‚ΠΎ Ссли ΡˆΠ°Ρ€ΠΈΠΊ двиТСтся Π±Π΅Π· вращСния (Ρ‚.Π΅. ΡΠΊΠΎΠ»ΡŒΠ·ΠΈΡ‚ ΠΏΠΎ повСрхности Π·Π΅Ρ€ΠΊΠ°Π»Π°), Ρ‚ΠΎ Π΅Π³ΠΎ Π΄Π²ΠΈΠΆΠ΅Π½ΠΈΠ΅ ΠΏΠΎΠ»Π½ΠΎΡΡ‚ΡŒΡŽ Π°Π½Π°Π»ΠΎΠ³ΠΈΡ‡Π½ΠΎ двиТСнию маятника с Π΄Π»ΠΈΠ½ΠΎΠΉ подвСса R - r . Π’ΠΎΠ³Π΄Π° ΠΈΠ· Ρ„ΠΎΡ€ΠΌΡƒΠ»Ρ‹ маятника

ΠΌΠΎΠΆΠ½ΠΎ Π½Π°ΠΉΡ‚ΠΈ ΠΈΠ½Ρ‚Π΅Ρ€Π΅ΡΡƒΡŽΡ‰ΡƒΡŽ нас Π²Π΅Π»ΠΈΡ‡ΠΈΠ½Ρƒ:

ΠŸΠ΅Ρ€ΠΈΠΎΠ΄ Π’ опрСдСляСтся с ΠΏΠΎΠΌΠΎΡ‰ΡŒΡŽ сСкундомСра, Π° r извСстно ΠΏΠΎ ΡƒΡΠ»ΠΎΠ²ΠΈΡŽ.

ΠŸΠΎΡΠΊΠΎΠ»ΡŒΠΊΡƒ ΠΎΠ±Ρ‹Ρ‡Π½ΠΎ Ρ‚Ρ€Π΅Π½ΠΈΠ΅ достаточно Π²Π΅Π»ΠΈΠΊΠΎ, Ρ‡Ρ‚ΠΎΠ±Ρ‹ ΡˆΠ°Ρ€ΠΈΠΊ двигался ΠΏΠΎ повСрхности Π·Π΅Ρ€ΠΊΠ°Π»Π° с Π²Ρ€Π°Ρ‰Π΅Π½ΠΈΠ΅ΠΌ, это Ρ€Π΅ΡˆΠ΅Π½ΠΈΠ΅ ΠΏΠ»ΠΎΡ…ΠΎ согласуСтся с ΠΎΠΏΡ‹Ρ‚ΠΎΠΌ. На самом Π΄Π΅Π»Π΅

ΠŸΡ€ΠΈΠ²Π΅Π΄Π΅ΠΌ ΠΏΡ€ΠΈΠΌΠ΅Ρ€ ΠΈΡΡΠ»Π΅Π΄ΠΎΠ²Π°Ρ‚Π΅Π»ΡŒΡΠΊΠΎΠΉ Π·Π°Π΄Π°Ρ‡ΠΈ Π½Π° вСсь ΡƒΡ€ΠΎΠΊ.

Π—Π°Π΄Π°Ρ‡Π° 14. ΠžΡΠΎΠ±Π΅Π½Π½ΠΎΡΡ‚ΠΈ колСбания ΠΊΡ€ΡƒΡ‚ΠΈΠ»ΡŒΠ½ΠΎΠ³ΠΎ маятника.

Π—Π°Π΄Π°Π½ΠΈΠ΅. Π˜ΡΡΠ»Π΅Π΄ΡƒΠΉΡ‚Π΅ особСнности колСбания ΠΊΡ€ΡƒΡ‚ΠΈΠ»ΡŒΠ½ΠΎΠ³ΠΎ маятника ΠΈ ΠΎΠΏΠΈΡˆΠΈΡ‚Π΅ основныС закономСрности Π΅Π³ΠΎ двиТСния.

ΠžΠ±ΠΎΡ€ΡƒΠ΄ΠΎΠ²Π°Π½ΠΈΠ΅: ΡˆΡ‚Π°Ρ‚ΠΈΠ² с ΠΌΡƒΡ„Ρ‚ΠΎΠΉ ΠΈ Π»Π°ΠΏΠΊΠΎΠΉ, ΠΎΡ‚Ρ€Π΅Π·ΠΊΠΈ ΠΌΠ΅Π΄Π½ΠΎΠΉ, ΡΡ‚Π°Π»ΡŒΠ½ΠΎΠΉ ΠΈ Π½ΠΈΡ…Ρ€ΠΎΠΌΠΎΠ²ΠΎΠΉ ΠΏΡ€ΠΎΠ²ΠΎΠ»ΠΎΠΊΠΈ Π΄Π»ΠΈΠ½ΠΎΠΉ ΠΎΠΊΠΎΠ»ΠΎ 1 ΠΌ ΠΈ Ρ€Π°Π·Π»ΠΈΡ‡Π½Ρ‹Ρ… Π΄ΠΈΠ°ΠΌΠ΅Ρ‚Ρ€ΠΎΠ², Π½Π°ΠΏΡ€ΠΈΠΌΠ΅Ρ€ 0,3, 0,50, 0,65, 1,0 ΠΌΠΌ, тонкая лСгкая дСрСвянная ΠΏΠ°Π»ΠΎΡ‡ΠΊΠ° Π΄Π»ΠΈΠ½ΠΎΠΉ 15-20 см , пластилин, скрСпка, Π»ΠΈΠ½Π΅ΠΉΠΊΠ°, транспортир, сСкундомСр.

ΠžΠ±Ρ‰ΠΈΠΉ Π²ΠΈΠ΄ ΠΊΡ€ΡƒΡ‚ΠΈΠ»ΡŒΠ½ΠΎΠ³ΠΎ маятника Π΄ΠΎΠ»ΠΆΠ΅Π½ Π±Ρ‹Ρ‚ΡŒ Π² соотвСтствии с рисунком 12. Π‘ΠΊΡ€Π΅ΠΏΠΊΠ°, изогнутая ΠΎΠΏΡ€Π΅Π΄Π΅Π»Π΅Π½Π½Ρ‹ΠΌ ΠΎΠ±Ρ€Π°Π·ΠΎΠΌ, слуТит для ΡƒΡ€Π°Π²Π½ΠΎΠ²Π΅ΡˆΠΈΠ²Π°Π½ΠΈΡ стСрТня с Π³Ρ€ΡƒΠ·Π°ΠΌΠΈ. Π’Ρ‹Π²Π΅Π΄Π΅Π½Π½Ρ‹ΠΉ ΠΈΠ· состояния равновСсия маятник Π½Π°Ρ‡ΠΈΠ½Π°Π΅Ρ‚ ΡΠΎΠ²Π΅Ρ€ΡˆΠ°Ρ‚ΡŒ Π²Ρ€Π°Ρ‰Π°Ρ‚Π΅Π»ΡŒΠ½ΠΎ-ΠΊΠΎΠ»Π΅Π±Π°Ρ‚Π΅Π»ΡŒΠ½ΠΎΠ΅ Π΄Π²ΠΈΠΆΠ΅Π½ΠΈΠ΅.

Π—Π°Ρ€Π°Π½Π΅Π΅ Π½ΡƒΠΆΠ½ΠΎ ΠΈΠ·Π³ΠΎΡ‚ΠΎΠ²ΠΈΡ‚ΡŒ ΠΈΠ· пластилина ΠΏΠ°Ρ€Ρ‹ ΡˆΠ°Ρ€ΠΈΠΊΠΎΠ² Ρ€Π°Π·Π½ΠΎΠΉ массы. ΠœΠ°ΡΡΡ‹ ΡˆΠ°Ρ€ΠΈΠΊΠΎΠ² ΠΏΡ€ΠΎΠΏΠΎΡ€Ρ†ΠΈΠΎΠ½Π°Π»ΡŒΠ½Ρ‹ ΠΊΡƒΠ±Ρƒ ΠΈΡ… Π΄ΠΈΠ°ΠΌΠ΅Ρ‚Ρ€ΠΎΠ², поэтому Π΅ΡΡ‚ΡŒ Π²ΠΎΠ·ΠΌΠΎΠΆΠ½ΠΎΡΡ‚ΡŒ Π²Ρ‹ΡΡ‚Ρ€ΠΎΠΈΡ‚ΡŒ ряд, Π½Π°ΠΏΡ€ΠΈΠΌΠ΅Ρ€: m 1 = 1, m 2 = 2,5, m 3 = 5,2, m 3 = 6,8, m 4 = 8,3 ΠΎΡ‚Π½. Π΅Π΄.

Π”ΠΈΠ°ΠΌΠ΅Ρ‚Ρ€ ΠΏΡ€ΠΎΠ²ΠΎΠ»ΠΎΠΊ ΠΌΠΎΠΆΠ½ΠΎ ΡΠΎΠΎΠ±Ρ‰ΠΈΡ‚ΡŒ учащимся Π·Π°Ρ€Π°Π½Π΅Π΅ ΠΈΠ»ΠΈ ΠΏΡ€Π΅Π΄ΠΎΡΡ‚Π°Π²ΠΈΡ‚ΡŒ ΠΈΠΌ Π²ΠΎΠ·ΠΌΠΎΠΆΠ½ΠΎΡΡ‚ΡŒ провСсти эти измСрСния ΡΠ°ΠΌΠΎΡΡ‚ΠΎΡΡ‚Π΅Π»ΡŒΠ½ΠΎ с ΠΏΠΎΠΌΠΎΡ‰ΡŒΡŽ ΡˆΡ‚Π°Π½Π³Π΅Π½Ρ†ΠΈΡ€ΠΊΡƒΠ»Ρ ΠΈΠ»ΠΈ ΠΌΠΈΠΊΡ€ΠΎΠΌΠ΅Ρ‚Ρ€Π°.

ΠŸΡ€ΠΈΠΌΠ΅Ρ‡Π°Π½ΠΈΠ΅. УспСх исслСдованиС Π²ΠΎ ΠΌΠ½ΠΎΠ³ΠΎΠΌ зависит ΠΎΡ‚ ΠΏΡ€Π°Π²ΠΈΠ»ΡŒΠ½ΠΎΠ³ΠΎ ΠΏΠΎΠ΄Π±ΠΎΡ€Π° оборудования, особСнно Π΄ΠΈΠ°ΠΌΠ΅Ρ‚Ρ€ΠΎΠ² Π²Ρ‹Π΄Π°Π½Π½Ρ‹Ρ… ΠΏΡ€ΠΎΠ²ΠΎΠ»ΠΎΠΊ. ΠšΡ€ΠΎΠΌΠ΅ Ρ‚ΠΎΠ³ΠΎ, ΠΆΠ΅Π»Π°Ρ‚Π΅Π»ΡŒΠ½ΠΎ, Ρ‡Ρ‚ΠΎΠ±Ρ‹ подвСс ΠΊΡ€ΡƒΡ‚ΠΈΠ»ΡŒΠ½ΠΎΠ³ΠΎ маятника находился Π²ΠΎ врСмя ΠΎΠΏΡ‹Ρ‚ΠΎΠ² Π² натянутом состоянии, для Ρ‡Π΅Π³ΠΎ массы Π³Ρ€ΡƒΠ·ΠΎΠ² Π΄ΠΎΠ»ΠΆΠ½Ρ‹ Π±Ρ‹Ρ‚ΡŒ достаточно большими.

Π’Π΅ΠΌΠ°Ρ‚ΠΈΠΊΠ° исслСдования ΠΊΡ€ΡƒΡ‚ΠΈΠ»ΡŒΠ½ΠΎΠ³ΠΎ маятника Π²Ρ‹Ρ‚Π΅ΠΊΠ°Π΅Ρ‚ ΠΈΠ· прСдполоТСния ΠΎ гармоничСском Ρ…Π°Ρ€Π°ΠΊΡ‚Π΅Ρ€Π΅ Π΅Π³ΠΎ ΠΊΠΎΠ»Π΅Π±Π°Π½ΠΈΠΉ. ΠžΠ±Ρ‰ΠΈΠΉ ΠΏΠ΅Ρ€Π΅Ρ‡Π΅Π½ΡŒ ΡΠΊΡΠΏΠ΅Ρ€ΠΈΠΌΠ΅Π½Ρ‚Π°Π»ΡŒΠ½Ρ‹Ρ… наблюдСний, ΠΊΠΎΡ‚ΠΎΡ€Ρ‹Π΅ ΠΌΠΎΠΆΠ½ΠΎ ΠΎΡΡƒΡ‰Π΅ΡΡ‚Π²ΠΈΡ‚ΡŒ ΠΏΠΎ Π΄Π°Π½Π½ΠΎΠΉ ΠΏΡ€ΠΎΠ±Π»Π΅ΠΌΠ΅ ΠΈ Π½Π° ΠΏΡ€Π΅Π΄Π»ΠΎΠΆΠ΅Π½Π½ΠΎΠΌ ΠΎΠ±ΠΎΡ€ΡƒΠ΄ΠΎΠ²Π°Π½ΠΈΠΈ, достаточно Π²Π΅Π»ΠΈΠΊ. ΠŸΡ€ΠΈΠ²Π΅Π΄Π΅ΠΌ Π½Π°ΠΈΠ±ΠΎΠ»Π΅Π΅ простыС ΠΈ доступныС.

  • - Зависит Π»ΠΈ ΠΏΠ΅Ρ€ΠΈΠΎΠ΄ ΠΊΠΎΠ»Π΅Π±Π°Π½ΠΈΠΉ ΠΎΡ‚ Π°ΠΌΠΏΠ»ΠΈΡ‚ΡƒΠ΄Ρ‹ (ΡƒΠ³Π»Π° ΠΏΠΎΠ²ΠΎΡ€ΠΎΡ‚Π°)?
  • - Зависит Π»ΠΈ ΠΏΠ΅Ρ€ΠΈΠΎΠ΄ ΠΊΠΎΠ»Π΅Π±Π°Π½ΠΈΠΉ ΠΎΡ‚ Π΄Π»ΠΈΠ½Ρ‹ подвСса маятника?
  • - Зависит Π»ΠΈ ΠΏΠ΅Ρ€ΠΈΠΎΠ΄ ΠΊΠΎΠ»Π΅Π±Π°Π½ΠΈΠΉ маятника ΠΎΡ‚ массы Π³Ρ€ΡƒΠ·ΠΎΠ²?
  • - Зависит Π»ΠΈ ΠΏΠ΅Ρ€ΠΈΠΎΠ΄ ΠΊΠΎΠ»Π΅Π±Π°Π½ΠΈΠΉ маятника ΠΎΡ‚ полоТСния Π³Ρ€ΡƒΠ·ΠΎΠ² Π½Π° стСрТнС?
  • - Зависит Π»ΠΈ ΠΏΠ΅Ρ€ΠΈΠΎΠ΄ ΠΊΠΎΠ»Π΅Π±Π°Π½ΠΈΠΉ ΠΎΡ‚ Π΄ΠΈΠ°ΠΌΠ΅Ρ‚Ρ€Π° ΠΏΡ€ΠΎΠ²ΠΎΠ»ΠΎΠΊΠΈ?

ЕстСствСнно, трСбуСтся Π½Π΅ просто однослоТно ΠΎΡ‚Π²Π΅Ρ‡Π°Ρ‚ΡŒ Π½Π° поставлСнныС вопросы, Π½ΠΎ ΠΈ ΠΈΡΡΠ»Π΅Π΄ΠΎΠ²Π°Ρ‚ΡŒ Ρ…Π°Ρ€Π°ΠΊΡ‚Π΅Ρ€ ΠΎΠΆΠΈΠ΄Π°Π΅ΠΌΡ‹Ρ… зависимостСй.

ΠŸΠΎΠ»ΡŒΠ·ΡƒΡΡΡŒ ΠΏΡ€ΠΈΡ‘ΠΌΠΎΠΌ Π°Π½Π°Π»ΠΎΠ³ΠΈΠΉ, Π²Ρ‹Π΄Π²ΠΈΠ³Π°Π΅ΠΌ Π³ΠΈΠΏΠΎΡ‚Π΅Π·Ρ‹ ΠΎ колСбаниях ΠΊΡ€ΡƒΡ‚ΠΈΠ»ΡŒΠ½ΠΎΠ³ΠΎ маятника, сравнивая Π΅Π³ΠΎ с матСматичСским маятником, ΠΈΠ·ΡƒΡ‡Π°Π΅ΠΌΡ‹ΠΌ ΠΏΠΎ школьной ΠΏΡ€ΠΎΠ³Ρ€Π°ΠΌΠΌΠ΅. Π—Π° основу Π±Π΅Ρ€Ρ‘ΠΌ ΠΏΠ΅Ρ€ΠΈΠΎΠ΄ ΠΊΠΎΠ»Π΅Π±Π°Π½ΠΈΠΉ ΠΈ Π΅Π³ΠΎ Π·Π°Π²ΠΈΡΠΈΠΌΠΎΡΡ‚ΡŒ ΠΎΡ‚ Ρ€Π°Π·Π»ΠΈΡ‡Π½Ρ‹Ρ… ΠΏΠ°Ρ€Π°ΠΌΠ΅Ρ‚Ρ€ΠΎΠ² маятника. НамСчаСм ΡΠ»Π΅Π΄ΡƒΡŽΡ‰ΠΈΠ΅ Π³ΠΈΠΏΠΎΡ‚Π΅Π·Ρ‹. ΠŸΠ΅Ρ€ΠΈΠΎΠ΄ ΠΊΠΎΠ»Π΅Π±Π°Π½ΠΈΠΉ ΠΊΡ€ΡƒΡ‚ΠΈΠ»ΡŒΠ½ΠΎΠ³ΠΎ маятника:

ΠŸΡ€ΠΈ ΠΌΠ°Π»Ρ‹Ρ… ΡƒΠ³Π»Π°Ρ… ΠΏΠΎΠ²ΠΎΡ€ΠΎΡ‚Π° Π½Π΅ зависит ΠΎΡ‚ Π°ΠΌΠΏΠ»ΠΈΡ‚ΡƒΠ΄Ρ‹;

  • - ΠΏΡ€ΠΎΠΏΠΎΡ€Ρ†ΠΈΠΎΠ½Π°Π»Π΅Π½ ΠΊΠΎΡ€Π½ΡŽ ΠΊΠ²Π°Π΄Ρ€Π°Ρ‚Π½ΠΎΠΌΡƒ ΠΈΠ· Π΄Π»ΠΈΠ½Ρ‹ подвСса - T ;
  • - ΠΏΡ€ΠΎΠΏΠΎΡ€Ρ†ΠΈΠΎΠ½Π°Π»Π΅Π½ ΠΊΠΎΡ€Π½ΡŽ ΠΊΠ²Π°Π΄Ρ€Π°Ρ‚Π½ΠΎΠΌΡƒ ΠΈΠ· массы Π³Ρ€ΡƒΠ·Π° - T ;
  • - ΠΏΡ€ΠΎΠΏΠΎΡ€Ρ†ΠΈΠΎΠ½Π°Π»Π΅Π½ Ρ€Π°ΡΡΡ‚ΠΎΡΠ½ΠΈΡŽ ΠΎΡ‚ Ρ†Π΅Π½Ρ‚Ρ€Π° подвСса Π΄ΠΎ Ρ†Π΅Π½Ρ‚Ρ€ΠΎΠ² Π³Ρ€ΡƒΠ·ΠΎΠ² - Tr ;
  • - ΠΎΠ±Ρ€Π°Ρ‚Π½ΠΎ ΠΏΡ€ΠΎΠΏΠΎΡ€Ρ†ΠΈΠΎΠ½Π°Π»Π΅Π½ ΠΊΠ²Π°Π΄Ρ€Π°Ρ‚Ρƒ Π΄ΠΈΠ°ΠΌΠ΅Ρ‚Ρ€Π° ΠΏΡ€ΠΎΠ²ΠΎΠ»ΠΎΠΊΠΈ - T1/d 2 .

ΠšΡ€ΠΎΠΌΠ΅ Ρ‚ΠΎΠ³ΠΎ, ΠΏΠ΅Ρ€ΠΈΠΎΠ΄ ΠΊΠΎΠ»Π΅Π±Π°Π½ΠΈΠΉ зависит ΠΎΡ‚ ΠΌΠ°Ρ‚Π΅Ρ€ΠΈΠ°Π»Π° подвСса: мСдь, ΡΡ‚Π°Π»ΡŒ, Π½ΠΈΡ…Ρ€ΠΎΠΌ. Π—Π΄Π΅ΡΡŒ Ρ‚Π°ΠΊΠΆΠ΅ имССтся ряд Π³ΠΈΠΏΠΎΡ‚Π΅Π·, ΠΏΡ€Π΅Π΄Π»Π°Π³Π°Π΅ΠΌ ΠΏΡ€ΠΎΠ²Π΅Ρ€ΠΈΡ‚ΡŒ ΠΈΡ… ΡΠ°ΠΌΠΎΡΡ‚ΠΎΡΡ‚Π΅Π»ΡŒΠ½ΠΎ.

1. Π˜Π·ΡƒΡ‡Π°Π΅ΠΌ Π·Π°Π²ΠΈΡΠΈΠΌΠΎΡΡ‚ΡŒ ΠΏΠ΅Ρ€ΠΈΠΎΠ΄Π° ΠΊΠΎΠ»Π΅Π±Π°Π½ΠΈΠΉ маятника ΠΎΡ‚ Π°ΠΌΠΏΠ»ΠΈΡ‚ΡƒΠ΄Ρ‹ (ΡƒΠ³Π»Π° ΠΏΠΎΠ²ΠΎΡ€ΠΎΡ‚Π°). Π Π΅Π·ΡƒΠ»ΡŒΡ‚Π°Ρ‚Ρ‹ ΠΈΠ·ΠΌΠ΅Ρ€Π΅Π½ΠΈΠΉ прСдставлСны Π² Ρ‚Π°Π±Π»ΠΈΡ†Π΅ 3:

Π’Π°Π±Π»ΠΈΡ†Π° 3

Π—Π°Π²ΠΈΡΠΈΠΌΠΎΡΡ‚ΡŒ ΠΏΠ΅Ρ€ΠΈΠΎΠ΄Π° ΠΊΠΎΠ»Π΅Π±Π°Π½ΠΈΠΉ маятника ΠΎΡ‚ Π°ΠΌΠΏΠ»ΠΈΡ‚ΡƒΠ΄Ρ‹

L = 60 см, m = 8,3 г, r = 12 см, d = 0,5 мм

Π’Ρ‹Π²ΠΎΠ΄ . Π’ ΠΏΡ€Π΅Π΄Π΅Π»Π°Ρ… Π΄ΠΎ 180 Π·Π°Π²ΠΈΡΠΈΠΌΠΎΡΡ‚ΡŒ ΠΏΠ΅Ρ€ΠΈΠΎΠ΄Π° ΠΊΠΎΠ»Π΅Π±Π°Π½ΠΈΠΉ ΠΊΡ€ΡƒΡ‚ΠΈΠ»ΡŒΠ½ΠΎΠ³ΠΎ маятника ΠΎΡ‚ Π°ΠΌΠΏΠ»ΠΈΡ‚ΡƒΠ΄Ρ‹ Π½Π΅ обнаруТиваСтся. Разброс Ρ€Π΅Π·ΡƒΠ»ΡŒΡ‚Π°Ρ‚ΠΎΠ² ΠΈΠ·ΠΌΠ΅Ρ€Π΅Π½ΠΈΠΉ ΠΌΠΎΠΆΠ½ΠΎ ΠΎΠ±ΡŠΡΡΠ½ΠΈΡ‚ΡŒ ΠΏΠΎΠ³Ρ€Π΅ΡˆΠ½ΠΎΡΡ‚ΡΠΌΠΈ измСрСния ΠΏΠ΅Ρ€ΠΈΠΎΠ΄Π° ΠΊΠΎΠ»Π΅Π±Π°Π½ΠΈΠΉ ΠΈ случайными ΠΏΡ€ΠΈΡ‡ΠΈΠ½Π°ΠΌΠΈ.

Π§Ρ‚ΠΎΠ±Ρ‹ Β«ΠΎΡ‚ΠΊΡ€Ρ‹Ρ‚ΡŒΒ» Π΄Ρ€ΡƒΠ³ΠΈΠ΅ зависимости Π½Π΅ΠΎΠ±Ρ…ΠΎΠ΄ΠΈΠΌΠΎ ΠΌΠ΅Π½ΡΡ‚ΡŒ Ρ‚ΠΎΠ»ΡŒΠΊΠΎ ΠΎΠ΄ΠΈΠ½ ΠΏΠ°Ρ€Π°ΠΌΠ΅Ρ‚Ρ€, оставляя всС Π΄Ρ€ΡƒΠ³ΠΈΠ΅ Π½Π΅ΠΈΠ·ΠΌΠ΅Π½Π½Ρ‹ΠΌΠΈ. ΠœΠ°Ρ‚Π΅ΠΌΠ°Ρ‚ΠΈΡ‡Π΅ΡΠΊΡƒΡŽ ΠΎΠ±Ρ€Π°Π±ΠΎΡ‚ΠΊΡƒ Ρ€Π΅Π·ΡƒΠ»ΡŒΡ‚Π°Ρ‚ΠΎΠ² Π»ΡƒΡ‡ΡˆΠ΅ всСго ΠΏΡ€ΠΎΠ²ΠΎΠ΄ΠΈΡ‚ΡŒ графичСски.

2. Π˜Π·ΡƒΡ‡Π°Π΅ΠΌ Π·Π°Π²ΠΈΡΠΈΠΌΠΎΡΡ‚ΡŒ ΠΏΠ΅Ρ€ΠΈΠΎΠ΄Π° ΠΊΠΎΠ»Π΅Π±Π°Π½ΠΈΠΉ маятника ΠΎΡ‚ Π΅Π³ΠΎ Π΄Π»ΠΈΠ½Ρ‹: Π’ = f(l). ΠŸΡ€ΠΈ этом Π½Π΅ мСняСм m, r, d. Π Π΅Π·ΡƒΠ»ΡŒΡ‚Π°Ρ‚Ρ‹ ΠΈΠ·ΠΌΠ΅Ρ€Π΅Π½ΠΈΠΉ прСдставлСны Π² Ρ‚Π°Π±Π»ΠΈΡ†Π΅ 4:

Π’Π°Π±Π»ΠΈΡ†Π° 4

Π—Π°Π²ΠΈΡΠΈΠΌΠΎΡΡ‚ΡŒ ΠΏΠ΅Ρ€ΠΈΠΎΠ΄Π° ΠΊΠΎΠ»Π΅Π±Π°Π½ΠΈΠΉ маятника ΠΎΡ‚ Π΄Π»ΠΈΠ½Ρ‹

m = 8,3 ΠΎΡ‚Π½. Π΅Π΄., r = 12 см, d = 0,5 ΠΌΠΌ

Π“Ρ€Π°Ρ„ΠΈΠΊ зависимости Π’ ΠΎΡ‚ l прСдставляСт собой ΠΊΡ€ΠΈΠ²ΡƒΡŽ Π²ΠΎΠ·Ρ€Π°ΡΡ‚Π°ΡŽΡ‰ΡƒΡŽ линию, ΠΏΠΎΡ…ΠΎΠΆΡƒΡŽ Π½Π° Π·Π°Π²ΠΈΡΠΈΠΌΠΎΡΡ‚ΡŒ, Π² соотвСтствии с рисунком 13Π° T 2 = l , Π² соотвСтствии с рисунком 13, Π± .

Π’Ρ‹Π²ΠΎΠ΄. ΠŸΠ΅Ρ€ΠΈΠΎΠ΄ ΠΊΠΎΠ»Π΅Π±Π°Π½ΠΈΠΉ ΠΊΡ€ΡƒΡ‚ΠΈΠ»ΡŒΠ½ΠΎΠ³ΠΎ маятника прямо ΠΏΡ€ΠΎΠΏΠΎΡ€Ρ†ΠΈΠΎΠ½Π°Π»Π΅Π½ ΠΊΠΎΡ€Π½ΡŽ ΠΊΠ²Π°Π΄Ρ€Π°Ρ‚Π½ΠΎΠΌΡƒ ΠΈΠ· Π΄Π»ΠΈΠ½Ρ‹ подвСса. НСкоторый разброс Ρ‚ΠΎΡ‡Π΅ΠΊ ΠΌΠΎΠΆΠ½ΠΎ ΠΎΠ±ΡŠΡΡΠ½ΠΈΡ‚ΡŒ ΠΏΠΎΠ³Ρ€Π΅ΡˆΠ½ΠΎΡΡ‚ΡΠΌΠΈ ΠΈΠ·ΠΌΠ΅Ρ€Π΅Π½ΠΈΠΉ ΠΏΠ΅Ρ€ΠΈΠΎΠ΄Π° ΠΊΠΎΠ»Π΅Π±Π°Π½ΠΈΠΉ ΠΈ Π΄Π»ΠΈΠ½Ρ‹ маятника

3. Π˜Π·ΡƒΡ‡Π°Π΅ΠΌ Π·Π°Π²ΠΈΡΠΈΠΌΠΎΡΡ‚ΡŒ ΠΏΠ΅Ρ€ΠΈΠΎΠ΄Π° ΠΊΠΎΠ»Π΅Π±Π°Π½ΠΈΠΉ маятника ΠΎΡ‚ массы Π³Ρ€ΡƒΠ·ΠΎΠ²: Π’=f(m). ΠŸΡ€ΠΈ этом Π½Π΅ мСняСм l, r, d. Π Π΅Π·ΡƒΠ»ΡŒΡ‚Π°Ρ‚Ρ‹ ΠΈΠ·ΠΌΠ΅Ρ€Π΅Π½ΠΈΠΉ прСдставлСны Π² Ρ‚Π°Π±Π»ΠΈΡ†Π΅ 5:

Π’Π°Π±Π»ΠΈΡ†Π° 5

Π—Π°Π²ΠΈΡΠΈΠΌΠΎΡΡ‚ΡŒ ΠΏΠ΅Ρ€ΠΈΠΎΠ΄Π° ΠΊΠΎΠ»Π΅Π±Π°Π½ΠΈΠΉ маятника ΠΎΡ‚ массы Π³Ρ€ΡƒΠ·ΠΎΠ²

l = 0,6 м, r = 12 см, d = 0,5 мм

Π“Ρ€Π°Ρ„ΠΈΠΊ зависимости Π’ ΠΎΡ‚ m прСдставляСт собой ΠΊΡ€ΠΈΠ²ΡƒΡŽ Π²ΠΎΠ·Ρ€Π°ΡΡ‚Π°ΡŽΡ‰ΡƒΡŽ линию, ΠΏΠΎΡ…ΠΎΠΆΡƒΡŽ Π½Π° Π·Π°Π²ΠΈΡΠΈΠΌΠΎΡΡ‚ΡŒ, Π² соотвСтствии с рисунком 14Π° . Π§Ρ‚ΠΎΠ±Ρ‹ ΡƒΠ±Π΅Π΄ΠΈΡ‚ΡŒΡΡ Π² этом, строим Π·Π°Π²ΠΈΡΠΈΠΌΠΎΡΡ‚ΡŒ T 2 =f(m) , Π² соотвСтствии с рисунком 14Π± .

Π’Ρ‹Π²ΠΎΠ΄. ΠŸΠ΅Ρ€ΠΈΠΎΠ΄ ΠΊΠΎΠ»Π΅Π±Π°Π½ΠΈΠΉ ΠΊΡ€ΡƒΡ‚ΠΈΠ»ΡŒΠ½ΠΎΠ³ΠΎ маятника прямо ΠΏΡ€ΠΎΠΏΠΎΡ€Ρ†ΠΈΠΎΠ½Π°Π»Π΅Π½ ΠΊΠΎΡ€Π½ΡŽ ΠΊΠ²Π°Π΄Ρ€Π°Ρ‚Π½ΠΎΠΌΡƒ ΠΈΠ· массы Π³Ρ€ΡƒΠ·ΠΎΠ². НСкоторый разброс Ρ‚ΠΎΡ‡Π΅ΠΊ ΠΌΠΎΠΆΠ½ΠΎ ΠΎΠ±ΡŠΡΡΠ½ΠΈΡ‚ΡŒ ΠΏΠΎΠ³Ρ€Π΅ΡˆΠ½ΠΎΡΡ‚ΡΠΌΠΈ ΠΈΠ·ΠΌΠ΅Ρ€Π΅Π½ΠΈΠΉ ΠΏΠ΅Ρ€ΠΈΠΎΠ΄Π° ΠΊΠΎΠ»Π΅Π±Π°Π½ΠΈΠΉ ΠΈ масс Π³Ρ€ΡƒΠ·ΠΎΠ², Π° Ρ‚Π°ΠΊΠΆΠ΅ случайными ΠΏΡ€ΠΈΡ‡ΠΈΠ½Π°ΠΌΠΈ.

4. Π˜Π·ΡƒΡ‡Π°Π΅ΠΌ Π·Π°Π²ΠΈΡΠΈΠΌΠΎΡΡ‚ΡŒ ΠΏΠ΅Ρ€ΠΈΠΎΠ΄Π° ΠΊΠΎΠ»Π΅Π±Π°Π½ΠΈΠΉ маятника ΠΎΡ‚ полоТСния Π³Ρ€ΡƒΠ·ΠΎΠ²: Π’ = f(r). ΠŸΡ€ΠΈ этом Π½Π΅ мСняСм l, m, d. Π Π΅Π·ΡƒΠ»ΡŒΡ‚Π°Ρ‚Ρ‹ ΠΈΠ·ΠΌΠ΅Ρ€Π΅Π½ΠΈΠΉ прСдставлСны Π² Ρ‚Π°Π±Π»ΠΈΡ†Π΅ 6:

Π’Π°Π±Π»ΠΈΡ†Π° 6

Π—Π°Π²ΠΈΡΠΈΠΌΠΎΡΡ‚ΡŒ ΠΏΠ΅Ρ€ΠΈΠΎΠ΄Π° ΠΊΠΎΠ»Π΅Π±Π°Π½ΠΈΠΉ маятника ΠΎΡ‚ полоТСния Π³Ρ€ΡƒΠ·ΠΎΠ²

m = 8,3 ΠΎΡ‚Π½.Π΅Π΄., l = 0,6 ΠΌ, d = 0,5 ΠΌΠΌ

Π’Ρ‹Π²ΠΎΠ΄. ΠŸΠ΅Ρ€ΠΈΠΎΠ΄ ΠΊΠΎΠ»Π΅Π±Π°Π½ΠΈΠΉ ΠΊΡ€ΡƒΡ‚ΠΈΠ»ΡŒΠ½ΠΎΠ³ΠΎ маятника прямо ΠΏΡ€ΠΎΠΏΠΎΡ€Ρ†ΠΈΠΎΠ½Π°Π»Π΅Π½ Ρ€Π°ΡΡΡ‚ΠΎΡΠ½ΠΈΡŽ r . НСкоторый разброс Ρ‚ΠΎΡ‡Π΅ΠΊ ΠΌΠΎΠΆΠ½ΠΎ ΠΎΠ±ΡŠΡΡΠ½ΠΈΡ‚ΡŒ ΠΏΠΎΠ³Ρ€Π΅ΡˆΠ½ΠΎΡΡ‚ΡΠΌΠΈ ΠΈΠ·ΠΌΠ΅Ρ€Π΅Π½ΠΈΠΉ ΠΏΠ΅Ρ€ΠΈΠΎΠ΄Π° ΠΊΠΎΠ»Π΅Π±Π°Π½ΠΈΠΉ ΠΈ расстояния r , Π° Ρ‚Π°ΠΊΠΆΠ΅ случайными ΠΏΡ€ΠΈΡ‡ΠΈΠ½Π°ΠΌΠΈ.

Π˜Π·ΡƒΡ‡Π°Π΅ΠΌ Π·Π°Π²ΠΈΡΠΈΠΌΠΎΡΡ‚ΡŒ ΠΏΠ΅Ρ€ΠΈΠΎΠ΄Π° ΠΊΠΎΠ»Π΅Π±Π°Π½ΠΈΠΉ маятника ΠΎΡ‚ Π΄ΠΈΠ°ΠΌΠ΅Ρ‚Ρ€Π° ΠΏΡ€ΠΎΠ²ΠΎΠ»ΠΎΠΊΠΈ: Π’ = f(d) , Π² соотвСтствии с рисунком 15. ΠŸΡ€ΠΈ этом Π½Π΅ мСняСм m, r, l .

Π Π΅Π·ΡƒΠ»ΡŒΡ‚Π°Ρ‚Ρ‹ ΠΈΠ·ΠΌΠ΅Ρ€Π΅Π½ΠΈΠΉ прСдставлСны Π² Ρ‚Π°Π±Π»ΠΈΡ†Π΅ 7.

Π’Π°Π±Π»ΠΈΡ†Π° 7

Π—Π°Π²ΠΈΡΠΈΠΌΠΎΡΡ‚ΡŒ ΠΏΠ΅Ρ€ΠΈΠΎΠ΄Π° ΠΊΠΎΠ»Π΅Π±Π°Π½ΠΈΠΉ маятника ΠΎΡ‚ Π΄ΠΈΠ°ΠΌΠ΅Ρ‚Ρ€Π° ΠΏΡ€ΠΎΠ²ΠΎΠ»ΠΎΠΊΠΈ

m = 8,3 ΠΎΡ‚Π½.Π΅Π΄., r = 12 см, l = 0,6 ΠΌ

Π“Ρ€Π°Ρ„ΠΈΠΊ зависимости Π’ ΠΎΡ‚ d прСдставляСт собой Π½ΠΈΡΠΏΠ°Π΄Π°ΡŽΡ‰ΡƒΡŽ ΠΊΡ€ΠΈΠ²ΡƒΡŽ, Π² соотвСтствии с рисунком 16Π° . МоТно ΠΏΡ€Π΅Π΄ΠΏΠΎΠ»ΠΎΠΆΠΈΡ‚ΡŒ, Ρ‡Ρ‚ΠΎ это Π·Π°Π²ΠΈΡΠΈΠΌΠΎΡΡ‚ΡŒ, Π³Π΄Π΅ n = 1, 2, 3 ΠΈ Ρ‚.Π΄. Для ΠΏΡ€ΠΎΠ²Π΅Ρ€ΠΊΠΈ этих ΠΏΡ€Π΅Π΄ΠΏΠΎΠ»ΠΎΠΆΠ΅Π½ΠΈΠΉ Π½Π΅ΠΎΠ±Ρ…ΠΎΠ΄ΠΈΠΌΠΎ ΡΡ‚Ρ€ΠΎΠΈΡ‚ΡŒ Π³Ρ€Π°Ρ„ΠΈΠΊΠΈ ΠΈ Ρ‚. Π΄. Из всСх Ρ‚Π°ΠΊΠΈΡ… Π³Ρ€Π°Ρ„ΠΈΠΊΠΎΠ² Π½Π°ΠΈΠ±ΠΎΠ»Π΅Π΅ Π»ΠΈΠ½Π΅ΠΉΠ½Ρ‹ΠΌ являСтся Π³Ρ€Π°Ρ„ΠΈΠΊ, Π² соотвСтствии с рисунком 16Π± .

Π’Ρ‹Π²ΠΎΠ΄. ΠŸΠ΅Ρ€ΠΈΠΎΠ΄ ΠΊΠΎΠ»Π΅Π±Π°Π½ΠΈΠΉ ΠΊΡ€ΡƒΡ‚ΠΈΠ»ΡŒΠ½ΠΎΠ³ΠΎ маятника ΠΎΠ±Ρ€Π°Ρ‚Π½ΠΎ ΠΏΡ€ΠΎΠΏΠΎΡ€Ρ†ΠΈΠΎΠ½Π°Π»Π΅Π½ ΠΊΠ²Π°Π΄Ρ€Π°Ρ‚Ρƒ Π΄ΠΈΠ°ΠΌΠ΅Ρ‚Ρ€Π° ΠΏΡ€ΠΎΠ²ΠΎΠ»ΠΎΠΊΠΈ подвСса. НСкоторый разброс Ρ‚ΠΎΡ‡Π΅ΠΊ ΠΌΠΎΠΆΠ½ΠΎ ΠΎΠ±ΡŠΡΡΠ½ΠΈΡ‚ΡŒ ΠΏΠΎΠ³Ρ€Π΅ΡˆΠ½ΠΎΡΡ‚ΡΠΌΠΈ ΠΈΠ·ΠΌΠ΅Ρ€Π΅Π½ΠΈΠΉ ΠΏΠ΅Ρ€ΠΈΠΎΠ΄Π° ΠΊΠΎΠ»Π΅Π±Π°Π½ΠΈΠΉ ΠΈ Π΄ΠΈΠ°ΠΌΠ΅Ρ‚Ρ€Π° ΠΏΡ€ΠΎΠ²ΠΎΠ»ΠΎΠΊΠΈ d , Π° Ρ‚Π°ΠΊΠΆΠ΅ случайными ΠΏΡ€ΠΈΡ‡ΠΈΠ½Π°ΠΌΠΈ.

ΠŸΡ€ΠΎΠ²Π΅Π΄Π΅Π½Π½Ρ‹Π΅ исслСдования ΠΏΠΎΠ·Π²ΠΎΠ»ΡΡŽΡ‚ ΡΠ΄Π΅Π»Π°Ρ‚ΡŒ Π²Ρ‹Π²ΠΎΠ΄ ΠΎ Ρ‚ΠΎΠΌ, Ρ‡Ρ‚ΠΎ ΠΏΠ΅Ρ€ΠΈΠΎΠ΄ ΠΊΠΎΠ»Π΅Π±Π°Π½ΠΈΠΉ ΠΊΡ€ΡƒΡ‚ΠΈΠ»ΡŒΠ½ΠΎΠ³ΠΎ маятника Π΄ΠΎΠ»ΠΆΠ΅Π½ Π²Ρ‹Ρ‡ΠΈΡΠ»ΡΡ‚ΡŒΡΡ ΠΏΠΎ Ρ„ΠΎΡ€ΠΌΡƒΠ»Π΅, Π³Π΄Π΅ k - коэффициСнт ΠΏΡ€ΠΎΠΏΠΎΡ€Ρ†ΠΈΠΎΠ½Π°Π»ΡŒΠ½ΠΎΡΡ‚ΠΈ, зависящий Ρ‚Π°ΠΊΠΆΠ΅ ΠΎΡ‚ ΡƒΠΏΡ€ΡƒΠ³ΠΈΡ… свойств ΠΌΠ°Ρ‚Π΅Ρ€ΠΈΠ°Π»Π° подвСса - ΠΌΠΎΠ΄ΡƒΠ»ΡŒ кручСния, ΠΌΠΎΠ΄ΡƒΠ»ΡŒ сдвига.

ОписаниС Ρ€Π°Π±ΠΎΡ‚Ρ‹: Π­Ρ‚Π° ΡΡ‚Π°Ρ‚ΡŒΡ ΠΌΠΎΠΆΠ΅Ρ‚ Π±Ρ‹Ρ‚ΡŒ ΠΏΠΎΠ»Π΅Π·Π½Π° учитСлям Ρ„ΠΈΠ·ΠΈΠΊΠΈ, Ρ€Π°Π±ΠΎΡ‚Π°ΡŽΡ‰ΠΈΠΌ Π² 7-9 классах ΠΏΠΎ ΠΏΡ€ΠΎΠ³Ρ€Π°ΠΌΠΌΠ°ΠΌ Ρ€Π°Π·Π»ΠΈΡ‡Π½Ρ‹Ρ… Π°Π²Ρ‚ΠΎΡ€ΠΎΠ². Π’ Π½Π΅ΠΉ ΠΏΡ€ΠΈΠ²Π΅Π΄Π΅Π½Ρ‹ ΠΏΡ€ΠΈΠΌΠ΅Ρ€Ρ‹ Π΄ΠΎΠΌΠ°ΡˆΠ½ΠΈΡ… ΠΎΠΏΡ‹Ρ‚ΠΎΠ² ΠΈ экспСримСнтов, ΠΏΡ€ΠΎΠ²ΠΎΠ΄ΠΈΠΌΡ‹Ρ… с ΠΏΠΎΠΌΠΎΡ‰ΡŒΡŽ дСтских ΠΈΠ³Ρ€ΡƒΡˆΠ΅ΠΊ, Π° Ρ‚Π°ΠΊΠΆΠ΅ качСствСнныС ΠΈ ΡΠΊΡΠΏΠ΅Ρ€ΠΈΠΌΠ΅Π½Ρ‚Π°Π»ΡŒΠ½Ρ‹Π΅ Π·Π°Π΄Π°Ρ‡ΠΈ, Π² Ρ‚ΠΎΠΌ числС ΠΈ с Ρ€Π΅ΡˆΠ΅Π½ΠΈΡΠΌΠΈ, распрСдСлСнныС ΠΏΠΎ классам обучСния. ΠœΠ°Ρ‚Π΅Ρ€ΠΈΠ°Π»ΠΎΠΌ Π΄Π°Π½Π½ΠΎΠΉ ΡΡ‚Π°Ρ‚ΡŒΠΈ ΠΌΠΎΠ³ΡƒΡ‚ Π²ΠΎΡΠΏΠΎΠ»ΡŒΠ·ΠΎΠ²Π°Ρ‚ΡŒΡΡ ΠΈ сами ΠΎΠ±ΡƒΡ‡Π°ΡŽΡ‰ΠΈΠ΅ΡΡ 7-9 классов, ΠΈΠΌΠ΅ΡŽΡ‰ΠΈΠ΅ ΠΏΠΎΠ²Ρ‹ΡˆΠ΅Π½Π½Ρ‹ΠΉ ΠΏΠΎΠ·Π½Π°Π²Π°Ρ‚Π΅Π»ΡŒΠ½Ρ‹ΠΉ интСрСс ΠΈ ΠΆΠ΅Π»Π°Π½ΠΈΠ΅ ΠΊ ΠΏΡ€ΠΎΠ²Π΅Π΄Π΅Π½ΠΈΡŽ ΡΠ°ΠΌΠΎΡΡ‚ΠΎΡΡ‚Π΅Π»ΡŒΠ½Ρ‹Ρ… исслСдований Π² Π΄ΠΎΠΌΠ°ΡˆΠ½ΠΈΡ… условиях.

Π’Π²Π΅Π΄Π΅Π½ΠΈΠ΅. ΠŸΡ€ΠΈ ΠΎΠ±ΡƒΡ‡Π΅Π½ΠΈΠΈ Ρ„ΠΈΠ·ΠΈΠΊΠ΅, ΠΊΠ°ΠΊ извСстно, большоС Π·Π½Π°Ρ‡Π΅Π½ΠΈΠ΅ ΠΈΠΌΠ΅Π΅Ρ‚ дСмонстрационный ΠΈ Π»Π°Π±ΠΎΡ€Π°Ρ‚ΠΎΡ€Π½Ρ‹ΠΉ экспСримСнт, яркий ΠΈ Π²ΠΏΠ΅Ρ‡Π°Ρ‚Π»ΡΡŽΡ‰ΠΈΠΉ, ΠΎΠ½ воздСйствуСт Π½Π° чувства Π΄Π΅Ρ‚Π΅ΠΉ, Π²ΠΎΠ·Π±ΡƒΠΆΠ΄Π°Π΅Ρ‚ интСрСс ΠΊ ΠΈΠ·ΡƒΡ‡Π°Π΅ΠΌΠΎΠΌΡƒ. Для создания интСрСса ΠΊ ΡƒΡ€ΠΎΠΊΠ°ΠΌ Ρ„ΠΈΠ·ΠΈΠΊΠΈ, особСнно Π² ΠΌΠ»Π°Π΄ΡˆΠΈΡ… классах, ΠΌΠΎΠΆΠ½ΠΎ, Π½Π°ΠΏΡ€ΠΈΠΌΠ΅Ρ€, Π΄Π΅ΠΌΠΎΠ½ΡΡ‚Ρ€ΠΈΡ€ΠΎΠ²Π°Ρ‚ΡŒ Π½Π° ΡƒΡ€ΠΎΠΊΠ°Ρ… дСтскиС ΠΈΠ³Ρ€ΡƒΡˆΠΊΠΈ, ΠΊΠΎΡ‚ΠΎΡ€Ρ‹Π΅ часто ΠΏΡ€ΠΎΡ‰Π΅ Π² ΠΎΠ±Ρ€Π°Ρ‰Π΅Π½ΠΈΠΈ ΠΈ эффСктивнСС, Ρ‡Π΅ΠΌ дСмонстрационноС ΠΈ Π»Π°Π±ΠΎΡ€Π°Ρ‚ΠΎΡ€Π½ΠΎΠ΅ ΠΎΠ±ΠΎΡ€ΡƒΠ΄ΠΎΠ²Π°Π½ΠΈΠ΅. ИспользованиС дСтских ΠΈΠ³Ρ€ΡƒΡˆΠ΅ΠΊ приносит Π±ΠΎΠ»ΡŒΡˆΡƒΡŽ ΠΏΠΎΠ»ΡŒΠ·Ρƒ, Ρ‚.ΠΊ. ΠΎΠ½ΠΈ ΠΏΠΎΠ·Π²ΠΎΠ»ΡΡŽΡ‚ ΠΎΡ‡Π΅Π½ΡŒ наглядно, Π½Π° Π·Π½Π°ΠΊΠΎΠΌΡ‹Ρ… с дСтства ΠΎΠ±ΡŠΠ΅ΠΊΡ‚Π°Ρ… Π΄Π΅ΠΌΠΎΠ½ΡΡ‚Ρ€ΠΈΡ€ΠΎΠ²Π°Ρ‚ΡŒ Π½Π΅ Ρ‚ΠΎΠ»ΡŒΠΊΠΎ Ρ‚Π΅ ΠΈΠ»ΠΈ ΠΈΠ½Ρ‹Π΅ физичСскиС явлСния, Π½ΠΎ ΠΈ проявлСниС физичСских Π·Π°ΠΊΠΎΠ½ΠΎΠ² Π² ΠΎΠΊΡ€ΡƒΠΆΠ°ΡŽΡ‰Π΅ΠΌ ΠΌΠΈΡ€Π΅ ΠΈ ΠΈΡ… ΠΏΡ€ΠΈΠΌΠ΅Π½Π΅Π½ΠΈΠ΅.

ΠŸΡ€ΠΈ ΠΈΠ·ΡƒΡ‡Π΅Π½ΠΈΠΈ Π½Π΅ΠΊΠΎΡ‚ΠΎΡ€Ρ‹Ρ… Ρ‚Π΅ΠΌ ΠΈΠ³Ρ€ΡƒΡˆΠΊΠΈ Π±ΡƒΠ΄ΡƒΡ‚ ΠΏΠΎΡ‡Ρ‚ΠΈ СдинствСнными наглядными пособиями. ΠœΠ΅Ρ‚ΠΎΠ΄ΠΈΠΊΠ° примСнСния ΠΈΠ³Ρ€ΡƒΡˆΠ΅ΠΊ Π½Π° ΡƒΡ€ΠΎΠΊΠ°Ρ… Ρ„ΠΈΠ·ΠΈΠΊΠ΅ подчиняСтся трСбованиям, ΠΏΡ€Π΅Π΄ΡŠΡΠ²Π»ΡΠ΅ΠΌΡ‹ΠΌ ΠΊ Ρ€Π°Π·Π»ΠΈΡ‡Π½Ρ‹ΠΌ Π²ΠΈΠ΄Π°ΠΌ школьного экспСримСнта:

1. Π˜Π³Ρ€ΡƒΡˆΠΊΠ° Π΄ΠΎΠ»ΠΆΠ½Π° Π±Ρ‹Ρ‚ΡŒ красочной, Π½ΠΎ Π±Π΅Π· Π½Π΅Π½ΡƒΠΆΠ½Ρ‹Ρ… для ΠΎΠΏΡ‹Ρ‚Π° Π΄Π΅Ρ‚Π°Π»Π΅ΠΉ. ВсС второстСпСнныС Π΄Π΅Ρ‚Π°Π»ΠΈ, Π½Π΅ ΠΈΠΌΠ΅ΡŽΡ‰ΠΈΠ΅ ΠΏΡ€ΠΈΠ½Ρ†ΠΈΠΏΠΈΠ°Π»ΡŒΠ½ΠΎΠ³ΠΎ значСния Π² Π΄Π°Π½Π½ΠΎΠΌ ΠΎΠΏΡ‹Ρ‚Π΅, Π½Π΅ Π΄ΠΎΠ»ΠΆΠ½Ρ‹ ΠΎΡ‚Π²Π»Π΅ΠΊΠ°Ρ‚ΡŒ внимания учащихся ΠΈ ΠΏΠΎΡ‚ΠΎΠΌΡƒ ΠΈΡ… Π»ΠΈΠ±ΠΎ Π½ΡƒΠΆΠ½ΠΎ Π·Π°ΠΊΡ€Ρ‹Ρ‚ΡŒ, Π»ΠΈΠ±ΠΎ ΡΠ΄Π΅Π»Π°Ρ‚ΡŒ ΠΌΠ΅Π½Π΅Π΅ Π·Π°ΠΌΠ΅Ρ‚Π½Ρ‹ΠΌΠΈ.

2. Π˜Π³Ρ€ΡƒΡˆΠΊΠ° Π΄ΠΎΠ»ΠΆΠ½Π° Π±Ρ‹Ρ‚ΡŒ Π·Π½Π°ΠΊΠΎΠΌΠΎΠΉ учащимся, Ρ‚.ΠΊ. ΠΏΠΎΠ²Ρ‹ΡˆΠ΅Π½Π½Ρ‹ΠΉ интСрСс ΠΊ конструкции ΠΈΠ³Ρ€ΡƒΡˆΠΊΠΈ ΠΌΠΎΠΆΠ΅Ρ‚ Π·Π°ΡΠ»ΠΎΠ½ΠΈΡ‚ΡŒ ΡΡƒΡ‚ΡŒ самой дСмонстрации.

3. Π‘Π»Π΅Π΄ΡƒΠ΅Ρ‚ Π·Π°Π±ΠΎΡ‚ΠΈΡ‚ΡŒΡΡ ΠΎ наглядности ΠΈ Π²Ρ‹Ρ€Π°Π·ΠΈΡ‚Π΅Π»ΡŒΠ½ΠΎΡΡ‚ΠΈ ΠΎΠΏΡ‹Ρ‚ΠΎΠ². Для этого Π½ΡƒΠΆΠ½ΠΎ Π²Ρ‹Π±ΠΈΡ€Π°Ρ‚ΡŒ ΠΈΠ³Ρ€ΡƒΡˆΠΊΠΈ Π½Π°ΠΈΠ±ΠΎΠ»Π΅Π΅ просто ΠΈ наглядно Π΄Π΅ΠΌΠΎΠ½ΡΡ‚Ρ€ΠΈΡ€ΡƒΡŽΡ‰ΠΈΠ΅ Π΄Π°Π½Π½ΠΎΠ΅ явлСниС.

4. ΠžΠΏΡ‹Ρ‚ Π΄ΠΎΠ»ΠΆΠ΅Π½ Π±Ρ‹Ρ‚ΡŒ ΡƒΠ±Π΅Π΄ΠΈΡ‚Π΅Π»ΡŒΠ½Ρ‹ΠΌ, Π½Π΅ ΡΠΎΠ΄Π΅Ρ€ΠΆΠ°Ρ‚ΡŒ Π½Π΅ относящихся ΠΊ Π΄Π°Π½Π½ΠΎΠΌΡƒ вопросу явлСний ΠΈ Π½Π΅ Π΄Π°Π²Π°Ρ‚ΡŒ ΠΏΠΎΠ²ΠΎΠ΄Π° ΠΊ Π½Π΅ΠΏΡ€Π°Π²ΠΈΠ»ΡŒΠ½ΠΎΠΌΡƒ Ρ‚ΠΎΠ»ΠΊΠΎΠ²Π°Π½ΠΈΡŽ.

Π˜Π³Ρ€ΡƒΡˆΠΊΠΈ ΠΌΠΎΠ³ΡƒΡ‚ Π±Ρ‹Ρ‚ΡŒ ΠΈΡΠΏΠΎΠ»ΡŒΠ·ΠΎΠ²Π°Π½Ρ‹ ΠΏΡ€ΠΈ ΠΏΡ€ΠΎΠ²Π΅Π΄Π΅Π½ΠΈΠΈ любого этапа ΡƒΡ‡Π΅Π±Π½ΠΎΠ³ΠΎ занятия: ΠΏΡ€ΠΈ объяснСнии Π½ΠΎΠ²ΠΎΠ³ΠΎ ΠΌΠ°Ρ‚Π΅Ρ€ΠΈΠ°Π»Π°, ΠΏΡ€ΠΈ Ρ„Ρ€ΠΎΠ½Ρ‚Π°Π»ΡŒΠ½ΠΎΠΌ экспСримСнтС, Ρ€Π΅ΡˆΠ΅Π½ΠΈΠΈ Π·Π°Π΄Π°Ρ‡ ΠΈ Π·Π°ΠΊΡ€Π΅ΠΏΠ»Π΅Π½ΠΈΠΈ ΠΌΠ°Ρ‚Π΅Ρ€ΠΈΠ°Π»Π°, Π½ΠΎ Π½Π°ΠΈΠ±ΠΎΠ»Π΅Π΅ цСлСсообразным, Π½Π° ΠΌΠΎΠΉ взгляд, являСтся использованиС ΠΈΠ³Ρ€ΡƒΡˆΠ΅ΠΊ Π² Π΄ΠΎΠΌΠ°ΡˆΠ½ΠΈΡ… экспСримСнтах, ΡΠ°ΠΌΠΎΡΡ‚ΠΎΡΡ‚Π΅Π»ΡŒΠ½Ρ‹Ρ… ΠΈΡΡΠ»Π΅Π΄ΠΎΠ²Π°Ρ‚Π΅Π»ΡŒΡΠΊΠΈΡ… Ρ€Π°Π±ΠΎΡ‚Π°Ρ…. ΠŸΡ€ΠΈΠΌΠ΅Π½Π΅Π½ΠΈΠ΅ ΠΈΠ³Ρ€ΡƒΡˆΠ΅ΠΊ ΠΏΠΎΠΌΠΎΠ³Π°Π΅Ρ‚ ΡƒΠ²Π΅Π»ΠΈΡ‡ΠΈΡ‚ΡŒ количСство Π΄ΠΎΠΌΠ°ΡˆΠ½ΠΈΡ… ΠΎΠΏΡ‹Ρ‚ΠΎΠ² ΠΈ ΠΈΡΡΠ»Π΅Π΄ΠΎΠ²Π°Ρ‚Π΅Π»ΡŒΡΠΊΠΈΡ… Ρ€Π°Π±ΠΎΡ‚, Ρ‡Ρ‚ΠΎ нСсомнСнно способствуСт Π²Ρ‹Ρ€Π°Π±ΠΎΡ‚ΠΊΠ΅ ΡΠΊΡΠΏΠ΅Ρ€ΠΈΠΌΠ΅Π½Ρ‚Π°Π»ΡŒΠ½Ρ‹Ρ… Π½Π°Π²Ρ‹ΠΊΠΎΠ² ΠΈ создаСт условия для творчСской Ρ€Π°Π±ΠΎΡ‚Ρ‹ Π½Π°Π΄ ΠΈΠ·ΡƒΡ‡Π°Π΅ΠΌΡ‹ΠΌ ΠΌΠ°Ρ‚Π΅Ρ€ΠΈΠ°Π»ΠΎΠΌ, ΠΏΡ€ΠΈ ΠΊΠΎΡ‚ΠΎΡ€ΠΎΠΌ Π³Π»Π°Π²Π½ΠΎΠ΅ усилиС Π½Π°ΠΏΡ€Π°Π²Π»Π΅Π½ΠΎ Π½Π΅ Π½Π° Π·Π°ΠΏΠΎΠΌΠΈΠ½Π°Π½ΠΈΠ΅ Ρ‚ΠΎΠ³ΠΎ, Ρ‡Ρ‚ΠΎ написано Π² ΡƒΡ‡Π΅Π±Π½ΠΈΠΊΠ΅, Π° Π½Π° постановку экспСримСнта ΠΈ ΠΎΠ±Π΄ΡƒΠΌΡ‹Π²Π°Π½ΠΈΠ΅ Π΅Π³ΠΎ Ρ€Π΅Π·ΡƒΠ»ΡŒΡ‚Π°Ρ‚Π°. ΠžΠΏΡ‹Ρ‚Ρ‹ с ΠΈΠ³Ρ€ΡƒΡˆΠΊΠ°ΠΌΠΈ Π±ΡƒΠ΄ΡƒΡ‚ для учащихся ΠΎΠ΄Π½ΠΎΠ²Ρ€Π΅ΠΌΠ΅Π½Π½ΠΎ ΠΈ ΡƒΡ‡Ρ‘Π±ΠΎΠΉ ΠΈ ΠΈΠ³Ρ€ΠΎΠΉ, ΠΏΡ€ΠΈΡ‡Ρ‘ΠΌ Ρ‚Π°ΠΊΠΎΠΉ ΠΈΠ³Ρ€ΠΎΠΉ, которая Π½Π΅ΠΏΡ€Π΅ΠΌΠ΅Π½Π½ΠΎ Ρ‚Ρ€Π΅Π±ΡƒΠ΅Ρ‚ усилия мысли.

ЭкспСримСнт Π² Ρ„ΠΈΠ·ΠΈΠΊΠ΅. ЀизичСский ΠΏΡ€Π°ΠΊΡ‚ΠΈΠΊΡƒΠΌ. Π¨ΡƒΡ‚ΠΎΠ² Π’.И., Π‘ΡƒΡ…ΠΎΠ² Π’.Π“., ΠŸΠΎΠ΄Π»Π΅ΡΠ½Ρ‹ΠΉ Π”.Π’.

М.: Π€ΠΈΠ·ΠΌΠ°Ρ‚Π»ΠΈΡ‚, 2005. - 184с.

ΠžΠΏΠΈΡΠ°Π½Ρ‹ ΡΠΊΡΠΏΠ΅Ρ€ΠΈΠΌΠ΅Π½Ρ‚Π°Π»ΡŒΠ½Ρ‹Π΅ Ρ€Π°Π±ΠΎΡ‚Ρ‹, входящиС Π² ΠΏΡ€ΠΎΠ³Ρ€Π°ΠΌΠΌΡƒ Ρ„ΠΈΠ·ΠΈΠΊΠΎ-матСматичСских Π»ΠΈΡ†Π΅Π΅Π² Π² Ρ€Π°ΠΌΠΊΠ°Ρ… физичСского ΠΏΡ€Π°ΠΊΡ‚ΠΈΠΊΡƒΠΌΠ°. ПособиС прСдставляСт собой ΠΏΠΎΠΏΡ‹Ρ‚ΠΊΡƒ создания Π΅Π΄ΠΈΠ½ΠΎΠ³ΠΎ руководства для провСдСния практичСских занятий Π² классах ΠΈ ΡˆΠΊΠΎΠ»Π°Ρ… с ΡƒΠ³Π»ΡƒΠ±Π»Π΅Π½Π½Ρ‹ΠΌ ΠΈΠ·ΡƒΡ‡Π΅Π½ΠΈΠ΅ΠΌ Ρ„ΠΈΠ·ΠΈΠΊΠΈ, Π° Ρ‚Π°ΠΊΠΆΠ΅ для ΠΏΠΎΠ΄Π³ΠΎΡ‚ΠΎΠ²ΠΊΠΈ ΠΊ ΡΠΊΡΠΏΠ΅Ρ€ΠΈΠΌΠ΅Π½Ρ‚Π°Π»ΡŒΠ½Ρ‹ΠΌ Ρ‚ΡƒΡ€Π°ΠΌ ΠΎΠ»ΠΈΠΌΠΏΠΈΠ°Π΄ высокого уровня.

Π’Π²ΠΎΠ΄Π½Ρ‹ΠΉ ΠΌΠ°Ρ‚Π΅Ρ€ΠΈΠ°Π» Ρ‚Ρ€Π°Π΄ΠΈΡ†ΠΈΠΎΠ½Π½ΠΎ посвящСн ΠΌΠ΅Ρ‚ΠΎΠ΄Π°ΠΌ ΠΎΠ±Ρ€Π°Π±ΠΎΡ‚ΠΊΠΈ ΡΠΊΡΠΏΠ΅Ρ€ΠΈΠΌΠ΅Π½Ρ‚Π°Π»ΡŒΠ½Ρ‹Ρ… Π΄Π°Π½Π½Ρ‹Ρ…. ОписаниС ΠΊΠ°ΠΆΠ΄ΠΎΠΉ ΡΠΊΡΠΏΠ΅Ρ€ΠΈΠΌΠ΅Π½Ρ‚Π°Π»ΡŒΠ½ΠΎΠΉ Ρ€Π°Π±ΠΎΡ‚Ρ‹ начинаСтся с тСорСтичСского ввСдСния. Π’ ΡΠΊΡΠΏΠ΅Ρ€ΠΈΠΌΠ΅Π½Ρ‚Π°Π»ΡŒΠ½ΠΎΠΉ части приводятся описания ΡΠΊΡΠΏΠ΅Ρ€ΠΈΠΌΠ΅Π½Ρ‚Π°Π»ΡŒΠ½Ρ‹Ρ… установок ΠΈ задания, Ρ€Π΅Π³Π»Π°ΠΌΠ΅Π½Ρ‚ΠΈΡ€ΡƒΡŽΡ‰ΠΈΠ΅ ΠΏΠΎΡΠ»Π΅Π΄ΠΎΠ²Π°Ρ‚Π΅Π»ΡŒΠ½ΠΎΡΡ‚ΡŒ Ρ€Π°Π±ΠΎΡ‚Ρ‹ учащихся ΠΏΡ€ΠΈ ΠΏΡ€ΠΎΠ²Π΅Π΄Π΅Π½ΠΈΠΈ ΠΈΠ·ΠΌΠ΅Ρ€Π΅Π½ΠΈΠΉ. ΠŸΡ€ΠΈΠ²ΠΎΠ΄ΡΡ‚ΡΡ ΠΎΠ±Ρ€Π°Π·Ρ†Ρ‹ Ρ€Π°Π±ΠΎΡ‡ΠΈΡ… Ρ‚Π°Π±Π»ΠΈΡ† для записи Ρ€Π΅Π·ΡƒΠ»ΡŒΡ‚Π°Ρ‚ΠΎΠ² ΠΈΠ·ΠΌΠ΅Ρ€Π΅Π½ΠΈΠΉ, Ρ€Π΅ΠΊΠΎΠΌΠ΅Π½Π΄Π°Ρ†ΠΈΠΈ ΠΏΠΎ ΠΌΠ΅Ρ‚ΠΎΠ΄Π°ΠΌ ΠΎΠ±Ρ€Π°Π±ΠΎΡ‚ΠΊΠΈ ΠΈ прСдставлСния Ρ€Π΅Π·ΡƒΠ»ΡŒΡ‚Π°Ρ‚ΠΎΠ² ΠΈ трСбования ΠΊ ΠΎΡ„ΠΎΡ€ΠΌΠ»Π΅Π½ΠΈΡŽ ΠΎΡ‚Ρ‡Π΅Ρ‚ΠΎΠ². Π’ ΠΊΠΎΠ½Ρ†Π΅ описаний ΠΏΡ€Π΅Π΄Π»Π°Π³Π°ΡŽΡ‚ΡΡ ΠΊΠΎΠ½Ρ‚Ρ€ΠΎΠ»ΡŒΠ½Ρ‹Π΅ вопросы, ΠΎΡ‚Π²Π΅Ρ‚Ρ‹ Π½Π° ΠΊΠΎΡ‚ΠΎΡ€Ρ‹Π΅ учащиСся Π΄ΠΎΠ»ΠΆΠ½Ρ‹ ΠΏΠΎΠ΄Π³ΠΎΡ‚ΠΎΠ²ΠΈΡ‚ΡŒ ΠΊ Π·Π°Ρ‰ΠΈΡ‚Π΅ Ρ€Π°Π±ΠΎΡ‚.

Для школ ΠΈ классов с ΡƒΠ³Π»ΡƒΠ±Π»Π΅Π½Π½Ρ‹ΠΌ ΠΈΠ·ΡƒΡ‡Π΅Π½ΠΈΠ΅ΠΌ Ρ„ΠΈΠ·ΠΈΠΊΠΈ.

Π€ΠΎΡ€ΠΌΠ°Ρ‚: djvu / zip

Π Π°Π·ΠΌΠ΅Ρ€: 2 ,6 Мб

/ Download Ρ„Π°ΠΉΠ»

Π’Π’Π•Π”Π•ΠΠ˜Π•

ЀизичСский ΠΏΡ€Π°ΠΊΡ‚ΠΈΠΊΡƒΠΌ являСтся Π½Π΅ΠΎΡ‚ΡŠΠ΅ΠΌΠ»Π΅ΠΌΠΎΠΉ Ρ‡Π°ΡΡ‚ΡŒΡŽ курса Ρ„ΠΈΠ·ΠΈΠΊΠΈ. ЯсноС ΠΈ Π³Π»ΡƒΠ±ΠΎΠΊΠΎΠ΅ усвоСниС основных Π·Π°ΠΊΠΎΠ½ΠΎΠ² Ρ„ΠΈΠ·ΠΈΠΊΠΈ ΠΈ Π΅Π΅ ΠΌΠ΅Ρ‚ΠΎΠ΄ΠΎΠ² Π½Π΅Π²ΠΎΠ·ΠΌΠΎΠΆΠ½ΠΎ Π±Π΅Π· Ρ€Π°Π±ΠΎΡ‚Ρ‹ Π² физичСской Π»Π°Π±ΠΎΡ€Π°Ρ‚ΠΎΡ€ΠΈΠΈ, Π±Π΅Π· ΡΠ°ΠΌΠΎΡΡ‚ΠΎΡΡ‚Π΅Π»ΡŒΠ½Ρ‹Ρ… практичСских занятий. Π’ физичСской Π»Π°Π±ΠΎΡ€Π°Ρ‚ΠΎΡ€ΠΈΠΈ учащиСся Π½Π΅ Ρ‚ΠΎΠ»ΡŒΠΊΠΎ ΠΏΡ€ΠΎΠ²Π΅Ρ€ΡΡŽΡ‚ извСстныС Π·Π°ΠΊΠΎΠ½Ρ‹ Ρ„ΠΈΠ·ΠΈΠΊΠΈ, Π½ΠΎ ΠΈ ΠΎΠ±ΡƒΡ‡Π°ΡŽΡ‚ΡΡ Ρ€Π°Π±ΠΎΡ‚Π΅ с физичСскими ΠΏΡ€ΠΈΠ±ΠΎΡ€Π°ΠΌΠΈ, ΠΎΠ²Π»Π°Π΄Π΅Π²Π°ΡŽΡ‚ Π½Π°Π²Ρ‹ΠΊΠ°ΠΌΠΈ ΡΠΊΡΠΏΠ΅Ρ€ΠΈΠΌΠ΅Π½Ρ‚Π°Π»ΡŒΠ½ΠΎΠΉ ΠΈΡΡΠ»Π΅Π΄ΠΎΠ²Π°Ρ‚Π΅Π»ΡŒΡΠΊΠΎΠΉ Π΄Π΅ΡΡ‚Π΅Π»ΡŒΠ½ΠΎΡΡ‚ΠΈ, учатся Π³Ρ€Π°ΠΌΠΎΡ‚Π½ΠΎΠΉ ΠΎΠ±Ρ€Π°Π±ΠΎΡ‚ΠΊΠ΅ Ρ€Π΅Π·ΡƒΠ»ΡŒΡ‚Π°Ρ‚ΠΎΠ² ΠΈΠ·ΠΌΠ΅Ρ€Π΅Π½ΠΈΠΉ ΠΈ критичСскому ΠΎΡ‚Π½ΠΎΡˆΠ΅Π½ΠΈΡŽ ΠΊ Π½ΠΈΠΌ.

Π”Π°Π½Π½ΠΎΠ΅ пособиС прСдставляСт собой ΠΏΠΎΠΏΡ‹Ρ‚ΠΊΡƒ создания Π΅Π΄ΠΈΠ½ΠΎΠ³ΠΎ руководства ΠΏΠΎ ΡΠΊΡΠΏΠ΅Ρ€ΠΈΠΌΠ΅Π½Ρ‚Π°Π»ΡŒΠ½ΠΎΠΉ Ρ„ΠΈΠ·ΠΈΠΊΠ΅ для вСдСния занятий Π² физичСских лабораториях ΠΏΡ€ΠΎΡ„ΠΈΠ»ΡŒΠ½Ρ‹Ρ… Ρ„ΠΈΠ·ΠΈΠΊΠΎ-матСматичСских школ ΠΈ Π»ΠΈΡ†Π΅Π΅Π². Оно рассчитано Π½Π° учащихся, Π½Π΅ ΠΎΠ±Π»Π°Π΄Π°ΡŽΡ‰ΠΈΡ… ΠΎΠΏΡ‹Ρ‚ΠΎΠΌ ΡΠ°ΠΌΠΎΡΡ‚ΠΎΡΡ‚Π΅Π»ΡŒΠ½ΠΎΠΉ Ρ€Π°Π±ΠΎΡ‚Ρ‹ Π² физичСской Π»Π°Π±ΠΎΡ€Π°Ρ‚ΠΎΡ€ΠΈΠΈ. ΠŸΠΎΡΡ‚ΠΎΠΌΡƒ описания Ρ€Π°Π±ΠΎΡ‚ Π²Ρ‹ΠΏΠΎΠ»Π½Π΅Π½Ρ‹ ΠΏΠΎΠ΄Ρ€ΠΎΠ±Π½ΠΎ ΠΈ ΠΎΠ±ΡΡ‚ΠΎΡΡ‚Π΅Π»ΡŒΠ½ΠΎ. ОсобоС Π²Π½ΠΈΠΌΠ°Π½ΠΈΠ΅ ΡƒΠ΄Π΅Π»Π΅Π½ΠΎ тСорСтичСскому обоснованию примСняСмых ΡΠΊΡΠΏΠ΅Ρ€ΠΈΠΌΠ΅Π½Ρ‚Π°Π»ΡŒΠ½Ρ‹Ρ… ΠΌΠ΅Ρ‚ΠΎΠ΄ΠΎΠ², вопросам ΠΎΠ±Ρ€Π°Π±ΠΎΡ‚ΠΊΠΈ Ρ€Π΅Π·ΡƒΠ»ΡŒΡ‚Π°Ρ‚ΠΎΠ² ΠΈΠ·ΠΌΠ΅Ρ€Π΅Π½ΠΈΠΉ ΠΈ ΠΎΡ†Π΅Π½ΠΊΠΈ ΠΈΡ… ΠΏΠΎΠ³Ρ€Π΅ΡˆΠ½ΠΎΡΡ‚Π΅ΠΉ.

ОписаниС ΠΊΠ°ΠΆΠ΄ΠΎΠΉ ΡΠΊΡΠΏΠ΅Ρ€ΠΈΠΌΠ΅Π½Ρ‚Π°Π»ΡŒΠ½ΠΎΠΉ Ρ€Π°Π±ΠΎΡ‚Ρ‹ начинаСтся с тСорСтичСского ввСдСния. Π’ ΡΠΊΡΠΏΠ΅Ρ€ΠΈΠΌΠ΅Π½Ρ‚Π°Π»ΡŒΠ½ΠΎΠΉ части ΠΊΠ°ΠΆΠ΄ΠΎΠΉ Ρ€Π°Π±ΠΎΡ‚Ρ‹ приводятся описания ΡΠΊΡΠΏΠ΅Ρ€ΠΈΠΌΠ΅Π½Ρ‚Π°Π»ΡŒΠ½Ρ‹Ρ… установок ΠΈ задания, Ρ€Π΅Π³Π»Π°ΠΌΠ΅Π½Ρ‚ΠΈΡ€ΡƒΡŽΡ‰ΠΈΠ΅ ΠΏΠΎΡΠ»Π΅Π΄ΠΎΠ²Π°Ρ‚Π΅Π»ΡŒΠ½ΠΎΡΡ‚ΡŒ Ρ€Π°Π±ΠΎΡ‚Ρ‹ учащихся ΠΏΡ€ΠΈ ΠΏΡ€ΠΎΠ²Π΅Π΄Π΅Π½ΠΈΠΈ ΠΈΠ·ΠΌΠ΅Ρ€Π΅Π½ΠΈΠΉ, ΠΎΠ±Ρ€Π°Π·Ρ†Ρ‹ Ρ€Π°Π±ΠΎΡ‡ΠΈΡ… Ρ‚Π°Π±Π»ΠΈΡ† для записи Ρ€Π΅Π·ΡƒΠ»ΡŒΡ‚Π°Ρ‚ΠΎΠ² ΠΈΠ·ΠΌΠ΅Ρ€Π΅Π½ΠΈΠΉ ΠΈ Ρ€Π΅ΠΊΠΎΠΌΠ΅Π½Π΄Π°Ρ†ΠΈΠΈ ΠΏΠΎ ΠΌΠ΅Ρ‚ΠΎΠ΄Π°ΠΌ ΠΎΠ±Ρ€Π°Π±ΠΎΡ‚ΠΊΠΈ ΠΈ прСдставлСния Ρ€Π΅Π·ΡƒΠ»ΡŒΡ‚Π°Ρ‚ΠΎΠ². Π’ ΠΊΠΎΠ½Ρ†Π΅ описаний ΠΏΡ€Π΅Π΄Π»Π°Π³Π°ΡŽΡ‚ΡΡ ΠΊΠΎΠ½Ρ‚Ρ€ΠΎΠ»ΡŒΠ½Ρ‹Π΅ вопросы, ΠΎΡ‚Π²Π΅Ρ‚Ρ‹ Π½Π° ΠΊΠΎΡ‚ΠΎΡ€Ρ‹Π΅ учащиСся Π΄ΠΎΠ»ΠΆΠ½Ρ‹ ΠΏΠΎΠ΄Π³ΠΎΡ‚ΠΎΠ²ΠΈΡ‚ΡŒ ΠΊ Π·Π°Ρ‰ΠΈΡ‚Π΅ Ρ€Π°Π±ΠΎΡ‚.

Π’ срСднСм Π·Π° ΡƒΡ‡Π΅Π±Π½Ρ‹ΠΉ Π³ΠΎΠ΄ ΠΊΠ°ΠΆΠ΄Ρ‹ΠΉ учащийся Π΄ΠΎΠ»ΠΆΠ΅Π½ Π²Ρ‹ΠΏΠΎΠ»Π½ΠΈΡ‚ΡŒ 10–12 ΡΠΊΡΠΏΠ΅Ρ€ΠΈΠΌΠ΅Π½Ρ‚Π°Π»ΡŒΠ½Ρ‹Ρ… Ρ€Π°Π±ΠΎΡ‚ Π² соотвСтствии с ΡƒΡ‡Π΅Π±Π½Ρ‹ΠΌ ΠΏΠ»Π°Π½ΠΎΠΌ.

Учащийся Π·Π°Ρ€Π°Π½Π΅Π΅ готовится ΠΊ Π²Ρ‹ΠΏΠΎΠ»Π½Π΅Π½ΠΈΡŽ ΠΊΠ°ΠΆΠ΄ΠΎΠΉ Ρ€Π°Π±ΠΎΡ‚Ρ‹. Он Π΄ΠΎΠ»ΠΆΠ΅Π½ ΠΈΠ·ΡƒΡ‡ΠΈΡ‚ΡŒ описаниС Ρ€Π°Π±ΠΎΡ‚Ρ‹, Π·Π½Π°Ρ‚ΡŒ Ρ‚Π΅ΠΎΡ€ΠΈΡŽ Π² объСмС, ΡƒΠΊΠ°Π·Π°Π½Π½ΠΎΠΌ Π² описании, порядок выполнСния Ρ€Π°Π±ΠΎΡ‚Ρ‹, ΠΈΠΌΠ΅Ρ‚ΡŒ ΠΏΡ€Π΅Π΄Π²Π°Ρ€ΠΈΡ‚Π΅Π»ΡŒΠ½ΠΎ ΠΏΠΎΠ΄Π³ΠΎΡ‚ΠΎΠ²Π»Π΅Π½Π½Ρ‹ΠΉ Π»Π°Π±ΠΎΡ€Π°Ρ‚ΠΎΡ€Π½Ρ‹ΠΉ ΠΆΡƒΡ€Π½Π°Π» с конспСктом Ρ‚Π΅ΠΎΡ€ΠΈΠΈ ΠΈ Ρ‚Π°Π±Π»ΠΈΡ†Π°ΠΌΠΈ, Π° Ρ‚Π°ΠΊΠΆΠ΅, Ссли это Π½Π΅ΠΎΠ±Ρ…ΠΎΠ΄ΠΈΠΌΠΎ, ΠΈΠΌΠ΅Ρ‚ΡŒ ΠΌΠΈΠ»Π»ΠΈΠΌΠ΅Ρ‚Ρ€ΠΎΠ²ΡƒΡŽ Π±ΡƒΠΌΠ°Π³Ρƒ для выполнСния ΠΏΡ€ΠΈΠΊΠΈΠ΄ΠΎΡ‡Π½ΠΎΠ³ΠΎ Π³Ρ€Π°Ρ„ΠΈΠΊΠ°.

ΠŸΠ΅Ρ€Π΅Π΄ Π½Π°Ρ‡Π°Π»ΠΎΠΌ выполнСния Ρ€Π°Π±ΠΎΡ‚Ρ‹ учащийся ΠΏΠΎΠ»ΡƒΡ‡Π°Π΅Ρ‚ допуск ΠΊ Ρ€Π°Π±ΠΎΡ‚Π΅.

ΠŸΡ€ΠΈΠΌΠ΅Ρ€Π½Ρ‹ΠΉ ΠΏΠ΅Ρ€Π΅Ρ‡Π΅Π½ΡŒ вопросов для получСния допуска:

1. ЦСль Ρ€Π°Π±ΠΎΡ‚Ρ‹.

2. ΠžΡΠ½ΠΎΠ²Π½Ρ‹Π΅ физичСскиС Π·Π°ΠΊΠΎΠ½Ρ‹, ΠΈΠ·ΡƒΡ‡Π°Π΅ΠΌΡ‹Π΅ Π² Ρ€Π°Π±ΠΎΡ‚Π΅.

3. Π‘Ρ…Π΅ΠΌΠ° установки ΠΈ ΠΏΡ€ΠΈΠ½Ρ†ΠΈΠΏ Π΅Π΅ дСйствия.

4. Π˜Π·ΠΌΠ΅Ρ€ΡΠ΅ΠΌΡ‹Π΅ Π²Π΅Π»ΠΈΡ‡ΠΈΠ½Ρ‹ ΠΈ расчСтныС Ρ„ΠΎΡ€ΠΌΡƒΠ»Ρ‹.

5. ΠŸΠΎΡ€ΡΠ΄ΠΎΠΊ выполнСния Ρ€Π°Π±ΠΎΡ‚Ρ‹.

УчащиСся, Π΄ΠΎΠΏΡƒΡ‰Π΅Π½Π½Ρ‹Π΅ ΠΊ Π²Ρ‹ΠΏΠΎΠ»Π½Π΅Π½ΠΈΡŽ Ρ€Π°Π±ΠΎΡ‚Ρ‹, обязаны ΡΠ»Π΅Π΄ΠΎΠ²Π°Ρ‚ΡŒ порядку выполнСния строго Π² соотвСтствии с описаниСм.

Π Π°Π±ΠΎΡ‚Π° Π² Π»Π°Π±ΠΎΡ€Π°Ρ‚ΠΎΡ€ΠΈΠΈ заканчиваСтся Π²Ρ‹ΠΏΠΎΠ»Π½Π΅Π½ΠΈΠ΅ΠΌ ΠΏΡ€Π΅Π΄Π²Π°Ρ€ΠΈΡ‚Π΅Π»ΡŒΠ½Ρ‹Ρ… расчСтов ΠΈ обсуТдСниСм ΠΈΡ… с ΠΏΡ€Π΅ΠΏΠΎΠ΄Π°Π²Π°Ρ‚Π΅Π»Π΅ΠΌ.

К ΡΠ»Π΅Π΄ΡƒΡŽΡ‰Π΅ΠΌΡƒ Π·Π°Π½ΡΡ‚ΠΈΡŽ учащийся ΡΠ°ΠΌΠΎΡΡ‚ΠΎΡΡ‚Π΅Π»ΡŒΠ½ΠΎ Π·Π°ΠΊΠ°Π½Ρ‡ΠΈΠ²Π°Π΅Ρ‚ ΠΎΠ±Ρ€Π°Π±ΠΎΡ‚ΠΊΡƒ ΠΏΠΎΠ»ΡƒΡ‡Π΅Π½Π½Ρ‹Ρ… ΡΠΊΡΠΏΠ΅Ρ€ΠΈΠΌΠ΅Π½Ρ‚Π°Π»ΡŒΠ½Ρ‹Ρ… Π΄Π°Π½Π½Ρ‹Ρ…, построСниС Π³Ρ€Π°Ρ„ΠΈΠΊΠΎΠ² ΠΈ ΠΎΡ„ΠΎΡ€ΠΌΠ»Π΅Π½ΠΈΠ΅ ΠΎΡ‚Ρ‡Π΅Ρ‚Π°.

На Π·Π°Ρ‰ΠΈΡ‚Π΅ Ρ€Π°Π±ΠΎΡ‚Ρ‹ учащийся Π΄ΠΎΠ»ΠΆΠ΅Π½ ΡƒΠΌΠ΅Ρ‚ΡŒ ΠΎΡ‚Π²Π΅Ρ‚ΠΈΡ‚ΡŒ Π½Π° всС вопросы ΠΏΠΎ Ρ‚Π΅ΠΎΡ€ΠΈΠΈ Π² ΠΏΠΎΠ»Π½ΠΎΠΌ объСмС ΠΏΡ€ΠΎΠ³Ρ€Π°ΠΌΠΌΡ‹, ΠΎΠ±ΠΎΡΠ½ΠΎΠ²Π°Ρ‚ΡŒ ΠΏΡ€ΠΈΠ½ΡΡ‚ΡƒΡŽ ΠΌΠ΅Ρ‚ΠΎΠ΄ΠΈΠΊΡƒ ΠΈΠ·ΠΌΠ΅Ρ€Π΅Π½ΠΈΠΉ ΠΈ ΠΎΠ±Ρ€Π°Π±ΠΎΡ‚ΠΊΠΈ Π΄Π°Π½Π½Ρ‹Ρ…, вывСсти ΡΠ°ΠΌΠΎΡΡ‚ΠΎΡΡ‚Π΅Π»ΡŒΠ½ΠΎ расчСтныС Ρ„ΠΎΡ€ΠΌΡƒΠ»Ρ‹. Π’Ρ‹ΠΏΠΎΠ»Π½Π΅Π½ΠΈΠ΅ Ρ€Π°Π±ΠΎΡ‚Ρ‹ Π½Π° этом Π·Π°Π²Π΅Ρ€ΡˆΠ°Π΅Ρ‚ΡΡ, выставляСтся ΠΎΠΊΠΎΠ½Ρ‡Π°Ρ‚Π΅Π»ΡŒΠ½Π°Ρ итоговая ΠΎΡ†Π΅Π½ΠΊΠ° Π·Π° Ρ€Π°Π±ΠΎΡ‚Ρƒ.

БСмСстровая ΠΈ годовая ΠΎΡ†Π΅Π½ΠΊΠΈ Π²Ρ‹ΡΡ‚Π°Π²Π»ΡΡŽΡ‚ΡΡ ΠΏΡ€ΠΈ ΡƒΡΠΏΠ΅ΡˆΠ½ΠΎΠΌ Π²Ρ‹ΠΏΠΎΠ»Π½Π΅Π½ΠΈΠΈ всСх Ρ€Π°Π±ΠΎΡ‚ Π² соотвСтствии с ΡƒΡ‡Π΅Π±Π½Ρ‹ΠΌ ΠΏΠ»Π°Π½ΠΎΠΌ.

ΠšΡƒΡ€Ρ "Π­ΠΊΡΠΏΠ΅Ρ€ΠΈΠΌΠ΅Π½Ρ‚Π°Π»ΡŒΠ½Π°Ρ Ρ„ΠΈΠ·ΠΈΠΊΠ°" практичСски Ρ€Π΅Π°Π»ΠΈΠ·ΠΎΠ²Π°Π½ Π½Π° комплСксном Π»Π°Π±ΠΎΡ€Π°Ρ‚ΠΎΡ€Π½ΠΎΠΌ ΠΎΠ±ΠΎΡ€ΡƒΠ΄ΠΎΠ²Π°Π½ΠΈΠΈ, Ρ€Π°Π·Ρ€Π°Π±ΠΎΡ‚Π°Π½Π½ΠΎΠΌ Π£Ρ‡Π΅Π±Π½ΠΎ-мСтодичСской Π»Π°Π±ΠΎΡ€Π°Ρ‚ΠΎΡ€ΠΈΠ΅ΠΉ Московского Ρ„ΠΈΠ·ΠΈΠΊΠΎ-тСхничСского института, Π²ΠΊΠ»ΡŽΡ‡Π°ΡŽΡ‰Π΅ΠΌ Π² сСбя Π»Π°Π±ΠΎΡ€Π°Ρ‚ΠΎΡ€Π½Ρ‹Π΅ комплСксы ΠΏΠΎ ΠΌΠ΅Ρ…Π°Π½ΠΈΠΊΠ΅ ΠΌΠ°Ρ‚Π΅Ρ€ΠΈΠ°Π»ΡŒΠ½ΠΎΠΉ Ρ‚ΠΎΡ‡ΠΊΠΈ, ΠΌΠ΅Ρ…Π°Π½ΠΈΠΊΠ΅ Ρ‚Π²Π΅Ρ€Π΄ΠΎΠ³ΠΎ Ρ‚Π΅Π»Π°, молСкулярной Ρ„ΠΈΠ·ΠΈΠΊΠ΅, элСктродинамикС, гСомСтричСской ΠΈ физичСской ΠΎΠΏΡ‚ΠΈΠΊΠ΅. Π’Π°ΠΊΠΎΠ΅ ΠΎΠ±ΠΎΡ€ΡƒΠ΄ΠΎΠ²Π°Π½ΠΈΠ΅ имССтся Π²ΠΎ ΠΌΠ½ΠΎΠ³ΠΈΡ… спСциализированных Ρ„ΠΈΠ·ΠΈΠΊΠΎ-матСматичСских ΡˆΠΊΠΎΠ»Π°Ρ… ΠΈ лицСях России.

Π’Π²Π΅Π΄Π΅Π½ΠΈΠ΅.

ΠŸΠΎΠ³Ρ€Π΅ΡˆΠ½ΠΎΡΡ‚ΠΈ физичСских Π²Π΅Π»ΠΈΡ‡ΠΈΠ½. ΠžΠ±Ρ€Π°Π±ΠΎΡ‚ΠΊΠ° Ρ€Π΅Π·ΡƒΠ»ΡŒΡ‚Π°Ρ‚ΠΎΠ² ΠΈΠ·ΠΌΠ΅Ρ€Π΅Π½ΠΈΠΉ.

ΠŸΡ€Π°ΠΊΡ‚ΠΈΡ‡Π΅ΡΠΊΠ°Ρ Ρ€Π°Π±ΠΎΡ‚Π° 1. Π˜Π·ΠΌΠ΅Ρ€Π΅Π½ΠΈΠ΅ объСма Ρ‚Π΅Π» ΠΏΡ€Π°Π²ΠΈΠ»ΡŒΠ½ΠΎΠΉ Ρ„ΠΎΡ€ΠΌΡ‹.

ΠŸΡ€Π°ΠΊΡ‚ΠΈΡ‡Π΅ΡΠΊΠ°Ρ Ρ€Π°Π±ΠΎΡ‚Π° 2. ИсслСдованиС прямолинСйного двиТСния Ρ‚Π΅Π» Π² ΠΏΠΎΠ»Π΅ Π·Π΅ΠΌΠ½ΠΎΠ³ΠΎ тяготСния Π½Π° машинС Атвуда.

ΠŸΡ€Π°ΠΊΡ‚ΠΈΡ‡Π΅ΡΠΊΠ°Ρ Ρ€Π°Π±ΠΎΡ‚Π° 3. Π‘ΡƒΡ…ΠΎΠ΅ Ρ‚Ρ€Π΅Π½ΠΈΠ΅. ΠžΠΏΡ€Π΅Π΄Π΅Π»Π΅Π½ΠΈΠ΅ коэффициСнта трСния скольТСния.

ВСорСтичСскоС Π²Π²Π΅Π΄Π΅Π½ΠΈΠ΅ ΠΊ Ρ€Π°Π±ΠΎΡ‚Π°ΠΌ ΠΏΠΎ колСбаниям.

ΠŸΡ€Π°ΠΊΡ‚ΠΈΡ‡Π΅ΡΠΊΠ°Ρ Ρ€Π°Π±ΠΎΡ‚Π° 4. Π˜Π·ΡƒΡ‡Π΅Π½ΠΈΠ΅ ΠΊΠΎΠ»Π΅Π±Π°Π½ΠΈΠΉ ΠΏΡ€ΡƒΠΆΠΈΠ½Π½ΠΎΠ³ΠΎ маятника.

ΠŸΡ€Π°ΠΊΡ‚ΠΈΡ‡Π΅ΡΠΊΠ°Ρ Ρ€Π°Π±ΠΎΡ‚Π° 5. Π˜Π·ΡƒΡ‡Π΅Π½ΠΈΠ΅ ΠΊΠΎΠ»Π΅Π±Π°Π½ΠΈΠΉ матСматичСского маятника. ΠžΠΏΡ€Π΅Π΄Π΅Π»Π΅Π½ΠΈΠ΅ ускорСния свободного падСния.

ΠŸΡ€Π°ΠΊΡ‚ΠΈΡ‡Π΅ΡΠΊΠ°Ρ Ρ€Π°Π±ΠΎΡ‚Π° 6. Π˜Π·ΡƒΡ‡Π΅Π½ΠΈΠ΅ ΠΊΠΎΠ»Π΅Π±Π°Π½ΠΈΠΉ физичСского маятника.

ΠŸΡ€Π°ΠΊΡ‚ΠΈΡ‡Π΅ΡΠΊΠ°Ρ Ρ€Π°Π±ΠΎΡ‚Π° 7. ΠžΠΏΡ€Π΅Π΄Π΅Π»Π΅Π½ΠΈΠ΅ ΠΌΠΎΠΌΠ΅Π½Ρ‚ΠΎΠ² ΠΈΠ½Π΅Ρ€Ρ†ΠΈΠΈ Ρ‚Π΅Π» ΠΏΡ€Π°Π²ΠΈΠ»ΡŒΠ½ΠΎΠΉ Ρ„ΠΎΡ€ΠΌΡ‹ ΠΌΠ΅Ρ‚ΠΎΠ΄ΠΎΠΌ ΠΊΡ€ΡƒΡ‚ΠΈΠ»ΡŒΠ½Ρ‹Ρ… ΠΊΠΎΠ»Π΅Π±Π°Π½ΠΈΠΉ.

ΠŸΡ€Π°ΠΊΡ‚ΠΈΡ‡Π΅ΡΠΊΠ°Ρ Ρ€Π°Π±ΠΎΡ‚Π° 8. Π˜Π·ΡƒΡ‡Π΅Π½ΠΈΠ΅ Π·Π°ΠΊΠΎΠ½ΠΎΠ² вращСния Ρ‚Π²Π΅Ρ€Π΄ΠΎΠ³ΠΎ Ρ‚Π΅Π»Π° Π½Π° крСстообразном маятникС ΠžΠ±Π΅Ρ€Π±Π΅ΠΊΠ°.

ΠŸΡ€Π°ΠΊΡ‚ΠΈΡ‡Π΅ΡΠΊΠ°Ρ Ρ€Π°Π±ΠΎΡ‚Π° 9. ΠžΠΏΡ€Π΅Π΄Π΅Π»Π΅Π½ΠΈΠ΅ ΠΎΡ‚Π½ΠΎΡˆΠ΅Π½ΠΈΡ молярных тСплоСмкостСй Π²ΠΎΠ·Π΄ΡƒΡ…Π°.

ΠŸΡ€Π°ΠΊΡ‚ΠΈΡ‡Π΅ΡΠΊΠ°Ρ Ρ€Π°Π±ΠΎΡ‚Π° 10. БтоячиС Π²ΠΎΠ»Π½Ρ‹. Π˜Π·ΠΌΠ΅Ρ€Π΅Π½ΠΈΠ΅ скорости Π²ΠΎΠ»Π½Ρ‹ Π² ΡƒΠΏΡ€ΡƒΠ³ΠΎΠΉ струнС.

ΠŸΡ€Π°ΠΊΡ‚ΠΈΡ‡Π΅ΡΠΊΠ°Ρ Ρ€Π°Π±ΠΎΡ‚Π° 11. ΠžΠΏΡ€Π΅Π΄Π΅Π»Π΅Π½ΠΈΠ΅ ΠΎΡ‚Π½ΠΎΡˆΠ΅Π½ΠΈΡ ср/с ΞΉ? для Π²ΠΎΠ·Π΄ΡƒΡ…Π° Π² стоячСй Π·Π²ΡƒΠΊΠΎΠ²ΠΎΠΉ Π²ΠΎΠ»Π½Π΅.

ΠŸΡ€Π°ΠΊΡ‚ΠΈΡ‡Π΅ΡΠΊΠ°Ρ Ρ€Π°Π±ΠΎΡ‚Π° 12. Π˜Π·ΡƒΡ‡Π΅Π½ΠΈΠ΅ Ρ€Π°Π±ΠΎΡ‚Ρ‹ элСктронного осциллографа.

ΠŸΡ€Π°ΠΊΡ‚ΠΈΡ‡Π΅ΡΠΊΠ°Ρ Ρ€Π°Π±ΠΎΡ‚Π° 13. Π˜Π·ΠΌΠ΅Ρ€Π΅Π½ΠΈΠ΅ частоты ΠΊΠΎΠ»Π΅Π±Π°Π½ΠΈΠΉ ΠΏΡƒΡ‚Π΅ΠΌ исслСдования Ρ„ΠΈΠ³ΡƒΡ€ ЛиссаТу.

ΠŸΡ€Π°ΠΊΡ‚ΠΈΡ‡Π΅ΡΠΊΠ°Ρ Ρ€Π°Π±ΠΎΡ‚Π° 14. ΠžΠΏΡ€Π΅Π΄Π΅Π»Π΅Π½ΠΈΠ΅ ΡƒΠ΄Π΅Π»ΡŒΠ½ΠΎΠ³ΠΎ сопротивлСния Π½ΠΈΡ…Ρ€ΠΎΠΌΠΎΠ²ΠΎΠΉ ΠΏΡ€ΠΎΠ²ΠΎΠ»ΠΎΠΊΠΈ.

ΠŸΡ€Π°ΠΊΡ‚ΠΈΡ‡Π΅ΡΠΊΠ°Ρ Ρ€Π°Π±ΠΎΡ‚Π° 15. ΠžΠΏΡ€Π΅Π΄Π΅Π»Π΅Π½ΠΈΠ΅ сопротивлСния ΠΏΡ€ΠΎΠ²ΠΎΠ΄Π½ΠΈΠΊΠΎΠ² компСнсационным ΠΌΠ΅Ρ‚ΠΎΠ΄ΠΎΠΌ Уитстона.

ΠŸΡ€Π°ΠΊΡ‚ΠΈΡ‡Π΅ΡΠΊΠ°Ρ Ρ€Π°Π±ΠΎΡ‚Π° 16. ΠŸΠ΅Ρ€Π΅Ρ…ΠΎΠ΄Π½Ρ‹Π΅ процСссы Π² кондСнсаторС. ΠžΠΏΡ€Π΅Π΄Π΅Π»Π΅Π½ΠΈΠ΅ Смкости.

ΠŸΡ€Π°ΠΊΡ‚ΠΈΡ‡Π΅ΡΠΊΠ°Ρ Ρ€Π°Π±ΠΎΡ‚Π° 17. ΠžΠΏΡ€Π΅Π΄Π΅Π»Π΅Π½ΠΈΠ΅ напряТСнности элСктричСского поля Π² цилиндричСском ΠΏΡ€ΠΎΠ²ΠΎΠ΄Π½ΠΈΠΊΠ΅ с Ρ‚ΠΎΠΊΠΎΠΌ.

ΠŸΡ€Π°ΠΊΡ‚ΠΈΡ‡Π΅ΡΠΊΠ°Ρ Ρ€Π°Π±ΠΎΡ‚Π° 18. ИсслСдованиС Ρ€Π°Π±ΠΎΡ‚Ρ‹ источника Π² Ρ†Π΅ΠΏΠΈ постоянного Ρ‚ΠΎΠΊΠ°.

ΠŸΡ€Π°ΠΊΡ‚ΠΈΡ‡Π΅ΡΠΊΠ°Ρ Ρ€Π°Π±ΠΎΡ‚Π° 19. Π˜Π·ΡƒΡ‡Π΅Π½ΠΈΠ΅ Π·Π°ΠΊΠΎΠ½ΠΎΠ² отраТСния ΠΈ прСломлСния свСта.

ΠŸΡ€Π°ΠΊΡ‚ΠΈΡ‡Π΅ΡΠΊΠ°Ρ Ρ€Π°Π±ΠΎΡ‚Π° 20. ΠžΠΏΡ€Π΅Π΄Π΅Π»Π΅Π½ΠΈΠ΅ фокусных расстояний ΡΠΎΠ±ΠΈΡ€Π°ΡŽΡ‰Π΅ΠΉ ΠΈ Ρ€Π°ΡΡΠ΅ΠΈΠ²Π°ΡŽΡ‰Π΅ΠΉ Π»ΠΈΠ½Π·.

ΠŸΡ€Π°ΠΊΡ‚ΠΈΡ‡Π΅ΡΠΊΠ°Ρ Ρ€Π°Π±ΠΎΡ‚Π° 21. Π―Π²Π»Π΅Π½ΠΈΠ΅ элСктромагнитной ΠΈΠ½Π΄ΡƒΠΊΡ†ΠΈΠΈ. ИсслСдованиС ΠΌΠ°Π³Π½ΠΈΡ‚Π½ΠΎΠ³ΠΎ поля солСноида.

ΠŸΡ€Π°ΠΊΡ‚ΠΈΡ‡Π΅ΡΠΊΠ°Ρ Ρ€Π°Π±ΠΎΡ‚Π° 22. ИсслСдованиС Π·Π°Ρ‚ΡƒΡ…Π°ΡŽΡ‰ΠΈΡ… ΠΊΠΎΠ»Π΅Π±Π°Π½ΠΈΠΉ.

ΠŸΡ€Π°ΠΊΡ‚ΠΈΡ‡Π΅ΡΠΊΠ°Ρ Ρ€Π°Π±ΠΎΡ‚Π° 23. Π˜Π·ΡƒΡ‡Π΅Π½ΠΈΠ΅ явлСния рСзонанса Π² Ρ†Π΅ΠΏΠΈ ΠΏΠ΅Ρ€Π΅ΠΌΠ΅Π½Π½ΠΎΠ³ΠΎ Ρ‚ΠΎΠΊΠ°.

ΠŸΡ€Π°ΠΊΡ‚ΠΈΡ‡Π΅ΡΠΊΠ°Ρ Ρ€Π°Π±ΠΎΡ‚Π° 24. Дифракция Π€Ρ€Π°ΡƒΠ½Π³ΠΎΡ„Π΅Ρ€Π° Π½Π° Ρ‰Π΅Π»ΠΈ. Π˜Π·ΠΌΠ΅Ρ€Π΅Π½ΠΈΠ΅ ΡˆΠΈΡ€ΠΈΠ½Ρ‹ Ρ‰Π΅Π»ΠΈ Β«Π²ΠΎΠ»Π½ΠΎΠ²Ρ‹ΠΌ ΠΌΠ΅Ρ‚ΠΎΠ΄ΠΎΠΌΒ».

ΠŸΡ€Π°ΠΊΡ‚ΠΈΡ‡Π΅ΡΠΊΠ°Ρ Ρ€Π°Π±ΠΎΡ‚Π° 25. Дифракция Π€Ρ€Π°ΡƒΠ½Π³ΠΎΡ„Π΅Ρ€Π°. Дифракционная Ρ€Π΅ΡˆΠ΅Ρ‚ΠΊΠ° ΠΊΠ°ΠΊ оптичСский ΠΏΡ€ΠΈΠ±ΠΎΡ€.

ΠŸΡ€Π°ΠΊΡ‚ΠΈΡ‡Π΅ΡΠΊΠ°Ρ Ρ€Π°Π±ΠΎΡ‚Π° 26. ΠžΠΏΡ€Π΅Π΄Π΅Π»Π΅Π½ΠΈΠ΅ показатСля прСломлСния стСкла Β«Π²ΠΎΠ»Π½ΠΎΠ²Ρ‹ΠΌΒ» ΠΌΠ΅Ρ‚ΠΎΠ΄ΠΎΠΌ.

ΠŸΡ€Π°ΠΊΡ‚ΠΈΡ‡Π΅ΡΠΊΠ°Ρ Ρ€Π°Π±ΠΎΡ‚Π° 27. ΠžΠΏΡ€Π΅Π΄Π΅Π»Π΅Π½ΠΈΠ΅ радиуса ΠΊΡ€ΠΈΠ²ΠΈΠ·Π½Ρ‹ Π»ΠΈΠ½Π·Ρ‹ Π² экспСримСнтС с ΠΊΠΎΠ»ΡŒΡ†Π°ΠΌΠΈ ΠΡŒΡŽΡ‚ΠΎΠ½Π°.

ΠŸΡ€Π°ΠΊΡ‚ΠΈΡ‡Π΅ΡΠΊΠ°Ρ Ρ€Π°Π±ΠΎΡ‚Π° 28. ИсслСдованиС поляризованного свСта.

Ρ„Π•Π”Π•Π ΠΠ›Π¬ΠΠžΠ• Π“ΠžΠ‘Π£Π”ΠΠ Π‘Π’Π’Π•ΠΠΠžΠ• ΠžΠ‘Π©Π•ΠžΠ‘Π ΠΠ—ΠžΠ’ΠΠ’Π•Π›Π¬ΠΠžΠ• Π£Π§Π Π•Π–Π”Π•ΠΠ˜Π• БРЕДНЯ ΠžΠ‘Π©Π•ΠžΠ‘Π ΠΠ—ΠžΠ’ΠΠ’Π•Π›Π¬ΠΠΠ― Π¨ΠšΠžΠ›Π

Π˜ΠœΠ•ΠΠ˜ Π°. Π½. Ρ€ΠΠ”Π˜Π©Π•Π’Π

Π“. ΠΊΠ£Π—ΠΠ•Π¦Πš - 12

Π­ΠšΠ‘ΠŸΠ•Π Π˜ΠœΠ•ΠΠ’ΠΠ›Π¬ΠΠ«Π• Π—ΠΠ”ΠΠΠ˜Π― ПО Π€Π˜Π—Π˜ΠšΠ•

1. Π˜Π·ΠΌΠ΅Ρ€Π΅Π½ΠΈΠ΅ модуля Π½Π°Ρ‡Π°Π»ΡŒΠ½ΠΎΠΉ скорости ΠΈ Π²Ρ€Π΅ΠΌΠ΅Π½ΠΈ тормоТСния Ρ‚Π΅Π»Π°, двиТущСгося ΠΏΠΎΠ΄ дСйствиСм силы трСния

ΠŸΡ€ΠΈΠ±ΠΎΡ€Ρ‹ ΠΈ ΠΌΠ°Ρ‚Π΅Ρ€ΠΈΠ°Π»Ρ‹: 1) брусок ΠΎΡ‚ Π»Π°Π±ΠΎΡ€Π°Ρ‚ΠΎΡ€Π½ΠΎΠ³ΠΎ Ρ‚Ρ€ΠΈΠ±ΠΎΠΌΠ΅Ρ‚Ρ€Π°, 2) Π΄ΠΈΠ½Π°ΠΌΠΎΠΌΠ΅Ρ‚Ρ€ ΡƒΡ‡Π΅Π±Π½Ρ‹ΠΉ, 3) Π»Π΅Π½Ρ‚Π° ΠΈΠ·ΠΌΠ΅Ρ€ΠΈΡ‚Π΅Π»ΡŒΠ½Π°Ρ с сантимСтровыми дСлСниями.

1. ΠŸΠΎΠ»ΠΎΠΆΠΈΡ‚Π΅ брусок Π½Π° стол ΠΈ Π·Π°ΠΌΠ΅Ρ‚ΡŒΡ‚Π΅ Π΅Π³ΠΎ Π½Π°Ρ‡Π°Π»ΡŒΠ½ΠΎΠ΅ ΠΏΠΎΠ»ΠΎΒ­ΠΆΠ΅Π½ΠΈΠ΅.

2. Π’ΠΎΠ»ΠΊΠ½ΠΈΡ‚Π΅ слСгка брусок Ρ€ΡƒΠΊΠΎΠΉ ΠΈ Π·Π°ΠΌΠ΅Ρ‚ΡŒΡ‚Π΅ Π΅Π³ΠΎ Π½ΠΎΠ²ΠΎΠ΅ ΠΏΠΎΠ»ΠΎΒ­ΠΆΠ΅Π½ΠΈΠ΅ Π½Π° столС (см. рис.).

3. Π˜Π·ΠΌΠ΅Ρ€ΡŒΡ‚Π΅ Ρ‚ΠΎΡ€ΠΌΠΎΠ·Π½ΠΎΠΉ ΠΏΡƒΡ‚ΡŒ бруска ΠΎΡ‚Π½ΠΎΡΠΈΡ‚Π΅Π»ΡŒΠ½ΠΎ стола._________

4. Π˜Π·ΠΌΠ΅Ρ€ΡŒΡ‚Π΅ ΠΌΠΎΠ΄ΡƒΠ»ΡŒ вСса бруска ΠΈ вычислитС Π΅Π³ΠΎ массу.__

5. Π˜Π·ΠΌΠ΅Ρ€ΡŒΡ‚Π΅ ΠΌΠΎΠ΄ΡƒΠ»ΡŒ силы трСния скольТСния бруска ΠΏΠΎ столу.___________________________________________________________

6. Зная массу, Ρ‚ΠΎΡ€ΠΌΠΎΠ·Π½ΠΎΠΉ ΠΏΡƒΡ‚ΡŒ ΠΈ ΠΌΠΎΠ΄ΡƒΠ»ΡŒ силы трСния скольТСния, вычислитС ΠΌΠΎΠ΄ΡƒΠ»ΡŒ Π½Π°Ρ‡Π°Π»ΡŒΠ½ΠΎΠΉ скорости ΠΈ врСмя тормоТСния бруска.______________________________________________

7. Π—Π°ΠΏΠΈΡˆΠΈΡ‚Π΅ Ρ€Π΅Π·ΡƒΠ»ΡŒΡ‚Π°Ρ‚Ρ‹ ΠΈΠ·ΠΌΠ΅Ρ€Π΅Π½ΠΈΠΉ ΠΈ вычислСний.__________

2. Π˜Π·ΠΌΠ΅Ρ€Π΅Π½ΠΈΠ΅ модуля ускорСния Ρ‚Π΅Π»Π°, двиТущСгося ΠΏΠΎΠ΄ дСйствиСм сил упругости ΠΈ трСния

ΠŸΡ€ΠΈΠ±ΠΎΡ€Ρ‹ ΠΈ ΠΌΠ°Ρ‚Π΅Ρ€ΠΈΠ°Π»Ρ‹: 1) Ρ‚Ρ€ΠΈΠ±ΠΎΠΌΠ΅Ρ‚Ρ€ Π»Π°Π±ΠΎΡ€Π°Ρ‚ΠΎΡ€Π½Ρ‹ΠΉ, 2) Π΄ΠΈΠ½Π°ΠΌΠΎΠΌΠ΅Ρ‚Ρ€ ΡƒΡ‡Π΅Π±Π½Ρ‹ΠΉ с фиксатором.

ΠŸΠΎΡ€ΡΠ΄ΠΎΠΊ выполнСния Ρ€Π°Π±ΠΎΡ‚Ρ‹

1. Π˜Π·ΠΌΠ΅Ρ€ΡŒΡ‚Π΅ ΠΌΠΎΠ΄ΡƒΠ»ΡŒ вСса бруска с ΠΏΠΎΠΌΠΎΡ‰ΡŒΡŽ Π΄ΠΈΠ½Π°ΠΌΠΎΠΌΠ΅Ρ‚Ρ€Π°._______

_________________________________________________________________.

2. Π—Π°Ρ†Π΅ΠΏΠΈΡ‚Π΅ Π΄ΠΈΠ½Π°ΠΌΠΎΠΌΠ΅Ρ‚Ρ€ Π·Π° брусок ΠΈ ΠΏΠΎΠ»ΠΎΠΆΠΈΡ‚Π΅ ΠΈΡ… Π½Π° Π»ΠΈΠ½Π΅ΠΉΠΊΡƒ Ρ‚Ρ€ΠΈΠ±ΠΎΠΌΠ΅Ρ‚Ρ€Π°. Π£ΠΊΠ°Π·Π°Ρ‚Π΅Π»ΡŒ Π΄ΠΈΠ½Π°ΠΌΠΎΠΌΠ΅Ρ‚Ρ€Π° установитС Π½Π° Π½ΡƒΠ»Π΅Π²ΠΎΠ΅ Π΄Π΅Π»Π΅Π½ΠΈΠ΅ ΡˆΠΊΠ°Π»Ρ‹, Π° фиксатор - ΠΎΠΊΠΎΠ»ΠΎ ΡƒΠΏΠΎΡ€Π° (см. рис.).

3. ΠŸΡ€ΠΈΠ²Π΅Π΄ΠΈΡ‚Π΅ брусок Π² Ρ€Π°Π²Π½ΠΎΠΌΠ΅Ρ€Π½ΠΎΠ΅ Π΄Π²ΠΈΠΆΠ΅Π½ΠΈΠ΅ вдоль Π»ΠΈΠ½Π΅ΠΉΠΊΠΈ Ρ‚Ρ€ΠΈΠ±ΠΎΠΌΠ΅Ρ‚Ρ€Π° ΠΈ ΠΈΠ·ΠΌΠ΅Ρ€ΡŒΡ‚Π΅ ΠΌΠΎΠ΄ΡƒΠ»ΡŒ силы трСния скольТСния. ________

_________________________________________________________________.

4. ΠŸΡ€ΠΈΠ²Π΅Π΄ΠΈΡ‚Π΅ брусок Π² ускорСнноС Π΄Π²ΠΈΠΆΠ΅Π½ΠΈΠ΅ вдоль Π»ΠΈΠ½Π΅ΠΉΠΊΠΈ Ρ‚Ρ€ΠΈΠ±ΠΎΠΌΠ΅Ρ‚Ρ€Π°, подСйствовав Π½Π° Π½Π΅Π³ΠΎ силой, большСй модуля силы трСния скольТСния. Π˜Π·ΠΌΠ΅Ρ€ΡŒΡ‚Π΅ ΠΌΠΎΠ΄ΡƒΠ»ΡŒ этой силы. __________________

_________________________________________________________________.

5. По ΠΏΠΎΠ»ΡƒΡ‡Π΅Π½Π½Ρ‹ΠΌ Π΄Π°Π½Π½Ρ‹ΠΌ вычислитС ΠΌΠΎΠ΄ΡƒΠ»ΡŒ ускорСния бруска._

_________________________________________________________________.

__________________________________________________________________

2. ΠŸΠ΅Ρ€Π΅ΠΌΠ΅ΡΡ‚ΠΈΡ‚Π΅ брусок с Π³Ρ€ΡƒΠ·Π°ΠΌΠΈ Ρ€Π°Π²Π½ΠΎΠΌΠ΅Ρ€Π½ΠΎ вдоль Π»ΠΈΠ½Π΅ΠΉΠΊΠΈ Ρ‚Ρ€ΠΈΠ±ΠΎΠΌΠ΅Ρ‚Ρ€Π° ΠΈ Π·Π°ΠΏΠΈΡˆΠΈΡ‚Π΅ показания Π΄ΠΈΠ½Π°ΠΌΠΎΠΌΠ΅Ρ‚Ρ€Π° с Ρ‚ΠΎΡ‡Π½ΠΎΡΡ‚ΡŒΡŽ Π΄ΠΎ 0,1 Н.__________________________________________________________.

3. Π˜Π·ΠΌΠ΅Ρ€ΡŒΡ‚Π΅ ΠΌΠΎΠ΄ΡƒΠ»ΡŒ пСрСмСщСния бруска с Ρ‚ΠΎΡ‡Π½ΠΎΡΡ‚ΡŒΡŽ Π΄ΠΎ 0,005 ΠΌ

ΠΎΡ‚Π½ΠΎΡΠΈΡ‚Π΅Π»ΡŒΠ½ΠΎ стола. ___________________________________________.

__________________________________________________________________

5. ВычислитС Π°Π±ΡΠΎΠ»ΡŽΡ‚Π½ΡƒΡŽ ΠΈ ΠΎΡ‚Π½ΠΎΡΠΈΡ‚Π΅Π»ΡŒΠ½ΡƒΡŽ ΠΏΠΎΠ³Ρ€Π΅ΡˆΠ½ΠΎΡΡ‚ΠΈ измСрСния Ρ€Π°Π±ΠΎΡ‚Ρ‹._______________________________________________

__________________________________________________________________

6. Π—Π°ΠΏΠΈΡˆΠΈΡ‚Π΅ Ρ€Π΅Π·ΡƒΠ»ΡŒΡ‚Π°Ρ‚Ρ‹ ΠΈΠ·ΠΌΠ΅Ρ€Π΅Π½ΠΈΠΉ ΠΈ вычислСний.__________

__________________________________________________________________

_________________________________________________________________

ΠžΡ‚Π²Π΅Ρ‚ΡŒΡ‚Π΅ Π½Π° вопросы:

1. Как Π½Π°ΠΏΡ€Π°Π²Π»Π΅Π½ Π²Π΅ΠΊΡ‚ΠΎΡ€ силы тяги ΠΎΡ‚Π½ΠΎΡΠΈΡ‚Π΅Π»ΡŒΠ½ΠΎ Π²Π΅ΠΊΡ‚ΠΎΡ€Π° пСрСмСщСния бруска?_____________________________________________

_________________________________________________________________.

2. Какой Π·Π½Π°ΠΊ ΠΈΠΌΠ΅Π΅Ρ‚ Ρ€Π°Π±ΠΎΡ‚Π°, ΡΠΎΠ²Π΅Ρ€ΡˆΠ΅Π½Π½Π°Ρ силой тяги ΠΏΠΎ ΠΏΠ΅Ρ€Π΅ΠΌΠ΅Ρ‰Π΅Π½ΠΈΡŽ бруска?____________________________________________

__________________________________________________________________

Π’Π°Ρ€ΠΈΠ°Π½Ρ‚ 2.

1. ΠŸΠΎΠ»ΠΎΠΆΠΈΡ‚Π΅ брусок с двумя Π³Ρ€ΡƒΠ·Π°ΠΌΠΈ Π½Π° Π»ΠΈΠ½Π΅ΠΉΠΊΡƒ Ρ‚Ρ€ΠΈΠ±ΠΎΠΌΠ΅Ρ‚Ρ€Π°. Π—Π° ΠΊΡ€ΡŽΡ‡ΠΎΠΊ бруска Π·Π°Ρ†Π΅ΠΏΠΈΡ‚Π΅ Π΄ΠΈΠ½Π°ΠΌΠΎΠΌΠ΅Ρ‚Ρ€, располоТив Π΅Π³ΠΎ ΠΏΠΎΠ΄ ΡƒΠ³Π»ΠΎΠΌ 30Β° ΠΊ Π»ΠΈΠ½Π΅ΠΉΠΊΠ΅ (см. рис.). Π£Π³ΠΎΠ» Π½Π°ΠΊΠ»ΠΎΠ½Π° Π΄ΠΈΠ½Π°ΠΌΠΎΠΌΠ΅Ρ‚Ρ€Π° ΠΏΡ€ΠΎΠ²Π΅Ρ€ΡŒΡ‚Π΅ с ΠΏΠΎΠΌΠΎΡ‰ΡŒΡŽ ΡƒΠ³ΠΎΠ»ΡŒΠ½ΠΈΠΊΠ°.

2. ΠŸΠ΅Ρ€Π΅ΠΌΠ΅ΡΡ‚ΠΈΡ‚Π΅ Ρ€Π°Π²Π½ΠΎΠΌΠ΅Ρ€Π½ΠΎ брусок с Π³Ρ€ΡƒΠ·Π°ΠΌΠΈ ΠΏΠΎ Π»ΠΈΠ½Π΅ΠΉΠΊΠ΅, сохраняя ΠΏΠ΅Ρ€Π²ΠΎΠ½Π°Ρ‡Π°Π»ΡŒΠ½ΠΎΠ΅ Π½Π°ΠΏΡ€Π°Π²Π»Π΅Π½ΠΈΠ΅ силы тяги. Π—Π°ΠΏΠΈΡˆΠΈΡ‚Π΅ показания Π΄ΠΈΠ½Π°ΠΌΠΎΠΌΠ΅Ρ‚Ρ€Π° с Ρ‚ΠΎΡ‡Π½ΠΎΡΡ‚ΡŒΡŽ Π΄ΠΎ 0,1 Н.____________________

_________________________________________________________________.

3. Π˜Π·ΠΌΠ΅Ρ€ΡŒΡ‚Π΅ ΠΌΠΎΠ΄ΡƒΠ»ΡŒ пСрСмСщСния бруска с Ρ‚ΠΎΡ‡Π½ΠΎΡΡ‚ΡŒΡŽ Π΄ΠΎ 0,005 ΠΌ ΠΎΡ‚Π½ΠΎΡΠΈΡ‚Π΅Π»ΡŒΠ½ΠΎ стола._______________________________________________

4. ВычислитС Ρ€Π°Π±ΠΎΡ‚Ρƒ силы тяги ΠΏΠΎ ΠΏΠ΅Ρ€Π΅ΠΌΠ΅Ρ‰Π΅Π½ΠΈΡŽ бруска ΠΎΡ‚Π½ΠΎΡΠΈΡ‚Π΅Π»ΡŒΠ½ΠΎ стола._______________________________________________

__________________________________________________________________

__________________________________________________________________.

5. Π—Π°ΠΏΠΈΡˆΠΈΡ‚Π΅ Ρ€Π΅Π·ΡƒΠ»ΡŒΡ‚Π°Ρ‚Ρ‹ ΠΈΠ·ΠΌΠ΅Ρ€Π΅Π½ΠΈΠΉ ΠΈ вычислСний.__________

__________________________________________________________________

ΠžΡ‚Π²Π΅Ρ‚ΡŒΡ‚Π΅ Π½Π° вопросы:

1. Как Π½Π°ΠΏΡ€Π°Π²Π»Π΅Π½ Π²Π΅ΠΊΡ‚ΠΎΡ€ силы тяги ΠΎΡ‚Π½ΠΎΡΠΈΡ‚Π΅Π»ΡŒΠ½ΠΎ Π²Π΅ΠΊΡ‚ΠΎΡ€Π° пСрСмСщСния бруска? ____________________________________________

_________________________________________________________________.

2. Какой Π·Π½Π°ΠΊ ΠΈΠΌΠ΅Π΅Ρ‚ Ρ€Π°Π±ΠΎΡ‚Π° силы тяги ΠΏΠΎ ΠΏΠ΅Ρ€Π΅ΠΌΠ΅Ρ‰Π΅Π½ΠΈΡŽ бруска?

_________________________________________________________________.

_________________________________________________________________

4. Π˜Π·ΠΌΠ΅Ρ€Π΅Π½ΠΈΠ΅ ΠšΠŸΠ” ΠΏΠΎΠ΄Π²ΠΈΠΆΠ½ΠΎΠ³ΠΎ Π±Π»ΠΎΠΊΠ°

П Ρ€ΠΈΠ±ΠΎΡ€Ρ‹ ΠΈ ΠΌΠ°Ρ‚Π΅Ρ€ΠΈΠ°Π»Ρ‹ : 1) Π±Π»ΠΎΠΊ, 2) Π΄ΠΈΠ½Π°ΠΌΠΎΠΌΠ΅Ρ‚Ρ€ ΡƒΡ‡Π΅Π±Π½Ρ‹ΠΉ, 3) Π»Π΅Π½Ρ‚Π° ΠΈΠ·ΠΌΠ΅Ρ€ΠΈΡ‚Π΅Π»ΡŒΠ½Π°Ρ с сантимСтровыми дСлСниями, 4) Π³Ρ€ΡƒΠ·Ρ‹ массой ΠΏΠΎ 100 Π³ с двумя ΠΊΡ€ΡŽΡ‡ΠΊΠ°ΠΌΠΈ – 3 ΡˆΡ‚., 5) ΡˆΡ‚Π°Ρ‚ΠΈΠ² с Π»Π°ΠΏΠΊΠΎΠΉ, 6) Π½ΠΈΡ‚ΡŒ Π΄Π»ΠΈΠ½ΠΎΠΉ 50 см с пСтлями Π½Π° ΠΊΠΎΠ½Ρ†Π°Ρ….

ΠŸΠΎΡ€ΡΠ΄ΠΎΠΊ выполнСния Ρ€Π°Π±ΠΎΡ‚Ρ‹

1. Π‘ΠΎΠ±Π΅Ρ€ΠΈΡ‚Π΅ установку с ΠΏΠΎΠ΄Π²ΠΈΠΆΠ½Ρ‹ΠΌ Π±Π»ΠΎΠΊΠΎΠΌ, ΠΊΠ°ΠΊ ΠΏΠΎΠΊΠ°Π·Π°Π½ΠΎ Π½Π° рисункС. Π§Π΅Ρ€Π΅Π· Π±Π»ΠΎΠΊ ΠΏΠ΅Ρ€Π΅Π±Ρ€ΠΎΡΡŒΡ‚Π΅ Π½ΠΈΡ‚ΡŒ. Один ΠΊΠΎΠ½Π΅Ρ† Π½ΠΈΡ‚ΠΈ Π·Π°Ρ†Π΅ΠΏΠΈΡ‚Π΅ Π·Π° Π»Π°ΠΏΠΊΡƒ ΡˆΡ‚Π°Ρ‚ΠΈΠ²Π°, Π²Ρ‚ΠΎΡ€ΠΎΠΉ - Π·Π° ΠΊΡ€ΡŽΡ‡ΠΎΠΊ Π΄ΠΈΠ½Π°ΠΌΠΎΠΌΠ΅Ρ‚Ρ€Π°. К ΠΎΠ±ΠΎΠΉΠΌΠ΅ Π±Π»ΠΎΠΊΠ° ΠΏΠΎΠ΄Π²Π΅ΡΡŒΡ‚Π΅ Ρ‚Ρ€ΠΈ Π³Ρ€ΡƒΠ·Π° массой ΠΏΠΎ 100 Π³.

2.Π’ΠΎΠ·ΡŒΠΌΠΈΡ‚Π΅ Π΄ΠΈΠ½Π°ΠΌΠΎΠΌΠ΅Ρ‚Ρ€ Π² Ρ€ΡƒΠΊΡƒ, располоТитС Π΅Π³ΠΎ Π²Π΅Ρ€Ρ‚ΠΈΠΊΠ°Π»ΡŒΠ½ΠΎ Ρ‚Π°ΠΊ, Ρ‡Ρ‚ΠΎΠ±Ρ‹ Π±Π»ΠΎΠΊ с Π³Ρ€ΡƒΠ·Π°ΠΌΠΈ повис Π½Π° нитях, ΠΈ ΠΈΠ·ΠΌΠ΅Ρ€ΡŒΡ‚Π΅ ΠΌΠΎΠ΄ΡƒΠ»ΡŒ силы натяТСния Π½ΠΈΡ‚ΠΈ._____________

___________________________________________

3.ΠŸΠΎΠ΄Π½ΠΈΠΌΠΈΡ‚Π΅ Ρ€Π°Π²Π½ΠΎΠΌΠ΅Ρ€Π½ΠΎ Π³Ρ€ΡƒΠ·Ρ‹ Π½Π° Π½Π΅ΠΊΠΎΡ‚ΠΎΡ€ΡƒΡŽ высоту ΠΈ ΠΈΠ·ΠΌΠ΅Ρ€ΡŒΡ‚Π΅ ΠΌΠΎΠ΄ΡƒΠ»ΠΈ ΠΏΠ΅Ρ€Π΅ΠΌΠ΅Ρ‰Π΅Π½ΠΈΠΉ Π³Ρ€ΡƒΠ·ΠΎΠ² ΠΈ Π΄ΠΈΠ½Π°ΠΌΠΎΠΌΠ΅Ρ‚Ρ€Π° ΠΎΡ‚Π½ΠΎΡΠΈΡ‚Π΅Π»ΡŒΠ½ΠΎ стола. ___________________________________________________________

_________________________________________________________________.

4.ВычислитС ΠΏΠΎΠ»Π΅Π·Π½ΡƒΡŽ ΠΈ ΡΠΎΠ²Π΅Ρ€ΡˆΠ΅Π½Π½ΡƒΡŽ Ρ€Π°Π±ΠΎΡ‚Ρ‹ ΠΎΡ‚Π½ΠΎΡΠΈΡ‚Π΅Π»ΡŒΠ½ΠΎ стола. ___________________________________________________________

__________________________________________________________________

5.ВычислитС ΠšΠŸΠ” ΠΏΠΎΠ΄Π²ΠΈΠΆΠ½ΠΎΠ³ΠΎ Π±Π»ΠΎΠΊΠ°. ________________________

ΠžΡ‚Π²Π΅Ρ‚ΡŒΡ‚Π΅ Π½Π° вопросы:

1.Какой Π²Ρ‹ΠΈΠ³Ρ€Ρ‹Ρˆ Π² силС Π΄Π°Π΅Ρ‚ ΠΏΠΎΠ΄Π²ΠΈΠΆΠ½Ρ‹ΠΉ Π±Π»ΠΎΠΊ?______________

2.МоТно Π»ΠΈ ΠΏΡ€ΠΈ ΠΏΠΎΠΌΠΎΡ‰ΠΈ ΠΏΠΎΠ΄Π²ΠΈΠΆΠ½ΠΎΠ³ΠΎ Π±Π»ΠΎΠΊΠ° ΠΏΠΎΠ»ΡƒΡ‡ΠΈΡ‚ΡŒ Π²Ρ‹ΠΈΠ³Ρ€Ρ‹Ρˆ Π² Ρ€Π°Π±ΠΎΡ‚Π΅? _______________________________________________

_________________________________________________________________

3.Как ΠΏΠΎΠ²Ρ‹ΡΠΈΡ‚ΡŒ ΠšΠŸΠ” ΠΏΠΎΠ΄Π²ΠΈΠΆΠ½ΠΎΠ³ΠΎ Π±Π»ΠΎΠΊΠ°?_____________________

____________________________________________________________________________________________________________________________________________________________________________________________________.

5. Π˜Π·ΠΌΠ΅Ρ€Π΅Π½ΠΈΠ΅ ΠΌΠΎΠΌΠ΅Π½Ρ‚Π° силы

П Ρ€ΠΈΠ±ΠΎΡ€Ρ‹ ΠΈ ΠΌΠ°Ρ‚Π΅Ρ€ΠΈΠ°Π»Ρ‹ : 1) ΠΆΠ΅Π»ΠΎΠ± Π»Π°Π±ΠΎΡ€Π°Ρ‚ΠΎΡ€Π½Ρ‹ΠΉ, 2) Π΄ΠΈΠ½Π°ΠΌΠΎΠΌΠ΅Ρ‚Ρ€ ΡƒΡ‡Π΅Π±Π½Ρ‹ΠΉ, 3) Π»Π΅Π½Ρ‚Π° ΠΈΠ·ΠΌΠ΅Ρ€ΠΈΡ‚Π΅Π»ΡŒΠ½Π°Ρ с сантимСтровыми дСлСниями, 4) пСтля ΠΈΠ· ΠΏΡ€ΠΎΡ‡Π½ΠΎΠΉ Π½ΠΈΡ‚ΠΈ.

ΠŸΠΎΡ€ΡΠ΄ΠΎΠΊ выполнСния Ρ€Π°Π±ΠΎΡ‚Ρ‹

1.ΠΠ°Π΄Π΅Π½ΡŒΡ‚Π΅ ΠΏΠ΅Ρ‚Π»ΡŽ Π½Π° ΠΊΠΎΠ½Π΅Ρ† ΠΆΠ΅Π»ΠΎΠ±Π° ΠΈ Π·Π°Ρ†Π΅ΠΏΠΈΡ‚Π΅ Π΅Π΅ Π΄ΠΈΠ½Π°ΠΌΠΎΠΌΠ΅Ρ‚Ρ€ΠΎΠΌ, ΠΊΠ°ΠΊ ΠΏΠΎΠΊΠ°Π·Π°Π½ΠΎ Π½Π° рисункС. Поднимая Π΄ΠΈΠ½Π°ΠΌΠΎΠΌΠ΅Ρ‚Ρ€, ΠΏΠΎΠ²ΠΎΡ€Π°Ρ‡ΠΈΠ²Π°ΠΉΡ‚Π΅ ΠΆΠ΅Π»ΠΎΠ± Π²ΠΎΠΊΡ€ΡƒΠ³ Π³ΠΎΡ€ΠΈΠ·ΠΎΠ½Ρ‚Π°Π»ΡŒΠ½ΠΎΠΉ оси, проходящСй Ρ‡Π΅Ρ€Π΅Π· Π΄Ρ€ΡƒΠ³ΠΎΠΉ Π΅Π³ΠΎ ΠΊΠΎΠ½Π΅Ρ†.

2.Π˜Π·ΠΌΠ΅Ρ€ΡŒΡ‚Π΅ ΠΌΠΎΠ΄ΡƒΠ»ΡŒ силы, Π½Π΅ΠΎΠ±Ρ…ΠΎΠ΄ΠΈΠΌΠΎΠΉ для вращСния ΠΆΠ΅Π»ΠΎΠ±Π°._

3.Π˜Π·ΠΌΠ΅Ρ€ΡŒΡ‚Π΅ ΠΏΠ»Π΅Ρ‡ΠΎ этой силы. ________________________________.

4.ВычислитС ΠΌΠΎΠΌΠ΅Π½Ρ‚ этой силы.______________________________

__________________________________________________________________.

5.ΠŸΠ΅Ρ€Π΅Π΄Π²ΠΈΠ½ΡŒΡ‚Π΅ ΠΏΠ΅Ρ‚Π»ΡŽ Π² сСрСдину ΠΆΠ΅Π»ΠΎΠ±Π°, ΠΈ снова ΠΈΠ·ΠΌΠ΅Ρ€ΡŒΡ‚Π΅ ΠΌΠΎΠ΄ΡƒΠ»ΡŒ силы, Π½Π΅ΠΎΠ±Ρ…ΠΎΠ΄ΠΈΠΌΠΎΠΉ для вращСния ΠΆΠ΅Π»ΠΎΠ±Π°, ΠΈ Π΅Π΅ ΠΏΠ»Π΅Ρ‡ΠΎ.______

___________________________________________________________________________________________________________________________________.

6.ВычислитС ΠΌΠΎΠΌΠ΅Π½Ρ‚ Π²Ρ‚ΠΎΡ€ΠΎΠΉ силы. ___________________________

_________________________________________________________________.

7.Π‘Ρ€Π°Π²Π½ΠΈΡ‚Π΅ вычислСнныС ΠΌΠΎΠΌΠ΅Π½Ρ‚Ρ‹ сил. Π‘Π΄Π΅Π»Π°ΠΉΡ‚Π΅ Π²Ρ‹Π²ΠΎΠ΄. _____

_______________________________________________________________________________________________________________________________________________________________________________________________________________________________________________________________________.

6. Β«Π˜Π·ΠΌΠ΅Ρ€Π΅Π½ΠΈΠ΅ ТСсткости ΠΏΡ€ΡƒΠΆΠΈΠ½Ρ‹.

ЦСль Ρ€Π°Π±ΠΎΡ‚Ρ‹: Π½Π°ΠΉΡ‚ΠΈ ΠΆΠ΅ΡΡ‚ΠΊΠΎΡΡ‚ΡŒ ΠΏΡ€ΡƒΠΆΠΈΠ½Ρ‹.

ΠœΠ°Ρ‚Π΅Ρ€ΠΈΠ°Π»Ρ‹ : 1) ΡˆΡ‚Π°Ρ‚ΠΈΠ² с ΠΌΡƒΡ„Ρ‚Π°ΠΌΠΈ ΠΈ Π»Π°ΠΏΠΊΠΎΠΉ; 2) ΡΠΏΠΈΡ€Π°Π»ΡŒΠ½Π°Ρ ΠΏΡ€ΡƒΠΆΠΈΠ½Π°.

ΠŸΠΎΡ€ΡΠ΄ΠΎΠΊ выполнСния Ρ€Π°Π±ΠΎΡ‚Ρ‹:

Π—Π°ΠΊΡ€Π΅ΠΏΠΈΡ‚Π΅ Π½Π° ΡˆΡ‚Π°Ρ‚ΠΈΠ²Π΅ ΠΊΠΎΠ½Π΅Ρ† ΡΠΏΠΈΡ€Π°Π»ΡŒΠ½ΠΎΠΉ ΠΏΡ€ΡƒΠΆΠΈΠ½Ρ‹ (Π΄Ρ€ΡƒΠ³ΠΎΠΉ ΠΊΠΎΠ½Π΅Ρ† ΠΏΡ€ΡƒΠΆΠΈΠ½Ρ‹ снабТСн стрСлкой - ΡƒΠΊΠ°Π·Π°Ρ‚Π΅Π»Π΅ΠΌ ΠΈ ΠΊΡ€ΡŽΡ‡ΠΊΠΎΠΌ).

Рядом с ΠΏΡ€ΡƒΠΆΠΈΠ½ΠΎΠΉ ΠΈΠ»ΠΈ Π·Π° Π½Π΅ΠΉ установитС ΠΈ Π·Π°ΠΊΡ€Π΅ΠΏΠΈΡ‚Π΅ Π»ΠΈΠ½Π΅ΠΉΠΊΡƒ с ΠΌΠΈΠ»Π»ΠΈΠΌΠ΅Ρ‚Ρ€ΠΎΠ²Ρ‹ΠΌΠΈ дСлСниями.

ΠžΡ‚ΠΌΠ΅Ρ‚ΡŒΡ‚Π΅ ΠΈ Π·Π°ΠΏΠΈΡˆΠΈΡ‚Π΅ Ρ‚ΠΎ Π΄Π΅Π»Π΅Π½ΠΈΠ΅ Π»ΠΈΠ½Π΅ΠΉΠΊΠΈ, ΠΏΡ€ΠΎΡ‚ΠΈΠ² ΠΊΠΎΡ‚ΠΎΡ€ΠΎΠ³ΠΎ приходится стрСлка-ΡƒΠΊΠ°Π·Π°Ρ‚Π΅Π»ΡŒ ΠΏΡ€ΡƒΠΆΠΈΠ½Ρ‹. __________________________

ΠŸΠΎΠ΄Π²Π΅ΡΡŒΡ‚Π΅ ΠΊ ΠΏΡ€ΡƒΠΆΠΈΠ½Π΅ Π³Ρ€ΡƒΠ· извСстной массы ΠΈ ΠΈΠ·ΠΌΠ΅Ρ€ΡŒΡ‚Π΅ Π²Ρ‹Π·Π²Π°Π½Π½ΠΎΠ΅ ΠΈΠΌ ΡƒΠ΄Π»ΠΈΠ½Π΅Π½ΠΈΠ΅ ΠΏΡ€ΡƒΠΆΠΈΠ½Ρ‹.________________________________

___________________________________________________________________

К ΠΏΠ΅Ρ€Π²ΠΎΠΌΡƒ Π³Ρ€ΡƒΠ·Ρƒ Π΄ΠΎΠ±Π°Π²ΡŒΡ‚Π΅ Π²Ρ‚ΠΎΡ€ΠΎΠΉ, Ρ‚Ρ€Π΅Ρ‚ΠΈΠΉ ΠΈ Ρ‚. Π΄. Π³Ρ€ΡƒΠ·Ρ‹, записывая ΠΊΠ°ΠΆΠ΄Ρ‹ΠΉ Ρ€Π°Π· ΡƒΠ΄Π»ΠΈΠ½Π΅Π½ΠΈΠ΅ /Ρ…/ ΠΏΡ€ΡƒΠΆΠΈΠ½Ρ‹. По Ρ€Π΅Π·ΡƒΠ»ΡŒΡ‚Π°Ρ‚Π°ΠΌ ΠΈΠ·ΠΌΠ΅Ρ€Π΅Π½ΠΈΠΉ Π·Π°ΠΏΠΎΠ»Π½ΠΈΡ‚Π΅ Ρ‚Π°Π±Π»ΠΈΡ†Ρƒ _____________________________________

___________________________________________________________________

__________________________________________________________________.

DIV_ADBLOCK195">

_______________________________________________________________.

3. Π’Π·Π²Π΅ΡΡŒΡ‚Π΅ брусок ΠΈ Π³Ρ€ΡƒΠ·.______________________________________

________________________________________________________________.

4.К ΠΏΠ΅Ρ€Π²ΠΎΠΌΡƒ Π³Ρ€ΡƒΠ·Ρƒ Π΄ΠΎΠ±Π°Π²ΡŒΡ‚Π΅ Π²Ρ‚ΠΎΡ€ΠΎΠΉ, Ρ‚Ρ€Π΅Ρ‚ΠΈΠΉ Π³Ρ€ΡƒΠ·Ρ‹, ΠΊΠ°ΠΆΠ΄Ρ‹ΠΉ Ρ€Π°Π· взвСшивая брусок ΠΈ Π³Ρ€ΡƒΠ·Ρ‹ ΠΈ измСряя силу трСния. _______________

____________________________________________________________________________________________________________________________________________________________________________________________.


5. По Ρ€Π΅Π·ΡƒΠ»ΡŒΡ‚Π°Ρ‚Π°ΠΌ ΠΈΠ·ΠΌΠ΅Ρ€Π΅Π½ΠΈΠΉ постройтС Π³Ρ€Π°Ρ„ΠΈΠΊ зависимости силы трСния ΠΎΡ‚ силы давлСния ΠΈ, ΠΏΠΎΠ»ΡŒΠ·ΡƒΡΡΡŒ ΠΈΠΌ, ΠΎΠΏΡ€Π΅Π΄Π΅Π»ΠΈΡ‚Π΅ срСднСС Π·Π½Π°Ρ‡Π΅Π½ΠΈΠ΅ коэффициСнта трСния ΞΌ ср. ______________________________-

_____________________________________________________________________________________________________________________________________________________________________________________________________.

Лабораторная Ρ€Π°Π±ΠΎΡ‚Π°

Π˜Π·ΠΌΠ΅Ρ€Π΅Π½ΠΈΠ΅ ТСсткости ΠΏΡ€ΡƒΠΆΠΈΠ½Ρ‹

ЦСль Ρ€Π°Π±ΠΎΡ‚Ρ‹ : Π½Π°ΠΉΡ‚ΠΈ ΠΆΠ΅ΡΡ‚ΠΊΠΎΡΡ‚ΡŒ ΠΏΡ€ΡƒΠΆΠΈΠ½Ρ‹ с ΠΏΠΎΠΌΠΎΡ‰ΡŒΡŽ измСрСния удлинСния ΠΏΡ€ΡƒΠΆΠΈΠ½Ρ‹ ΠΏΡ€ΠΈ ΡƒΡ€Π°Π²Π½ΠΎΠ²Π΅ΡˆΠΈΠ²Π°Π½ΠΈΠΈ силы тяТСсти Π³Ρ€ΡƒΠ·Π° силой упругости ΠΏΡ€ΡƒΠΆΠΈΠ½Ρ‹ ΠΈ ΠΏΠΎΡΡ‚Ρ€ΠΎΠΈΡ‚ΡŒ Π³Ρ€Π°Ρ„ΠΈΠΊ зависимости силы упругости Π΄Π°Π½Π½ΠΎΠΉ ΠΏΡ€ΡƒΠΆΠΈΠ½Ρ‹ ΠΎΡ‚ Π΅Π΅ удлинСния.

ΠžΠ±ΠΎΡ€ΡƒΠ΄ΠΎΠ²Π°Π½ΠΈΠ΅: Π½Π°Π±ΠΎΡ€ Π³Ρ€ΡƒΠ·ΠΎΠ²; Π»ΠΈΠ½Π΅ΠΉΠΊΠ° с ΠΌΠΈΠ»Π»ΠΈΠΌΠ΅Ρ‚Ρ€ΠΎΠ²Ρ‹ΠΌΠΈ дСлСниями; ΡˆΡ‚Π°Ρ‚ΠΈΠ² с ΠΌΡƒΡ„Ρ‚ΠΎΠΉ ΠΈ Π»Π°ΠΏΠΊΠΎΠΉ; ΡΠΏΠΈΡ€Π°Π»ΡŒΠ½Π°Ρ ΠΏΡ€ΡƒΠΆΠΈΠ½Π° (Π΄ΠΈΠ½Π°ΠΌΠΎΠΌΠ΅Ρ‚Ρ€).

Вопросы для самоподготовки

1. Как ΠΎΠΏΡ€Π΅Π΄Π΅Π»ΠΈΡ‚ΡŒ силу тяТСсти Π³Ρ€ΡƒΠ·Π°?_________________________

__________________________________________________________________

__________________________________________________________________

__________________________________________________________________

__________________________________________________________________

__________________________________________________________________

4. Π“Ρ€ΡƒΠ· Π½Π΅ΠΏΠΎΠ΄Π²ΠΈΠΆΠ½ΠΎ висит Π½Π° ΠΏΡ€ΡƒΠΆΠΈΠ½Π΅. Π§Ρ‚ΠΎ ΠΌΠΎΠΆΠ½ΠΎ ΡΠΊΠ°Π·Π°Ρ‚ΡŒ Π² этом случаС ΠΎ силС тяТСсти Π³Ρ€ΡƒΠ·Π° ΠΈ ΠΎ силС упругости ΠΏΡ€ΡƒΠΆΠΈΠ½Ρ‹? _________

__________________________________________________________________

__________________________________________________________________

5. Как с ΠΏΠΎΠΌΠΎΡ‰ΡŒΡŽ ΡƒΠΊΠ°Π·Π°Π½Π½ΠΎΠ³ΠΎ оборудования ΠΌΠΎΠΆΠ½ΠΎ ΠΈΠ·ΠΌΠ΅Ρ€ΠΈΡ‚ΡŒ ΠΆΠ΅ΡΡ‚ΠΊΠΎΡΡ‚ΡŒ ΠΏΡ€ΡƒΠΆΠΈΠ½Ρ‹? _____________________________________________

__________________________________________________________________

__________________________________________________________________

6. Как, зная ΠΆΠ΅ΡΡ‚ΠΊΠΎΡΡ‚ΡŒ, ΠΏΠΎΡΡ‚Ρ€ΠΎΠΈΡ‚ΡŒ Π³Ρ€Π°Ρ„ΠΈΠΊ зависимости силы упругости ΠΎΡ‚ удлинСния ΠΏΡ€ΡƒΠΆΠΈΠ½Ρ‹?________________________________

__________________________________________________________________

__________________________________________________________________

ΠŸΡ€ΠΈΠΌΠ΅Ρ‡Π°Π½ΠΈΠ΅ . ΠŸΡ€ΠΈΠΌΠΈΡ‚Π΅ ускорСниС свободного падСния Ρ€Π°Π²Π½Ρ‹ΠΌ (10 Β±0,2) ΠΌ/с2, массу ΠΎΠ΄Π½ΠΎΠ³ΠΎ Π³Ρ€ΡƒΠ·Π° (0,100 Β± 0,002) ΠΊΠ³, массу Π΄Π²ΡƒΡ… Π³Ρ€ΡƒΠ·ΠΎΠ² - (0,200Β±0,004) ΠΊΠ³ ΠΈ Ρ‚. Π΄. Достаточно ΡΠ΄Π΅Π»Π°Ρ‚ΡŒ Ρ‚Ρ€ΠΈ ΠΎΠΏΡ‹Ρ‚Π°.

Лабораторная Ρ€Π°Π±ΠΎΡ‚Π°

Β«Π˜Π·ΠΌΠ΅Ρ€Π΅Π½ΠΈΠ΅ коэффициСнта трСния скольТСния»

ЦСль Ρ€Π°Π±ΠΎΡ‚Ρ‹ : ΠΎΠΏΡ€Π΅Π΄Π΅Π»ΠΈΡ‚ΡŒ коэффициСнт трСния.

ΠœΠ°Ρ‚Π΅Ρ€ΠΈΠ°Π»Ρ‹: 1) дСрСвянный брусок; 2) дСрСвянная Π»ΠΈΠ½Π΅ΠΉΠΊΠ°; 3) Π½Π°Π±ΠΎΡ€ Π³Ρ€ΡƒΠ·ΠΎΠ².

ΠŸΠΎΡ€ΡΠ΄ΠΎΠΊ выполнСния Ρ€Π°Π±ΠΎΡ‚Ρ‹

ΠŸΠΎΠ»ΠΎΠΆΠΈΡ‚Π΅ брусок Π½Π° Π³ΠΎΡ€ΠΈΠ·ΠΎΠ½Ρ‚Π°Π»ΡŒΠ½ΠΎ Ρ€Π°ΡΠΏΠΎΠ»ΠΎΠΆΠ΅Π½Π½ΡƒΡŽ Π΄Π΅Ρ€Π΅Π²ΡΠ½Π½ΡƒΡŽ Π»ΠΈΠ½Π΅ΠΉΠΊΡƒ. На брусок ΠΏΠΎΡΡ‚Π°Π²ΡŒΡ‚Π΅ Π³Ρ€ΡƒΠ·.

ΠŸΡ€ΠΈΠΊΡ€Π΅ΠΏΠΈΠ² ΠΊ бруску Π΄ΠΈΠ½Π°ΠΌΠΎΠΌΠ΅Ρ‚Ρ€, ΠΊΠ°ΠΊ ΠΌΠΎΠΆΠ½ΠΎ Π±ΠΎΠ»Π΅Π΅ Ρ€Π°Π²Π½ΠΎΠΌΠ΅Ρ€Π½ΠΎ тянитС Π΅Π³ΠΎ вдоль Π»ΠΈΠ½Π΅ΠΉΠΊΠΈ. Π—Π°ΠΌΠ΅Ρ‚ΡŒΡ‚Π΅ ΠΏΡ€ΠΈ этом ΠΏΠΎΠΊΠ°Π·Π°Π½ΠΈΠ΅ Π΄ΠΈΠ½Π°ΠΌΠΎΠΌΠ΅Ρ‚Ρ€Π°. ____________________________________________________

__________________________________________________________________

Π’Π·Π²Π΅ΡΡŒΡ‚Π΅ брусок ΠΈ Π³Ρ€ΡƒΠ·._________________________________________

К ΠΏΠ΅Ρ€Π²ΠΎΠΌΡƒ Π³Ρ€ΡƒΠ·Ρƒ Π΄ΠΎΠ±Π°Π²ΡŒΡ‚Π΅ Π²Ρ‚ΠΎΡ€ΠΎΠΉ, Ρ‚Ρ€Π΅Ρ‚ΠΈΠΉ Π³Ρ€ΡƒΠ·Ρ‹, ΠΊΠ°ΠΆΠ΄Ρ‹ΠΉ Ρ€Π°Π· взвСшивая бру­сок ΠΈ Π³Ρ€ΡƒΠ·Ρ‹ ΠΈ измСряя силу трСния._________________

_________________________________________________________________

_________________________________________________________________

По Ρ€Π΅Π·ΡƒΠ»ΡŒΡ‚Π°Ρ‚Π°ΠΌ ΠΈΠ·ΠΌΠ΅Ρ€Π΅Π½ΠΈΠΉ Π·Π°ΠΏΠΎΠ»Π½ΠΈΡ‚Π΅ Ρ‚Π°Π±Π»ΠΈΡ†Ρƒ:


5. По Ρ€Π΅Π·ΡƒΠ»ΡŒΡ‚Π°Ρ‚Π°ΠΌ ΠΈΠ·ΠΌΠ΅Ρ€Π΅Π½ΠΈΠΉ постройтС Π³Ρ€Π°Ρ„ΠΈΠΊ зависимости силы трСния ΠΎΡ‚ силы давлСния ΠΈ, ΠΏΠΎΠ»ΡŒΠ·ΡƒΡΡΡŒ ΠΈΠΌ, ΠΎΠΏΡ€Π΅Π΄Π΅Π»ΠΈΡ‚Π΅ срСднСС Π·Π½Π°Ρ‡Π΅Π½ΠΈΠ΅ коэффициСнта трСния ΞΌ. ________________________________

__________________________________________________________________

__________________________________________________________________

6. Π‘Π΄Π΅Π»Π°ΠΉΡ‚Π΅ Π²Ρ‹Π²ΠΎΠ΄.

Лабораторная Ρ€Π°Π±ΠΎΡ‚Π°

Π˜Π·ΡƒΡ‡Π΅Π½ΠΈΠ΅ капиллярных явлСний, обусловлСнных повСрхностным натяТСниСм Тидкости.

ЦСль Ρ€Π°Π±ΠΎΡ‚Ρ‹ : ΠΈΠ·ΠΌΠ΅Ρ€ΠΈΡ‚ΡŒ срСдний Π΄ΠΈΠ°ΠΌΠ΅Ρ‚Ρ€ капилляров.

ΠžΠ±ΠΎΡ€ΡƒΠ΄ΠΎΠ²Π°Π½ΠΈΠ΅ : сосуд с ΠΏΠΎΠ΄ΠΊΡ€Π°ΡˆΠ΅Π½Π½ΠΎΠΉ Π²ΠΎΠ΄ΠΎΠΉ, полоска Ρ„ΠΈΠ»ΡŒΡ‚Ρ€ΠΎΠ²Π°Π»ΡŒΠ½ΠΎΠΉ Π±ΡƒΠΌΠ°Π³ΠΈ Ρ€Π°Π·ΠΌΠ΅Ρ€ΠΎΠΌ 120 Ρ… 10 ΠΌΠΌ, полоска Ρ…Π»ΠΎΠΏΡ‡Π°Ρ‚ΠΎΠ±ΡƒΠΌΠ°ΠΆΠ½ΠΎΠΉ Ρ‚ΠΊΠ°Π½ΠΈ Ρ€Π°Π·ΠΌΠ΅Ρ€ΠΎΠΌ 120 Ρ… 10 ΠΌΠΌ, Π»ΠΈΠ½Π΅ΠΉΠΊΠ° ΠΈΠ·ΠΌΠ΅Ρ€ΠΈΡ‚Π΅Π»ΡŒΠ½Π°Ρ.

Π‘ΠΌΠ°Ρ‡ΠΈΠ²Π°ΡŽΡ‰Π°Ρ ΠΆΠΈΠ΄ΠΊΠΎΡΡ‚ΡŒ втягиваСтся Π²Π½ΡƒΡ‚Ρ€ΡŒ капилляра. ΠŸΠΎΠ΄ΡŠΡ‘ΠΌ Тидкости Π² капиллярС происходит Π΄ΠΎ Ρ‚Π΅Ρ… ΠΏΠΎΡ€, ΠΏΠΎΠΊΠ° Ρ€Π΅Π·ΡƒΠ»ΡŒΡ‚ΠΈΡ€ΡƒΡŽΡ‰Π°Ρ сила, Π΄Π΅ΠΉΡΡ‚Π²ΡƒΡŽΡ‰Π°Ρ Π½Π° ΠΆΠΈΠ΄ΠΊΠΎΡΡ‚ΡŒ Π²Π²Π΅Ρ€Ρ…, FΠ² Π½Π΅ уравновСсится силой тяТСсти mg столба Тидкости высотой h:

По Ρ‚Ρ€Π΅Ρ‚ΡŒΠ΅ΠΌΡƒ Π·Π°ΠΊΠΎΠ½Ρƒ ΠΡŒΡŽΡ‚ΠΎΠ½Π° сила FΠ², Π΄Π΅ΠΉΡΡ‚Π²ΡƒΡŽΡ‰Π°Ρ Π½Π° ΠΆΠΈΠ΄ΠΊΠΎΡΡ‚ΡŒ, Ρ€Π°Π²Π½Π° силС повСрхностного натяТСния FΠΏΠΎΠ², Π΄Π΅ΠΉΡΡ‚Π²ΡƒΡŽΡ‰Π΅ΠΉ Π½Π° стСнку капилляра ΠΏΠΎ Π»ΠΈΠ½ΠΈΠΈ соприкосновСния Π΅Ρ‘ с ΠΆΠΈΠ΄ΠΊΠΎΡΡ‚ΡŒΡŽ:

Π’Π°ΠΊΠΈΠΌ ΠΎΠ±Ρ€Π°Π·ΠΎΠΌ, ΠΏΡ€ΠΈ равновСсии Тидкости Π² капиллярС (рисунок 1)

FΠΏΠΎΠ² = mg. (1)

Π‘ΡƒΠ΄Π΅ΠΌ ΡΡ‡ΠΈΡ‚Π°Ρ‚ΡŒ, Ρ‡Ρ‚ΠΎ мСниск ΠΈΠΌΠ΅Π΅Ρ‚ Ρ„ΠΎΡ€ΠΌΡƒ полусфСры, радиус ΠΊΠΎΡ‚ΠΎΡ€ΠΎΠΉ r Ρ€Π°Π²Π΅Π½ радиусу капилляра. Π”Π»ΠΈΠ½Π° ΠΊΠΎΠ½Ρ‚ΡƒΡ€Π°, ΠΎΠ³Ρ€Π°Π½ΠΈΡ‡ΠΈΠ²Π°ΡŽΡ‰Π΅Π³ΠΎ ΠΏΠΎΠ²Π΅Ρ€Ρ…Π½ΠΎΡΡ‚ΡŒ Тидкости, Ρ€Π°Π²Π½Π° Π΄Π»ΠΈΠ½Π΅ окруТности:

Π’ΠΎΠ³Π΄Π° сила повСрхностного натяТСния Ρ€Π°Π²Π½Π°:

FΠΏΠΎΠ² = Οƒ2Ο€r, (2)

Π³Π΄Π΅ Οƒ – повСрхностноС натяТСниС Тидкости.

рисунок 1

Масса столба Тидкости ΠΎΠ±ΡŠΡ‘ΠΌΠΎΠΌ V = Ο€r2h Ρ€Π°Π²Π½Π°:

m = ρV = ρ Ο€r2h. (3)

ΠŸΠΎΠ΄ΡΡ‚Π°Π²Π»ΡΡ Π²Ρ‹Ρ€Π°ΠΆΠ΅Π½ΠΈΠ΅ (2) для FΠΏΠΎΠ² ΠΈ массы (3) Π² условиС равновСсия Тидкости Π² капиллярС, ΠΏΠΎΠ»ΡƒΡ‡ΠΈΠΌ

Οƒ2Ο€r = ρ Ο€r2hg,

ΠΎΡ‚ΠΊΡƒΠ΄Π° Π΄ΠΈΠ°ΠΌΠ΅Ρ‚Ρ€ капилляра

D = 2r = 4Οƒ/ ρgh. (4)

ΠŸΠΎΡ€ΡΠ΄ΠΎΠΊ выполнСния Ρ€Π°Π±ΠΎΡ‚Ρ‹.

Полосками Ρ„ΠΈΠ»ΡŒΡ‚Ρ€ΠΎΠ²Π°Π»ΡŒΠ½ΠΎΠΉ Π±ΡƒΠΌΠ°Π³ΠΈ ΠΈ Ρ…Π»ΠΎΠΏΡ‡Π°Ρ‚ΠΎΠ±ΡƒΠΌΠ°ΠΆΠ½ΠΎΠΉ Ρ‚ΠΊΠ°Π½ΠΈ ΠΎΠ΄Π½ΠΎΠ²Ρ€Π΅ΠΌΠ΅Π½Π½ΠΎ ΠΏΡ€ΠΈΠΊΠΎΡΠ½ΠΈΡ‚Π΅ΡΡŒ ΠΊ повСрхности ΠΏΠΎΠ΄ΠΊΡ€Π°ΡˆΠ΅Π½Π½ΠΎΠΉ Π²ΠΎΠ΄Ρ‹ Π² стаканС (рисунок 2), наблюдая поднятиС Π²ΠΎΠ΄Ρ‹ Π² полосках.

Как Ρ‚ΠΎΠ»ΡŒΠΊΠΎ прСкратится ΠΏΠΎΠ΄ΡŠΡ‘ΠΌ Π²ΠΎΠ΄Ρ‹, полоски Π²Ρ‹Π½ΡŒΡ‚Π΅ ΠΈ ΠΈΠ·ΠΌΠ΅Ρ€ΡŒΡ‚Π΅ Π»ΠΈΠ½Π΅ΠΉΠΊΠΎΠΉ высоты h1 ΠΈ h2 поднятия Π² Π½ΠΈΡ… Π²ΠΎΠ΄Ρ‹.

ΠΠ±ΡΠΎΠ»ΡŽΡ‚Π½Ρ‹Π΅ ΠΏΠΎΠ³Ρ€Π΅ΡˆΠ½ΠΎΡΡ‚ΠΈ измСрСния Ξ” h1 ΠΈ Ξ” h2 ΠΏΡ€ΠΈΠ½ΠΈΠΌΠ°ΡŽΡ‚ Ρ€Π°Π²Π½Ρ‹ΠΌΠΈ ΡƒΠ΄Π²ΠΎΠ΅Π½Π½ΠΎΠΉ Ρ†Π΅Π½Π΅ дСлСния Π»ΠΈΠ½Π΅ΠΉΠΊΠΈ.

Ξ” h1 = 2 ΠΌΠΌ; Ξ” h2 = 2 ΠΌΠΌ.

РассчитайтС Π΄ΠΈΠ°ΠΌΠ΅Ρ‚Ρ€ капилляров ΠΏΠΎ Ρ„ΠΎΡ€ΠΌΡƒΠ»Π΅ (4).

D2 = 4Οƒ/ ρgh2.

Для Π²ΠΎΠ΄Ρ‹ Οƒ Β± Δσ = (7, 3 Β± 0, 05)Ρ…10-2 Н/ ΠΌ.

РассчитайтС Π°Π±ΡΠΎΠ»ΡŽΡ‚Π½Ρ‹Π΅ ΠΏΠΎΠ³Ρ€Π΅ΡˆΠ½ΠΎΡΡ‚ΠΈ Ξ” D1 ΠΈ Ξ” D2 ΠΏΡ€ΠΈ косвСнном ΠΈΠ·ΠΌΠ΅Ρ€Π΅Π½ΠΈΠΈ Π΄ΠΈΠ°ΠΌΠ΅Ρ‚Ρ€Π° капилляров.

рисунок 2

Ξ” D1 = D1(Δσ/ Οƒ + Ξ” h1/ h1);

Ξ” D2 = D2(Δσ/ Οƒ + Ξ” h2/ h2).

ΠŸΠΎΠ³Ρ€Π΅ΡˆΠ½ΠΎΡΡ‚ΡΠΌΠΈ Ξ” g ΠΈ Ξ” ρ ΠΌΠΎΠΆΠ½ΠΎ ΠΏΡ€Π΅Π½Π΅Π±Ρ€Π΅Ρ‡ΡŒ.

ΠžΠΊΠΎΠ½Ρ‡Π°Ρ‚Π΅Π»ΡŒΠ½Ρ‹ΠΉ Ρ€Π΅Π·ΡƒΠ»ΡŒΡ‚Π°Ρ‚ измСрСния Π΄ΠΈΠ°ΠΌΠ΅Ρ‚Ρ€Π° капилляров ΠΏΡ€Π΅Π΄ΡΡ‚Π°Π²ΡŒΡ‚Π΅ Π² Π²ΠΈΠ΄Π΅